You are on page 1of 360

Патофізіологія системи крові

У жінки 42 років з пухлиною наднирників, що продукує альдостерон, артеріальний тиск крові


складає 160/90 мм.рт.ст. Обстеження не виявило відхилень в клітинному складі крові, лише
величина гематокриту є 0,32. Яке порушення загального об’єму крові є в хворої?
@Гіперволемія олігоцитемічна
У дитини з опікової хворобою (площа опіків біля 25 % поверхні тіла), гематокрит складає 0,65.
Яке порушення загального об’єму крові має місце в даному випадку?
A. @Гіповолемія поліцитемічна

У хворого з залізодефіцитною анемією гематокрит складає 0,31. Як можна охарактеризувати


зміни загального об’єму крові в даному випадку?
@Ізоволемія олігоцитемічна
4. При пораненні пацієнт втратив 15 % об’єму крові. Як можна охарактеризувати зміни загального
об’єму крові через 5 діб після крововтрати?
@Ізоволемія олігоцитемічна
5. На 7 добу після гострої шлункової кровотечі в 50-річного чоловіка з’явились ознаки нетермінових
механізмів компенсації. Які механізми компенсації відносяться до нетермінових?
@Збільшення синтезу білка в печінці

6. При пораненні пацієнт втратив 15 % об’єму крові. Який з нижченаведених механізмів забезпечить
відновлення кисневої ємності крові?
A. @Посилення еритропоезу

7. При пораненні пацієнт втратив 10 % об’єму крові. Який з нижченаведених механізмів швидко
компенсує зниження об’єму крові?
@Надходження тканинної рідини до судин

8. При пораненні пацієнт втратив 15 % об’єму крові. Який з нижченаведених механізмів швидко
нормалізує артеріальний тиск?
@Спазм кровоносних судин

9. При пораненні пацієнт втратив 15 % об’єму крові. Який з нижченаведених механізмів швидко
забезпечить компенсацію гіпоксії?
A. @Збільшення хвилинного об’єму серця
10. Хворий скаржиться на біль у животі, кал з домішками крові, загальну слабкість. При обстеженні
виявлено пухлину товстої кишки і анемію: гемоглобін 4,4 ммоль/л, еритроцити 2,8 • 10 12/л,
колірний показник 0,7. У мазку крові: анізоцитоз, пойкілоцитоз, помірна кількість
поліхроматофільних еритроцитів. Яка анемія в хворого?
@Хронічна постгеморагічна

11. При обстеженні пацієнта, що скаржиться на головний біль та порушення зору, виявлено:
еритроцити 7 • 1012/л, гемоглобін 12 ммоль/л, лейкоцити – 6,8 • 10 9/л, тромбоцити – 220 • 10 9/л.
Охарактеризуйте зміни в крові пацієнта:
@Еритроцитоз
12. На восьмому місяці вагітності в жінки розвинулася мегалобластна анемія. Які клітини
периферичної крові свідчать про мегалобластичний тип кровотворення?
@Мегалоцити

13. На шостому місяці вагітності в жінки розвинулася залізодефіцитна анемія. Які клітини
периферичної крові свідчать про дефіцит гемоглобіну в еритроцитах?
@Гіпохромні

14. На шостому місяці вагітності в жінки розвинулася гіпохромна анемія. Які клітини периферичної
крові свідчать про низький вміст гемоглобіну в еритроцитах при залізодефіцитній анемії?
@Анулоцити

15. У хворого вміст гемоглобіну в крові складає 4,5 ммоль/л, еритроцитів – 2,5 Т/л, в достатній
кількості присутні клітини, які свідчать про активність фізіологічних механізмів регенерації
«червоної» крові. Які це клітини периферичної крові?
@Поліхроматофільні еритроцити

16. Тиждень тому пацієнт втратив 15 % об’єму крові. При обстеженні: зменшення вмісту
еритроцитів, гемоглобіну, гематокриту, кольорового показника крові, ретикулоцитов - 10%.
Дайте характеристику стану «червоної крові» в пацієнта:
@Анемія регенераторна

17. При анеміях в периферичній крові виявляються регенеративні, дегенеративні і патологічні


форми еритроцитів. Назвіть регенеративну форму еритроцитів.
@Ретикулоцити

18. Пацієнтка С., 17 років, скаржиться на загальну слабкість, хронічну втому. При обстеженні в неї
виявлено анемію. Які з регенеративних форм еритроцитів не можуть бути в периферичній крові
при анемії?
@Еритробласти

19. У крові хворої після хіміотерапії: гемоглобін 5,0 ммоль/л, еритроцити 2,5 Т/л, КП 1,0,
ретикулоцити 0,01%. Визначте анемію в хворої:
@Арегенераторна

20. Для з’ясування стану кісткового мозку при анеміях бажано оцінити його регенераторну
здатність. Визначення кількості яких клітин в крові дозволяє це зробити?
@Ретикулоцити, нормобласти

21. Пацієнт скаржиться на загальну слабкість, швидку втому при невеликому навантаженні. У його
крові зменшено вміст еритроцитів і гемоглобіну. Наявність вільного гемоглобіну в крові
дозволяє діагностувати такий механізм анемії в хворого:
@Гемоліз
22. Привертає увагу жовтувато-лимонний відтінок шкіри в жінки. У крові: гемоглобін 4,6 ммоль/л,
еритроцити 2,8 Т/л, КП 0,82, ретикулоцити 7%; збільшено вміст непрямого білірубіну. Склад
«білої крові» не змінений. Для яких захворювань системи крові така картина крові є типовою:
@Гемолітичні анемії

23. Пацієнт скаржиться на загальну слабкість, втомлюваність, жовтяницю. У крові зменшено вміст
анемія, ретикулоцитів - 5%, збільшення вмісту непрямого білірубіну, гемоглобінурія. Зазначте
механізм анемії в хворого:
@Гемоліз інтраваскулярний

24. Пацієнт скаржиться слабість, жовтушність шкіри. У крові зменшено вміст гемоглобіну,
еритроцитів, КП крові 0,8, ретикулоцитів 5%; кількість лейкоцитів і тромбоцитів в межах норми.
Що може бути механізмом цих змін в крові пацієнта?
@Гемоліз еритроцитів

25. У дитини з наявним резус-антигеном на еритроцитах, що народилася від резус-негативної


матері, зменшено вміст гемоглобіну в крові. Алергічні реакції якого типу (за Кумбсом і Джелом)
лежать в основі цього захворювання?
@II типу (гуморальні цитотоксичні )

26. Пацієнтка скаржиться на напади болю в спині, які супроводжуються появою сечі бурого
кольору; обстеження показало пароксизмальну нічну гемоглобінурію. Надайте патогенетичну
характеристику анемії в хворої:
@Гемолітична набута

27. Точкова мутація гена β-ланцюга гемоглобіну є причиною анемії. Вкажіть типову для даної анемії
патологічну форму гемоглобіну.
@Hb S

28. У пацієнтки з жовчнокам’яною хворобою діагностовано спадкову анемію (мембранопатію). Для


яких анемій холелітіаз є типовим ускладненням?
@Гемолітичні

29. Лікування препаратами заліза не дало позитивного ефекту в хворої на мікроцитарну, гіпохромну
анемію. Подальше обстеження дозволило встановити діагноз таласемії. Дайте характеристику
цієї анемії:
@Гемоглобінопатія

30. У хворої на системний червоний вовчак в крові визначаються антиеритроцитарні антитіла,


зменшення кількості еритроцитів і вмісту гемоглобіну. Надайте етіопатогенетичну
характеристику цієї анемії:
@Гемолітична набута імунна

31. Лікування гіпохромної анемії в пацієнта препаратами заліза була неефективним. При подальшому
обстеженні встановлено діагноз залізорефрактерна, сидеробластна анемія. Дефіцит якої
речовини може бути її причиною?
@Вітаміну В6

32. Через деякий час після резекції пухлини тонкої кишки в хворого виявлено фолієводефіцитну
анемію. Які клітини є типовими в периферичній крові при цьому?
@Мегалобласти
33. У працівника типографії лікування гіпохромної анемії препаратами заліза не було ефективним.
При подальшому обстеженні діагностована залізорефрактерна анемія. Порушення синтезу яких
речовин є причиною цієї анемії?
@Порфіринів

34. У жінки, яка хворіє на атрофічний гастрит, виявлено мегалобластну анемію і лейкопенію.
Порушення синтезу якої речовини є причиною виникнення мегалобластного типу
кровотворення?
@ДНК

35. Хвора скаржиться на слабкість, тривалі менорагії. Об’єктивно: фіброміома матки і гіпохромна
гіпорегенераторна анемія. Який механізм виникнення анемія в хворої?
@Втрата заліза

36. Жінку, яка хворіє на атрофічний гастрит, турбує її швидка втомлюваність, задишка, відчуття
оніміння в кінцівках. При обстеженні виявлено гіперхромну макроцитарну анемію, лейкопенію.
Що є причиною зменшення кількості еритроцитів і лейкоцитів у крові хворої?
@Дефіцит вітаміну В12

37. Жінка скаржиться на зниження працездатності, сонливість, задишку при швидкій ході. При
обстеженні: еритроцити – 4,0 Т/л, гемоглобін – 4,8 ммоль/л, КП – 0,6; анулоцитоз, мікроцитоз.
Що може бути причиною такого стану крові?
@Дефіцит заліза

38. У хворого, якому 5 років тому зробили резекцію шлунка, діагностовано гіперхромну анемію. Що
спричинило розвиток анемії у хворого?
@Дефіцит внутрішнього фактора Касла

39. У хворої на лімфогранулематоз після променевої терапії: гемоглобін 6,0 ммоль/л, еритроцити 3,0
•1012/л, КП 1,0, ретикулоцити 0,1%. Визначте анемію в хворої:
@Гіпопластична

40. У жінки, віком 55 років, яка тривалий час страждає на залізодефіцитну анемію, виявлені
дистрофічні зміни в міокарді. Що могло стати причиною дистрофії міокарда у хворої?
@Зниження активності залізовмісних ферментів

41. У приймальне відділення лікарні поступила дівчинка 9 років з болем у животі. Обстеження
показало гострий апендицит. Як змінюється клітинний склад периферичної крові при гострому
нормергічному запаленні?
@Нейтрофільний лейкоцитоз
42. Дослідження показало, що загальна кількість лейкоцитів в крові пацієнта становить 3,0 Г/л,
лімфоцити складають 10%. Що може бути причиною змін клітинного складу "білої" крові у
цьому випадку?
@Опромінення

43. Загальна кількість лейкоцитів у крові пацієнта складає 10 Г/л; при цьому нейтрофілів - 82%.
Вкажіть можливу причину такого складу "білої" крові:
@Травма, опіки

44. У крові пацієнта загальна кількість лейкоцитів складає 3,0 Г/л; при цьому лімфоцити складають
80%. Охарактеризуйте зміни клітинного складу "білої" крові в зазначеному випадку:
@Нейтропенія

45. Мати хлопчика 4-х років скаржиться на те, що дитина погано їсть, дратівлива. Обстеження
показало, що хлопчик інфікований аскаридами. Які зміни з боку периферичної крові є типовими
для гельмінтозів?
@Еозинофілія

46. До лікарні поступив чоловік 47 років із загрудинним болем. За результатами ЕКГ в нього
діагностовано інфаркт міокарда. Які зміни клітинного складу периферичної крові індуковані
некротичними змінами в міокарді?
@Нейтрофілія

47. До лікарні поступила жінка з опіками. Як в типових випадках при гострому пошкодженні
тканин змінюється клітинний склад периферичної крові?
@Нейтрофілія

48. Юнак звернувся до лікаря зі скаргами на нежить, біль у горлі, гарячку. При обстеженні, у крові:
лейкоцити 10 Г/л, лімфоцити - 55%, нейтрофіли - 45%. Інфекція якого типу найбільш ймовірно
є причиною гострого респіраторного захворювання в пацієнта?
@Віруси

49. Загальна кількість лейкоцитів у крові пацієнта складає 10 Г/л; при цьому лімфоцити складають
56%, нейтрофіли 40 %. Охарактеризуйте зміни клітинного складу "білої" крові в зазначеному
випадку:
@Лімфоцитоз

50. Загальна кількість лейкоцитів у крові пацієнта 7 Г/л, еозинофіли складають 15%. Що може бути
причиною такого порушення складу "білої" крові?
@Глистяна інфекція
У
50. У пацієнта з макроцитарною гіпорегенераторною анемією вміст лейкоцитів в крові складає 4 Г/л,
лімфоцитів 40%, нейтрофілів 40%. Ядра нейтрофілів містять 5-7 сегментів. Вкажіть зміни "білої"
крові у пацієнта.
@Ядерний нейтрофільний зсув вправо

51. Загальна кількість лейкоцитів у крові пацієнта 10 Г/л; при цьому нейтрофілів 75%, серед них
10% - паличкоядерні. Охарактеризуйте зміни клітинного складу "білої" крові в цьому випадку:
@Ядерний зсув нейтрофілів вліво

52. Охарактеризуйте ядерне зрушення нейтрофілів у хворого на гостру бактеріальну пневмонію:


лейкоцити 11 Г/л, лімфоцити - 27%, нейтрофіли - 70%, з них: ю- 2%, п/я - 8%, с/я - 58%.
@Вліво регенераторне

53. У крові хворого на бактеріальну пневмонію збільшено вміст нейтрофілів. Яке зрушення ядер
нейтрофілів периферичної крові є типовим для гострого нормергічного запалення?
@Вліво регенераторне

54. Пацієнт скаржиться на часті інфекційні захворювання. При обстеженні: лейкоцити 3,8 Г/л:
лімфоцити - 40 %, нейтрофіли: п/я — 1 %, с/я — 49 %, гіперсегментація ядер нейтрофілів. Як
можна визначити зрушення "білої" крові у пацієнта?
@ Вправо

55. У дитини з гострим апендицитом у крові: лейкоцити – 11 Г/л, лімфоцити - 23%, нейтрофіли: юні
- 4%, п/я - 8%, с/я - 58%. Яке зрушення нейтрофілів периферичної крові у дитини?
@ Вліво регенеративне

56. У юнака з ангіною у крові: Лейк. - 11•109/л, Лімф. - 21%, Мон. - 5%, Еоз. - 2 %, Баз. - 0%. Нейтр.:
юні - 5%, п/я - 10%, с/я - 57%. Яке зрушення нейтрофілів периферичної крові є типовим для
гострого запалення?
@ Вліво регенеративне

57. При обстеженні у пацієнтки виявлена лейкопенія. У формулі: лімфоцити - 34%, нейтрофіли: юні
- 0%, п/я - 20%, с/я- 39%. Яке зрушення нейтрофілів периферичної крові спостерігається в
даному випадку?
@ Вліво дегенеративне

58. У пацієнта хворого на крупозну пневмонію у крові: лейкоцити 16 Г/л. Лімфоцити - 26%,
нейтрофіли: мієлоцити - 2%, юні - 6%, п/я - 12%, с/я - 50%. Яке зрушення нейтрофілів
периферичної крові у хворого?
@ Вліво гіперрегенеративне

59. У хворого на гострий бронхіт у крові: лейкоцити – 11 Г/л, лімфоцити - 23%, нейтрофіли: юні -
4%, п/я - 8%, с/я- 58%. Лейкоцити з токсичною зернистістю. Яке зрушення нейтрофілів
периферичної крові спостерігається в даному випадку?
@ Вліво регенеративно-дегенеративне
У

60. крові пацієнта: знижено вміст гемоглобіну та еритроцитів, лейкоцитів 27 Г/л, гранулоцитоз,
ядерне зрушення нейтрофілів вліво до промієлоцитів. Що може бути причиною вищезазначених
змін?
@Хронічний мієлолейкоз

61. Хворий 67 р., скаржиться на втомлюваність, схуднення, жовтушність шкіри і склер. При
обстеженні: анемія, лейкоцити — 54 Г/л, тромбоцити — 200 Г/л. Серед лейкоцитів 90%
складають малі та середні лімфоцити. Що може бути причиною такого стану?
@Хронічний лімфолейкоз

62. В абсолютній більшості випадків хронічного мієлолейкозу в клітинах пухлинного клону є


"Філадельфійська" хромосома. Яка подія відбувається із стовбуровою кровотворною клітиною
кісткового мозку внаслідок зазначеної хромосомної аберації?
@Трансформація

63. Трансплантація стовбурових клітин кісткового мозку здатна повністю відновити кровотворну і
лімфатичну системи реципієнта, що був під дією опромінення або цитотоксичних
хіміопрепаратів. Вкажіть потенційне джерело великої кількості стовбурових кровотворних
клітин:
@Пуповинна кров

64. Хворий на гострий лейкоз після кірсу хіміотерапії почіває себе задовільно.
Що можна трактувати як показник стану ремісії при гострому лейкозі?
@Бласти в кістковому мозку менше 5%
65. Хвора скаржиться на часті інфекційні захворювання останнім часом. При обстеженні: анемія,
тромбоцити — 220 Г/л, лейкоцити — 10 Г/л; 70% складають нейтрофіли, з них: 45% -
промієлоцити, 25% - сегментоядерні. Що може бути причиною такого стану в системі крові?
@Гострий лейкоз

66. Хвора Р., 47 р., скаржиться на втомлюваність, носові кровотечі, біль у кістках. При обстеженні:
еритроцити — 2,7 Т/л, гемоглобін — 4,0 ммоль/л, лейкоцити — 14 Г/л, тромбоцити — 70 Г/л.
Серед лейкоцитів крові 7% складають бласти. Що може бути причиною цих змін?
@Гострий лейкоз

67. У крові пацієнта зменшено вміст всіх клітин: анемія, нейтропенія, тромбоцитопенія.
Дослідження клітинного складу кісткового мозку показало гострий мієлолейкоз. Що є
специфічною ознакою гострих лейкозів?
@Збільшення кількості незрілих клітин в кістковому мозку

68. У хворого виявлені анемія, тромбоцитопенія, гранулоцитоз. Загальна кількість лейкоцитів у


крові складає 57 Г/л, ядерний зсув нейтрофілів вліво до промієлоцитів. У кістковому мозку -
гранулоцитоз. Яка патологія наявна в хворого?
@Хронічний мієлолейкоз
69. Пацієнту, який скаржиться на загальну слабкість та біль в кістках, встановлено діагноз
"Хронічний мієлолейкоз". Назвіть хромосомну аномалію, типову для більшості випадків цього
захворювання.
@Транслокація 9 на 22

70. крові пацієнта: анемія, тромбоцитопенія, загальна кількість лейкоцитів складає 3,0 г/л,
лімфоцити складають 60%. Що може бути причиною вищеописаної зміни складу "білої" крові:
@Гострий лейкоз

71. Чоловік 50 р. звернувся до лікаря з приводу збільшення лімфатичного вузла на шиї. Будь-які
неприємні відчуття в цьому місці в нього відсутні. За результатами біопсії діагностовано
злоякісну пухлину з клітини лімфатичного вузла. Як звуться такі пухлини системи крові?
@Лімфома

72. Хворому на лейкоз зроблено аналіз крові. Що найімовірніше вказує на наявність гострого
лейкозу?
@Наявність лейкемічного провалу

73. Після третього курсу хіміотерапії у хворого на хронічний мієлолейкоз відсутні будь–які
позитивні зміни. Яка властивість пухлин пов’язана з розвитком їх резистентності до
хіміотерапії?
@Прогресія

74. У 1908 році Елерман і Банг вперше довели, що віруси можуть викликати лейкоз у курей.
Стосовно якого онковірусу доведено, що він здатен викликати лейкоз у людини?
@Вірус HTLV

75. У хворого на хронічний мієлолейкоз визначається гіперплазія ясен, стоматит. Дослідження


біоптату слизової оболонки ротової порожнини виявило її лейкоцитарну інфільтрацію. З якою
властивістю пухлини пов’язане ураження ротової порожнини у хворого?
@Метастазування

76. У постраждалого після опромінення виник гострий лейкоз. Який механізм трансформації
характерний для фізичних канцерогенів?
@Мутаційний

77. У хворого на хронічний мієлолейкоз на фоні резистентності пухлини до хіміотерапії виник


бластний криз, що було розцінено лікарем як прояв пухлинної прогресії. Що складає суть
пухлинної прогресії?
@Малігнізація

78. У 1907 році Елерман і Банг в експерименті викликали лейкоз у курей шляхом введення їм
безклітинного фільтрату, отриманого з крові курки. Який механізм канцерогенезу вмикають
лише онкогенні віруси?
@Епігеномний
У

79. Стан хворого на хронічний мієлолейкоз раптово значно погіршився, звичайні протипухлинні
препарати перестали діяти. Що можна трактувати як гематологічний прояв пухлинної прогресії
хронічного мієлолейкозу?
@Лейкемічний провал
81. У хворого на цингу спостерігаються кровоточивість ясен та петехії на шкірі. Що спричинює
порушення гемостазу при цьому захворюванні?
@Порушення синтезу колагену

82. У дівчинки після перенесеної ангіни з’явився петехіальний висип на шкірі кінцівок та тулуба.
Об’єктивно: кількість тромбоцитів 80 Г/л, антитромбоцитарні антитіла. Алергічні реакції якого
типу (за Кумбсом і Джелом) лежать в основі цього захворювання?
@II типу (гуморальні цитотоксичні )

83. У хворого порушена адгезія тромбоцитів до колагену та спостерігаються кровотечі з дрібних


судин. Порушення якої ланки гемостазу можна припустити у хворого?
@Судинно-тромбоцитарної

84. У хворого діагностовано тромбоцитопенію. Які клінічні прояви типові для порушень
тромбоцитарно – судинного гемостазу?
@Петехії, екхімози (синці)

85. У хворої при обстеженні виявили тромбоцитопатію. Вкажіть, яка зміна відіграє важливу роль в
патогенезі тромбоцитопатій?
@Продукція патологічних тромбоцитів кістковим мозком

86. Перед проведенням оперативного втручання виявлено, що в людини час кровотечі збільшений
до 10 хв. Дефіцит яких формених елементів у складі крові може бути причиною таких змін?
@Тромбоцитів

87. У пацієнтки тривале вживання аспірину викликало крововиливи. Об’єктивно:


тромбоцитопенія з порушенням функціональної активності тромбоцитів. Тромбоцитопатія в
даному випадку зумовлена пригніченням активності:
@Циклооксигенази

88. У хворого діагностовано зменшення продукції ендотелієм судин фактора Віллебранда. Яке
порушення судинно-тромбоцитарного гемостазу спостерігається при цьому?
@Порушення адгезії тромбоцитів
89. У дитини з геморагічним висипом, що виник після ГРВІ, діагностовано геморагічний васкуліт
(хвороба Шенляйн – Геноха). Алергічні реакції якого типу (за Кумбсом і Джелом) лежать в
основі цього захворювання?
@III типу (імунокомплексні)

90. У дитини з геморагічним висипом, що виник після ГРВІ, діагностовано геморагічний васкуліт
(хвороба Шенляйн – Геноха). Що спричинює порушення гемостазу при цьому захворюванні?
@Пошкодження судинної стінки

91. У жінки, що страждає на жовчно-кам’яну хворобу виявлено геморрагічний синдром, зумовлений


дефіцитом вітаміну К. Який з перелічених факторів є неповноцінним при гіповітамінозі К?
@Стюарта-Прауера (ф. X)
92. У дитини з бронхіальною астмою спостерігається підвищена кровоточивість, сповільнення
процесу згортання крові, що зумовлено вивільненням однієї з біологічно активних речовин з
тканинних базофілів. Яка речовина призвела до патології згортання крові в даному випадку?
@Гепарин

93. У пацієнта при обстеженні було виявлено тромбофілію (прискорення процесу зсідання крові ).
Що могло стати причиною порушення?
@Дефіцит інгібіторів протеолітичних ферментів

94. У хворого із захворюванням печінки виявлено зниження вмісту протромбіну в крові. Це


призведе до порушення перш за все:
@Другої фази коагуляційного гемостазу

95. У хворого, що страждає на стрептококову інфекцію, розвинувся геморагічний синдром. Яка


причина підвищеної кровоточивості при цьому?
@Посилений фібриноліз

96. У хворого на хронічний гепатит видалили зуб. Кровотечу, що виникла після цього, не вдавалося
зупинити протягом 2-х годин. Проведене дослідження гемостазу встановило зменшення вмісту
декількох факторів згортання крові. Який вид гемостазу порушений у цьому випадку?
@Коагуляційний

97. У хворого 45 років діагностована механічна жовтяниця, що супроводжується стеатореєю та


геморагіями на шкірі. У крові - зменшення вмісту протромбіну та інших факторів зсідання крові.
Яка причина порушення коагуляційного гемостазу в даного хворого?
@Порушення всмоктування в кишечнику вітаміну К

98. У хлопчика 7 років, після падіння з велосипеда виник гемартроз колінного суглоба. Введення
кріопреципітату та відкачування крові з суглоба призвело до значного поліпшення стану дитини.
Про яке захворювання слід думати?
@Гемофілія А

99. У хлопчика з вираженим геморагічним синдромом в плазмі крові відсутній антигемофильній


глобулін А (фактор VIII). Яка фаза гемостазу первинно порушена у цієї дитини?
@Внутрішній шлях активації протромбінази

100. Хлопчик страждає на гемофілію. Якими клінічними ознаками проявляються порушення


коагуляційного гемостазу?
@Гематомами, тривалими кровотечами

101. У постраждалого внаслідок ДТП через деякий час після перенесеної політравми розвинувся
синдром дисемінованого внутрішньосудинного зсідання крові (ДВЗ). Який фактор був
ініціатором цього ускладнення?
@Тканинний тромбопластин (ф. III)

102. У хворого на гострий панкреатит розвинувся синдром дисемінованого внутрішньосудинного


зсідання крові (ДВЗ). Яка речовина була ініціатором цього ускладнення?
@Трипсин

103. У хворого на політравму з гострою нирковою недостатністю, стан ускладнився внутрішньою


кровотечею. Що є головною ланкою патогенезу цієї стадії ДВЗ - синдрому?
@Споживання факторів згортання крові

104. У хворого з опіковою хворобою як ускладнення розвинувся ДВЗ - синдром. Яку стадію ДВЗ -
синдрому можна запідозрити, якщо відомо, що кров хворого згортається менше ніж за 3 хв.?
@Гіперкоагуляції

105. У хворого з травмою розвинувся ДВЗ - синдром. Які зміни гемостазу спостерігаються в ІІ фазу
ДВЗ -синдрому? @Гіпокоагуляція

106. У пацієнта після переливання несумісної крові виник ДВЗ – синдром. Що є головною ланкою в
патогенезі цього ускладнення при гемолізі?
@Надходження у кров внутрішньоклітинних протеаз

106. У вагітної жінки відбулося відшарування плаценти, розвилася перша стадія ДВЗ - синдрому,
що характеризується прискоренням процесу згортання крові. Який фактор спричинив
виникнення цього синдрому в даному випадку?
@Надлишок у крові тромбопластину

107. У хворого на тлі хронічної ниркової недостатності розвився ДВЗ-синдром. При обстеженні
виявлено збільшення часу згортання крові, тромбоцитопенію, підвищення рівня фібрин-
мономерних комплексів та продуктів деградації фібрину. Про яку стадію ДВЗ-синдрому слід
думати?
@Гіпокоагуляції

108. У пацієнта перебіг токсичної гемолітичної анемії ускладнено розвитком гострої ниркової
недостатності. Що є головною ланкою в патогенезі цього ускладнення при гемолізі?
@ДВЗ - крові

109. У хворого на хронічний лімфолейкоз виникли геморагії внаслідок розвитку ДВЗ - синдрому.
Які зміни показників периферичної крові будуть спостерігатися при цьому?
@Гіпокоагуляція, тромбоцитопенія

111. У жінки, 22 років, діагностовано міокардит, є ознаки серцевої недостатності. Який вид
серцевої недостатності за патогенезом виник у хворої? A. Від перевантаження серця підвищеним
опором відтоку крові.
@Від ушкодження міокарда.

112. У хворого наслідком гіпертонічного кризу стала гостра серцева недостатність. Який головний
механізм виникнення серцевої недостатності у цьому випадку?
@ Перевантаження серця опором.
113. У хворого з недостатністю мітрального клапана виникла гостра серцева недостатність. Який
патофізіологічний варіант недостатності серця спостерігається в цьому випадку? @Від
перевантаження серця.
114. У хворого 65 років діагностовано повну атріо-вентрикулярну блокаду, яка ускладнилась
серцевою недостатністю. Який патогенетичний варіант серцевої недостатності у хворого?
@Міокардіальна, аритмічна.

115. У хворого спостерігається різке підвищення артеріального тиску за рахунок зміни тонусу
судин. Який компенсаторний механізм забезпечує збільшення сили скорочень міокарду у цьому
випадку?
@Гомеометричний.
116. У хворого після гостро перенесеного захворювання діагностовано недостатність мітрального
клапану, але ознак серцевої недостатності не виявлено. Який негайний механізм компенсації
забезпечує в такому випадку гіперфункцію серця?
@Гетерометричний механізм

117. У чоловіка, 25 років, виявлено недостатність мітрального клапана без порушення кровообігу.
Який довготривалий механізм забезпечує компенсацію?
@Гіпертрофія міокарда.

118. Через місяць після виникнення експериментальної артеріальної гіпертензії в собаки товщина
стінки лівого шлуночка серця збільшилась в 1,7 рази. Надалі маса серця не збільшувалась,
хвилинний об’єм серця нормалізувався.
Яка стадія гіпертрофії міокарда спостерігається у тварини?
@Завершеної гіпертрофії.

119. У тварини з недостатністю аортальних клапанів розвинулась гіпертрофія лівого шлуночка


серця. В окремих його ділянках визначаються локальні контрактури. Накопичення якої речовини в
кардіоміоцитах зумовило контрактури?
@Кальцію.

120. У хворого з недостатністю мітрального клапана виникла гіпертрофія лівого шлуночка серця.
Який механізм розвитку гіпертрофії є провідним?
@Активація генетичного аппарату кардіоміоцитів

121. У хворого 25 років після перенесеного гострого ендокардиту виникла недостатність клапанів
аорти, що викликала гіпертрофію лівого шлуночка серця. Через вісім тижнів маса серця збільшилась
на 65 % і ріст її зупинився. Що викликало припинення подальшої гіпертрофії?
@Нормалізація навантаження на одиницю м’язової маси серця

122. Через 1 годину після накладання кільця, що звужує аорту, у собаки зросла сила та частота
серцевих скорочень, а об’єм циркулюючої крові та товщина стінки лівого шлуночка не відрізнялися
від вихідних показників. Яка стадія гіпертрофії міокарда спостерігається у тварини? @Аварійна.

123. У дитини 14 років з відкритою артеріальною протокою маса серця в 3 рази перевищує вікову
норму. Стан дитини прогресивно погіршується: спостерігаються стійкий ціаноз, задишка, періодичні
запаморочення. Яка особливість гіпертрофованого міокарда сприяє виникненню декомпенсації?
@Зниження енергозабезпечення міокардіоцитів.

124. У дитини 14 років з відкритою артеріальною протокою маса серця в 3 рази перевищує вікову
норму. Стан дитини прогресивно погіршується: спостерігаються стійкий ціаноз, задишка, періодичні
запаморочення. Яка стадія гіпертрофії міокарда спостерігається у дитини? @Прогресуючого
кардіосклерозу.

125. У тварини з мітральним стенозом розвинулась гіпертрофія лівого передсердя. Який показник
свідчить про перехід гіпертрофії у другу стадію?
@Нормалізація інтенсивності функціонування структур.
126. У собаки з аортальним стенозом розвинулась гіпертрофія лівого шлуночка серця. Який
показник свідчить про наявність першої стадії гіпертрофії? @Збільшення інтенсивності
функціонування структур.

127. У хворого розвинулася гостра лівошлуночкова недостатність, яка швидко ускладнилася


набряком легень. Що є головною ланкою в патогенезі кардіогенного набряку легень?
@Збільшення гідростатичного тиску крові

128. У хворого на гіпертонічну хворобу під час фізичного навантаження з’явились відчуття нестачі
повітря, ціаноз, дистанційні вологі хрипи. Що є найтяжчим клінічним проявом лівошлуночкової
недостатності серця?
@Набряк легень

129. У хворого на гіпертонічну хворобу під час фізичного навантаження з’явились відчуття
м’язової слабкості, нестачі повітря. Який рефлекс запобігає розвитку набряку легень при
лівошлуночкової недостатності серця?
@ Китаєва

130. У хворого на правошлуночкову серцеву недостатність виникли асцит та набряки. Який


основний патогенетичний механізм розвитку набряків у цього хворого?
@Збільшення гідростатичного тиску крові у венах

131. У хворого на ревматизм розвинувся міокардит з проявами недостатності кровообігу. Яке


порушення гемодінаміки спостерігається при цьому? @Зниження систолічного артеріального тиску.

132. Хворий, 39 років, з хронічною серцевою недостатністю скаржиться на задишку при


невеликому фізичному навантаженні, набряки на ногах. Що може бути свідченням декомпенсації
функції серця у хворого?
@Тахікардія

133. Хвора 45 років скаржиться на задишку при невеликому фізичному навантажені, набряки на
ногах. Діагностовано недостатність кровообігу. Який гемодинамічний показник свідчить про
декомпенсацію серця?
@Зменшення хвилинного об’єму серця

134. У хворого з міокардіодистрофією розширені порожнини серця, знижена сила серцевих


скорочень, збільшилися кінцевий систолічний об’єм шлуночків та кровонаповнення вен. Про який
функціональний стан серця свідчать ці порушення?
@Міогенна дилатація

135. Хворий 65 років скаржиться на загальну слабість, серцебиття й задишку при помірному
фізичному навантаженні, набряки та ціаноз нижніх кінцівок. Діагностовано серцеву недостатність.
Чим зумовлений ціаноз у хворого?
@Збільшенням кількості відновленого гемоглобіну в крові
136. У підлітка після перенесеного інфекційного захворювання спостерігається коливання частоти
серцевих скорочень залежно від фази дихання. Який механізм виникнення дихальної аритмії у
хворого?
@Коливання тонусу блукаючого нерва

137. Дані ЕКГ пацієнта з гіпертонічною хворобою: ритм синусовий, правильний, частота серцевих
скорочень - 92 за 1 хв., тривалість інтервалу Р-Q 0,19 с., комплекс QRS не змінений. Яка функція
серця порушена?
@Автоматизм

138. Після тяжкої інфекції у хворого розвинувся міокардит з ушкодженням провідної системи
серця і виникненням синдрому Морганьї – Адамса - Стокса. Яке порушення ритму у пацієнта?
@Перехід неповної атріовентрикулярної блокади в повну.
139. У хворого виявлено екстрасистолію. Дані ЕКГ: відсутній зубець Р, комплекс QRS
деформований і розширений, наявна повна компенсаторна пауза. Вкажіть локалізацію ектопічного
вогнища збудження в цьому випадку?
@Шлуночки.

140. У хворого на дифузний токсичний зоб часто виникають серцебиття, слабкість, дефіцит пульсу.
На ЕКГ виявлено відсутність зубців Р, замість них численні хвилі f, неоднакові інтервали R-R. Яке
порушення серцевого ритму у хворого?
@Миготіння передсердь

141. Нормальний ритм серця у хворої раптово перервався нападом скорочень частотою 200 уд/хв.
тривалістю до 5 хв. Назвіть можливий наслідок тривалої пароксизмальної тахікардії.
@Зменшення серцевого викиду

142. На ЕКГ лікар виявив у окремих серцевих циклах скорочення інтервалу Т-Р, двофазний зубець
Р, комплекс QRS без порушень, неповна компенсаторна пауза. Для якої аритмії характерні такі зміни
на ЕКГ?
@Передсердної екстрасистолії

143. У хворого, що страждає на важку форму порушення водно-сольового обміну, настала зупинка
серця в діастолі. Що є найбільш імовірною причиною зупинки серця в діастолі?
@Гіперкаліємія

144. На ЕКГ у хворої на міокардит виявлено подовження PQ інтервалу до 0,4 сек. Яка властивість
провідної системи порушена?
@Провідність

145. Під час операційного втручання трапилося рефлекторне посилення впливу блукаючого нерва
на серце. Що при цьому може виникнути?
@Зупинка серця

146. Чоловік, 57 років, скаржиться на біль у ділянці серця, який виник після негативних емоцій.
Нітрогліцерин достатньо швидко усунув біль. Який механізм ішемії найбільш імовірний?
@Ангіоспастичний.
147. У хворого, що скаржиться на стенокардію у спокої, рівень холестерину у крові становить 8
ммоль/л. Що, найімовірніше, є причиною вінцевої недостатності у хворого? @Атеросклероз
вінцевих артерій
148. При ішемії у міокардіоцитах виникає контрактура міофібрил, активація фосфоліпази А 2 та
роз’єднання окиснення з фосфоруванням. Який механізм пошкодження клітин домінує у цьому
випадку?
@Кальцієвий

149. Емоційний стрес у пацієнта похилого віку був причиною виникнення гострого болю за
грудиною. Лікар швидкої медичної допомоги встановив ішемічну хворобу серця. Що є причиною
розвитку стенокардії у хворого?
@Гостра вінцева недостатність

150. У експерименті на коронарну артерію наклали лігатуру, а через 60 хв. зняли, що призвело до
збільшення зони некрозу порівняно з очікуваною. Що лежить в основі даного явища?
@Реперфузійний синдром

151. У хворого, який раптово помер, патологоанатомічно виявили обтуруючий тромб у лівій
коронарній артерії, але не виявили некрозу. Що, найімовірніше, стало причиною ранньої серцевої
смерті у хворого?
@Фібриляція шлуночків

152. Відновлення коронарного кровообігу у хворого на інфаркт міокарда супроводжувалося


зниженням скорочувальної здатності серця і погіршенням загального стану пацієнта. Які реакції
визначають збільшення пошкодження кардіоміоцитів в умовах реперфузії?
@Вільнорадикальні

153. Через 3 тижні після ангіни у хворого виник осередковий некроз міокарда. У крові виявили
міокардіотоксичні антитіла. Алергічні реакції якого типу лежать в основі виникнення некрозу серця
у даному випадку?
@II типу (гуморальні цитотоксичні)

154. У хворого з інфарктом міокарда артеріальний тиск різко знизився до 70/40 мм рт. ст. Який
первинний механізм розвитку артеріальної гіпотензії?
@Зменшення хвилинного об’єму серця.

155. У хворого з інфарктом міокарда розвинулася серцева недостатність. Який патогенетичний


механізм її розвитку?
@Зменшення маси функціонуючих кардіоміоцитів
156. Після перев’язки однієї з гілок коронарних артерій у собаки розвинувся інфаркт міокарда,
який супроводжувався проявами резорбційнонекротичного синдрому. Яка характерна
ознака розвитку цього синдрому?
@Збільшення вмісту креатинфосфокінази в крові.
157. Хворий з інфарктом міокарда скаржиться на гарячку, лабораторно виявлені нейтрофільний
лейкоцитоз, підвищення ШОЕ. Яка речовина викликала ці зміни?
@Інтерлейкін 1

158. У пацієнта, у якого 1,5 міс тому виник інфаркт міокарда, діагностовано синдром Дресслера з
характерною тріадою: перикардитом, плевритом, пневмонією. Який головний механізм цього
ускладнення?
@Автоімунне ураження.

159. У тварини, якій у порожнину перикарда вводили фізіологічний розчин виникла гостра
тампонада серця. Який процес первинно порушується при цьому виді недостатності серця?
@Діастолічне розширення серця

160. У тварини, якій у порожнину перикарда вводили фізіологічний розчин виникла гостра
тампонада серця. Який компенсаторний механізм спрацьовує при цьому виді недостатності серця?
@Тахікардія

161. Хворий 67 років страждає на атеросклероз судин головного мозку. При обстеженні виявлено
гіперліпопротеїнемію IIб типу. Який клас ліпопротеїнів плазми крові значно підвищений у хворого?
@Ліпопротеїни низької щільності

162. Після тривалого введення великих доз ергокальциферолу у тварини виникли склерозування та
кальциноз середньої оболонки артерій. Який патологічний процес моделювали в експерименті?
@Артеріосклероз Менкеберга

163. У дитини 10 років з сухожильними ксантомами та тяжкими нападами стенокардії виявлено


значне збільшення вмісту ліпопротеїнів низької щільності у плазмі крові. Який дефект лежить в
основі виникнення первинної гіперліпопротеїнемії у хворого?
@Дефект рецептора апоВ/Е

164. У хворого при ангіографії виявили значне атеросклеротичне ураження ниркових артерій. До
яких наслідків призводить ця патологія?
@Артеріальна гіпертензія

165. У хворого на цукровий діабет виявлено інтенсивний прогресуючий розвиток атеросклерозу,


що проявлялося розвитком ішемічної хвороби серця, стенозуванням ниркових артерій, порушеннями
мозкового кровообігу. Що є найбільш імовірною причиною розвитку атеросклерозу?
@Глікерування апоВ/Е

166. У тварини змоделювали артеріосклероз Менкеберга. Що є його основними проявами?


@Кальциноз та склероз медії

167. Хворий 67 років страждає на атеросклероз судин головного мозку. При обстеженні виявлено
гіперхолестеринемію. Який клас ліпопротеїнів плазми крові найбільш імовірно значно знижений у
хворого?
@Ліпопротеїни високої щільності

168. У кроля, якого тримали на раціоні з великим вмістом тригліцеридів, виникло


атеросклеротичне ураження артерій. Що було головною причиною розвитку атеросклерозу в даному
випадку?
@Ендогенна гіперхолестеринемія

169. У хворого на цукровий діабет виявлено інтенсивний прогресуючий розвиток атеросклерозу,


що проявлялося розвитком ішемічної хвороби серця, стенозуванням ниркових артерій, порушеннями
мозкового кровообігу. Що є найбільш імовірною причиною розвитку атеросклерозу?
@Поява модифікованих ліпопротеїнів

170. У хворого при ангіографії виявили значне атеросклеротичне ураження внутрішніх сонних
артерій. До яких наслідків може призвести ця патологія?
@Ішемічний інсульт

171. У хворого, 43 років, артеріальна гіпертензія є наслідком помірного збільшення серцевого


викиду і загального периферичного опору. Який гемодинамічний варіант артеріальної гіпертензії має
місце в цьому випадку? @Еукінетичний.

172. Дівчина астенічної конституції скаржиться на загальну слабкість, швидку втомлюваність,


зменшення працездатності. Об’єктивно: пульс - 98 за 1 хв., АТ - 90/60 мм. рт. ст. Який механізм
зумовив гіпотензію у хворої?
@Зниження периферійного судинного опору

173. У хворого виявлено артеріальну гіпертензію з гіпокінетичним типом кровообігу. Який фактор
найбільш істотно впливає на артеріальний тиск?
@Периферичний судинний опір

174. У хворого з гіпертонічною хворобою різко підвищився артеріальний тиск до 180/105 мм рт. ст.
Яке ускладнення, найбільш вірогідно, може виникнути у хворого в цьому випадку?
@Гостра серцева недостатність

175. При критичному зниженні температури тіла в третьому періоді лихоманки на фоні тахікардії
АТ = 80 / 60 мм рт. ст. До якої форми порушення судинного тонусу належить це явище?
@Гострої гіпотензії

176. У хворого з гіпертонічною хворобою різко підвищився артеріальний тиск до 210/150 мм рт. ст.
Яке ускладнення, найбільш вірогідно, може виникнути у хворого в цьому випадку?
@Геморагічний інсульт

177. У чоловіка, що тривалий час страждає на артеріальну гіпертензію, останнім часом систолічний
тиск почав знижуватися, а діастолічний залишається підвищеним. Який гемодинамічний тип
артеріальної гіпертензії у хворого? @Гіпокінетичний.
178. Хворий страждає на гіпертонічну хворобу 5 років. Яке ускладнення, найбільш вірогідно, може
виникнути в цьому випадку?
@Гіпертрофія лівого шлуночка серця

179. У чоловіка, що 15 років хворіє на гіпертонічну хворобу, виявлено гіпокінетичний тип


кровообігу. Що є причиною стійкого збільшення судинного опору при цьому?
@Артеріолосклероз

180. У хворого, 18 років, артеріальна гіпертензія є наслідком значного збільшення серцевого


викиду при нормальному периферичному опорі судин. Який гемодинамічний варіант артеріальної
гіпертензії має місце в цьому випадку?
@Гіперкінетичний.
181. У хворого на стійку артеріальну гіпертензію значно погіршився стан під час лікування артриту
індометацином. Що призвело до посилення артеріальної гіпертензії?
@Гіпопродукція простагландинів.

182. У тварини, якій видалили обидві нирки, на фоні регулярного гемодіалізу виникла стійка
артеріальна гіпертензія. Що призвело до підвищення артеріального тиску?
@Гіпопродукція простагландинів і кінінів.

183. Артеріальна гіпертензія у хворого зумовлена вираженим атеросклерозом ниркових артерій.


Гіперактивність якої системи є головною ланкою в патогенезі цієї форми гіпертензії?
@Ренін-ангіотензинової

184. У хворого на феохромоцитому (пухлину мозкової речовини наднирників) розвинулася


артеріальна гіпертензія. Що стало причиною підвищення артеріального тиску?
@Збільшення хвилинного об’єму серця

185. У хворого на мікседему розвинулася артеріальна гіпотензія. Що стало причиною зниження


артеріального тиску?
@Зниження хвилинного об’єму серця

186. У жінки 25 років, що скаржиться на болі в шиї та голові, виявлені при обстеженні
остеохондроз шийного відділу хребта та нестійка артеріальна гіпертензія. Який різновид
симптоматичної гіпертензії у хворої? @Центрально-ішемічна.

187. У хворого на синдром Конна (первинний гіперальдостеронізм) розвинулася артеріальна


гіпертензія. Що стало причиною підвищення артеріального тиску?
@Гіпернатріємія

188. У хворого на синдром Іценка - Кушинга розвинулася артеріальна гіпертензія. Що є


механізмом підвищення артеріального тиску?
@Гіперволемія
189. У чоловіка з хворобою Аддісона травма викликала значне погіршення стану. Які зміни в
системі кровообігу характерні для таких хворих? @Гіпотензія
190. У хворого на Базедову хворобу розвинулася артеріальна гіпертензія. Що стало причиною
підвищення артеріального тиску?
@Збільшення хвилинного об’єму серця

191. У недоношеної дитини при народженні діагностували рестриктивну дихальну недостатність.


Яка причина лежить в основі недостатності дихання в цьому випадку?
@Дефіцит сурфактанта

192. При обстеженні у хворого виявили обструктивну форму дихальної недостатності. Яка причина
могла б її викликати?
@Запалення бронхів

193. У непритомного юнака, що отруївся морфіном, спостерігається рідке поверхневе дихання. Яка
дихальна недостатність виникла при цьому?
@Вентиляційна дисрегуляторна

194. У хворого з поліомієлітом діагностовано значні патологічні зміни альвеолярної вентиляції.


Порушення функції якої ланки системи зовнішнього дихання спостерігається в цьому випадку?
@Мотонейронів спинного мозку

195. Хворий з мітральним стенозом скаржиться на важку задишку, ціаноз, кашель. Яка дихальна
недостатність спостерігається при цьому?
@Перфузійна

196. При обстеженні хворого виявили перфузійну форму дихальної недостатності. Яка причина
могла її викликати?
@Емболія легеневої артерії

197. У шахтаря виявлений фіброз легенів, який супроводжується порушенням альвеолярної


вентиляції. Який провідний механізм виникнення вентиляційної недостатності в цьому випадку?
@Зменшення розтяжності легенів

198. У підлітка, 12 років, виник важкий напад бронхіальної астми: виражена експіраторна задишка,
блідість шкіри. Який вид порушення альвеолярної вентиляції має місце?
@Обструктивний
199. У хворого на цукровий діабет виникла діабетична кома, яка супроводжувалася глибоким
гучним диханням. Як називається такий тип дихання?
@Гіперпное

200. У хворого, 62 років, після мозкового крововиливу спостерігається наростання глибини і


частоти дихання, а потім зменшення до повного припинення, після чого цикл дихальних рухів
відновлюється. Який тип дихання виник у хворого?
@Чейна-Стокса
201. У результаті виробничої травми у хворого виявлено перелом декількох ребер. Який характер
дихання за звичай спостерігається в таких випадках?
@Тахіпное

202. Анафілактичний шок у морської свинки зумовив зміну характеру дихання. Який тип дихання
спостерігається в цьому випадку?
@Експіраторна задишка

203. Хворий з черепно-мозковою травмою госпіталізований у важкому стані. Його дихання


характеризується судомним подовженим вдихом, який переривається рідкими короткими
видихами. Для якого типу дихання це характерно?
@Апнейстичного

204. У хворого з двостороннім запаленням легенів з’явилась задишка. Назвіть найбільш суттєву
ознаку для визначення диспное.
@Відчуття нестачі повітря

205. У тварин було перерізано обидва блукаючих нерва. Яким стане дихання в цьому випадку?
@Рідким та глибоким

206. При обстеженні у хворого діагностували обструкцію нижніх дихальних шляхів. Яке
порушення дихання виникло?
@Експіраторна задишка

207. Хворий був госпіталізований в стані діабетичної коми. Дихання шумне, часте, після глибокого
вдиху йде посилений видих. Який тип дихання спостерігається у цього хворого? @Куссмауля E.
Гаспінг
208. У хворого в стані агонії після короткочасної зупинки дихання почали реєструватися поодинокі
зітхання, сила і частота яких поступово зменшувалися. Якому типу дихання відповідають
подібні прояви?
@Гаспінг-диханню

209. У хворого на дифтерію виник набряк гортані. Назвіть характерний тип дихання при цьому.
@Рідке та глибоке (стенотичне)

210. Під час операції неадекватно висока вентиляція легенів ускладнилась розвитком судом у
пацієнта. Який розлад кислотно-основного стану може бути причиною їх розвитку?
@Алкалоз газовий

211. У хворого 76 років виявлено значне збільшення залишкового об’єму легенів. Яке
захворювання може викликати ці зміни?
@Емфізема
212. Під час аускультації хворого попросили глибоко дихати. Після 5 хвилин інтенсивних
дихальних рухів хворий відчув запаморочення. Це викликано:
@Гіпокапнією

213. У травмпункт доставили потерпілого з травматичним ураженням грудної клітки. На


рентгенограмі виявлено газ у плевральній порожнині. Назвіть характерну для пневмотораксу
ознаку.
@Зниження транспульмонального тиску

214. У хворого внаслідок патологічного процесу збільшилась товщина альвеолокапілярної


мембрани. Безпосереднім наслідком цього буде зменшення:
@Дифузійної здатності легенів

215. У хворого на фіброзно-кавернозний туберкульоз легенів різко посилились кашель з


виділенням гнійно-слизової мокроти з домішками крові та слабкість. Яка причина вентиляційної
недостатності, що виникла у хворого?
@Зменшення кількості функціонуючих альвеол

216. Хворий на хронічний бронхіт скаржиться на задишку під час фізичного навантаження,
постійний кашель з виділенням мокроти. При обстеженні діагностовано ускладнення – емфізема
легенів. Вона зумовлена зменшенням:
@Еластичних властивостей легенів

217. У хворого на туберкульоз легенів спостерігається значний дефіцит сурфактанту. Які


патологічні зміни в легеневій тканині можуть виникнути в цьому випадку?
@Ателектаз

218. У хворого внаслідок хронічного захворювання органів дихання на тлі задишки, тахікардії та
ціанозу під час дослідження газового складу крові виявлено розвиток гіпоксемії та гіперкапнії.
Яке порушення зовнішнього дихання спостерігається у цього хворого?
@Гіповентиляція

219. У хворого в стані спокою значно збільшена робота дихальних м’язів. Що з нижче
перерахованого може бути причиною цього явища?
@Обструкція дихальних шляхів

220. У робітника цементного заводу, що хворіє на пневмоконіоз, спостерігається задишка,


акроціаноз. Що є причиної цього?
@Зниження дифузійної здатності легенів

221. Після отруєння фосфорорганічними речовинами у хворого виникло тривале підвищення


слиновиділення. До якого порушення в організмі може призвести гіперсалівація?
@Нейтралізації шлункового соку
222. У чоловіка, якому 5 років тому видалили шлунок, стоматолог виявив атрофічно-запальні
процеси слизової ротової порожнини: яскраво-червоний, гладенький («лакований») язик, гінгівіт.
Чим зумовлені ці зміни?
@Дефіцитом вітаміну В12.

223. У тварини в експерименті індукували розвиток карієсу. Який найважливіший компенсаторний


механізм включається при розвитку цього захворювання?
@Утворення вторинного дентину

224. Хворий звернувся до лікаря зі скаргами на гнійні виділення з ясенних кишень і розхитування
зубів. Що є причиною гнійного пародонтиту у пацієнта? @Бактеріальна інфекція

225. До гастроентерологічного відділення потрапив хворий 57 років з підозрою на синдром


Золлінгера - Еллісона, про що свідчило різке збільшення рівня гастрину у сироватці крові. Яке
порушення секреторної функції шлунка найбільш імовірне в цього хворого?
@Гіперсекреція гіперацидна

226. У хворого, який отримав тривалий курс лікування глюкокортикоїдами, виявлені виразки
шлунка. Який механізм є головним у їх розвитку?
@Зменшення регенерації епітелію шлунка

227. Хворий скаржиться на нудоту, печію, біль у епігастральній області, закріп, схуднення. При
обстеженні загальна НСl - 96 ммоль/л, вільна - 60 ммоль/л. Який стан кислотоутворюючої
функції у даного хворого?
@Гіперацидний

228. Для моделювання виразки шлунку тварині ввели в шлункові артерії атофан, який викликає їх
склерозування. Який механізм пошкодження слизової оболонки шлунка є провідним у даному
випадку?
@Гіпоксичний

229. У пацієнта 35 років, хворого на невроз, виявили виразкову хворобу. Який чинник,
найімовірніше, міг стати причинним у виникненні цього захворювання?
@Порушення кірково-підкіркових взаємовідносин

230. У хворого з синдромом Zollinger-Ellison (пухлина з G-клітин з локалізацією в підшлунковій


залозі) відзначається збільшення секреції HCl та пептичні виразки. Яка з перерахованих речовин
викликає цей комплекс симптомів?
@Гастрин

231. У хворої В., яка довгий час приймала саліцилати для лікування ревматизму, під час
фіброгастроскопії знайдена дуоденальна виразка. Який провідний механізм розвитку виразкової
хвороби в даному випадку?
@ Зниження синтезу простагландинів
232. У жінки 52 років з хворобою Іценка - Кушинга підвищена кислотність шлункового соку. Який
механізм забезпечив підвищення секреції в шлунку?
@Зниження синтезу простагландинів

233. Аналіз матеріалу біопсії слизової оболонки шлунка людини, хворої на хронічний гастрит,
показав різке зменшення кількості парієнтальних клітин. Як це відбивається на показниках
аналізу шлункового соку?
@Зменшення кислотності

234. У хворого зі скаргами на оперізуючий біль в епігастральній області, повторну блювоту різко
знизився системний артеріальний тиск. При лабораторному обстеженні знайдено підвищений
вміст діастази в сечі. Про яке ускладнення гострого панкреатиту повинен подумати лікар у
першу чергу?
@Панкреатичний шок

235. У хворої зі скаргами на оперізуючий біль в епігастральній області при лабораторному


обстеженні знайдено підвищений вміст діастази в сечі, а також стеаторея. Для якої форми
патології травного каналу найбільш характерні такі явища?
@Гострий панкреатит

236. У жінки віком 67 років, яка тривалий час страждала на холецистит, після їжі раптово виник
різкий біль у верхній частині живота, нудота, блювання. Встановлено діагноз – гострий
панкреатит. Що є основною ланкою патогенезу цього захворювання?
@Передчасна активація ферментів підшлункової залози

237. У хворого виявлений синдром подразненої кишки, який характеризується значним


посиленням моторики. Виділення якого інкрету найбільш імовірно збільшується при цьому?
@Мотиліну

238. До дитячої лікарні потрапила 8-місячна дитина з діареєю, здуттям живота, гіпотрофією.
Симптоми зявилися і почали наростати після введення в раціон харчування борошняних
виробів. Діагностовано целіакія. Який процес у травному каналі порушений?
@Всмоктування в кишках
239. Хворий, що лікувався антибіотиками з приводу пневмонії, скаржиться на прискорене
виділення рідких випорожнень із специфічним запахом, біль і бурчання в животі. Що,
найімовірніше, стало причиною розвитку проносу?
@Дисбактеріоз

240. У новонародженої дитини діагностована спадкова недостатність лактази. Який процес при
цьому був порушений у дитини?
@Мембранне травлення

241. У хворого, що тривалий час страждає на хронічний ентероколіт, після вживання молока
виникли метеоризм, діарея, коліки. З нестачею якого ферменту в кишечнику це пов'язано?
@Лактаза
242. У хворого після апендектомії припинилося просування химуса. Який вид кишкової
непрохідності за механізмом виник у даному випадку?
@Паралітична

243. У хворого з ущемленою паховою грижею порушене просування химуса. Який вид кишкової
непрохідності має місце в даному випадку?
@Странгуляційна

244. Хворий з високою кишковою непрохідністю скаржиться на біль у животі, багаторазову


блювоту з жовчю, задишку. Об'єктивно: ЧСС – 110 за 1 хв., АТ – 90/50 мм рт. ст., гематокрит -
0,52, гіпонатріємія. Що є провідним механізмом порушень кровообігу у хворого?
@Дегідратація, гіповолемія

245. Хворий з високою кишковою непрохідністю скаржиться на біль у животі, багаторазову


блювоту з жовчю, задишку. Об'єктивно: ЧСС – 110 за 1 хв., АТ – 90/50 мм рт. ст., гематокрит -
0,52, гіпонатріємія. Яке порушення кислотно-основного гомеостазу в хворого?
@Видільний ацидоз

246. У хворого з жовтяницею виявлено: збільшення в крові вмісту прямого білірубіну та


холестерину, білірубінурія, ахолія, поява в крові жовчних кислот. Для якого виду жовтяниці
характерні такі зміни?
@Механічної

247. У жінки, 57 років, після нападу печінкової коліки з'явилася жовтяниця. УЗД показало
закупорку загальної жовчної протоки каменем. Підвищення кількості якої речовини в крові
свідчить про механічну жовтяницю?
@Прямого білірубіну

248. Хворий відзначає часті проноси, особливо після вживання жирної їжі, втрату маси тіла.
Лабораторні дослідження показали наявність стеатореї; кал ахолічний. Що може бути причиною
такого стану?
@Обтурація жовчних шляхів

249. У хворого з жовтяницею виявлені гіпербілірубінемія за рахунок прямого і непрямого


білірубіну, у сечі – прямий білірубін, уробілиноген. Є також гіпохолестеринемія,
гіпопротеїнемія, уповільнення згортання крові. Для якого виду жовтяниці характерні такі зміни?
@Паренхіматозної

250. У хворого з жовтяницею діагностовано синдром Криглера - Наджара. Яке порушення обміну
жовчних пігментів призвело до жовтяниці у хворого?
@Знижена коньюгація білірубіну гепатоцитами

251. Чоловік скаржиться на гарячку, нудоту, біль у правому підребер'ї. Об'єктивно: жовті шкіра та
склери, печінка збільшена, сеча темна, кал знебарвлений, гіпербілірубінемія (білірубін прямий та
непрямий), білірубінурія, уробілінурія, гіпопротеїнемія. Для якого з названих станів характерні
такі зміни?
@Паренхіматозної жовтяниці

252. У хворого з жовтяницею діагностовано синдром Жильбера. Яке порушення обміну жовчних
пігментів призвело до жовтяниці у хворого?
@Знижене захоплення білірубіну гепатоцитами

253. У хворого розвинулась імунна гемолітична анемія. Який показник сироватки крові
збільшиться найбільше?
@Непрямий білірубін

254. До лікаря звернувся пацієнт зі скаргами на пожовтіння склер і шкіри, потемніння сечі і калу.
Аналіз крові: еритроцити - 2,5 • 1012/л, Нв - 80 г/л, кольоровий показник – 0,9, ретикулоцитоз.
Яка жовтяниця розвинулася у цього пацієнта? @Гемолітична
255. У новонародженого з гемолітичною хворобою розвинулася енцефалопатія. Збільшення якої
речовини в крові викликало ураження ЦНС?
@Білірубіну, не зв'язаного з альбуміном

256. В експерименті після дії жовчі на серце жаби спостерігається брадикардія. Який механізм
лежить в основі цього ефекту?
@Активація n. vagus жовчними кислотами

257. У хворого з жовтяницею спостерігається артеріальна гіпотензія, брадикардія, свербіння шкіри,


дратівливість, сонливість вдень і безсоння вночі. Як називається такий синдром?
@Холемічний

258. Жінка, 55 років, скаржиться на швидку стомлюваність, дратівливість, безсоння вночі і


сонливий стан вдень, свербіння шкіри. Попередній діагноз: жовчнокам'яна хвороба із
закупоркою каменем загальної жовчної протоки. Чим зумовлені зміни з боку нервової системи у
хворої?
@Холалемією

259. У хворого з жовтяницею з’явилися порушення травлення, стеаторея, знебарвлення калу,


порушення згортання крові, ознаки авітамінозу. Про розвиток якого синдрому свідчать дані
зміни?
@Ахолічного

260. У хворого, який страждає на жовчнокам’яну хворобу, внаслідок обтурації жовчовивідних


шляхів спостерігається знебарвлений жирний кал. Відсутність якого компонента жовчі зумовлює
стеаторею?
@Жовчних кислот

261. У хворого через наявність каменю у загальній жовчній протоці припинилося надходження
жовчі в кишечник. Порушення якого з процесів спостерігається при цьому?
@Перетравлення жирів
262. У хворого з патологією печінки є ознаки геморагічного синдрому. Який можливий механізм
його виникнення?
@Порушення утворення протромбіну

263. У хворого з алкогольним цирозом печінки скарги на загальну слабкість, задишку. Встановлено
зниження артеріального тиску, асцит, розширення поверхневих вен передньої стінки живота,
спленомегалію. Яке порушення гемодинаміки спостерігається у хворого?
@Синдром портальної гіпертензії

264. У хворого, що страждає на цироз печінки, спостерігається зниження артеріального тиску,


розширення поверхневих вен передньої стінки живота, спленомегалія, асцит. Який механізм є
провідним у виникненні асциту?
@Застійний

265. У хворого 35 років, що переніс гепатит і продовжував приймати алкоголь, розвинулися ознаки
цирозу печінки з асцитом і набряками на нижніх кінцівках. Які зміни складу крові стали
вирішальними в розвитку набряку?
@Гіпоальбумінемія

266. Хвора на хронічний гепатит скаржиться на підвищену чутливість до барбітуратів, які раніше
вона переносила без симптомів інтоксикації. З порушенням якої функції печінки це пов’язане
найбільше?
@Метаболічної

267. Скарги і об'єктивні дані дозволяють припустити наявність у хворого запального процесу в
жовчному міхурі, порушення колоїдних властивостей жовчі, вірогідність утворення жовчних
каменів. Яка речовина, головним чином, може зумовити утворення каменів?
@Холестерин

268. У людини унаслідок хронічного захворювання печінки істотно порушена її білковосинтезуюча


функція. Який параметр гомеостазу зменшиться найбільш вірогідно?
@Онкотичний тиск плазми крові

269. У хворого на цироз печінки низький артеріальний тиск. Який механізм порушення
гемодинаміки при цьому?
@Депонування крові в портальному руслі

270. Тварині після попередньої підготовки (фістула Екка – Павлова) видалили печінку. Що стане
причиною загибелі тварини?
@Гіпоглікемічна кома
271. У хворого на аденому простати знизилась швидкість клубочкової фільтрації. Який механізм
цього ускладнення?
@Підвищення гідростатичного тиску в капсулі клубочка

272. 43-річний хворий скаржиться на масивні набряки та артеріальну гіпертензію. У сечі: відносна
щільність 1,017, білок 4,0 г/л, Ер – 15-18 в полі зору (вилужені). Який нирковий процес
переважно порушений у хворого?
@Фільтрація

273. У хворого А., 38 років, на 3-му році захворювання на системний червоний вовчак виявилось
дифузне ураження нирок, що супроводжувалося нефротичним синдромом. Який механізм
розвитку протеїнурії у хворого?
@Автоімунне ушкодження клубочків нефронів

274. При обстеженні хворого встановлено, що кліренс ендогенного креатиніну після 24-х
годинного збору сечі становить 50 мл/хв (при нормі – 110-150 мл/хв.). Про порушення якого
ниркового процесу свідчить наявність такої ознаки?
@Клубочкової фільтрації

275. У хворого на мієломну хворобу з вираженою гіперпарапротеїнемією знизилась швидкість


клубочкової фільтрації (ШКФ). Який фактор зумовив зменшення ШКФ в даному випадку?
@Підвищення онкотичного тиску крові

276. У хворого, віком 58 років, з гострою серцевою недостатністю, спостерігалось зменшення


добової кількості сечі – олігурія. Який механізм цього явища?
@Зниження гідростатичного тиску крові в клубочкових капілярах

277. Хворий після автомобільної катастрофи непритомний, АТ 70/40 мм рт. ст. За добу виділяє біля
300 мл сечі. Який механізм порушення сечоутворення у даному випадку?
@Зменшення клубочкової фільтрації

278. У хворого з гострою нирковою недостатністю в стадії поліурії азотемія не тільки не


зменшилась, але й продовжувала підвищуватись. Що в даному випадку викликало поліурію?
@ Зменшення реабсорбції

279. У результаті порушення техніки безпеки відбулося отруєння сулемою (хлорид ртуті). Через 2
дні добовий діурез становив 200 мл. У хворого зявилися головний біль, блювання, судоми,
задишка, у легенях – вологі хрипи. Як називається така патологія?
@Гостра ниркова недостатність

280. Хворий доставлений у лікарню із шлунковою кровотечею в тяжкому стані. АТ - 80/60 мм рт.
ст. Діурез 60-80 мл сечі за добу з відносною щільністю 1,028-1,036. Який механізм зумовив
зменшення добового діурезу?
@Зниження гідростатичного тиску в капілярах клубочків
281. У хворого, який був госпіталізований після того, як перебував під завалом протягом 4 годин,
спостерігається різке зниження діурезу, гіперазотемія, ознаки набряку головного мозку. В якій
стадії гострої ниркової недостатності перебуває хворий? @Олігоануричній

282. У хворого після тяжкої травми грудної клітки розвинувся шок та з’явилися ознаки гострої
ниркової недостатності (ГНН). Що є провідним механізмом розвитку ГНН в даному випадку?
@Зменшення артеріального тиску

283. У хворого, що тривалий час страждає на хронічний гломерулонефрит, виникла азотемія при
помірній поліурії. Що є головною ланкою в патогенезі цих симптомів при хронічній нирковій
недостатності?
@Зменшення маси діючих нефронів

284. У чоловіка 48 років, що давно хворіє на хронічний гломерулонефрит, обстеження виявило


наявність хронічної недостатності нирок. Чим пояснюється гіперазотемія при хронічній нирковій
недостатності?
@Зниженням клубочкової фільтрації

285. У хворого з хронічною нирковою недостатністю встановлено зменшення кліренсу за інуліном


до 60 мл/хв. З порушенням якої функції нирок це пов’язано?
@Клубочкової фільтрації

A.
B.
C.
D.
E.
286. У хворого на хронічну ниркову недостатність з’явилися анорексія, диспепсія, порушення
ритму серця, свербіння шкіри. Який механізм розвитку цих порушень є головним?
@Гіперазотемія

287. У хворого, що тривалий час страждає на цукровий діабет І типу, виникли азотемія, олігурія,
гіпо-, ізостенурія, протеїнурія. Що є головною ланкою в патогенезі цих симптомів при хронічній
нирковій недостатності?
@Зменшення маси діючих нефронів

288. У хворого з хронічною патологією нирок після проведення проби Зимницького виявлено
гіпостенурію. Які зміни сечовиділення будуть спостерігатися при цьому?
@Поліурія

289. У хворого М. діагностовано хронічний гломерулонефрит. Внаслідок значних склеротичних


змін маса функціонуючих нефронів зменшилася до 10%. Яке з перерахованих нижче порушень
зумовлює уремічний синдром?
@Азотемія

290. Білому щуру ввели під шкіру сулему в дозі 5 мг/кг маси тіла. Через 24 години в плазмі крові
концентрація креатиніну збільшилася в декілька разів. Який механізм ретенційної гіперазотемії в
даному випадку?
@Зниження клубочкової фільтрації

291. У результаті гострої ниркової недостатності у хворого виникла олігурія. Яка добова кількість
сечі відповідає даному симптому?
@100-500 мл

292. В результаті гострої ниркової недостатності у хворого виникла анурія. Яка добова кількість
сечі відповідає даному симптому?
@50-100 мл

293. У хворого на первинний нефротичний синдром встановлений вміст загального білку крові 40
г/л. Яка причина зумовила гіпопротеїнемію?
@Протеїнурія

294. У хворого на хронічний пієлонефрит розвинулась ізостенурія. Яка питома вага сечі має бути у
цього хворого?
@1,010
295. Хворій Н. 3 роки тому був поставлений діагноз хронічний гломерулонефрит. Останні 6 місяців
зявились набряки. Що головним чином спричинює їх розвиток?
@Протеїнурія

296. У 20-річного хлопця через 2 тижня після перенесеної лакунарної ангіни був поставлений
діагноз: гострий гломерулонефрит. Які патологічні зміни в складі сечі найбільш імовірні?
@Протеїнурія
297. В експерименті кролю ввели нефроцитотоксичну сироватку морської свинки. Яке
захворювання нирок моделювалося в цьому досліді?
@Гострий дифузний гломерулонефрит

298. У хворого з гломерулонефритом виявлено: АТ – 185/105 мм рт.ст., анемія, лейкоцитоз,


гіперазотемія, гіпопротеїнемія. Який показник свідчить про ускладнення гломерулонефриту
нефротичним синдромом?
@Гіпопротеїнемія

299. У хворого на гострий гломерулонефрит спостерігається протеїнурія з молекулярною масою


білка понад 70000 D. Яке порушення спричинило це явище?
@Ушкодження клубочкової мембрани

300. У дитини 5 років через 2 тижні після перенесеної ангіни виник гострий дифузний
гломерулонефрит, що характеризувався олігурією, протеїнурією, гематурією, збільшенням ОЦК.
Порушення якого з ниркових процесів найбільш суттєве для виникнення цих симптомів?
@Клубочкової фільтрації

301. У хворого Н., 25 років, після перенесеної інфекції розвинувся нецукровий діабет. Дефіцит
якого гормону призвів до розвитку даної патології?
@Вазопресину

302. Хворий скаржиться на постійну спрагу, знижений апетит. Випиває за добу 9 л. Поліурія, сеча
знебарвлена, відносна щільність – 1,005. Найбільш імовірною причиною розвитку даної
патології у хворого є пошкодження:
@Гіпоталамічних ядер
303. У дитини 2,5 років спостерігається затримка у фізичному розвитку, поганий сон та апетит,
спрага, поліурія. Цукор у сечі не виявляється. Яка ендокринна патологія є причиною порушень
водно-мінерального обміну?
@Гіпосекреція антидіуретичного гормону

304. У хворого 40 років у зв'язку з ураженням гіпоталамо-гіпофізарного провідного шляху виникла


поліурія (10-12 л за добу), полідипсія. При дефіциті якого гормону виникають такі порушення?
@Вазопресину

305. До лікаря звернувся чоловік 27 років. При огляді було виявлено збільшення кистей рук,
ступнів та нижньої щелепи. Крім того є імпотенція, атрофія яєчок. Функції якої залози
порушені?
@Передньої частини гіпофізу

306. Хворий звернувся до лікаря зі скаргами на біль у голові, зміни у кінцівках, збільшення рук та
ніг. Зовні: масивні надбрівні дуги, губи. При надлишку якого гормону виявляються такі зміни?
@Соматотропного
307. Батьки хлопчика 15 років звернулися до лікаря зі скаргами на значне відставання сина в рості.
Об’єктивно: статура правильна, зріст 98 см, інтелект не порушений. При нестачі якого гормону
з'являються такі зміни?
@Соматотропного

308. У хворого В., 46 років, виявлено непропорційне збільшення кистей рук, ступнів, носа, вух,
надбрівних дуг. Лабораторно - гіперглікемія, глюкозурія, порушення тесту толерантності до
глюкози. Причиною цукрового діабету є:
@Гіперсекреція соматотропного гормону

309. У хворої, що страждає на гіпертиреоз, підвищені основний обмін, температура тіла, стійкість
до гіпотермії. Що можна зазначити як головну ланку в патогенезі цих проявів?
@Збільшення споживання О2 мітохондріями

310. Хворій Д., 50 років, був поставлений діагноз мікседема. Порушення утворення яких гормонів
призводить до розвитку даної патології?
@Тироксину і трийодтироніну
311. Пацієнт скаржиться на напади серцебиття, пітливість і дратівливість. Обстеження показало
наявність у нього дифузного токсичного зобу. Імунні реакції якого типу (за Кумбсом і Джеллом)
є головною ланкою в патогенезі Базедової хвороби?
@V типу (стимулюючі)

312. У ліквідатора аварії на ЧАЕС через деякий час зявились скарги на підвищену збудливість,
нервозність, серцебиття, зниження маси тіла, постійну слабкість, тремтіння тіла, відчуття жару.
Гіперфункція якої залози може бути причиною вказаних змін:
@Щитоподібної залози

313. У хворого з базедовою хворобою виявлено тахікардію та ізольовану систолічну артеріальну


гіпертензію. Збільшення чутливості до якого гормону є головною ланкою в патогенезі цих
проявів при гіпертиреозі?
@Адреналіну

314. У пацієнта 20 років після хірургічного видалення щитоподібної залози спостерігається


гіпоплазія емалі, порушення утворення дентину. Недостатність якого гормону стала причиною
вказаної патології?
@Тиреокальцитоніну

315. У жительки Закарпаття з’явились слабкість, сонливість, апатія, зниження пам’яті, набряки.
Після обстеження встановлений діагноз “ендемічний зоб”. Брак якої речовини у воді та їжі може
призвести до цієї хвороби?
@Йоду

316. У жительки Закарпаття діагностовано гіпотиреоїдний зоб. Вміст якого гормону в крові
збільшується при ендемічному зобі?
@ТТГ
317. Дитина 3-х років млява, малорухлива, різко відстає від однолітків у фізичному і психічному
розвитку, понижені основний обмін і температура тіла. Яке захворювання у дитини?
@ Кретинізм

318. У хворого на правошлуночкову серцеву недостатність є асцит та набряки. Яке ендокринне


порушення має значення в патогенезі серцевих набряків?
@Вторинний гіперальдостеронізм
319. У хворого, 39 років, з набряками на ногах, задишкою і іншими ознаками правошлуночкової
серцевої недостатності діагностована гіперпродукція альдостерону. Чим викликана ця зміна?
@ Гіперпродукцією реніну

320. У хворого виявлена аденома клубочкової зони кори наднирників. Внаслідок цього розвинувся
первинний гіперальдостеронізм або хвороба Конна. На обмін якого іону впливає ця пухлина?
@Натрію

321. У хворого на аденому клубочкової зони кори наднирників (хворобу Конна) спостерігаються
м’язова слабкість, атонія кишок, аритмії серця, парестезії. Дефіцит якого катіону є головною
ланкою в патогенезі цих розладів?
@Калію

322. У хворого на аденому клубочкової зони кори наднирників (хворобу Конна) спостерігаються
артеріальна гіпертензія, м’язова слабкість, аритмії серця, судоми. Яке порушення кислотно-
основного гомеостазу в хворого?
@Видільний алкалоз

323. Після адреналектомії у тварини виникла зупинка серця в діастолі і вона загинула. Яке
порушення викликало зупинку серця?
@Гіперкаліемія

324. Після двостороннього видалення наднирників у собаки зявилась м’язова слабкість,


адинамія, зниження температури тіла, гіпоглікемія. Які ще можуть бути відмічені прояви
наднирникової недостатності?
@Артеріальна гіпотонія

325. У хворого Аддісонова або бронзова хвороба. Недостатність якого гормону має місце при
цьому патологічному процесі?
@Альдостерону

326. У хворого, що протягом року лікувався преднізолоном, при поступовому зменшенні дози
з’явилися швидка фізична і психічна втомлюваність, артеріальна гіпотензія, схуднення,
прогресуюча гіперпігментація шкіри. Що лежить в основі цього симптомокомплексу?
@Хронічна гіпофункція кори наднирників
327. Після перенесеного сепсису у хворого зявився бронзовий колір шкіри, характерний для
Аддісонової хвороби. Механізм гіперпігментації полягає у підвищенні секреції гормону:
@Меланоцитстимулюючого
328. Хворий з Аддісоновою хворобою має виражену схильність до гіпоглікемії. Чим спричинена
гіпоглікемія у хворого?
@Пригніченням глюконеогенезу

329. У хворого з частими ортостатичними колапсами діагностовано хвороба Аддісона. Зменшення


чутливості до якого гормону є головною ланкою в патогенезі цих проявів при гіпокортицизмі?
@Адреналіну

330. У хворого на ожиріння виявили синдром Іценка – Кушинга. Яке порушення метаболізму є
головною ланкою в патогенезі ожиріння в хворих на кортизолпродукуючі пухлини?
@Гіперглікемія

331. У хворої виявлені ожиріння обличчя, шиї і верхньої частини тулуба, стрії на животі,
гірсутизм, гіперглікемія і гіпертензія. Гіперпродукція якого гормону може бути причиною таких
змін?
@Кортизол

332. Хвора з діагнозом хвороба Іценка - Кушінга скаржиться на м’язову слабкість, гіпертензію,
ожиріння верхньої частини тулуба, шиї та обличчя. Первинна гіперпродукція якого гормону
зумовлює дану хворобу?
@Кортикотропіну

333. У хворої 44 років виявлені ознаки гіперкортицизму. Що допоможе диференціювати хворобу


Іценка – Кушінга від синдрому Іценка – Кушінга?
@Рівень кортикотропіну в крові

334. У хворого виявлено при гормон-продукуючій пухлині пучкової зони кори наднирників. Вміст
у крові якого гормону зменшується в цьому випадку? @АКТГ

335. У хворого відмічається періодичні напади головного болю з серцебиттям і сильним


потовиділенням. При обстеженні виявлені гіпертензія, гіперглікемія, підвищення основного
обміну, тахікардія. Для якої патології наднирників характерні ці симптоми?
@Гіперфункції мозкової речовини наднирників

336. У хворої з феохромоцитомою після психічного навантаження виникає тахікардія,


підвищується артеріальний тиск, з’являється різкий біль у надчеревній ділянці. Ці приступи
зумовлені:
@Масивним викидом катехоламінів наднирниками

337. У дівчинки діагностований адреногенітальний синдром з ознаками вірилізації. Надмірна


секреція яких гормонів наднирників зумовила вірилізацію? @Андрогенів

338. У дівчинки діагностований адреногенітальний синдром. Гіперпродукція якого гормону стала


причиною надлишку андрогенів?
@АКТГ

339. У дівчинки діагностований адреногенітальний синдром. Первинний генетичний дефект


синтезу яких гормонів наднирників зумовив дану патологію?
@Глюкокортикоїдів

340. У чоловіка 42 років, що довгий час перебував у стані стресу, у сечі значно збільшений вміст
17-кетостероїдів, що в першу чергу свідчить про підвищення секреції:
@АКТГ

341. Чоловік 25 років надійшов до лікарні через годину після автомобільної аварії. Спостерігається
картина гострого стресу без виражених ушкоджень. Яке явище в периферійній крові буде
характерним для цього стану?
@Еозинопенія

342. У спортсмена, який тривалий час вживав анаболічні стероїди, виник гіпогонадизм. Чим
зумовлений цей стан?
@Гіпопродукцією гонадотропіну

343. У жінки після тяжкого психоемоційного стресу виникла аменорея. Функції якого органу
порушені?
@Гіпоталамуса

344. Через 1–2 доби після видалення у собаки прищитоподібних залоз спостерігалось: в’ялість,
спрага, різке підвищення нервово-м’язової збудливості з розвитком тетанії. Яке порушення
обміну електролітів виникло?
@Гіпокальціємія

345. Хворому 45-ти років при оперативному втручанні на щитоподібній залозі випадково видалили
прищитоподібні залози. Це призвело до:
@Тетанії
346. У пацієнта 20 років спостерігається гіпоплазія емалі, порушення утворення дентину.
Гіперпродукція якого гормону стала причиною вказаної патології?
@Паратгормону

347. Жінка, що хворіє на цукровий діабет 1 типу, знаходиться у стані крайнього збудження;
агресивна, відмовляється від обстеження. Шкіра бліда, волога. Руки тремтять. Пульс - 90/хв, AT -
135/85 мм рт.ст., ЧД - 18/хв. Запаху ацетону або алкоголю немає. Що найбільш імовірно наявне у
хворої?
@Гіпоглікемія

348. У дівчини 14 років, яка скаржиться на значне схуднення, спрагу, поліурію, виявлено
гіперглікемію натщесерце та зниження толерантності до глюкози. При нестачі якого гормону
може розвинутись такий стан?
@Інсуліну

349. У хворого діагностовано цукровий діабет 1 типу. Яка ендокринопатія за патогенезом виникла
у хворого?
@Залозиста

350. У хворого вперше виявили цукровий діабет 2 типу. У крові гіперглікемія натщесерце 8
ммоль/л, гіперінсулінемія. Яка ендокринопатія за патогенезом виникла у хворого?
@Периферична

351. Відомо, що при раптовому припиненні надходження кисню в головний мозок, непритомність
настає через 6-7 секунд. Яке патологічне явище лежить в основі порушення функцій клітин
мозку в даному випадку?
@Енергодефіцит

352. Відомо, що при зниженні рівня глюкози в крові дуже швидко виникає гіпоглікемічна кома.
Яке патологічне явище лежить в основі порушення функцій клітин мозку в даному випадку?
@Енергодефіцит

353. У хворого з хворобою Крейтцфельда-Якоба розвинулася спонгіоформна губчаста


енцефалопатія. До якої групи належить інфекційний агент, що викликав це захворювання?
@Пріони
354. З нейронів ніколи не утворюються пухлини, оскільки вони не діляться і в них неможлива одна
з стадій канцерогенезу. Яка це стадія?: @Промоція

355. У головному мозку ніколи не буває фізіологічної артеріальної гіперемії. Це явище зумовлене
тим, що мозковий кровообіг підпорядкований: Добовим змінам активності мозку
@Метаболічним потребам нейронів

356. У хворого діагностовано автоімунний енцефаліт. Чим може бути зумовлене це захворювання?
@Пошкодженням гематоенцефалічного бар’єру

357. У хворого з черепно-мозковою травмою виник енцефаліт. Який механізм призвів до


автоімунного ушкодження мозку?
@Демаскування антигену

358. У дитини 6 місяців з уповільненим моторним і психічним розвитком відзначається збліднення


шкірних покривів, волосся, райдужної оболонки ока, позитивна проба Фелінга (з 3% розчином
трихлороцтового заліза). Яке з зазначених спадкових захворювань виявлено в дитини?
@Фенілкетонурія

359. Після автомобільної катастрофи у хворого діагностована травма середньої третини плеча з
неповним розривом серединного нерва. Окрім рухових і сенсорних порушень нижче місця
травми, хворий скаржиться на пекучий, нестерпний біль. Який вид болю у хворого?
@Каузалгія.

360. Хворий з анемією Аддісона – Бірмера скаржиться на відчуття „повзання мурашок” по тілу. Як
називається це порушення чутливості?
@Парестезія
361. Після травматичного видалення зуба хворий скаржиться на тупий, не чітко локалізований, сильний
біль у яснах, підвищення температури тіла до 37,5 0С. Діагностовано альвеоліт. Який вид болю у даного
хворого?
@Протопатичний

362. Хворий 50 років з цукровим діабетом 1-го типу скаржиться на повну втрату чутливості пальців ніг. Як
називається таке порушення чутливості?
@Анестезія

363. Хворому 4 місяці тому ампутували ногу. Зараз його дуже турбує відчуття сильного болю у відсутній
кінцівці. Як можна визначити цей вид болю?
@Патологічний

364. Дитина випадково доторкнулася до гарячої праски, скрикнула та відсмикнула руку. Який вид болю
виник у цьому випадку?
@Епікритичний

365. У хворого порушена координація рухів; рухи розмашисті, непропорційні, хода «півняча», «п'яна»,
пальцем важко попасти в ціль. З ураженням яких нервових структур пов'язано це порушення?
@Мозочка

366. У тварини після видалення одного з відділів головного мозку рухи втратили плавність, точність. Як
називається описане явище?
@Атаксія

367. У тварини після перерізання n. oftalmicus з'явилися екзофтальм, звуження зіниці, в'ялість повік. Через
деякий час рогівка помутніла, утворилася виразка. Порушення якої функції нервового волокна виникло в
даному випадку? @Трофічної

368. У хворого відсутні довільні і мимовільні рухи нижніх кінцівок, що супроводжуються атонією і
атрофією м'язів. Який параліч має місце в даному випадку?
@Периферичний

369. У хворого відмічені швидкі мимовільні стереотипні скорочення м’язів обличчя, що імітують мигання і
примружування. До якої форми розладів рухової функції нервової системи слід віднести описане
порушення?
@Гіперкінез

370. 78-річному хворому поставлений діагноз «паркінсонізм». Нестача якого медіатора в хвостатому ядрі
головного мозку є головною ланкою в патогенезі цієї хвороби?
@Дофаміну
371. У пацієнта під час гіпертонічного кризу стали неможливими довільні рухи правою ногою, пасивні
рухи збереглися в повному обсязі. При пальпації визначається високий тонус м'язів кінцівки. Яка форма
порушення рухової функції має місце у хворого?
@Центральний параліч

372. У дитини 12 років спостерігаються швидкі, несподівані, розмашисті, позбавлені стереотипності рухи.
У них беруть участь м'язи обличчя і кінцівок. М'язовий тонус понижений. Яка форма порушень рухової
функції нервової системи виникла?
@Хорея

373. У хворого виникли мимовільні „червоподібні” рухи в пальцях рук і ніг (згинання і розгинання пальців,
що настають один за одним; перерозгинання середніх і кінцевих фаланг), які посилюються при довільних
рухах. Як називається така форма порушення рухової функції нервової системи?
@Атетоз

374. У жінки після інсульту відсутні довільні рухи у верхній і нижній кінцівках справа. Тонус м'язів і
рефлекси в них підвищені. Спостерігаються патологічні рефлекси. Які клітини мозку пошкоджені у
хворої?
@Пірамідні клітини кори

375. У хворого на поліомієліт відсутні довільні і мимовільні рухи правої ноги, що супроводжуються
атонією і атрофією м'язів. Які клітини мозку пошкоджені у хворого?
@Альфа-мотонейрони рухових ядер

376. У хворого з печінковою недостатністю з’явилася сонливість, а потім він знепритомнів. Накопичення
якої речовини порушує енергозабезпечення нейронів при печінковій енцефалопатії?
@Аміак
377. У хворого з печінковою недостатністю з’явилася сонливість, а потім він знепритомнів. Яка речовина
гальмує синаптичну передачу в мозку при печінковій енцефалопатії?
@ГАМК

378. Хворого на цукровий діабет 1 типу доставили у клініку в коматозному стані. Об’єктивно: дихання
шумне, часте, глибоке, запах ацетону з рота. Вміст глюкози в крові 15,2 ммоль/л, кетонових тіл – 10,6
ммоль/л. Для якого ускладнення даного захворювання характерні такі показники?
@Кетоацидотична кома

379. Хворому на цукровий діабет І типу ввели інсулін. Через деякий час у хворого з’явилися
загальна слабкість, гостре почуття голоду, дратівливість, запаморочення, кома. Який основний
механізм виникнення коми у пацієнта? @Вуглеводне голодування головного мозку.

380. У хворого на цукровий діабет розвинулась гіперглікемічна кома (рівень глюкози 54 ммоль/л). Яке
патологічне явище лежить в основі порушення функцій клітин мозку в даному випадку?
@Гіпогідратація клітин мозку
381. Хворий 35 років доставлений у стаціонар непритомним, дихання часте, глибоке (типу Куссмауля), АТ
90/45 мм рт. ст., з рота запах ацетону. Глюкоза крові – 18 ммоль/л, у сечі – глюкозурія, кетонурія. Яка
кома виникла у даного хворого?
@Кетоацидотична

382. Хворий 55 років доставлений у стаціонар непритомним, дихання часте, поверхневе, АТ 90/45 мм рт.
ст., шкіра в'яла, суха. Лабораторно: глюкоза крові – 55 ммоль/л, Na – 150 ммоль/л, Cl – 15 ммоль/л Яка
кома виникла у даного хворого?
@Гіперосмолярна

383. Хворий 68 років доставлений у стаціонар непритомним, дихання часте, глибоке, АТ 90/45 мм рт. ст.,
шкіра в'яла, суха. Лабораторно: глюкоза крові – 15 ммоль/л, молочна кислота крові – 12 ммоль/л. Яка
кома виникла у даного хворого?
@Лактацидотична

384. У хворого на хронічну ниркову недостатність з’явилися анорексія, нудота, блювота, порушення ритму
серця, свербіння шкіри, психічні порушення аж до коми. Який механізм розвитку уремічної коми є
головним?
@Гіперазотемія

385. У хворого, що раптово знепритомнів, у плазмі крові Na + – 180 ммоль/л, осмолярність – 340 мосмоль/л,
глюкоза – 5 ммоль/л. Чим, найімовірніше, зумовлене виникнення коми у хворого?
@Дегідратацією нейронів

386. У хворого на хворобу Аддісона внутрішньовенна інфузія глюкози призвела до «водного отруєння» та
гіпоосмолярної коми. Чим зумовлене виникнення коми у хворого?
@Набуханням нейронів

387. Хворого доставлено до медичної установи в коматозному стані. Зі слів супровідних удалося
встановити, що хворий знепритомнів під час тренування на завершальному етапі марафонської дистанції.
Яка кома найбільш імовірна в цього хворого?
@Гіпоглікемічна

388. Хворий сонливий, свідомість затуманена, реакція на сильні подразники загальмована. М'язові
фібрилярні посмикування. Дихання типу Чейна-Стокса з аміачним запахом. При аускультації серця -
шум тертя перикарда. У крові знижена кількість сечовини. Який вид коми розвинувся у хворого?
@Печінкова

389. У лікарню швидкої допомоги поступив чоловік із закритим переломом стегна. Постраждалий у
збудженому стані. Що вимагає негайної корекції в патогенезі травматичного шоку у пацієнта? @ Біль.

390. Після введення анестетика в пацієнта розвинувся анафілактичний шок. Який механізм визначає
розвиток гострої недостатності кровообігу при цьому?
@Зниження судинного опору

391. У патогенезі гострої недостатності кровообігу при анафілактичному шоку є розширення артеріол та
підвищення проникності капілярів. Який з медіаторів зумовлює розвиток даних порушень?
@Гістамін
392. У хворого після введення лідокаїну розвинувся анафілактичний шок. Які медіатори анафілаксії
спричинюють безпосередньо обструкцію нижніх дихальних шляхів і гостру недостатність дихання?
@Гістамін, лейкотрієни

393. У хворого на інфаркт міокарда з’явилися клінічні ознаки кардіогенного шоку. Який із вказаних нижче
патогенетичних механізмів є первинним у розвитку шоку?
@Зниження насосної функції серця

394. Хворий, 52 років, був доставлений в клініку з симптомами гострого панкреатиту та явищами
панкреатичного шоку. Що є основним механізмом розвитку панкреатичного шоку?
@Надходження активованих протеаз у кров

395. Хворий, 52 років, був доставлений в клініку з симптомами гострого панкреатиту та явищами
панкреатичного шоку. Активація якої системи є причиною гіпотензії при панкреатичному шоці?
@Калікреїн-кінінової

396. Під час легеневої кровотечі у хворого на туберкульоз пацієнт втратив 45 % загального об’єму крові.
Яке ускладнення, найбільш вірогідно, може виникнути у хворого?
@Постгеморагічний шок

397. Під час легеневої кровотечі у хворого на туберкульоз пацієнт втратив 45 % загального об’єму крові.
Який із вказаних нижче патогенетичних механізмів є первинним у розвитку постгеморагічного шоку?
@Гіповолемія

398. У хворого з опіками 40 % поверхні тіла розвинувся опіковий шок. Який механізм розвитку шоку
домінує на його початку?
@Больовий

399. Потерпілого з джутом на стегні привезли через 3 години після травми в задовільному стані. Після
зняття джгута стан хворого різко погіршився: з'явився різкий набряк тканини стегна, частий пульс,
холодний піт, артеріальний тиск знизився до 80/60 мм рт.ст. Яке ускладнення розвинулось?
@Токсемічний шок
400. Масивне тривале розчавлювання м'яких тканин потерпілого в результаті шахтного вибуху призвело до
появи у хворого олігурії, гіпоізостенурії, протеїнурії, міоглобінурії, гіперкаліємії, гіпонатріємії. Який
провідний механізм порушення функції нирок у цьому випадку?
@Розвиток токсемії
1. У працівників підприємства по виробленню килимових покриттів відмічені риніт, дерматит,
бронхіальна астма. За якою класифікацією ці хвороби можуть бути віднесені до одної групи?
@Патогенетична

2. Перебіг гарячки, гіпоксії, голодування, запалення не залежать від виду, віку та статі тварини. Яку назву
мають патологічні зміни, що розвиваються за однаковими основними закономірностями?
@Типовий патологічний процес

3. Після того, як хлопчик промочив ноги, у нього підвищилася температура тіла, з'явився біль у горлі.
Діагностовано гостре респіраторне захворювання. Який фактор є причиною хвороби в даному разі?
@Інфекція

4. Стан хворого, який протягом 4 років хворіє на пептичну виразку шлунка, особливо погіршується восени
і навесні, коли виникають печія, нудота, біль у животі. Як можна характеризувати стан хворого в період
між загостреннями хвороби?
@Ремісія

5. Хлопчик, 12 років, після повернення зі школи почав скаржитися на головний біль, озноб, біль у м’язах,
втрату апетиту, втому. Для якого періоду хвороби характерні такі симптоми?
@Продромальний

6. Хворому, що страждає на карієс, ускладнений гострим гнійним запаленням навколозубної тканини


(періодонтитом), який супроводжується підвищенням температури тіла, було зроблено екстракцію зуба.
Яку зміну в даного хворого можна віднести до поняття «патологічний стан»?
@Відсутність зуба
7. Чоловік 40 років у результаті переохолодження занедужав на крупозну пневмонію. Яка причина
виникнення такої форми запалення легенів?
@Пневмокок

8. При рентгенологічному обстеженні у хворого виразковою хворобою виявлено стеноз воротаря. Це


порушення є:
@Патологічним станом

9. У жінки, 55 років, стоматолог виявив кілька зубів, уражених карієсом. Яким нозологічним поняттям
можна визначити цей стан? @Патологічний процес.

10. У здорової людини, яка перебувала в контакті з хворим на дифтерію, при бактеріологічному
дослідженні змиву з носоглотки виявлено збудника дифтерії. Який період хвороби спостерігається в
даному випадку?
@Латентний

11. Хто із зазначених нижче вчених уперше ввів у експериментальну практику метод хронічного
експерименту й вважається засновником цього методу ? A. @Павлов І.П.
12. Хто із перерахованих нижче вчених у своїх дослідженнях широко використовував порівняльно-
патологічний метод? A. @Мєчніков І.І.
A.
1. До приймального відділення надійшов чоловік, що постраждав під час вибуху в шахті. Загальний стан
задовільний. Свідомість збережена. На обличчі, руках і тулубі визначаються ділянки гіперемованої шкіри
з пухирями. Який фактор переважно чинив дію на шахтаря? @Термічний

2. Чоловіка приблизно 50 років було знайдено на вулиці при температурі повітря +3оС. Об'єктивно:
температура тіла 35,2оС, хворий промерзлий, не може самостійно рухатися. Температура пальців ніг
20oС. ЧД - 16/хв., АТ - 100/60 мм рт. ст. Тони серця приглушені, ЧСС 55 уд./хв. Найімовірніше у хворого
спостерігається:
@Загальне переохолодження в стадії декомпенсації

3. Який механізм компенсує надмірну втрату солей натрію в організмі при тривалому потінні в умовах
жаркого мікроклімату?
@Підвищення вироблення альдостерону

4. У разі дії на організм низької температури підвищується активність механізмів терморегуляції. Який з
нижченаведених механізмів найефективніше обмежує тепловіддачу?
@Спазм судин шкіри.

5. У хворого з опіками 40 % поверхні тіла розвинувся опіковий шок. Який механізм розвитку шоку домінує
на його початку?
@Больовий

6. При температурі повітря -230С у селянина шкіра бліда, холодна, суха, температура тіла залишається
нормальною. Назвіть основний критерій виникнення стадії декомпенсації при гіпотермії.
@Зниження температури ядра тіла нижче норми

7. Який механізм тепловіддачі найбільш ефективно спрацьовує при перебуванні людини в умовах 40%
вологості й температури навколишнього середовища +35оС? @Потовиділення

8. У робітника, який влітку працював у щільному костюмі, різко підвищилася температура тіла, з’явилися
задишка, тахікардія, запаморочення, судоми; потім він знепритомнів. Що стало причиною тяжкого стану
робітника? A. @Зниження тепловіддачі.
9. При температурі повітря +330С у робітника "гарячого" цеху температура тіла залишається нормальною.
Назвіть основний механізм, що попереджує розвиток стадії декомпенсації:
@Посилення потовиділення

10.У разі дії на організм високої температури підвищується активність механізмів терморегуляції. Який з
нижченаведених механізмів сприяє посиленню тепловіддачі? @Розширення судин шкіри.

11.В умовах високої температури повітря зменшується здатність стабілізувати системний артеріальний тиск
при переході з горизонтального положення у вертикальне (ортостатична стійкість). Основна причина
цього полягає в тому, що стає неможливим достатнє:
@Рефлекторне звуження артеріальних судин опору
12.У разі дії на організм низької температури підвищується активність механізмів терморегуляції. Який з
нижченаведених механізмів сприяє посиленню теплопродукції?
@Скорочення скелетних м’язів.

13.У хлопчика 10 років після тривалого перебування на пляжі виникли почервоніння і набряк шкіри плечей,
обличчя і спини, пекучий біль у почервонілих ділянках, підвищення температури тіла до 37,2 оС. Яке
порушення спостерігається у хлопчика? @Опіки

14.Під час ліквідації аварії на ЧАЕС у пожежника з'явилися нудота, блювання, пронос із кров'ю. АТ 100/60
мм. рт. ст., Ер. 4 Т/л, гемоглобін 140 г/л. Яка форма гострої променевої хвороби розвинулася в
постраждалого?
@Кишкова

15.Ліквідатор аварії на ЧАЕС одержав дозу опромінення 3 Гр. Які зміни в крові спостерігались у перший
день після опромінення?
@Лімфопенія

16.Під час ліквідації наслідків аварії на Чорнобильській АЕС робітник одержав дозу опромінення 5 Гр.
Скаржиться на головний біль, нудоту, запаморочення. Які зміни кількості лейкоцитів можна очікувати у
хворого через 10 годин після опромінення?
@Нейтрофільний лейкоцитоз.

17.Через декілька тижнів після опромінення (7 Гр) у хворого виникла кровотеча. Які зміни в крові мають
вирішальне значення в патогенезі геморагічного синдрому при гострій променевій хворобі?
@Тромбоцитопенія

18.Ліквідатор наслідків аварії на АЕС отримав дозу іонізуючого опромінення 6 Гр. Які зміни лейкоцитарної
формули слід очікувати через 10 діб?
@Агранулоцитоз, лімфопенія

19.Через 8 діб після опромінення у потерпілого розвинувся виразково-некротичний стоматит. У крові:


еритроцити - 3,0 • 1012/л, гемоглобін - 5,6 ммоль/л, лейкоцити - 1,2 • 109/л, тромбоцити – 80 • 109/л. Для
якого періоду кістково-мозкової форми гострої променевої хвороби це характерно?
@Розгорнутих клінічних ознак

20.Через 4 доби після опромінення потерпілий почуває себе задовільно. У крові: еритроцити - 4,2 • 10 12/л,
гемоглобін - 8,0 ммоль/л, лейкоцити - 5,2 • 10 9/л, тромбоцити – 150 • 109/л. Для якого періоду гострої
променевої хвороби, кістковомозкової форми, характерні описані зміни?
@Прихований

21.Ліквідатор аварії на ЧАЕС отримав дозу опромінення 5 Гр. Через тиждень у нього виявлено
агранулоцитоз. Який патогенетичний механізм є провідним у його виникненні?
@Пригнічення лейкопоезу

22.У хворого на променеву хворобу з’явилися ознаки геморагічного синдрому. Який механізм має
найбільше значення у патогенезі цього синдрому?
@Гіпопродукція тромбоцитів кістковим мозком

23.Працівника АЕС доставили в клініку після одноразового опромінення зі скаргами на головний біль,
непритомність, підвищення температури тіла, слабість, блювання, пронос. В аналізі крові - лейкоцитоз із
лімфопенією. Який період променевої хвороби найбільш імовірний у пацієнта?
@Первинних реакцій
24. Через добу після опромінення (3,5 Гр) у потерпілого в крові: еритроцити - 4,7 • 10 12/л, гемоглобін - 9,0
ммоль/л, лейкоцити – 11 • 109/л, тромбоцити –
270 • 109/л. У мазку крові: базофіли – 1 %, еозинофіли – 3 %, нейтрофіли – 81 %, лімфоцити – 11 %, моноцити
– 4 %. З яким періодом гострої променевої хвороби пов’язані зазначені зрушення?
@Первинних реакцій

25.Пацієнт потрапив до лікарні через 5 діб після опромінення (3 Гр). Недостатність якої функціональної
системи є головною ланкою патогенезу гострої променевої хвороби у хворого?
@Крові

26.Чоловік потрапив до лікарні через 3 доби після впливу іонізуючого випромінювання в дозі 3 Гр. Зміни з
боку якого органу стали головними в патогенезі порушень у цьому випадку?
@Червоного кісткового мозку.

27.Електрик, випадково доторкнувся до оголеного електропроводу обома руками і загинув. Що спричинило


смерть у даному випадку?
@Фібриляція серця

28.Електрик, випадково доторкнувся до оголеного електропроводу однією рукою і отримав опіки. Що


спричинило ураження у даному випадку?
@Електротермічна дія струму

29.Електрик, випадково доторкнувся до оголеного електропроводу однією рукою і не зміг його відпустити,
доки не вимкнули струм. Що спричинило ураження у даному випадку?
@Електромеханічна дія струму

30.При розгерметизації літака на висоті 15 км льотчик загинув. Причиною смерті було гостре порушення
кровообігу. Який вид емболії найбільш вірогідно виник при цьому?
@Газова

31.При зануренні на 100 м у аквалангіста з'явилися порушення вищої нервової діяльності і координації
рухів. Що найбільш ймовірно викликає ці розлади?
@Гіперсатурація азоту
32. Після занурення водолаза на глибину 60 м у нього з’явилися симптоми порушення функцій
ЦНС: збудження, ейфорія, ослаблення уваги, професійні помилки. Токсичною дією на нейрони якої
речовини зумовлені ці симптоми? A. @Азоту.

33. Під час аварійного підйому з глибини у водолаза розвинулися судоми із знепритомненням. Що є
головною ланкою в патогенезі синдрому декомпресії?
@Газова емболія

34.При розвитку гірської хвороби розлади функцій організму багато в чому залежать від ступеня чутливості
різних органів до гіпоксії. Вкажіть, який орган або система має найбільшу чутливість до нестачі кисню ?
@Головний мозок

35.На якій висоті над рівнем моря виникає закипання рідини в організмі при нормальній температурі тіла
людини?
@19 000 м

36.Альпініст протягом декількох діб підіймався в гору. На висоті 5 000 м почали турбувати тахіпное,
тахікардія, біль у вухах, головний біль розпираючого характеру. Вкажіть можливу причину явищ, що
спостерігались.
@Зниження барометричного тиску

37.У туристів, які піднялися на висоту 3000 м, дихання стало частим і глибоким. Ці зміни є наслідком
стимуляції:
@Хеморецепторів каротидних синусів

38.У водолаза, що проводив роботи на великій глибині, при швидкому поверненні його в умови
нормального атмосферного тиску з'явився біль у суглобах, свербіж шкіри, порушення зору,
непритомність. Як називається описане явище ?
@Хвороба декомпресії

39.При підйомі в гори на висоті 5 000 м в учасників альпіністської групи з'явилися скарги на задишку,
прискорене серцебиття, головний біль, запаморочення, шум у вухах. Який патологічний фактор викликав
зазначені явища?
@Гіпоксемія
40.При розгерметизації літака на висоті 6 км у льотчика виникли біль у вухах та носова кровотеча. Яка
можлива причина вказаних симптомів? A. @Зниження барометричного тиску повітря.

41.У водолаза, що проводив роботи на великій глибині, при швидкому поверненні його в умови
нормального атмосферного тиску з'явився біль у суглобах, свербіж шкіри, порушення зору,
непритомність. Як називається описане явище ?
@Кесонна хвороба
42.При розвитку гірської хвороби розвиваються зміни ряду гомеостатичних показників організму. Яка зміна
носить власне патологічний характер?
@Гіпоксемія

43.Після вимушеного швидкого підняття водолаза з глибини на поверхню у нього з’явилися симптоми
кесонної хвороби: біль у суглобах, свербіння шкіри, мерехтіння в очах, запаморочення свідомості. Яким
видом емболії зумовлений розвиток цих симптомів?
@Газова

1. До медико-генетичної консультації звернулася жінка з приводу ризику захворювання на гемофілію у


свого сина. Її чоловік страждає на дане захворювання з народження. Жінка та її батьки здорові стосовно
гемофілії. Яка імовірність народження хворого хлопчика в цій сім'ї?
@Усі хлопчики будуть здорові

2. До медико-генетичної консультації звернулася жінка з приводу ризику захворювання на фосфопенічний


рахіт у своєї дочки. Її мати страждає на дане захворювання з народження. Жінка та її чоловік здорові.
Яка імовірність народження хворої дівчинки у цій сім'ї?
@Усі дівчатка будуть здорові

3. До медико-генетичної консультації звернулася жінка з приводу ризику захворювання на дальтонізм у


своєї дочки. Її батько страждає на дане захворювання з народження. Жінка та її чоловік здорові
стосовно дальтонізму. Яка імовірність народження хворої дівчинки в цій сім'ї?
@Усі дівчатка будуть здорові
4. До медико-генетичної консультації звернулася жінка з приводу ризику захворювання на дальтонізм у
своєї дитини. Її чоловік страждає на дане захворювання з народження. Жінка та її батьки здорові
стосовно дальтонізму. Яка імовірність народження хворої дитини в цій сім'ї?
@Усі діти будуть здорові

5. До медико-генетичної консультації звернулася жінка з приводу ризику захворювання на гемофілію у


своєї дитини. Її батько страждає на дане захворювання з народження. Жінка та її чоловік здорові
стосовно гемофілії. Яка імовірність народження хворої дитини в цій сім'ї?
@25 % дітей будуть хворими

6. Яка вірогідність народження хворих синів у здорової жінки, батько якої страждає на атрофію здорових
нервів (тип успадкування такий, як при гемофілії), а чоловік здоровий?
@50%

7. Яка вірогідність народження хворих дочок у подружньої пари, де чоловік хворіє на гіпофосфатемічний
рахіт, а жінка здорова? A. @100%.
8. Чоловік хворіє на брахідактилію, а дружина здорова. Відомо, що його мати була здоровою, а батько
хворів на брахідактилію. Яка вірогідність народження дітей з брахідактилією у цієї подружньої пари?
@50%

9. До лікаря-генетика звернулися батьки юнака 18 років. Об’єктивно виявлені виражена розумова


відсталість та гіпогонадизм. Попередній діагноз: синдром Клайнфельтера. Який метод медичної
генетики дозволяє підтвердити цей діагноз?
@Цитологічний
10. Під час обстеження юнака з розумовою відсталістю виявлено євнухоїдну будову тіла, недорозвиненість
статевих органів. Який метод генетичного дослідження слід застосувати для уточнення діагнозу?
@Цитологічний

11. У пологовому будинку народилася дитина з численними порушеннями як зовнішніх так і внутрішніх
органів – серця, нирок, травної системи. Був встановлений попередній діагноз – синдром Дауна. Який
метод генетичного дослідження слід застосувати для підтвердження діагнозу?
@Цитологічний

12. Лікар запідозрив у дитини синдром Шерешевського – Тернера. Який метод генетичного обстеження
дасть можливість встановити цей діагноз?
@Цитологічний

13. У населення України частота гетерозигот за геном фенілкетонурії становить 3 %. Який метод
генетичного дослідження використовується для раннього виявлення фенілкетонурії у новонароджених?
@Біохімічний

14. При дослідженні каріотипу 7-річного хлопчика виявлено 46 хромосом. Одне плече 15-ї хромосоми
коротше від іншого. Який вид мутації має місце у цієї дитини? A. @Делеція

15. У клітинах людини відбулося відновлення пошкодженої ділянки молекули ДНК за непошкодженим
ланцюгом з допомогою специфічних ферментів. Яку назву має це явище? @Репарація

16. У клітинах людини під дією ультрафіолетового випромінювання відбулося пошкодження молекули
ДНК, але спрацювала система репарації ДНК. Який з названих ферментів входить до цієї системи?
@Лігаза

17. Внаслідок впливу гама-випромінювання ділянка ланцюга ДНК подвоїлась. Який з наведених видів
мутацій виник у ланцюзі ДНК?
@Дуплікація
18. Під час медико-генетичного консультування родини зі спадковою патологією виявлено, що аномалія
проявляється через покоління у чоловіків. Який тип успадковування притаманний для цієї хвороби?
@Х-зчеплене рецесивне

19. Під час диспансерного обстеження хлопчика 7 років встановлено діагноз: дальтонізм. Батьки здорові,
колірний зір нормальний. У дідуся по материнській лінії виявлена така сама аномалія. Який тип
успадкування цієї патології?
@Рецесивний, зчеплений зі статтю

20. Під час медико-генетичного консультування родини зі спадковою патологією виявлено, що аномалія
проявляється у кожному поколінні у чоловіків. Який тип успадковування притаманний для цієї
хвороби? @Y-зчеплене
21. Під час медико-генетичного консультування родини з ахондроплазією виявлено, що аномалія
проявляється у кожному поколінні як у чоловіків, так і в жінок. Який тип успадковування притаманний
для цієї хвороби?
@Аутосомне домінантне

22. Під час медико-генетичного консультування дитини з фенілкетонурією виявлено, що це захворювання у


сім'ї діагностоване вперше. Який тип успадковування притаманний для цієї спадкової аномалії?
@Аутосомне рецесивне

23. Аналіз родоводу встановив аутосомно-домінантний тип успадкування захворювання. Яка з названих
нижче хвороб має такий тип успадковання? A. @Полідактилія.

24. Аналіз родоводу встановив полігенний тип успадкування захворювання. Яка з названих нижче хвороб
має такий тип успадковання? A. @Цукровий діабет.

25. Дитина страждає на спадкову хворобу. Яка хвороба з названих нижче успадковується за рецесивним
типом, зчепленим з Х-хромосомою?
@Гіпогаммаглобулінемія Брутона.

26. Дитина страждає на спадкову хворобу. Яка з названих нижче хвороб передається за аутосомно-
рецесивним типом? A. @Алкаптонурія.

27. Дитина страждає на спадкову хворобу. Яка з названих нижче хвороб передається за аутосомно-
домінантним типом? A. @Полідактилія.

28. Дитина страждає на спадкову хворобу. Яка з названих нижче хвороб передається за рецесивним типом,
зчепленим з Х-хромосомою? A. @Гемофілія

29. Дитина страждає на спадкову хворобу. Яка з названих нижче хвороб передається за аутосомно-
рецесивним типом? A. @Альбінізм.

30. У хворої людини в ядрах соматичних клітин виявлено одне тільце Барра (статевий хроматин). При
якому варіанті каріотипу це спостерігається? @47, XX, 15+

31. У хворого виявлено синдром Клайнфельтера. Який у нього каріотип? A. @47, XXY

32. У хворої людини в ядрах соматичних клітин виявлено одне тільце Барра (статевий хроматин). При
якому варіанті каріотипу це спостерігається? @47, XXY

33. У хворого виявлено “ускладнений” синдром Клайнфельтера. Який каріотип може бути у нього? @49,
XXXXY.

34. У дитини діагностовано вроджений сифіліс. Виберіть найбільш точне визначення вроджених хвороб.
@Хвороби, з якими людина народжується.

35. Дитина народилась зі спадковою хворобою. Яка хвороба є водночас вродженою і спадковою?
@Брахідактилія.

36. Спадкова хороба вперше виявила себе у дорослому віці. Яка хвороба є спадковою, але не вродженою?
@Хорея Гентінгтона.

37. Дитина народилась з патологією. Яка хвороба є водночас вродженою і не спадковою?


@Гемолітична хвороба новонароджених (резус-конфлікт)

38. У хлопчика діагностовано хворобу Дауна. Яка зміна у хромосомах є причиною цієї хвороби?
@Трисомія 21
39. Чоловік 28 років звернувся до лікаря з приводу безпліддя. Має високий зріст, довгі кінцівки. У ядрах
букальних епітеліоцитів пацієнта виявлено по одному тільцю Барра. Для якої патології характерні
зазначені порушення?
@Синдром Клайнфельтера

40. У жінки 45 років народився хлопчик з розщепленням верхньої щелепи (“заяча губа” та “вовча паща”).
Під час додаткового обстеження виявлені значні порушення з боку нервової, серцево-судинної систем
та зору. При дослідженні каріотипу діагностована трисомія по 13 хромосомі. Який синдром має місце у
хлопчика?
@Патау

41. До медико-генетичної консультації звернулася жінка з приводу відхилень фізичного та статевого


розвитку. Під час мікроскопії клітин слизової оболонки ротової порожнини статевого хроматину не
виявлено. Для якої хромосомної патології це характерно?
@Синдром Шерешевського—Тернера

42. У експериментального собаки слабкий тип конституції. За якою класифікацією виділено цей тип?
@Павлова

43. Хворий на шизофренію має астенічний тип конституції. За якою класифікацією виявляється такий
зв'язок між будовою тіла та схильністю до психічних захворювань?
@Кречмера

44. Юнак, 18 років, високий на зріст, з міцними кістками та розвиненими м’язами. Який у нього тип
конституції за Сиго?
@М’язовий
45. Хлопець 18 років, зріст 180см, вага 130 кг, дещо надмірне відкладення жиру в області живота і стегон,
страждає запорами. Який у нього тип конституції за Сиго? @Травний

46. Хлопець, 15 років високий на зріст, худий, з тонкими кістками, часто хворіє на респіраторні
захворювання. Який у нього тип конституції за Богомольцем?
@Астенічний

47. У чоловіка атлетичний тип конституції за класифікацїєю Кречмера. Серед хворих на які хвороби
частіше зустрічається цей тип конституції? A. @Епілепсію.

48. У хворого виявлено астенічний тип конституції за класифікацією Кречмера. Серед хворих на які
хвороби частіше зустрічається цей тип конституції? A. @Шизофренію.

49. У хворого гіперстенічний тип конституції за М.В.Чорноруцьким. Який принцип покладений в основу
цієї класифікації?
@Особливості основних функцій організму і обміну речовин.
50. У дитини з екзематозними ураженнями шкіри виявлено ексудативно-катаральний діатез. Що таке
діатез?
@Аномалія конституції з неадекватною реакцією організму на подразники.

51. У дитини з хронічним тонзилітом та аденоїдними вегетаціями виявлено лімфатико-гіпопластичного


діатез. Що ще характерне для цього діатезу? A. @Затримка інволюції вилочкової залози.

52. Людина має астенічний тип конституції за класифікацією О.О.Богомольця. Що характерне для цього
типу? @Тонка, ніжна сполучна тканина.

108.Людина має мозковий тип конституції за класифікацією Сиго. Який ще тип конституції відноситься до
цієї класифікації? A. @Травний.

109. Людина має астенічний тип конституції за класифікацією О.О.Богомольця Який принцип покладений в
основу цієї класифікації? A. @Особливості структури сполучної тканини.
A.
1. Хворі з парціальною недостатністю В-системи імунітету і, відповідно, гуморальної відповіді,
страждають на різні бактеріальні інфекції. У той же час, вони захищені від вірусів, рикетсій, хламідії,
найпростіших та грибів. Яка властивість мікроорганізмів є визначальною для ввімкнення конкретного
типу імунної відповіді – гуморальної або клітинної?
@Розташування відносно клітини (поза-, внутрішньо-)

2. У хворого на бактеріальну (стафілококкову) пневмонію в крові виражений лейкоцитоз, високі титри


антитіл G. Які властивості збудника інфекції є причиною ввімкнення гуморальної відповіді імунної
системи?
@Позаклітинне розміщення

3. У хворого на кавернозний туберкульоз легені в крові лімфо-, моноцитоз. Утворення каверни є


наслідком гіперергічного розвитку запалення в легені, під час якого лімфоцити – хелпери намагаються
допомогти макрофагам знищувати мікобактерії. Які властивості збудника інфекції є причиною
ввімкнення клітинної відповіді імунної системи?
@Внутрішньоклітинне розміщення

4. У крові хворого на вітряну віспу визначаються лімфо-, моноцитоз. Які властивості вірусної інфекції є
причиною ввімкнення клітинної відповіді імунної системи?
@Внутрішньоклітинне розміщення

5. Місце запалення при хламідійній інфекції характеризується вираженою інфільтрацією тканини


лімфоцитами і моноцитами. Які властивості хламідійної інфекції є причиною ввімкнення клітинної
відповіді імунної системи?
@Внутрішньоклітинне розміщення

6. При вірусній інфекції віруси здатні виживати і розмножуватися в клітині. Які саме властивості вірусних
білків дозволяють їм це робити?
@Аналогія з клітинними білками

7. Стратегією виживання деяких вірусів є їх здатність спричинювати надмірну проліферацію інфікованих


клітин та виживати в несприятливих умовах імунної відповіді. Такі перетворення вірус-інфікованих
клітин визначаються як пухлинна трансформація, а віруси, здатні її спричинювати – онкогенні. Які
властивості вірусних білків дозволяють їм це робити? @Вмикати поділ, уникати апоптозу
8. Які віруси не мають онкогенних властивостей (можливості індукції пухлинної трансформації клітин)?
@Ентеро

9. Губчаста енцефалопатія підтвердила діагноз хвороби Крейтцфельда-Якоба в померлого. До якої групи


належить інфекційний агент, що спричинює це захворювання?
@Пріони

10. Спонгіоформна енцефалопатія у хворих на хворобу Крейтцфельда-Якоба спричинена дією пріонного


білка PrPSc. Чим відрізняється патогенний білок PrPSc від нативного білка людини PrPC?
@Конформацією

11. У чоловіка 46 років, хворого на деменцію, діагностовано губчасту енцефалопатію. Що є головною


ланкою в патогенезі дегенеративних змін нейронів при повільних пріонових інфекціях?
@Перевантаження нейронів власними спотвореними протеїнами PrPC

12. У хворих з повільними інфекціями (хворобою Крейтцфельда-Якоба та ін.) практично відсутня імунна
відповідь на пріони. Чим зумовлена така реакція імунної системи? @Толерантністю до власного білка

13. Причиною інфекційного мононуклеозу є вірус Епстайна - Барра. Який білок на поверхні лімфоцитів цей
вірус використовує для проникнення до клітин?
@Рецептори комплементу 2 типу (СR2)

14. У ВІЛ–інфікованих людей відбувається поступове зменшення кількості лімфоцитів–хелперів. Який


білок на поверхні лімфоцитів ВІЛ використовує для проникнення до клітин? @СD 4

15. У хворого на пневмоцистну пневмонію діагностовано СНІД. Який власний білок ВІЛ використовує для
проникнення в клітини - мішені? @gp120

16. Після інфікування у хворого розвинувся хронічний активний гепатит В. Що є механізмом розвитку
цього захворювання? @Імунне ураження гепатоцитів
17. У хворого виявлений дифузний шкірний і прогресуючий вісцеральний лейшманіоз. Який фермент
дозволяє лейшманії швидко знешкоджувати активні форми кисню і уникати фагоцитозу?
@Супероксиддисмутаза

18. У дитини, яка часто хворіє на піодермію, діагностовано порушення фагоцитозу, що спричинено
спадковим дефектом обміну полісахаридів у фагоцитах (хвороба Альдера). Які саме клітини недієздатні
при цьому? @Нейтрофіли

19. Лікування дитини сироваткою проти кору привело до одужання маленького пацієнта. Зі створенням
якого виду резистентності пов’язано одужання дитини при цьому? @Специфічна пасивна

20. Дівчина, яка довго знаходилася на холоді, змерзла. Щоб зігрітися, вона намагається активно рухатися.
Який універсальний адаптивний механізм забезпечує підтримання температури її тіла в межах норми за
таких надзвичайних умов? A. @Стрес

21. Чоловік з переломом кінцівки втратив велику кількість крові. При цьому в нього артеріальний тиск
крові 100/60 мм. рт. ст., частота серцевих скорочень 96 за 1 хв. Який універсальний адаптивний
механізм забезпечує діяльність системи кровообігу в межах норми за таких надзвичайних умов?
@Стрес
22. Загальний стан хворого з невеликим інфарктом лівого шлуночка відносно задовільний. Артеріальний
тиск крові 115/75 мм. рт. ст., частота серцевих скорочень 96 за 1 хв. Який універсальний адаптивний
механізм забезпечує діяльність системи кровообігу в межах норми за таких надзвичайних умов?
A. @Стрес

23. Після крововтрати артеріальний тиск крові в пацієнта є 105/65 мм. рт. ст., частота серцевих скорочень
98 за 1 хв. Які гормони вмикають адаптивні перебудови в організмі при цьому?
@Адреналін, кортизол, АДГ

24. У дитини, яка хворіє на піодермію, встановлено порушення фагоцитозу, яке пов’язують з дефіцитом
НАДФ - оксидази в нейтрофілах. Яка стадія фагоцитозу порушена при цьому?
A. @Перетравлення
25. У дитини після вакцинації виникла стійкість до дифтерії. Як найповніше охарактеризувати
резистентність, яка є в основі цієї стійкості:
A. @Набута специфічна активна

26. Хворий спадковим синдромом Чедіака - Хігасі страждає від гнійничкових уражень шкіри, часто хворіє
на гнійні отити, тонзиліти. Дефект яких органел фагоцитів є причиною недостатності фагоцитозу при
цьому синдромі?
A. @Лізосоми

27. Собака захворів чумкою, тоді як його господар залишився здоровим. Який вид реактивності лежить в
основі такої стійкості господаря?
A. @Видова

28. У пацієнта часто виникає ангіоневротичний набряк ока або губи. Зі спадковим дефіцитом якого білка
пов’язаний розвиток набряку Квінке?
A. @Інгібітора комплементу С1

1. У піддослідних шимпанзе викликали стан стану первинної недостатності клітинного імунітету. Яка з
експериментальних методик може бути застосована для цього?
A. @Видалення тимуса

2. Хворому на гострий лейкоз проведена операція з трансплантації донорського кісткового мозку. Яке
ускладнення операції може створювати реальну загрозу для життя хворого в післяопераційному
періоді?
A. @«Трансплантат проти хазяїна»

3. П’ятирічний хлопчик з народження хворіє на різні інфекції (вірусні, бактеріальні, грибкові). При
обстеженні в нього виявлено зменшення кількості лімфоцитів, відсутність IgА, замала кількість IgG. Як
можна визначити порушення функції імунної системи у хворого?
A. @Первинний комбінований імунодефіцит

4. Дитина 1 року постійно хворіє на вірусні та грибкові інфекції. При обстеженні виявлено відсутність Т -
лімфоцитів. Що може бути причиною такого стану імунітету в дитини?
A. @Гіпоплазія тимусу
5. Хлопчик 9 місяців страждає на гнійні інфекції. При обстеженні: в кістковому мозку - нормальна
кількість пре-В-клітин, відсутність плазматичних клітин; в крові - відсутність В-лімфоцитів та антитіл,
нормальна кількість Т-клітин. Кваліфікуйте стан імунної системи в дитини:
A. @Вроджена недостатність гуморального імунітету

6. Однорічний хлопчик постійно страждає на гнійні інфекції. Обстеження показало в дитини спадкове
захворювання імунної системи – гіпогаммаглобулінемію Бруттона. Як успадковується патологічний
ген, що є причиною цього захворювання?
A. @Зчеплено з Х – хромосомою

7. Хлопчик 1,5 років постійно хворіє на інфекції дихальних шляхів, травного каналу. Спадкова
недостатність продукції яких антитіл є найбільш вірогідною причиною частого інфікування дитини?
A. @Ig A

8. Батьків 6-ти річного хлопчика турбує зниження апетиту і поганий сон у дитини. При обстеженні
з’ясовано, що хлопчик інфікований аскаридами. Антитіла якого класу сприяють еміграції еозинофілів
до місця глистяної інвазії і, таким чином, мають значення в захисті від гельмінтів?
A. @Ig E

9. У ВІЛ-інфікованого пацієнта діагностовано зниження функції антигенпрезентуючих клітин. Які це


клітини? @Макрофаги

10. У ВІЛ-інфікованого пацієнта діагностовано зниження функції антигенпрезентуючих клітин. З


молекулами клітинної поверхні якого типу відбувається презентація антигенів Т-хелперам
антигенпрезентуючими клітинами? @МНС ІІ

11. Однією з причин недостатньої імунної відповіді на Herpes інфекцію є здатність Herpes вірусів
зменшувати експресію білків на поверхні інфікованих клітин. За якими молекулами відбувається
ідентифікація інфікованих клітин як мішеней лімфоцитами- кілерами? @МНС І

12. Після інтенсивної хіміотерапії злоякісної пухлини у хворого виникло порушення імунної системи. Який
вид недостатності імунітету виник у нього?
A. @Вторинний комбінований імунодефіцит
13. Пацієнту з вираженим кандидозом ротової порожнини рекомендовано подальше обстеження для
вирішення питання про можливу ВІЛ- інфекцію. Зі зменшенням кількості яких клітин в організмі
пов'язаний розвиток СНІДу? @Т CD4
14. Чоловік потрапив до лікарні з симптомами менінгіту. При обстеженні: криптококовий менінгіт;
недостатність функції макрофагів, критичне зменшення кількості CD4 Т лімфоцитів в крові та величини
співвідношення CD4/CD8 до 0,5. Кваліфікуйте стан імунної системи в пацієнта: @СНІД

15. Хворий скаржиться на загальну слабкість, постійний кашель, пітливість, підвищення температури тіла.
При обстеженні діагностовано пневмоцистну пневмонію, СНІД. Первинна недостатність функції яких
клітин є причиною опортуністичної інфекції при СНІДі?
A. @Макрофаги, Т - хелпери

16. У хворого на ожиріння, гіперглікемію, інфекцію, що постійно рецидивує, виявлено пухлину кори
наднирників, що продукує надмірну кількість гормонів. Надлишок яких гормонів є причиною
недостатності імунітету в пацієнта?
A. @Кортизолу

17. У патогенезі ураження ЦНС при СНІДі має значення вторинна інфекція, запалення і токсична дія
цитокінів на нейрони. Які клітини транспортують ВІЛ до ЦНС? @Макрофаги

18. Огляд показав множинний карієс у дитини. Недостатня продукція яких антитіл може бути причиною
полегшеного інфікування ротової порожнини Strеptococcus mutans з подальшим порушенням їх
фагоцитозу нейтрофілами?
A. @Ig A, G

19. Хлопчик, 2 років, народжений від ВІЛ – інфікованої матері, також ВІЛ – інфікований. Він регулярно
піддається обстеженню з метою завчасної діагностики СНІДу. Зменшення кількості яких клітин в крові
є достовірним критерієм розвитку СНІДу?
A. @CD4 Т

20. Немовля з вродженим стенозом аорти страждає на грибкову і вірусну інфекцію. При обстеженні:
нормальна кількість пре-В- і плазматичних клітин у кістковому мозку; нормальна кількість В-,
відсутність Т-клітин у циркуляції. Який стан імунітету у дитини?
A. @Синдром Ді Джорджи

21. Немовля з гіпоплазією тимуса та прищитовидних залоз страждає на тетанію, інфекції. При обстеженні:
нормальна кількість пре-В- і плазматичних клітин у кістковому мозку; нормальна кількість В-,
відсутність Т-клітин у циркуляції. Яка інфекція є типовою для такого стану імунітету? @Віруси

22. У чоловіка, що вже місяць хворіє на пневмонію, виявляються збільшені лімфатичні вузли в ділянці шиї,
під пахвами. В крові хворого майже відсутні зрілі нейтрофіли. Дослідження кісткового мозку показало
«Гострий лейкоз». Інфекція якого типу є закономірною для станів з недостатністю нейтрофілів та
фагоцитозу? A. @Піогенні коки
23. Після курсу терапії лімфогранулематозу в пацієнта розвинулася пневмонія. Який з терапевтичних
заходів став причиною недостатності імунітету?
A. @Хіміотерапія

24. У хлопчика з лімфомою розвинулася важка імунодепресія після застосування хіміотерапії і гормонів під
час лікування лімфоми. Які гормони спричинюють апоптоз лімфоцитів?
A. @Глюкокортикоїди
1. Провідною ланкою в патогенезі автоімунних хвороб є зняття толерантності імунної системи до власних
антигенів. Що може бути причиною цього порушення імунної системи?
A. @Презентація автоантигенів

2. Провідною ланкою в патогенезі автоімунних хвороб є зняття толерантності імунної системи до власних
антигенів. Що може бути причиною цього порушення імунної системи?
A. @Модифікація автоантигену

3. Яке автоімунне захворювання розвивається за загальним з "сироватковою хворобою" механізмом?


A. @Системний червоний вовчак

4. Системний червоний вовчак (СЧВ) є автоімунним захворюванням, при якому надвелика кількість
імунних комплексів автоантиген-антитіло-комплемент спричинюють пошкодження і запалення нирок,
суглобів, шкіри, ЦНС, інших тканин. На який автоантиген реагує імунна система при СЧВ?
A. @Ядерні білки (нуклеопротеїди)

5. При хворобі Базедова щитоподібна залоза продукує надмірну кількість гормонів Т 3, Т4. Що є причиною
гіперфункції тироцитів при цьому автоімунному захворюванні?
A. @Стимуляція антитілами G

6. Імунна система не толерантна до органів і тканин, які «приховані» від неї за спеціалізованими
гематопаренхіматозними бар’єрами. При пошкодженні бар’єру якого органа може виникати автоімунне
захворювання?
A. @Головний мозок

7. У хворого з травмою ока виникло автоімунне запалення у другому оці, що призвело до значного
зниження зору. Який механізм призвів до автоімунного ушкодження ока?
A. @Демаскування автоантигену

8. З метою попередження розвитку реакції «трансплантат проти хазяїна», пацієнту з пересадженим


кістковим мозком призначено курс імунодепресивної терапії. Які гормони застосовують з цією метою?
@Глюкокортикоїди
9. Автоімунний інсуліт, що є причиною розвитку цукрового діабету 1 типу, розвивається за механізмом
автоімунних реакцій 4-го типу, за Кумбсом і Джеллом. Що є причиною пошкодження β-клітин
островців підшлункової залози?
A. @Т-лімфоцити

10. При цукровому діабеті 1 типу в організмі створюється важкий дефіцит інсуліну з причини зменшення
кількості В-клітин в островках Лангенганса, що, в 95% випадків, є наслідком автоімунного запалення
(інсуліту). Зазначте механізм знищення В-клітин островків підшлункової залози при цьому:
A. @Агресія кілерів
11. Жінка скаржиться на погіршення пам’яті, сухість шкіри, набряклість обличчя. Діагностовано
гіпофункцію щитоподібної залози з причини автоімунного тиреоїдиту, що розвивається за механізмом
реакцй 4-го типу, за Кумбсом і Джеллом. Що є механізмом знищення тироцитів при автоімунному
тироїдиті:
A. @Агресія кілерів
1. У хворої на системний червоний вовчак виявлені протеїнурія, гематурія, набряки, ураження шкіри та
суглобів. Алергічні реакції якого типу (за Кумбсом і Джеллом) є причиною тканинного пошкодження в
патогенезі цього захворювання?
A. @Опосередковані імунними комплексами

2. Через годину після вживання амідопірину пацієнт скаржиться на біль в горлі, неможливість ковтання
слини. При обстеженні: некротична ангіна, у крові є антилейкоцитарні антитіла, нейтропенія. Алергічні
реакції якого типу, за класифікацією Кумбса і Джелла, лежать в основі цієї патології?
A. @Опосередковані IgG, цитотоксичні

3. До лікаря звернулася жінка зі скаргами на те, що після вживання горіхів і меду у неї виникає
уртикарний висип на обличчі та шиї. Алергічні реакції якого типу за Кумбсом і Джелом має місце у
цьому випадку?
A. @Опосередковані IgE

4. Через місяць після встановлення зубних протезів у пацієнта з’явилися локальні (в місці металевих
коронок) почервоніння й ущільнення ясен. Які алергічні реакції визначають розвиток гінгівіту в цьому
випадку?
A. @Опосередковані макрофагами і Т- лімфоцитами

5. У хворого на правець через 2 тижні після введення значної дози лікувальної (кінської) сироватки
виникли уртикарний висип, біль в суглобах та м’язах, гарячка. Алергічна реакція якого типу
розвинулася у хворого?
A. @Опосередкована імунними комплексами

6. Пацієнт відмічає почервоніння, ущільнення шкіри долоней після контакту з пральним порошком.
Алергічні реакції якого типу є причиною розвитку контактного дерматиту?
A. @Опосередковані макрофагами і Т- лімфоцитами

7. У зубного техніка виражений контактний дерматит. Алергічні реакції якого типу, за Кумбсом і
Джеллом, визначають розвиток цього захворювання?
A. @Опосередковані макрофагами і Т- лімфоцитами

8. У хворого на сифіліс на другому тижні від початку терапії великими дозами пеніціліну, з'явилися
симптоми гломерулонефриту, артриту, уртикарні висипи на шкірі, що було схоже на "сироваткову
хворобу". Алергічні реакції якого типу, за Кумбсом і Джеллом, є причиною ускладнення
протимікробної терапії у пацієнта?
A. @Опосередковані імунними комплексами

9. У хворого з вітамін В12–дефицитною анемією, що резистентна до терапії вітаміном В 12, виявлена велика
кількість IgG проти рецепторів ентероцитів, що забезпечують рецептор-залежний ендоцитоз комплексу
«вітамін В12–внутрішній фактор Касла». Імунні реакції якого типу (за Кумбсом і Джеллом) є головною
ланкою в патогенезі перніціозної анемії в пацієнта?
A. @Опосередковані IgG клітинні дисфункції

10. На 8 день після введення протиправцевої сироватки з приводу брудної рани стопи у пацієнта
температура тіла підвищилася до 38 оС, з'явилися біль в суглобах, висип, свербіж, протеїнурія.
Алергічна реакція якого типу розвинулася в цьому випадку?
A. @Опосередкована імунними комплексами

11. Після введення антитіл до білків базальної мембрани ниркових клубочків у тварини розвинувся гострий
гломерулонефрит. Алергічні реакції якого типу, за класифікацією Кумбса і Джелла, лежить в основі цієї
патології?
A. @Опосередковані IgG, цитотоксичні

12. Через декілька хвилин після ін’єкції ліків у пацієнта з’явилися відчуття нестачі повітря, утруднення
видоху, шкіра вкрилася потом, артеріальний тиск крові знизився до 80/40 мм.рт.ст. Алергійні реакції
якого типу, за Кумбсои і Джелом, зумовили вказані патологічні симптоми?
A. @I типу, системні

13. Пацієнт скаржиться на напади серцебиття, пітливість, дратівливість. Обстеження показало в нього
дифузний токсичний зоб. Імунні реакції якого типу, за Кумбсом і Джеллом, є причиною гіперфункції
щитоподібної залози при Базедовій хворобі?
A. @Опосередковані IgG клітинні дисфункції

14. У дитини визначається позитивна реакція Манту. Імунні реакції якого типу, за Кумбсом і Джеллом, є
причиною почервоніння, ущільнення і некротичних змін шкіри у місці введення туберкуліну?
A. @IV типу
15. Жінка звернулася зі скаргами на свербіж та печіння в очах, сльозотечу, виділення з носа. Симптоми
з’явилися під час поїздки за місто. Діагностовано поліноз. Алергічні реакції якого типу, за Кумбсом і
Джелом, спричиюють розвиток полінозу? A. @I типу, локальні

16. Ділянка шкіри в місці контакту з металевим браслетом почервоніла, ущільнилась. До сплаву, з якого
виготовлений браслет, входять солі нікелю і хрому. Алергічні реакції якого типу, за Кумбсом і
Джеллом, є причиною контактного дерматиту в пацієнта?
A. @Опосередковувані макрофагами і лімфоцитами

17. У хворого з протеїнурією, гематурією, набряками та артеріальною гіпертензією встановлено діагноз


«Гострий гломерулонефрит». Алергічні реакції якого типу, за Кумбсом і Джеллом, є причиною
гломерулонефриту у 80% випадків?
A. @Опосередковувані імунними комплексами
18. У паціента з недостатністю дихання діагностовано атопічну бронхіальну астму. Алергічні реакції якого
типу, за Кумбсом і Джеллом, є причиною обструкції нижніх дихальних шляхів і недостатності
альвеолярної вентиляції в хворого?
A. @Опосередковувані IgЕ

19. Через тиждень після застосування протиправцевої сироватки у пацієнта підвищилася температура тіла,
з'явився висип, напухли суглоби, у сечі з'явився білок. Алергічні реакції якого типу (за Кумбсом і
Джеллом) є головною ланкою в патогенезі сироваткової хвороби у пацієнта?
A. @III типу
192.У пацієнта з ідіопатичною тромбоцитопенічною пурпурою у крові визначається високий титр
антитромбоцитарних IgG. Імунні реакції якого типу, за Кумбсом і Джеллом, визначають розвиток
тромбоцитопенії у цього хворого?
A. @ІI типу

1. Усі члени сім'ї пацієнта Д., 22 років, страждають від алергічних реакцій анафілактичного типу. Яким
терміном описують спадкову схильність до розвитку анафілактичних реакцій?
A. @Атопія

2. У Каті З. вживання в їжу червоної риби завжди супроводжується відчуттям дискомфорту, болями в
животі, іноді проносом. Як називається локальна анафілактична реакція в травному тракті?
A. @Алергічний гастроентерит

3. При будь-якому контакті з сухим кормом для рибок Женя Т. миттєво починає задихатися. Вкажіть
клінічний прояв алергічної реакції анафілактичного типу в нижніх дихальних шляхах:
A. @Атопічна бронхіальна астма

4. У деяких людей є генетичні передумови розвитку певних анафілактичних реакцій на харчові антигени.
Як називаються клінічні прояви локальної анафілаксії в шкірі?
A. @Кропив'янка

197.Через 10 днів після введення протидифтерійної сироватки у дитини з'явився висип на шкірі, який
супроводжувався сильним свербінням, підвищилася температура тіла до 38 оC, з'явився біль у суглобах. Яке
ускладнення виникло?
A. @Сироваткова хвороба

1. У дитини після вживання червоної малини виникли уртикарні висипи на шкірі, які супроводжувалися
сильним свербінням, підвищилася температура тіла до 37,5 оC. Яке захворювання виникло?
A. @Кропив’янка

2. Під час внутрішньовенної ін’єкції ліків шкіра хворого вкрилася холодним потом, кінцівки захолонули,
з’явилися задишка і утруднення дихання. Назвіть системну анафілактичну реакцію, що розвинулася у
пацієнта: @Анафілактичний шок
3. Одразу після внутрішньом’язового введення антибіотика у пацієнта з'явилися задуха, відчуття страху,
акроціаноз. Назвіть системну анафілактичну реакцію у пацієнта:
A. @Анафілактичний шок

4. У місцях розвитку анафілактичних реакцій виражені гіперемія, накопичення ексудату, набряки і


свербіння. Який медіатор анафілаксії визначає розвиток вищезазначених симптомів?
A. @Гістамін
202.У жінки з бронхіальною астмою після контакту з собакою виник напад задухи. Які медіатори анафілаксії
є причиною тривалого бронхоспазму?
A. @Лейкотриєни

1. У патогенезі гострої недостатності дихання при анафілактичному шоку є бронхоспазм, гіперсекреція


слизу, набряк слизової оболонки нижніх дихальних шляхів. Які з медіаторів зумовлюють розвиток
даних порушень?
A. @Гістамін, лейкотрієни

2. Під час внутрішньовенної інекції ліків у пацієнта розвинувся анафілактичний шок. Які медіатори
анафілаксії спричинюють безпосередньо обструкцію нижніх дихальних шляхів і гостру недостатність
дихання при цьому?
A. @Гістамін, лейкотрієни

3. Катехоламіни / адреноміметики можуть істотно поліпшити прохідність нижніх дихальних шляхів і тому
використовуються для надання невідкладної допомоги пацієнтам з алергічними реакціями:
A. @Анафілактичний шок

4. Вже багато років жінка страждає на алергійний риніт. Вкажіть головну ланку в патогенезі цього
захворювання:
A. @Алерген – індукована дегрануляція тканинних базофілів
207.У Маші Д., 7 років, алергія на волоський горіх. Вживання в їжу будь-якого продукту, що містить горіхи,
призводить до появи у дитини висипу, що свербить. З якими антитілами пов'язаний розвиток алергічних
реакцій анафілактичного типу? A. @IgE

1. Для постановки туберкулінової проби дитині внутрішньошкірно введено туберкулін. Через 24 години в
місці введення туберкуліну відзначена виражена гіперемія, ущільнення шкіри з некрозом в центрі. Який
механізм лежить в основі розвитку даних змін?
A. @Клітинна цитотоксичність

2. Для постановки туберкулінової проби дитині внутрішньошкірно введено туберкулін. Через 24 години в
місці введення відзначена виражена гіперемія, ущільнення тканини. Що спричинює пошкодження і
запалення у цьому випадку?
A. @Т-лімфоцити, макрофаги

3. У жінки віком 45 років, яка тривалий час хворіє на атопічну бронхіальну астму, виник напад ядухи. Що
є причиною нестачі повітря в хворої при цьому?
A. @Спазм, обтурація дрібних бронхів

4. Одазу після введення анестетика в пацієнта розвинувся анафілактичний шок. Який механізм визначає
розвиток гострої недостатності кровообігу при цьому? A. @Вазодилатація

5. Хворому М. перед введенням лікувальної дози протиправцевої сироватки було зроблено пробу на
чутливість до неї. Проба виявилася позитивною. Яким чином найбільш правильно провести специфічну
гіпосенсибілізацію за Безредкою?
A. @Введенням малих доз сироватки

6. Дитині 10 років, що під час гри порізала ногу відламком скла, з метою попередження ускладнень
протиправцеву сироватку вводили за Безредкою. Який механізм лежить в основі подібного способу
гіпосенсибілізації?
A. @Зв'язування специфічних IgE

7. Під час парентерального введення ліків у пацієнта розвинувся анафілактичний шок. Антитіла якого
класу відіграють головну роль у патогенезі анафілактичного шоку у людини?
A. @IgE
8. У резус-негативної дитини, народженої від резус-позитивної матері, виникла гемолітична хвороба
новонароджених. Антитіла якого класу спричинюють гемоліз еритроцитів і, таким чином, відіграють
головну роль у патогенезі захворювання? A. @IgG

9. У тварини при моделюванні феномену Артюса на шкірі з’явився осередок геморагічного некрозу.
Активація якої системи призводить до пошкодження судинної стінки при імунокомплексних реакціях?
A. @Комплементу
217.У пацієнта з некротичною ангіною при обстеженні у крові виявлені нейтропенія й антилейкоцитарні
антитіла. Що є одним з механізмів пошкодження нейтрофілів у цьому випадку?
A. @Комплемент-залежний цитоліз
1. У експерименті кроля опромінили рентгенівськими променями і отримали гостру променеву хворобу.
Вкажіть патогенетичний варіант пошкодження клітин у цьому випадку:
A. @Насильне

2. У експерименті кроля тримали в барокамері за умов розрідженого повітря. Вкажіть патогенетичний


варіант пошкодження клітин у цьому випадку:
A. @Цитопатичне

3. У експерименті кроля тримали на дієті, що не містить вітамінів С і Е. Вкажіть патогенетичний варіант


пошкодження клітин у цьому випадку:
A. @Цитопатичне

4. У експерименті кроля опромінили рентгенівськими променями і отримали гостру променеву хворобу.


Який механізм викликав активацію перекисного окислення ліпідів у цьому випадку?
A. @Посилення утворення вільних радикалів

5. У експерименті кроля тримали у барокамері за умов підвищеного тиску кисню. Який механізм викликав
активацію перекисного окислення ліпідів у цьому випадку?
A. @Посилення утворення вільних радикалів

223.У експерименті кролю вводили великі дози вітаміну Д. Який механізм викликав активацію перекисного
окислення ліпідів у цьому випадку?
A. @Посилення утворення первинних вільних радикалів

1. У експерименті кроля тримали на дієті, що не містить вітамінів С і Е. Який механізм викликав


активацію перекисного окислення ліпідів у цьому випадку?
A. @Недостатність первинних антиоксидантів

2. При інфаркті міокарду в клітинах серцевого м’яза активуються ліпідні механізми пошкодження клітин.
Який з перерахованих механізмів відноситься до ліпідних? A. @Активація перекисного окислення
ліпідів

3. При інфаркті міокарду в клітинах серцевого м’яза підвищується концентрація іонів кальцію, що значно
посилює їх пошкодження. Який з перерахованих механізмів відноситься до кальцієвих? @Активація
мембранних фосфоліпаз
4. При ішемії в клітинах підвищується концентрація іонів водню, що значно посилює їх пошкодження.
Який з перерахованих механізмів відноситься до ацидотичних?
@Підвищення проникності мембран

5. При інфаркті міокарду в клітинах серцевого м’яза підвищується концентрація іонів кальцію, що значно
посилює їх пошкодження. Який з перерахованих механізмів відноситься до кальцієвих?
A. @Роз’єднання окиснення та фосфорилювання

6. При гіпоксії у клітинах порушується робота іонних насосів, що значно посилює пошкодження. Який з
перерахованих механізмів пошкодження клітин відноситься до електролітно-осмотичних?
A. @Втрата клітиною потенціалу спокою

7. При голодуванні в клітинах порушується структура та функція білків, що значно посилює пошкодження
клітини. Який з перерахованих механізмів відноситься до протеїнових?
A. @Зменшення активності ферментів

8. При дефіциті вітаміну В12 в клітинах порушується синтез азотистих основ, що значно посилює
пошкодження клітини. Який з перерахованих механізмів відноситься до нуклеїнових?
A. @Зменшення реплікації

9. При ішемії у міокардіоцитах виникає контрактура міофібрил, роз’єднання окиснення та фосфорування


та активація фосфоліпази А2. Які механізми пошкодження клітин переважають у цьому випадку?
A. @Кальцієві

10. При дії іонізуючої радіації у клітинах значно активується перекисне окиснення ліпідів. Які механізми
пошкодження клітин переважають у цьому випадку? A. @Ліпідні

11. У клітині виникли підвищення проникності мембран, активація лізосомних ферментів, порушення
активності ферментів та структурних білків. Які механізми пошкодження клітин переважають у цьому
випадку?
A. @Ацидотичні
12. У клітині виникли внутрішньоклітинний набряк, осмотичне розтягнення мембран та втрата потенціалу
спокою. Які механізми пошкодження клітин переважають у цьому випадку?
A. @Електолітно-осмотичні
1. У кроля після видалення правого верхнього шийного симпатичного вузла спостерігаються почервоніння
й підвищення температури шкіри правої половини голови. Який вид артеріальної гіперемії розвинувся в
кролика?
A. @Нейропаралітична

2. У хворого на хронічну серцеву недостатність часто виникають головний біль, запаморочення та


знепритомнення. Який вид порушень місцевого кровообігу виник у головному мозку в цьому випадку?
A. @Ішемія

3. Експериментатор при подразненні вагуса спостерігав збільшення коронарного кровотоку. Який вид
артеріальної гіперемії виник при цьому?
A. @Нейротонічна

4. Експериментатор при подразненні horda tympani спостерігав гіперемію піднижньощелепної слинної


залози. Який вид артеріальної гіперемії виник при цьому? A. @Нейротонічна

5. У хворого на хронічну серцеву недостатність ноги набряклі, ціанотичні, холодні на дотик. Який вид
порушень місцевого кровообігу є головним у виникненні цих змін?
A. @Венозна гіперемія

241.Відновлення коронарного кровообігу у хворого з тромбозом вінцевої артерії супроводжувалося


зниженням скорочувальної здатності серця і погіршенням загального стану пацієнта. Яке явище стало
причиною ускладнення?
A. @Реперфузійний синдром

1. У хворого з облітеруючим ендартеріїтом після гангліонарної симпатектомії спостерігається позитивний


лікувальний ефект, пов’язаний з розвитком артеріальної гіперемії нижніх кінцівок. Який вид гіперемії
виник у пацієнта?
A. @Нейропаралітична

2. У чоловіка, 57 років, який скаржиться на біль у ділянці серця після негативних емоцій, встановлено
ішемічну хворобу серця. Який механізм ішемії найбільш імовірний?
A. @Ангіоспастичний.
3. Після нанесення ацетилхоліну на вухо кролика місцево виникло почервоніння цієї ділянки шкіри. Який
вид артеріальний гіперемії виник у кролика?
A. @Нейротонічна

4. Тривале перебування на холоді викликало збліднення шкіри вуха у дитини 5 років. Яка речовина
зумовила розвиток ішемії?
A. @Норадреналін
5. Відновлення коронарного кровообігу у хворого на інфаркт міокарда супроводжувалося зниженням
скорочувальної здатності серця і погіршенням загального стану пацієнта. Які реакції визначають
збільшення пошкодження кардіоміоцитів в умовах реперфузії?
A. @Вільнорадикальні

6. У хворої внаслідок емболії малого кола кровообігу виникли гіпертензія в малому колі, гіпотензія у
великому колі кровообігу та збільшення центрального венозного тиску. Що є основною причиною
порушення кровообігу в цьому випадку?
A. @Правошлуночкова недостатність

7. Внутрішньовенне введення тромбопластину тварині призвело до тромбозу. Який механізм лежить в


основі його розвитку?
A. @Активація системи зсідання крові

8. Відомо, що у венах тромби утворюються в п'ять разів частіше, ніж у артеріях, а у венах нижніх кінцівок
- у три рази частіше, ніж у венах верхніх кінцівок. Вкажіть, з яким патогенетичним фактором, що
зумовлює процес тромбоутворення, пов'язане це явище?
A. @Уповільненням кровообігу

9. У хворої внаслідок емболії малого кола кровообігу виникли гіпертензія в малому колі, гіпотензія у
великому колі кровообігу та збільшення центрального венозного тиску. Що є основною причиною
правошлуночкової недостатності в цьому випадку?
A. @Спазм артеріол легень

251.Низка патологічних процесів (атеросклероз судин, запалення, механічна травма, некроз тканин)
супроводжується тромбоутворенням. Який патогенний фактор у механізмі тромбоутворення відіграє роль
ініціатора при цих процесах?
A. @Ушкодження ендотелію судин

1. У хворого з пораненням яремної вени встановлено зниження артеріального, збільшення центрального


венозного тиску, ціаноз. Яке порушення периферичного кровообігу виникло у хворого?
A. @Емболія малого кола кровообігу

2. У хворого 73-х років з закритим переломом правої стегнової кістки діагностовано емболію судин
головного мозку. Назвіть вид емболії, що найбільш часто виникає при переломах трубчастих кісток:
A. @Жирова

3. У хворої внаслідок емболії малого кола кровообігу виникли гіпертензія в малому колі, гіпотензія у
великому колі кровообігу та збільшення центрального венозного тиску. Що є основною причиною
смерті в цьому випадку?
A. @Правошлуночкова недостатність
4. Ембол, що утворився у великому колі кровообігу знову потрапив у судини великого кола, минаючи
мале коло. Як називається такий вид емболії?
A. @Парадоксальна

5. Нанесення гіпертонічного розчину NaCl на брижу жаби призвело до швидкого уповільнення і зупинки
течії крові в судинах. Яке порушення кровообігу спостерігалося при цьому?
A. @Стаз істинний

6. У хворої на бешиху виникла облітерація лімфатичних судин, що призвело до «слоновості». Який


різновид порушень мікроциркуляції виник у хворої?
A. @Механічна лімфатична недостатність

7. У експерименті після внутрішньовенного введення декстрину в мікросудинах утворилась велика


кількість агрегатів еритроцитів, що суттєво порушили мікроциркуляцію. Який головний механізм
виникнення цих змін?
A. @Порушення суспензійної стабільності крові

8. У хворої на рак молочної залози після видалення реґіонарних лімфовузлів виник набряк руки. Який
різновид порушень мікроциркуляції виник у хворої?
A. @Механічна лімфатична недостатність

9. У хворого на хронічну серцеву недостатність виникло порушення лімфотоку в нижніх кінцівках. Який
різновид порушень мікроциркуляції виник у хворого?
A. @Механічна лімфатична недостатність

10. У хворого на цироз печінки різко знизився артеріальний тиск. Який патологічний механізм є
ініціатором порушення гемодинаміки при цьому?
A. @Збільшення депонування крові в портальному руслі

11. У експерименті після внутрішньовенного введення метилцелюлози в мікросудинах утворилась велика


кількість агрегатів еритроцитів, що суттєво порушили мікроциркуляцію. Який головний механізм
виникнення цих змін?
A. @Порушення суспензійної стабільності крові

12. У хворого з алкогольним цирозом печінки раптово виникли загальна слабкість, задишка, набряки,
асцит, розширення шкірних вен передньої стінки живота. Яке порушення периферичного кровообігу
має місце у цього хворого?
A. @Емболія ворітної вени
13. У експерименті після внутрішньовенного введення метилцелюлози в мікросудинах утворилась велика
кількість агрегатів еритроцитів.. Яке порушення мікроциркуляції виникло?
A. @Сладж

14. У хворої на варикозне розширення вен після невдалої операції виникла облітерація лімфатичних судин,
що призвело до «слоновості». Який різновид порушень мікроциркуляції виник у хворої?
A. @Механічна лімфатична недостатність

266. У місці опіку з’явилась ділянка некрозу. Який фактор викликає первинну альтерацію?
A. @Флогогенний агент

267. Після опіку в зоні гіперемії та набряку шкіри у хворого з’явилася ділянка некрозу. Який механізм
спричинює посилення руйнівних явищ в осередку запалення?
A. @Вторинна альтерація
268. У дитини з гнійним апендицитом виявлене значне ушкодження тканини апендикса. Які фактори
зумовлюють розвиток вторинної альтерації в осередку запалення?
A. @Лізосомні ферменти

269. У центрі опіку з’явилась ділянка некрозу. Які медіатори запалення можуть брати участь у вторинній
альтерації?
A. @Вільні радикали

270. Хворий скаржиться на біль у місці опіку. Які біологічно активні речовини є медіаторами болю при
запаленні?
A. @Гістамін, кініни

271. В осередку запалення шкіра набрякла, червона, гаряча. Що спричинює місцеве почервоніння тканини в
місці запалення?
A. @Розширення артеріол

272. На брижі жаби під мікроскопом спостерігали розширення судин, прискорення кровотоку, осьовий рух
крові в судинах у місці запалення. Яка артеріальна гіперемія розвинулась при цьому?
A. @Метаболічна

273. На брижі жаби в осередку запалення спостерігали артеріальну гіперемію. Який фактор є головним у
патогенезі цього порушення кровообігу?
A. @Дія біологічно активних речовин

274. У місці опіку спостерігається почервоніння. Які медіатори запалення викликають артеріальну
гіперемію?
A. @Гістамін

275. При позитивній реакції Манту шкіра в місці введення туберкуліну ущільнена, червоного кольору. Яка
речовина може бути причиною почервоніння тканини?
A. @NO

276. Дерматит супроводжувався почервонінням. Які речовини можуть цьому сприяти? A. @Простагландини

277.При позитивній реакції Манту шкіра в місці введення туберкуліну червоного кольору. Вплив якого іону
на артеріоли може бути причиною почервоніння тканини?
A. @К+

1. В осередку гострого запалення активно взаємодіють різні клітини. Більшість з них є


універсальним джерелом неспецифічних медіаторів гострого запалення, які утворюються з фосфоліпідів
клітинних мембран. Які це медіатори?
A. @Ейкозаноїди

2. Основними медіаторами гострого запалення є похідні арахідонової кислоти. Назвіть ці медіатори?


A. @Простагландини, лейкотрієни

3. У осередку запалення шкіри спостерігається набряк. Які медіатори запалення підвищують проникність
судин?
A. @Гістамін

4. У осередку гострого запалення спостерігається набряк. Які медіатори запалення підвищують


проникність судин?
A. @Простагландини

5. Через дві години після травми в місці пошкодження утворився набряк. Що зумовлює ексудацію при
запаленні?
A. @Медіатори

6. У хворого отримали 100 мл гнійного ексудату з плевральної порожнини. Який механізм зумовлює
ексудацію в даному випадку?
A. @Підвищення судинної проникності

7. У місці гострого запалення тканина набрякла. Що сприяє ексудації в осередку запалення?


A. @Підвищення гідростатичного тиску крові
8. При ангіні мигдалики у хворого червоні набряклі. Який механізм зумовлює ексудацію в осередку
запалення?
A. @Підвищення судинної проникності

9. У місці опіку з’явився набряк. Який механізм сприяє ексудації в осередку запалення?
A. @Підвищення онкотичного тиску в тканині

10. У пацієнта карієс ускладнився пульпітом, що супроводжується нестерпним болем. Що є основною


причиною виникнення болю у даного хворого?
A. @Ексудація

288.Хворий з фурункулом на руці скаржиться на сильний біль, набряк та почервоніння у місці запалення,
неможливість нормальних рухів рукою, гарячку, сонливість. Який з цих симптомів відноситься до місцевих
проявів запалення?
A. @Набряк

1. У хворого на гострий апендицит виявили у крові лейкоцитоз, підвищення ШОЕ, появу білків гострої
фази запалення. Яка речовина зумовила ці загальні прояви запалення?
A. @Інтерлейкін 1

2. У хворого на остеомієліт спостерігаються гарячка, лейкоцитоз, підвищення ШОЕ, міалгії, артралгії,


сонливість. Які клітини вивільнюють медіатор запалення, що зумовлює появу цих загальних реакцій
організму?
A. @Макрофаги

3. Хворий з фурункулом на руці скаржиться на сильний біль, набряк та почервоніння у місці запалення,
неможливість нормальних рухів рукою, гарячку, сонливість. Який з цих симптомів відноситься до
загальних проявів запалення?
A. @Гарячка (febris)

4. У зоні запалення лівої гомілки на другу добу відзначається: припухлість, почервоніння, біль, жар. Який
провідний механізм зумовив місцеве підвищення температури?
A. @Артеріальна гіперемія

293.При запаленні артеріальна гіперемія переходить у венозну. Які фактори цьому сприяють?
A. @Активація згортання крові.
1. В експерименті одержано алергічне запалення шкіри у тварини. Спостерігається гіперемія, набряк та
еміграція моноцитів в осередок запалення. З дією якого медіатора пов’язаний хемотаксис моноцитів?
A. @Лімфокіни
2. Під час другої стадії фагоцитозу об’єкт фіксується до поверхні фагоциту. Які речовини стимулюють цей
процес і є опсонінами при запаленні?
A. @Імуноглобуліни

3. Після поглинання об’єкту фагоцитозу відбувається його знищення спеціальними бактерицидними


механізмами. Яка ферментативна система нейтрофілів є основою їх бактерицидності?
A. @Мієлопероксидазна система

4. Після поглинання мікроорганізмів фагоцитами відбувається їх знищення бактерицидними речовинами.


Назвіть такі речовини?
A. @Активні кисневі радикали

5. У стадію прилипання фагоцитозу об’єкт фіксується до поверхні фагоцита. Який механізм цього явища?
A. @Опсонізація

6. Внаслідок гіперонкії в місці запалення з'явився набряк. Який механізм є провідним у виникненні
підвищеного онкотичного тиску?
A. @Скупчення макромолекул у тканинній рідині

7. Внаслідок гіперонкії в місці запалення з'явився набряк. Який механізм є провідним у виникненні
підвищеного онкотичного тиску в тканинній рідині?
A. @Підвищення проникності судинної стінки

8. Під час запалення спостерігається вихід лейкоцитів з судин у тканину. Які гуморальні медіатори
стимулюють хемотаксис лейкоцитів у осередок запалення?
A. @Компоненти комплементу С3а, С5а

9. У хворого з позитивною реакцією Манту шкіра інфільтрована макрофагами і лімфоцитами. Які


медіатори спричинюють це явище?
A. @Лімфокіни
10. Під час запалення спостерігається вихід лейкоцитів з судин у тканину. Які медіатори стимулюють
адгезію лейкоцитів до стінок судин (крайове стояння)?
A. @Селектини

304.При вивченні під мікроскопом розвитку запалення на язиці жаби спостерігали крайове стояння
лейкоцитів та їх еміграцію крізь судинну стінку. Який фактор є провідним у цьому процесі?
A. @Поява адгезивних білків на ендотелії в осередку запалення

1. У порожнину абсцесу, індукованого скипидаром у піддослідної тварини, ввели смертельну дозу


правцевого токсину, а тварина не загинула. Яка ймовірна причина такого результату досліду?
A. @Формування бар’єру навколо вогнища запалення.

2. У зоні запалення виникла інтенсивна деструкція клітин. Яке місцеве фізикохімічне явище буде
пов'язане з цим?
A. @Гіперкалійіонія

3. У зоні запалення виникла інтенсивна протезна деструкція клітин. Яке місцеве фізико-хімічне явище
буде пов'язане з цим?
A. @Гіперонкія

4. У експерименті на кролях було виявлено підвищення кількості макромолекул у міжклітинному


середовищі при запаленні. Яка фізико-хімічна зміна виникла? A. @Гіперонкія.

5. У експерименті на кролях було виявлено підвищення активності гліколізу в осередку запалення. Яка
фізико-хімічна зміна виникла? A. @Ацидоз.

6. У експерименті було виявлено підвищення кількості мікромолекул у позаклітинному середовищі при


запаленні. Яка фізико-хімічна зміна виникла? A. @Гіперосмія.

7. У експерименті на брижі жаби було виявлено ексудацію в осередок запалення. Який механізм першої
фази підвищення проникності судинної стінки? A. @Заокруглення ендотеліоцитів.

8. Після незначної травми структура пошкодженої тканини повністю відновилася. Як називається такий
різновид проліферації?
A. @Регенерація

9. Після значної травми сформувався рубець і структура пошкодженої тканини повністю не відновилася.
Як називається такий різновид проліферації?
A. @Репарація

10. У хворого з рваною раною кількість гною значно зменшилась, на дні рани з’явились грануляції. Які
біологічно-активні речовини стимулюють проліферацію в осередку запалення?
A. @Фактори росту

315.У місці операційного розрізу сформувався рубець. Проліферація яких клітин у осередку запалення
супроводжується продукцією білків сполучної тканини?
A. @Фібробласти

1. При дослідженні запальної тканини виявлено підвищення інтенсивності анаболічних процесів. Для якої
стадії запалення найбільш характерна ця зміна?
A. @Проліферації

2. У пацієнта утворився келоїдний рубець. Яка стадія запалення була порушена ?


A. @Проліферація

3. У хворого на цукровий діабет трофічна виразка довго не загоюється. Яка стадія запалення порушена в
даному випадку? A. @Проліферація.

4. У дитини, 6 років, погіршення перебігу запалення верхніх дихальних шляхів зумовлене


гіперпродукцією прозапальних гормонів. Який з гормонів має прозапальну дію?
A. @Альдостерон.

5. У клінічній практиці глюкокортикоїди застосовуються як протизапальні засоби.


Який механізм їх протизапальної дії зменшує вторинну альтерацію? A. @Стабілізація
мембран.
6. У клінічній практиці широке застосування як протизапальні засоби отримали глюкокортикоїди. Який
механізм їх протизапальної дії зменшує проліферацію? @Пригнічення синтезу білка.

7. У клінічній практиці широке застосування як протизапальні засоби отримали глюкокортикоїди. Який


механізм їх протизапальної дії зменшує ексудацію? @Зменшення утворення ейкозаноїдів.

8. Пацієнт з бронхітом відкашлює багато мокроти. У нього в анамнезі – ендокринопатія. Який гормон має
прозапальну дію і посилює ексудацію?
@Альдостерон

9. Після місцевого застосування глюкокортикоїдів у хворого на екзему значно зменшились набряк,


почервоніння, біль. Яким чином глюкокортикоїди зменшують ексудацію?
@Пригнічують продукцію ейкозаноїдів

10. В досліді Конгейма спостерігали крайове стояння лейкоцитів. Що зумовлює початкову стадію еміграції
лейкоцитів?
@Адгезивні білки

1. Відомо, що первинні пірогени ініціюють утворення вторинних. Яким чином вторинні пірогени
спричинюють підвищення температури?
@Впливають на роботу центру терморегуляції

2. Жінка 46 років скаржиться на загальну слабкість, озноб, біль у горлі. Об'єктивно: почервоніння в зоні
мигдаликів. Температура тіла 38,60С. Які з перерахованих клітин є головним джерелом ендогенних
пірогенів, що викликають лихоманку у хворої?
@Макрофаги
3. У дитини із запаленням легенів спостерігається підвищення температури тіла. Яка з перерахованих
біологічно активних речовин відіграє провідну роль у виникненні цього явища? @Інтерлейкін - 1

4. Хворому на грип лікар у комплексній терапії не призначив жарознижувальних препаратів, виходячи із


захисної дії підвищеної температури. Який з механізмів такої дії підвищеної температури включається
при лихоманці?
A. @Активація продукції інтерферону

5. У хворої на гостру пневмонію з’явився озноб з підвищенням температури тіла до 39 °C, загальна
слабкість, сухий кашель. Який з перерахованих медіаторів запалення має властивості ендогенного
пірогену? A. @Інтерлейкін 1.

6. При гарячці первинні пірогени зумовлюють синтез вторинних. Вкажіть місце утворення останніх.
A. @Макрофаги

7. З метою отримання гарячки тварині ввели ендотоксин. Вкажіть, як зміниться чутливість нейронів
центру терморегуляції до холодових та теплових сигналів? A. @До теплових – знизиться, до холодових
– підвищиться

8. Експериментальній тварині з метою відтворення лихоманки було введено пірогенал. Який механізм при
цьому запускає процес підвищення температури тіла?
A. @Підвищення установчої точки нейронів терморегуляторного центру

9. Розвиток лихоманки у хворого супроводжувався зсувом установчої точки терморегуляторного центру


на більш високий рівень із послідовним чергуванням таких стадій:
A. @Incrementi, fastigii, decrementi

10. У хворого спостерігається збліднення шкірних покривів, «гусяча шкіра», тахіпное, тахікардія. Якій
стадії лихоманки відповідає даний стан?
A. @Incrementi

11. Після введення пірогеналу у хворого 36 років підвищилася температура тіла, шкірні покриви стали
блідими, холодними на дотик, з'явився озноб, збільшилося споживання кисню. Який механізм регуляції
температури в описаному періоді лихоманки є первинним?
A. @Знижується тепловіддача

12. В патогенезі гарячки розрізняють три стадії. Вкажіть, як змінюється терморегуляція в першій стадії
гарячки?
A. @Теплопродукція зростає, тепловіддача знижується
13. У хворого на грип відзначається блідість шкірних покривів, «гусяча шкіра», озноб. Яке співвідношення
процесів утворення й віддачі тепла в описаному періоді лихоманки?
A. @Теплопродукція перевищує тепловіддачу

14. У хворого на ГРВІ блідість шкіри змінилася гіперемією, виникло відчуття жару, шкіра на дотик стала
гарячою. Якій стадії лихоманки відповідає цей стан? A. @Fastigii

15. У хворого на крупозну пневмонію різко підвищилася температура до 39,5 оС та утримувалася такою
протягом 7 діб. Вкажіть, як змінюється терморегуляція у другій стадії гарячки?
A. @Підвищена теплопродукція відповідає збільшеній тепловіддачі

16. У хворого на ГРВІ блідість шкіри змінилася гіперемією, виникло відчуття жару, шкіра на дотик стала
гарячою. Яке співвідношення процесів утворення й віддачі тепла в описаному періоді лихоманки?
A. @Теплопродукція дорівнює тепловіддачі на більш високому рівні

17. При обстеженні хворого виявлені такі клінічні ознаки: шкірні покриви рожеві, теплі на дотик, сухі, ЧСС
– 92 у хвилину, ЧД – 22 за хвилину, температура тіла – 39,2 оС. Яке співвідношення процесів утворення
й віддачі тепла в описаному періоді лихоманки?
A. @Теплопродукція дорівнює тепловіддачі

18. У патогенезі гарячки розрізняють три стадії. Вкажіть, як змінюється терморегуляція у третій стадії
гарячки?
A. @Тепловіддача перевищує теплопродукцію

19. При обстеженні хворого виявлено такі об'єктивні дані: шкірні покриви гіперемовані, вологі на дотик,
відзначається поліурія, полідипсія, температура тіла - 37,2 0С. Якій стадії лихоманки відповідає цей
стан?
A. @Decrementi
20. У хворого з лихоманкою внаслідок лікування температура тіла почала швидко знижуватися. Яка
причина зниження температури при цьому?
A. @Зниження утворення пірогенів

21. Після різкого зниження температури хворий знепритомнів. Який механізм призвів до зниження
артеріального тиску і виникнення колапсу?
A. @Розширення судин

22. У хворого на ГРВІ з’явилось профузне потовиділення. Шкіра червона, гаряча, волога. Яке
співвідношення процесів утворення й віддачі тепла в описаному періоді лихоманки?
A. @Тепловіддача перевищує теплопродукцію
23. При обстеженні хворого виявлено такі об'єктивні дані: шкірні покриви гіперемовані, вологі на дотик,
відзначається поліурія, полідипсія, температура тіла - 37,2 0С. Якій стадії лихоманки відповідає цей
стан?
A. @Падіння температури

24. У хворого на туберкульоз протягом двох місяців ранкова температура тіла була в межах 36,4 – 36,9 0С,
до вечора вона підвищувалася до 37,3 – 37,9 0С. Яка лихоманка за рівнем температури відзначалася у
хворого?
A. @Субфебрильна

25. У хворого з гарячкою температура трималася на рівні 39,6 оС протягом 7 діб. Після цього критично
знизилася і була нормальною. Через 6 днів приступ гарячки повторився. Для якого захворювання
характерна така температурна крива? A. @Поворотний тиф

26. У хворого з гарячкою температура трималася на рівні 38,6-39,4 оС протягом 6 діб. Після цього
температура літично знизилась і хворий одужав. Для якого захворювання характерна така температурна
крива?
A. @Пневмонія

27. У хворого з гарячкою температура протягом доби декілька разів коливалася від 37,2 оС до 40,8 оС. Для
якого захворювання характерна така температурна крива? A. @Сепсис
28. У хворого з гарячкою температура різко піднялася до 40,1 оС, утримувалася на високому рівні протягом
2 годин, а потім критично знизилася до 35,9 оС. Через день приступ гарячки повторився. Для якого
захворювання характерна така температурна крива?
A. @Малярія

29. У хворого з гарячкою температура трималася на рівні 39,6 оС протягом 7 діб. Після цього критично
знизилася і була нормальною. Через 6 днів приступ гарячки повторився. Який тип температурної кривої
у хворого? A. @Febris recurrens.

30. У хворого після переохолодження температура тіла підвищилася до 39,7 °C і коливалася від 39 °C до
39,8 °C протягом 3 діб. Який тип температурної кривої у хворого?
A. @Febris continua.

31. У хворого з гарячкою температура протягом доби декілька разів коливалася від 37,1 оС до 41,1 оС. Який
тип температурної кривої у хворого? A. @Febris hectica.

32. У хворого протягом доби температура тіла підвищується і залишається високою від 1 до 3 годин, а
потім знижується до нормального рівня. Таке підвищення температури тіла спостерігається періодично
через 3 дні на четвертий. Який це тип температурної кривої? A. @Febris remittens.
1. Чоловік тривалий час працює в нафтопереробній промисловості. Який з наведених класів канцерогенів
зустрічається у його оточенні?
A. @Поліциклічні ароматичні вуглеводні

2. З анамнезу пацієнта з раком легень відомо, що протягом 20 років він палив до 30 цигарок на добу. До
якої групи належать канцерогени тютюнового диму? A. @Поліциклічні ароматичні вуглеводні

3. У працівника складу, де зберігали арахіс, виникла пухлина печінки. Яка речовина найімовірніше є
причиною виникнення пухлини у хворого?
A. @Афлатоксин

4. Хворий на рак печінки довгий час контактував з диметиламіноазобензолом (ДАБ). Чим пояснюється
виникнення пухлини саме в цьому органі?
A. @Органотропністю ДАБ

5. Епідеміологічне дослідження пухлин показало високу кореляцію розвитку пухлин легенів з палінням
тютюну. З дією якого канцерогену найбільш імовірно пов'язане виникнення цього виду патології?
A. @3,4-бензпірену

6. У експерименті пухлини часто індукують шляхом введення безклітинного фільтрату з пухлини. Хто
вперше успішно поставив такі досліди?
A. @Еллерман і Банг

7. За даними гінекологічного обстеження у жінки діагностовано рак шийки матки. Який вірус може бути
віднесеним до етіологічних факторів розвитку пухлини в даному випадку?
A. @Герпеса

8. У 1910 році Раус в експерименті викликав саркому в курей шляхом введення їм безклітинного
фільтрату з саркоми. Який метод експериментального моделювання використовував автор?
A. @Індукування

9. У 1908 році Елерман і Банг вперше довели, що віруси можуть викликати лейкоз у курей. Стосовно яких
онковірусів доведено, що вони здатні викликати пухлини у людини?
A. @Вірус Епстайна – Барра

10. Одну групу щурів тримали в умовах прямого сонячного опромінення, а іншу – в закритих камерах. У
тварин, які перебували у відкритих камерах, виникли пухлини непокритих шерстю частин шкіри. Вплив
якого фактора зумовив канцерогенез?
A. @Ультрафіолетового опромінення
11. У 1915 році Ішикава та Ямагіва в експерименті викликали рак шкіри втираючи в неї кам’яновугільну
смолу протягом 6 місяців. Який механізм канцерогенезу характерний для хімічних канцерогенів?
A. @Мутаційний

12. У 1910 році Раус в експерименті викликав саркому в курей шляхом введення їм безклітинного
фільтрату, отриманого з саркоми курки. Який механізм канцерогенезу вмикають лише онкогенні
віруси?
A. @Епігеномний

13. У постраждалого після опромінення виник гострий лейкоз. Який механізм трансформації характерний
для фізичних канцерогенів?
A. @Мутаційний

14. Дослідження каріотипу трансформованих клітин хворого на хронічний мієлолейкоз показало наявність
філадельфійської хромосоми. Яка хромосомна аберація спричинює активацію протоонкогену і
лейкозогенез у цьому випадку?
A. @Транслокація

15. Експериментальне вивчення геному пухлинних клітин ретинобластоми показало, що у механізмах


канцерогенезу може мати значення:
A. @Делеція генів інгібіторів клітинного поділу (генів супресорів)

16. Спостереження за ростом пухлинних клітин в культурі тканин виявило необмеженість їх поділу. Які
зміни в пухлинних клітинах зумовлюють відсутність ліміту Хейфліка?
A. @Дерепресія гену теломерази
17. Аналіз експериментальних даних по вивченню онкобілків показав, що їх функція полягає в тому, що
вони
A. @Є стимуляторами мітозу

18. Шкіру тварини змащували мікрокількостями метилхолантрену. Пізніше ту саму ділянку шкіри
змащували кротоновим маслом, і це призвело до виникнення пухлини. З якою стадією канцерогенезу
пов’язана дія кротонового масла?
A. @Промоція

19. У досліді Беренблюма – Моттрама доведено, що кротонове масло, яке саме не є канцерогеном, може
бути промотором і сприяє розвитку пухлин. Який ефект на клітину справляє промотор?
A. @Стимулює до поділу

20. У експерименті виявлено, що в пухлинних клітинах значно активується синтез певних груп білків. Що
таке онкобілки?
A. @Продукти онкогенів
21. Після третього курсу хіміотерапії у хворого на злоякісну пухлину легені відсутні будь-які позитивні
зміни. Яка властивість пухлин зумовлює розвиток їх резистентності до хіміотерапії?
A. @Прогресія

22. Пацієнта турбує те, що шкіра навкруги пігментного невуса почервоніла. Дослідження показало
інвазивний ріст пухлини у хворого. Що можна зазначити як механізм пухлинної прогресії?
A. @Додаткові мутації в трансформованих клітинах

23. Після обстеження хворому поставлено діагноз „Рак печінки”. Який білок в крові може свідчити про
наявність злоякісної пухлини у хворого?
A. @Альфа- фетопротеїн

24. Гістологічне дослідження тканини раку шийки матки показало високий ступінь злоякісності пухлини у
хворої. Що з нижчезазначеного є показником морфологічної анаплазії пухлини?
A. @Високе співвідношення ядро/цитоплазма
25. У хворого виявлено недиференційований рак шлунка з вираженими ознаками анаплазії. Що таке
анаплазія пухлинної тканини?
A. @Дедиференціювання пухлинних клітин

26. На саркомі Ієнсена досліджено, що споживання глюкози з артерії, що веде до пухлини, значно
збільшується, також наявний приріст вмісту молочної кислоти у відвідній вені. Про що свідчить це
явище?
A. @Посилення гліколізу

384.У хворого виявлено низькодиференційований рак шкіри. Вкажіть прояв анаплазії пухлин.
A. @Антигенне спрощення

1. Клітини гепатоми не здатні до синтезу жовчних кислот. Як може бути названа така властивість
пухлинних клітин?
A. @Функціональна анаплазія

2. Біохімічний аналіз пухлини, вилученої у хворого, виявив інтенсивний гліколіз навіть в умовах
достатнього надходження кисню до клітини. Така особливість обміну речовин пухлинної клітини є
проявом:
A. @Біохімічної анаплазії

3. У хворого на рак легень виявлено гіперпродукцію пухлиною адренокортикотропного гормону. Яка


властивість пухлини зумовлює такий розвиток подій?
A. @Метаплазія
4. Під час видалення пухлини м’якої тканини встановлено, що скупчення пухлинних клітин
спостерігаються не тільки в осередку, але й серед оточуючих тканин, вздовж судин та нервів. Який тип
росту пухлини виявлено в цьому випадку?
A. @Інфільтративний

5. Хворому встановлено діагноз: злоякісна пухлина легенів. Яка характеристика пухлинного росту
найбільш імовірно свідчить про його злоякісність?
A. @Інфільтративний
B.
1. При обстеженні жінки 50 років, що тривалий час перебувала на рослинній дієті, виявлено гіпотермію,
гіпотонію, м'язову слабкість, негативний азотистий баланс. Який фактор найімовірніше викликав такий
стан пацієнтки?
A. @Недостатня кількість білків у раціоні

2. При обстеженні чоловіка 45 років, що перебував тривалий час на рослинній дієті, виявлений негативний
азотистий баланс. Яка особливість раціону стала причиною цього явища?
A. @Недостатня кількість білків

3. У людини внаслідок тривалого голодування швидкість клубочкової фільтрації зросла на 20%. Що є


причиною зміни фільтрації в зазначених умовах?
A. @Зменшення онкотичного тиску плазми крові

4. У тварини, яка тривалий час знаходилася переважно на вуглеводній їжі, виникли набряки. Що стало їх
причиною?
A. @Гіпопротеїнемія

5. У хворого зі стенозом пілоричного відділу шлунка, який утруднює прийом їжі, спостерігається
схуднення, м'язова слабкість, набряки нижніх кінцівок. Який вид голодування у хворого?
A. @Неповне (недоїдання)

395.У дитини, хворої на квашиоркор, спостерігаються порушення ороговіння і злущування епідермісу,


набряки. Що є найбільш важливим механізмом виникнення цих порушень?
A. @Білкове голодування

1. У дитини, яка тривалий час голодувала, спостерігаються порушення росту, відсталість розумового
розвитку, тяжке виснаження, депігментація шкіри і волосся, набряки. Як називається ця форма білково-
калорійної недостатності?
A. @Аліментарний маразм

2. У чоловіка, яка тривалий час голодував, спостерігаються тяжке виснаження, депігментація шкіри і
волосся, набряки. Як називається ця форма білково-калорійної недостатності?
A. @Аліментарна дистрофія
3. В експерименті в умовах абсолютного голодування водна недостатність частково компенсується
утворенням ендогенної води. Яке з наведених речовин при окисненні дає найбільшу кількість
оксидаційної води?
A. @Жири

4. Пацієнт 28 років голодує 48 годин. Які речовини використовуються м'язовими тканинами як джерело
енергії в цьому випадку?
A. @Глюкоза
400.У голодуючої тварини в першу годину повного голодування енергетичні потреби забезпечуються
вуглеводами. Якому клінічному періоду голодування відповідають ці зміни?
A. @Байдужості

1. У щура в експерименті при повному голодуванні з водою спостерігаються незначне підвищення


основного обміну, дихальний коефіцієнт рівний 1. Тварина активно шукає їжу. Якому періоду
голодування відповідають ці зміни?
A. @Збудження

2. У чоловіка 45 років на 4 добу від початку голодування зберігається відчуття голоду, відзначається
загальна слабість, пригнічення психіки. Глюкоза крові 2,8 ммоль/л, дихальний коефіцієнт 1,0. В якому
періоді голодування за патофізіологічною характеристикою перебуває пацієнт?
A. @Неощадливої витрати енергії

3. У тварини під час повного голодування основний обмін на 10% нижчий, ніж в нормі. Дихальний
коефіцієнт дорівнює 1,0. Азотистий баланс негативний. У якому періоді голодування за
патофізіологічною класифікацією перебуває тварина?
A. @Неекономної трати енергії

4. У щура при повному голодуванні з водою спостерігаються незначне підвищення основного обміну,
дихальний коефіцієнт 1,0. Тварина активно шукає їжу. Які зміни обміну речовин характерні для цього
періоду голодування?
A. @Активація глікогенолізу

405.Після тривалого голодування в крові людини відзначається підвищення вмісту кетонових тіл. Що
зумовлює це явище?
A. @Надлишок ацетилКоА

1. При голодуванні (у другому періоді) у хворих може підвищуватися кількість ліпідів крові. Яка форма
гіперліпемії наявна в цьому випадку?
A. @Траспортна

2. В експериментальної тварини, якій відтворено повне харчове голодування, основний обмін знижений,
дихальний коефіцієнт дорівнює 0,7. У крові відзначається кетонемія, у сечі - кетонурія. Який клінічний
період голодування характеризують такі зміни?
A. @Пригнічення

3. Хворий на злоякісну пухлину стравоходу протягом тижня не приймав їжу. Яким чином змінився
гормональний статус у цього хворого?
A. @Підвищилася секреція глюкагону
4. У щурів під час повного голодування дихальний коефіцієнт становить 0,7. Азотистий баланс
негативний. У крові - гіперліпацидемія. У якому періоді голодування за патофізіологічною
характеристикою перебувають тварини?
A. @Максимального пристосування

410.Під час голодування в організмі виникає перехід на ендогенне харчування. Що з перерахованого


найімовірніше варто очікувати на 8 день повного голодування дорослої людини?
A. @Інтенсифікація кетогенезу в печінці

1. У щура в експерименті на 2-у добу повного голодування з водою дихальний коефіцієнт 0,7, основний
обмін знижений на 15%. Тварина пасивна. Які зміни обміну речовин характерні для цього періоду
голодування?
A. @Активація β-окислення жирних кислот

2. У щура в експерименті на 6-у добу повного голодування з водою спостерігаються незначне підвищення
основного обміну, дихальний коефіцієнт рівний 0,8. Тварина вкрай виснажена, практично не рухається.
Які зміни обміну речовин характерні для цього періоду голодування?
A. @Активація протеолізу
3. У щурів при повному голодуванні дихальний коефіцієнт становить 0,8. У сечі багато азоту, калію,
фосфору. Тварини виснажені, різко загальмовані. Яка речовина використовується як джерело енергії у
цьому періоді голодування?
A. @Білки

4. Внаслідок тривалого голодування у людини з’явились ознаки інтоксикації, набряки по всьому тілу,
збільшилось виділення із сечею азоту та калію. Посилення якого процесу зумовило ці явища?
A. @Катаболізму білка

1. У хворої Т. 42 років під час нападу бронхіальної астми при вивченні газового складу крові виявлено
наявність гіперкапнії, гіпоксемії та газового ацидозу. Який вид гіпоксії спостерігається в цьому
випадку?
A. @Дихальна

2. Після аварії на хімічному виробництві внаслідок отруєння нітросполуками до лікарні вступили 18


працівників підприємства. Потерпілі скаржилися на різку слабість, головний біль, задишку,
запаморочення. У чому полягає причина розвитку гіпоксії?
A. @Утворенні метгемоглобіну

3. Під час пожежі із закритого приміщення, повного диму, винесли в непритомному стані чоловіка
приблизно 50 років. Які зміни газового складу крові ймовірні в цьому випадку?
A. @Зменшення кисневої ємності крові
4. До лікарні звернувся хворий 28 років зі скаргами на слабість, головний біль, зниження працездатності,
задишку під час ходьби. При обстеженні газового складу крові виявлено збільшення артеріовенозної
різниці за киснем. Яка з перерахованих причин можливо призвела до розвитку гіпоксії в цьому
випадку?
A. @Серцева недостатність

5. Група відпочиваючих у кардіологічному санаторії пішла на екскурсію в гори на висоту 1200 м над
рівнем моря. Через 2 години від початку походу в одного з екскурсантів з'явилися тахікардія, задишка.
Який з перерахованих видів гіпоксії призвів до цих порушень?
A. @Циркуляторна
6. Після марафонського бігу спортсмен почуває себе дуже стомленим. У нього діагностовано гіпоксію
напруження. Які зміни показників крові в цих умовах наявні?
A. @Підвищення вмісту молочної кислоти

7. Судово-медичний експерт при розтині трупа 18-річної дівчини встановив, що смерть настала внаслідок
отруєння ціаністим калієм. Який фермент найбільшою мірою гальмується ціанідами?
A. @Цитохромоксидаза

8. До лікарні машиною швидкої допомоги доставлено з вулиці чоловіка 60 років у непритомному стані.
Шкірні покриви бліді. Пульс і дихання прискорені. Кількість еритроцитів 2,52 Г/л, вміст гемоглобіну –
72 г/л, результати газового аналізу: рО2 – 75 мм рт.ст., киснева емність крові – 10 об.%. Який вид
гіпоксії наявний? A. @Гемічна

9. Робітниця виробництва анілінових барвників доставлена в медичний пункт із ознаками отруєння


аніліном. Обличчя синюшне, шкірні покриви і слизові оболонки ціанотичні. Спостерігається задишка.
Пульс - 100 уд/хв. Яка причина розвитку гіпоксії найбільш імовірна в цьому випадку?
A. @Утворення метгемоглобіну

10. Чоловік 37 років скаржиться на задишку, кашель із виділенням мокротиння, загальну слабість,
підвищення температури до 390С. Занедужав після переохолодження. АТ - 110/60 мм рт.ст. Пульс - 98
уд/хв. У процесі обстеження виявлено крупозну пневмонію. Який вид гіпоксії наявний у цьому
випадку?
A. @Дихальна

11. При дослідженні впливу гострої гіпоксичної гіпоксії на білих щурів виявлено такі прояви захисно-
пристосувальних реакцій: тахіпное з гіпокапнією, тахікардія, збільшення кількості еритроцитів і
гемоглобіну. Яким зрушенням кривої дисоціації оксигемоглобіну супроводжуються ці явища?
@Вліво
12. Чоловік 70 років страждає на хронічну серцеву недостатність. При обстеженні виявлено ціаноз,
задишку, періодичні напади ядухи. Який вид гіпоксії , обумовлений даною патологією міг виникнути у
хворого?
@Циркуляторна застійна
13. У хворого після перенесеного грипу виникло запалення серцевого м'яза - міокардит. Об'єктивно:
задишка, тахікардія, ціаноз, які свідчать про розвиток гіпоксії. Визначте тип гіпоксії :
@Циркуляторна

14. У члена високогірної експедиції на висоті 6 км виникло запаморочення, слабість. Альпініст


знепритомнів, дихання зупинилося. Ці порушення виникли внаслідок:
@Надмірного виведення СО2 з організму

429.Потерпілого доставили в лікарню з гаража, де він перебував у непритомному стані при працюючому
моторі автомобіля. Попередній діагноз - отруєння чадним газом. Розвиток гіпоксії в потерпілого пов'язаний з
накопиченням у крові:
@Карбоксигемоглобіну

1. У пацієнта з гострим інфарктом міокарда з'явилися ознаки гіпоксії - тахікардія, задишка, ціаноз,
Вкажіть, який вид гіпоксії у хворого?
@Циркуляторна

2. При підйомі на «висоту» у барокамері в щура з'явилися часте дихання, тахікардія, зниження напруги
рО2 у крові. Яка форма гіпоксії наявна в цьому випадку?
@Гіпоксична

3. До відділення реанімації доставлено хворого, у крові якого виявлено високий вміст сульфгемоглобіну.
Який тип гіпоксії наявний у цьому випадку?
@Гемічний

4. Лікар-дослідник у складі альпіністської експедиції піднявся на висоту 5 км. На 3 день перебування в


нього з'явилися ознаки гірської хвороби: задишка, головний біль, втрата апетиту, загальна слабість,
ціаноз. Який тип гіпоксії наявний у цьому випадку?
@Гіпоксичний

5. До клініки доставлено потерпілого у результаті отруєння окисом вуглецю. Яка форма гіпоксії тут
наявна?
@Гемічна
6. При проведенні рятувальних робіт із шахти було винесено шахтаря у непритомному стані без зовнішніх
ознак ушкодження. У повітрі шахти містився у великій кількості метан. Який вид гіпоксії розвинувся в
потерпілих?
@Гіпоксична
7. При клінічному обстеженні жителів гірського селища, розташованого на висоті 3000 метрів, виявлено
підвищену кількість еритроцитів у крові. Причиною цього є:
@Підвищення утворення еритропоетинів

8. Людина постійно живе високо в горах. Яку зміну показників крові може бути виявлено в неї?
@Збільшення кількості еритроцитів

9. Водій після роботи заснув у гаражі у машині із працюючим двигуном. Прокинувшись, він відчув
головний біль, почалося блювання. Утворення якої сполуки в крові стало причиною цього стану?
@Карбоксигемоглобіну

10. Пацієнт звернувся зі скаргами на напади утрудненого дихання, запаморочення. З'ясувалося, що він
працює на хімічному підприємстві з виробництва синильної кислоти. З порушенням функції якого
ферменту можуть бути пов'язані ці симптоми?
@Цитохромоксидази

11. В експерименті на кролі через 2 тижні після звуження ниркової артерії виявлено збільшення кількості
еритроцитів і гемоглобіну в крові внаслідок стимуляції еритропоезу еритропоетинами. Що підсилює
утворення еритропоетинів? A. @Гіпоксія

12. У приміщенні відзначається підвищений вміст вуглекислого газу. Як зміниться дихання (глибина і
частота) у людини, що увійшла до цього приміщення?
@Збільшиться глибина й частота

13. Хвора 13 років перебуває на стаціонарному лікуванні в гематологічному відділенні обласної дитячої
лікарні з діагнозом залізодефіцитна анемія. Який тип гіпоксії є в цієї хворої?
@Гемічна
14. Велика травма стегна (з руйнуванням м'яких тканин, нервових стовбурів, судин), що супроводжується
масивною кровотечею, зумовила розвиток травматичного шоку. Який вид гіпоксії виникає в цьому
випадку?
@Змішана

15. Декількох працівників виробництва з виготовлення вибухівки доставили в лікарню із симптомами


отруєння. Спектральний аналіз крові показав, що це пов'язане з дією бертолетової солі. Який тип
гіпоксії розвивається при цьому?
@Гемічна

16. Чоловік, якого винесли під час пожежі із заповненої димом кімнати, перебуває в непритомному стані.
Який вид гіпоксії виник у постраждалого? A. @Гемічна.

17. Хворий на крупозну пневмонію скаржиться на задишку, кашель із виділенням мокротиння, загальну
слабість, гарячку. Який вид гіпоксії наявний у цьому випадку?
@Дихальна

18. У хворого внаслідок передозування наркотичної речовини (уретану) рО2 у венозній крові 80 мм рт.ст., а
артеріовенозна різниця за киснем зменшилась. Який вид гіпоксії наявний в цьому випадку?
@Тканинна

19. У хворого 60 років шкірні покриви бліді, пульс і дихання прискорені. Кількість еритроцитів 2,52 Г/л,
вміст гемоглобіну – 72 г/л. Який вид гіпоксії наявний? A. @Гемічна

20. При гострій дихальній гіпоксії у щурів виявлені гіпоксемія, гіперкапнія, ацидоз, тахіпное, тахікардія,
збільшення кількості еритроцитів і гемоглобіну. Яке зрушення кривої дисоціації оксигемоглобіну буде
у щурів? @Вправо

21. У хворого з хронічною правошлуночковою серцевою недостатністю виявлені ознаки циркуляторної


гіпоксії. Які зміни газового складу крові найбільш ймовірні в цьому випадку? @Збільшення
артеріовенозної різниці за киснем

22. У хворого зі скаргами на задишку, втомлюваність у крові знижені кількість еритроцитів, гемоглобіну,
колірний показник,у мазку мікроанізоцитоз, пойкілоцитоз. Що є головним критерієм розвитку гемічної
гіпоксії? A. @Зниження кисневої ємності крові.

23. В альпініста на висоті 4000 м над рівнем моря виникла ейфорія, неадекватна оцінка обставин,
галюцинації. Яка причина розвитку цих ознак гірської хвороби? A. @Зниження парціального тиску
кисню в повітрі.

24. У лікарню доставлено непритомного чоловіка після ремонту автомобіля в гаражному приміщенні.
Утворення якої сполуки має провідне значення в патогенезі гемічної гіпоксії в цього пацієнта?
@Карбоксигемоглобіну

25. Після аварії в цеху при роботі з нітросполуками до лікарні потрапили 18 робітників зі скаргами на різку
слабість, головний біль, задишку, запаморочення. У чому полягає причина розвитку гіпоксії?
@Утворенні метгемоглобіну

1. У тварин, які були адаптовані до холоду, відбувається порушення спряження процесів окислення
субстратів і синтезу АТФ. Це явище називається:
@Роз'єднання окислення і фосфорилювання

2. У хворого зі скаргами на дратівливість, тахікардію, тремтіння пальців рук, екзофтальм виявлене


збільшення основного обміну на 20%. Яке ендокринне порушення зумовило ці зміни?
@Гіпертиреоз

3. У хворого з ожирінням недостатньо розвинені вторинні статеві ознаки. Виявлене зменшення основного
обміну на 10%. Яке ендокринне порушення зумовило ці зміни?
@Гіпогонадизм

4. У хворого з хронічною серцевою недостатністю, зумовленою ревматичною вадою серця, основний


обмін підвищений на 30%. Який чинник викликав цю зміну?
@Посилення роботи серця і органів дихання

5. В експерименті тварині ввели динітрофенол. В результаті значно зросло виділення тепла організмом.
Вкажіть причину цього порушення.
@Роз'єднання окислення і фосфорилювання

460. У хворого зі скаргами на зниження пам’яті, сонливість, зниження температури тіла, ожиріння, набряки
виявлене зменшення основного обміну на 30%. Яке ендокринне порушення зумовило ці зміни?
@Гіпотиреоз

1. У хворих на цукровий діабет виникають значні порушення енергетичного обміну. Вкажіть первинну
зміну, що зумовлює ці порушення.
@Зменшення потужності циклу Кребса

2. У хворих з бері-бері (гіповітамінозом В1) спостерігаються судоми і психози, зумовлені порушенням


енергозабезпечення нейронів головного мозку. Який процес порушується в першу чергу в цьому
випадку?
@Цикл Кребса

3. У хворих з пелагрою (гіповітамінозом РР) один з головних механізмів захворювання пов'язаний з


порушенням енергетичного обміну. Робота яких ферментів порушується в першу чергу в цьому
випадку?
@Дихального ланцюга

4. У хворих з арибофлавінозом (гіповітамінозом В2) один з головних механізмів захворювання пов'язаний


з порушенням енергетичного обміну. Робота яких ферментів порушується в першу чергу в цьому
випадку?
@Дихального ланцюга

465. Дитина, хвора на цукровий діабет І типу, натще ввела собі інсулін. Через 15 хвилин у неї з’явилися дріж,
інтенсивне потовиділення, запаморочення. Що в даному випадку зумовило розвиток гіпоглікемії в дитини?
@Збільшення споживання глюкози тканинами

1. Дитина, хвора на цукровий діабет І типу, натще ввела собі інсулін. Через 15 хвилин у неї з’явилися
дріж, інтенсивне потовиділення, запаморочення. Яка зміна гомеостазу викликала ці зміни в організмі
дитини?
@Гіпоглікемія
2. У хворого на цукровий діабет після випадкового передозування інсуліну виникли гостре відчуття
голоду, тахікардія, підвищення АТ, блідість шкіри, сильна пітливість. Чим зумовлені прояви з боку
системи кровообігу?
@Підвищенням активності симпатичної нервової системи

3. У хлопчика, хворого на цукровий діабет, що ввів чергову дозу інсуліну, але не поїв, виникли гостре
почуття голоду, дратівливість, агресивна поведінка, згодом запаморочення, судоми. Чим зумовлені
зміни з боку центральної нервової системи?
@Зменшенням рівня глюкози в крові

4. У хворого на цукровий діабет після інтенсивного м'язового навантаження на тлі звичайних доз інсуліну
виникли гостре відчуття голоду, тахікардія, підвищення АТ, блідість шкіри, сильна пітливість. Нижче
якого рівня глюкози в крові виникає гіпоглікемічна кома?
@2,5 ммоль/л

5. У хворого з недостатністю кори надниркових залоз (хворобою Аддісона) спостерігається виражена


схильність до гіпоглікемії. Чим це зумовлено?
@Дефіцитом контрінсулярних гормонів

6. У хворого з недостатністю аденогіпофіза (пангіпопітуітаризмом) спостерігається виражена схильність


до гіпоглікемії. Чим це зумовлено?
@Дефіцитом контрінсулярних гормонів

7. У хворого з аденомою пучкової зони кори надниркових залоз (синдромом Іценка – Кушинга)
спостерігається виражена схильність до гіперглікемії. Чим це зумовлено?
@Надлишком контрінсулярних гормонів

8. У хворого з еозинофільною аденомою гіпофіза (акромегалією) спостерігається виражена схильність до


гіперглікемії. Чим це зумовлено?
@Надлишком контрінсулярних гормонів

9. У хворого з пухлиною з β-клітин острівців Лангерганса спостерігається виражена схильність до


гіпоглікемії. Чим це зумовлено?
@Надлишком інсуліну

475.У другий період повного голодування з водою в організмі порушується обмін речовин, у крові
збільшується кількість ліпідів, з’являються кетонові тіла, але вміст глюкози зберігається на рівні нижньої
межі норми. Що забезпечує еуглікемію? A. @Глюконеогенез

1. Хворому на цукровий діабет І типу введено інсулін. Через деякий час у хворого з’явилися загальна
слабкість, гостре почуття голоду, дратівливість, запаморочення. Який основний механізм розвитку
проявів гіпоглікемії у пацієнта? @Вуглеводне голодування головного мозку.
2. У дитини з затримкою фізичного та психічного розвитку, схильністю до гіпоглікемії, гіперліпідемією,
лактацидемією, гепатомегалією та остеопорозом діагностовано хворобу Гірке (глікогеноз). Що
спричинило гіпоглікемію у хворого? @Зниження глікогенолізу

3. У хворого з аденомою пучкової зони кори надниркових залоз (синдромом Іценка – Кушинга)
гіперглікемія зумовлена:
@Активацією глюконеогенезу в печінці

4. Відразу після больового нападу у хворого рівень глюкози у крові склав 7,5 ммоль/л. Чим можна
пояснити гіперглікемію?
@Підвищенням активності симпатичної нервової системи

5. У хворого з феохромоцитомою (пухлиною мозкової речовини наднирників) спостерігається підвищення


рівня глюкози в крові. Гіперглікемія зумовлена:
@Активацією глікогенолізу в печінці

6. У хлопчика 7 років через 2 години після вживання глюкози її вміст у крові складає 20 ммоль/л.
Виявлено цукровий діабет 1 типу (інсулінозалежний). Що є причиною гіперглікемії при цьому
захворюванні?
@Абсолютний дефіцит інсуліну

7. У чоловіка 47 років знижена толерантність до вуглеводів. Виявлено цукровий діабет 2 типу. Який
механізм веде до гіперглікемії при цьому захворюванні?
@Зниження споживання глюкози тканинами
8. У хворого на цукровий діабет рівень глюкози в крові складає 32 ммоль/л. Яке з перерахованих
порушень обміну речовин є безпосереднім наслідком гіперглікемії? @Гіперосмія крові

9. У хворого на цукровий діабет рівень глюкози в крові складає 15 ммоль/л. Яке з перерахованих
порушень обміну речовин є безпосереднім наслідком гіперглікемії? @Глюкозурія

10. У хворого на цукровий діабет виявлено глюкозурію. При якому мінімальному (пороговому) рівні
глюкози в крові виникає глюкозурія?
@8,88 ммоль/л

11. У хворого на цукровий діабет виявлено глюкозурію. Яке порушення обміну речовин є безпосереднім
наслідком глюкозурії?
@Поліурія

12. У хворого на цукровий діабет виявлено глюкозурію і поліурію. Яке порушення обміну речовин є
безпосереднім наслідком поліурії?
@Зневоднення
13. У хворого на цукровий діабет рівень глюкози в крові складає 15 ммоль/л. Яке з перерахованих
порушень обміну речовин є безпосереднім наслідком гіперглікемії? @Глікерування білків

14. У хворого на цукровий діабет 1-го типу спостерігаються виражені порушення жирового обміну. Яке
порушення жирового обміну характерне для цього захворювання?
@Кетонемія

15. У хворого на цукровий діабет 2-го типу спостерігаються виражені порушення жирового обміну. Яке
порушення жирового обміну характерне для цього захворювання?
@Ожиріння

491.У хворого на цукровий діабет 2-го типу спостерігаються виражені порушення жирового обміну. Яке
порушення жирового обміну характерне для цього захворювання? @Гіперхолестеринемія
1. У хворого на цукровий діабет 2-го типу спостерігаються виражені порушення жирового обміну. Яке
порушення жирового обміну характерне для цього захворювання? @Гіперліпопротеїнемія

2. У хлопчика 7 років, що значно схуд за останній місяць, виявлено цукровий діабет 1 типу. Що є
причиною схуднення при цьому захворюванні?
@Абсолютний дефіцит інсуліну

3. У хворого на цукровий діабет знижені процеси регенерації, довго не загоюються рани. Які зміни в
обміні речовин це спричинюють? @Пригнічення синтезу білків.

4. У хворого на цукровий діабет 1-го типу спостерігаються виражені порушення білкового обміну. Яке з
перерахованих порушень білкового обміну характерне для цього захворювання? @Негативний
азотистий баланс

496.У хворого на цукровий діабет 1-го типу часто спостерігаються порушення водноелектролітного обміну.
Яке порушення водно-електролітного обміну характерне для цього захворювання?
@Гіпогідрія

1. У хворого на цукровий діабет 1-го типу часто спостерігається зневоднення. Що зумовлює це


порушення?
@Поліурія

2. Хворий 68 років доставлений у стаціонар непритомним, дихання часте, глибоке, ЧСС – 110 уд/хв., АТ
90/45 мм рт. ст., шкіра в'яла, суха. Лабораторно: глюкоза крові – 15 ммоль/л, молочна кислота крові – 12
ммоль/л. Що є найбільш вірогідною причиною лактацидемії у хворого?
@Гіпоксія

3. У хворого на цукровий діабет може виникати гіперлактацидемія. Що зумовлює це порушення?


@Глікогеноліз у м’язах
4. У хворого на цукровий діабет може виникати гіпоксія. Що зумовлює це порушення?
@Зневоднення
5. У експерименті тварині вводили β-цитотропні віруси. Який патологічний стан моделювали в даному
експерименті? @Абсолютну інсулінову недостатність

6. У експерименті тварині вводили антитіла до інсулінових рецепторів. Який патологічний стан


моделювали в даному експерименті?
@Відносну інсулінову недостатність

7. У експерименті тварині вводили великі дози кортизолу. Який патологічний стан моделювали у даному
експерименті?
@Стероїдний цукровий діабет

8. У хворого М. виявлений цукровий діабет 2-го типу. Яка з перерахованих ознак характерна для даного
захворювання?
@Інсулінорезистентність тканин

9. У дівчини 14 років, яка скаржиться на значне схуднення, спрагу, поліурію діагностовано цукровий
діабет 1-го типу. Який патогенетичний механізм є провідним у виникненні цього захворювання?
@Аутоімунне ушкодження β-клітин

10. У хворого на аденому гіпофіза (хворобу Іценка – Кушинга) спостерігаються гіперглікемія, глюкозурія,
поліурія, поліфагія, полідипсія. Який вид діабету спостерігається у хворого?
@Вторинний цукровий діабет

11. У чоловіка 64 років, який скаржиться на спрагу, поліурію, ожиріння діагностовано цукровий діабет 2-го
типу. Який патогенетичний механізм є провідним у виникненні цього захворювання?
@Інсулінорезистентність тканин

12. У хворого М. виявлено цукровий діабет 1-го типу. Яка з перерахованих ознак характерна для даного
захворювання?
@Абсолютна інсулінова недостатність

13. У дівчини 14 років, яка скаржиться на значне схуднення, спрагу, поліурію виявлено антитіла проти
острівцевих клітин. Який варіант цукрового діабету у хворої? @1-й тип
14. У вагітної жінки 34 років, яка скаржиться на спрагу, поліурію, вперше виявлено глюкозурію та
зниження толерантності до глюкози. Який варіант цукрового діабету у хворої?
@4-й тип

15. У хворого на аденому кори наднирника (синдром Іценка – Кушинга) спостерігаються гіперглікемія,
глюкозурія, поліурія, поліфагія, полідипсія. Який вид діабету спостерігається у хворого?
@3-й тип
512.У хлопчика з еозинофільною аденомою гіпофіза на фоні значного збільшення швидкості росту із
збереженням нормальних пропорцій тіла спостерігається виражена схильність до гіперглікемії. Який варіант
цукрового діабету у хворого? A. @3-й тип

1. Жінки 34 років перенесла гострий панкреатит, після чого в неї виникли спрага та поліурія. При
обстеженні виявленї глюкозурія та зниження толерантності до глюкози. Який варіант цукрового діабету
у хворої?
@3-й тип

2. У чоловіка 64 років, який скаржиться на спрагу, поліурію, ожиріння діагностовано цукровий діабет з
інсулінорезистентністю тканин. Який варіант цукрового діабету у хворого? @2-й тип

3. Після перенесеного епідемічного паротиту дитина почала худнути, постійно відчуває спрагу; наявні
поліурія, глюкозурія. Вміст глюкози в крові становить 16 ммоль/л, кетонових тіл - 400 мкмоль/л. Що
спричинює цукровий діабет у дитини?
@Абсолютний дефіцит інсуліну

4. У хворого виявили глюкозурію на фоні нормальних рівнів глюкози в крові. Який варіант цукрового
діабету у хворого?
@Нирковий

517.У хворого на цукровий діабет розвинулась гіперглікемічна кома (рівень глюкози 54 ммоль/л). Яке
патологічне явище лежить в основі порушення функцій клітин мозку в даному випадку? @Гіпогідратація
клітин мозку

1. У хворого на цукровий діабет 1 типу виникла кома. Який характер коми є найімовірнішим у даному
випадку?
@Кетонемічна

2. Хворий 35 років доставлений у стаціонар непритомним, дихання часте, глибоке (типу Куссмауля), АТ
90/45 мм рт. ст., з рота запах ацетону. Глюкоза крові – 18 ммоль/л, у сечі – глюкозурія, кетонурія. Яка
кома виникла у даного хворого?
@Кетоацидотична
3. Хворий 55 років доставлений у стаціонар непритомним, дихання часте, поверхневе, АТ 90/45 мм рт. ст.,
шкіра в'яла, суха. Лабораторно: глюкоза крові – 55 ммоль/л, Na – 150 ммоль/л, Cl – 15 ммоль/л Яка кома
виникла у даного хворого? A. @Гіперосмолярна

4. Чоловік С., 45 років, хворий на цукровий діабет 2 типу, має вдруге інфаркт міокарда. Яке ускладнення
діабету прискорює розвиток ішемічної хвороби серця в хворого? @Макроангіопатія
522.У хворого на цукровий діабет 1 типу під час кетонемічної коми розвинулося патологічне гіперпное. Що
спричинило розвиток дихання Куссмауля у хворого?
@Ацидоз метаболічний

1. Жінка, хвора на цукровий діабет 1 типу, скаржиться на погіршення зору. Обстеження показало в неї
діабетичну ретинопатію. Яке ускладнення діабету може бути причиною втрати зору?
@Мікроангіопатія

2. Хворий 68 років доставлений у стаціонар непритомним, дихання часте, глибоке, ЧСС – 110 уд/хв., АТ
90/45 мм рт. ст., шкіра в'яла, суха. Лабораторно: глюкоза крові – 15 ммоль/л, молочна кислота крові – 12
ммоль/л. Яка кома виникла у даного хворого?
@Лактацидотична

3. Дівчина, хвора на цукровий діабет, чекає на донорську нирку. Яке ускладнення діабету є причиною
хронічної ниркової недостатності?
@Мікроангіопатія

4. У чоловіка 55 років, хворого на цукровий діабет 2-го типу, після незначної травми ступні виникла
гангрена ноги. Яке ускладнення діабету є головною причиною гангрени? @Макроангіопатія
527.У чоловіка 55 років, хворого на цукровий діабет 2-го типу, після незначної травми ступні виникла
гангрена ноги. Діагностовано атеросклероз стегнових артерій. Який механізм прискорює розвиток
атеросклерозу при цукровому діабеті? A. @Глікерування апопротеїнів

1. До лікарні доставили хворого на цукровий діабет у стані непритомності. Дихання типу Куссмауля,
артеріальний тиск 80/50 мм рт. ст., запах ацетону з рота. Накопиченням в організмі яких речовин можна
пояснити виникнення цих розладів? @Кетонових тіл.

2. У дитини з затримкою розвитку, розумовою відсталістю, катарактою та цирозом печінки виявлено


галактозурію. Активність якого фермента знижена у хворого?
@Галактозо-1-фосфат-уридилтрансферази

3. У дитини з ураженням печінки та нирок після вживання фруктів діагностовано фруктозурію та


гіпоглікемією. Активність якого ферменту знижена у хворого?
@Фруктозодифосфат-альдолази

4. У дитини з затримкою фізичного та психічного розвитку, гіпоглікемією, гіперліпідемією,


лактацидемією, гепатомегалією та остеопорозом діагностовано хворобу Гірке. Яка речовина
накопичується у тканинах хворого?
@Глікоген

532.У чоловіка з гротескними рисами обличчя, множинними деформаціями кісток та суглобів, ураженням
печінки, серця та селезінки діагностовано мукополісахаридоз. Накопичення якої речовини в тканинах
спричинило захворювання?
@Глікозаміногліканів

1. У новонародженої дитини після годування молоком спостерігаються диспепсія, блювота. При годуванні
розчином глюкози ці явища зникають. Недостатня активність якого ферменту призводить до даних
розладів?
@Лактази

2. У новонародженої дитини діагностована спадкова недостатність сахарази. Який процес при цьому
порушений у дитини?
@Мембранне травлення
1. У хворої з обтураційною жовтяницею при обстеженні виявлена стеаторея. Відсутність якого
компоненту жовчі в кишках призвела до порушення всмоктування та перетравлення жирів?
@Жовчних кислот

2. Хворий 67 років страждає на атеросклероз судин головного мозку. При обстеженні виявлено
гіперліпідемію. Підвищення вмісту яких ліпопротеїнів плазми крові найбільше сприяє атерогенезу?
@Ліпопротеїнів низької щільності
3. Хворий 67 років страждає на атеросклероз судин головного мозку. При обстеженні виявлено
гіперліпідемію. Зниження вмісту яких ліпопротеїнів плазми крові найбільше сприяє атерогенезу?
@Ліпопротеїнів високої щільності

4. У хворого з еруптивними ксантомами, гепатоспленомегалією, абдомінальними коліками та


панкреатитом виявлено значне збільшення вмісту хіломікронів у плазмі крові. Який дефект лежить в
основі виникнення первинної гіперліпопротеїнемії у хворого?
@Дефіцит ліпопротеїнліпази

5. У хворого з еруптивними ксантомами, гепатоспленомегалією, абдомінальними коліками та


панкреатитом виявлено значне збільшення вмісту хіломікронів у плазмі крові. Яка первинна
гіперліпопротеїнемія (ГЛП) у хворого?
@ГЛП I типу

6. У дитини 10 років з сухожильними ксантомами та тяжкими нападами стенокардії виявлено значне


збільшення вмісту ліпопротеїнів низької щільності у плазмі крові. Яка первинна гіперліпопротеїнемія
(ГЛП) у хворого?
@ГЛП IIа типу

7. У дитини 10 років з сухожильними ксантомами та тяжкими нападами стенокардії виявлено значне


збільшення вмісту ліпопротеїнів низької щільності у плазмі крові. Який дефект лежить в основі
виникнення первинної гіперліпопротеїнемії у хворого?
@Дефект рецептора апоВ/Е

8. У хворого 45 років при обстеженні виявлений вміст холестерину в одиниці об’єму крові 5,8 ммоль/л.
Який ризик розвитку атеросклерозу в хворого?
@Середній

9. У хворого 45 років при обстеженні виявлений вміст тригліцеридів в одиниці об’єму крові 2,8 ммоль/л.
Який ризик розвитку атеросклерозу в хворого?
@Високий

10. У хворого 45 років при обстеженні виявлений вміст ЛПНЩ в одиниці об’єму крові 5 г/л. Який ризик
розвитку атеросклерозу в хворого?
@Середній

545.У хворої 45 років при обстеженні виявлений вміст ЛПВЩ в одиниці об’єму крові 6,1 г/л. Який ризик
розвитку атеросклерозу в хворої?
@Низький

1. У хворих на цукровий діабет часто виникає дефіцит ліпокаїчної субстанції. Яке порушення буде
наслідком цієї зміни?
@Жирова інфільтрація печінки

2. У хворого встановлений діагноз – вторинне ожиріння. Що могло стати його причиною?


@Гіперпродукція кортизолу

3. У мишей лінії Цукера (ob/ob) спонтанно виникає первинне ожиріння з ненаситним апетитом. Який
механізм виникнення ожиріння у цих мишей?
@Первинна недостатність лептину

4. У хворого встановлений діагноз – гіперпластичне ожиріння. Що могло стати його причиною?


@Перегодовування в грудному віці

5. У хворого встановлений діагноз – вторинне ожиріння. Що могло стати його причиною?


@Енцефаліт
6. У мишей лінії db/db спонтанно виникає ненаситний апетит, гіпотермія та гіпометаболізм, і розвивається
ожиріння, при якому маса тіла значно перевищує норму. Який механізм виникнення ожиріння?
@Вторинна недостатність лептину

7. Після введення ауротіоглюкози у частини тварин виникли гіперфагія, посилення ліпогенезу, зниження
термогенезу, викликаного вживанням їжі, та ожиріння. Який механізм виникнення ожиріння?
@Ураження вентромедіальних ядер гіпоталамуса

8. У людини діагностовано первинне ожиріння. З порушенням утворення якого гормону воно пов’язане?
@Лептину

9. У людини, яка перехворіла важкою формою грипу, з’явилися ожиріння, судинні кризи, сильний
головний біль. Який вид ожиріння (за етіологією) виник у хворого?
@Церебральне

10. У мишей зі спадковим ожирінням і цукровим діабетом 2-го типу виявлено гіперглікемію та зниження
кількості інсулінових рецепторів у міоцитах. Який первинний патогенетичний механізм посилення
ліпогенезу в цих тварин? A. @Гіперінсулінемія.

556.У хворих на первинне ожиріння значно збільшені апетит та споживання їжі. Яка речовина стимулює
центр голоду і активізує харчову поведінку людини?
@Нейропептин Y

1. У хворих з ожирінням у крові збільшені рівень фібриногену, загальна коагуляцій на активність та


знижений рівень гепарину. Які порушення можуть виникнути у хворого при таких змінах?
@Тромбози
2. Хвора на нейрогенну анорексію вкрай виснажена. Маса тіла дівчини нижча за нормальну на 48%. Як
називається такий стан?
@Ендогенна кахексія
3. У хворого із опіками 35% площі шкіри та зниженням маси на 38% діагностовано опікову хворобу у
стадії опікового виснаження. Яка речовина, що пригнічує центр голоду та стимулює катаболізм,
викликала виснаження у хворого?
@Кахексин

4. У хворого із злоякісною пухлиною спостерігається значне схуднення і виснаження. Яка речовина, що


пригнічує центр голоду та стимулює катаболізм, викликала виснаження у хворого?
@Кахексин

561.У хворого на цукровий діабет 1-го типу Спостерігається значна втрата маси тіла. Яка зміна обміну
речовин викликала схуднення у хворого?
@Відсутність анаболічної дії інсуліну
A.
562.У хворого на цироз печінки встановлене значне зниження в крові вмісту альбумінів. Яке порушення в
організмі найбільш вірогідно може бути наслідком такої гіпопротеїнемії? @Набряки

1. У хворого з гепатитом вміст загального білка в крові становить 40 г/л, вміст альбумінів і фібриногену
знижений. Які зміни якісного білкового складу крові наявні в цьому випадку?
@Гіпопротеїнемія

2. Чоловік 65 років, що страждає на подагру, скаржиться на болі в зоні нирок. При ультразвуковому
дослідженні встановлено наявність ниркових каменів. Яка зміна веде до утворення ниркових каменів у
цьому випадку?
@Гіперурикемія

3. Через місяць після видалення у хворого значного відрізка тонкої кишки у нього досліджували кров на
вміст білка в плазмі, альбумін склав 12 г/л. Яка зміна білкового складу крові виникла?
@Гіпопротеїнемія

4. Пацієнтка 60 років звернулася до лікаря зі скаргами на болі в малих суглобах ніг і рук. Суглоби
збільшені, мають вигляд потовщених вузлів. У сироватці крові підвищений вміст уратів. Порушення
обміну якої речовини може бути причиною розвитку цього стану?
@Пуринів

5. У хворого з інфекційним захворюванням концентрація білка в крові складає 120 г/л. Яка кількісна зміна
білкового складу крові наявна?
@Гіперпротеїнемія

6. До лікаря звернувся чоловік 65 років зі скаргами на гострий біль у великих пальцях ніг. Він любить і
часто вживає пиво. Виникла підозра на подагру. Вміст якої речовини необхідно визначити в крові для
підтвердження діагнозу?
@Сечової кислоти

7. У хворого з вірусним гепатитом А вміст загального білка в крові становить 60 г/л, вміст альбумінів і
фібриногену знижений, гамма-глобуліну підвищений. Які зміни якісного білкового складу крові наявні
в цьому випадку?
@Диспротеїнемія

8. У дитини 10 років з сечокам’яною хворобою, уповільненням росту та мегалобластною анемією,


резистентною до вітамінотерапії, виявлено оротоацидурію. Синтез якої речовини порушений у хворого?
@Уридину

9. У людини, що виконувала важку фізичну роботу в умовах підвищеної температури навколишнього


середовища, змінився білковий склад плазми крові. Яка форма зміни білкового складу крові буде
спостерігатися при цьому?
@Відносна гіперпротеїнемія

572.Альбіноси погано переносять вплив сонця: засмага не розвивається, а з’являються опіки. Порушення
метаболізму якої амінокислоти лежить в основі цього явища?
@Тирозину

1. При аналізі сечі тримісячної дитини виявлено підвищену кількість гомогентизинової кислоти, сеча при
стоянні на повітрі набуває темного забарвлення. Для якого з перелічених нижче захворювань характерні
описані зміни?
@Алкаптонурії

2. При клінічному обстеженні у вагітної жінки виявлено збільшення вмісту фенілаланіну в крові. Що з
названого нижче може виникнути у дитини?
@Олігофренія

3. Відомо, що фенілкетонурія виникає внаслідок мутації гена, відповідального за перетворення


амінокислоти фенілаланіну й розпаду її до кінцевих продуктів обміну - СО 2 і Н2О. Вкажіть, який шлях
обміну фенілаланіну призведе до розвитку фенілкетонурії?
@Фенілаланін –> фенілпіруват –> кетокислоти

4. При обстеженні у хворого було встановлено діагноз алкаптонурії - спадкового захворювання,


зумовленого дефіцитом ферменту:
@Оксидази гомогентизинової кислоти
577.У дитини 6 місяців з уповільненим моторним і психічним розвитком відзначається посвітління шкірних
покривів, волосся, райдужної оболонки ока, позитивна проба Фелінга (з 3% розчином трихлороцтового
заліза). Яке з зазначених спадкових захворювань виявлено в дитини?
@Фенілкетонурія

1. У хворого 25 років білі волосся та шкіра, рожево-червоні очі. Скаржиться на поганий зір вдень та
підвищену чутливість шкіри до сонця – фотодерматит. Дефіцит якого фермента зумовлює це
захворювання?
@Тирозинази

2. У дитини на 6-му місяці життя з’явилося тяжке ураження центральної нервової системи, посвітління
шкіри та волосся. Порушення метаболізму якої амінокислоти лежить в основі цього явища?
@Фенілаланіну

3. У дитини з розумовою відсталістю встановлено кретинізм. Який механізм є головним у розвитку


пошкодження ЦНС при цьому?
@Зменшення синтезу тироксину

4. У дитини 8 місяців виникли прояви пелагри. Порушення метаболізму якої амінокислоти лежить в
основі цього явища?
@Триптофану

582.У хворого на адреногенітальний синдром спостерігається позитивний азотистий баланс. Який механізм
зумовив це явище?
@Гіперпродукція андрогенів

1. У хворих з печінковою комою накопичується амоній, який зумовлює порушення енергозабезпечення


нейронів головного мозку. Який процес порушується в першу чергу в цьому випадку?
@Цикл Кребса

2. Пацієнт, в якого наявна лихоманка протягом трьох тижнів, втратив вагу. Визначення залишкового азоту
крові показало, що він становить 50 ммоль/л. Який азотистий баланс у хворого? @Негативний
3. У хворого на цукровий діабет залишковий азот крові становить 35 ммоль/л. Який вид гіперазотемії
виник у хворого?
@Продукційна

4. Пацієнт, що одужує після тяжкої форми крупозної пневмонії, почав потроху відновлювати вагу. Який
азотистий баланс у хворого?
@Позитивний

5. У хворого на хронічну ниркову недостатність у стадії уремії залишковий азот крові становить 60
ммоль/л. Який вид гіперазотемії виник у хворого?
@Ретенційна

6. У хворого на еозинофільну аденому гіпофіза спостерігається позитивний азотистий баланс. Який


механізм зумовив це явище?
@Гіперпродукція гормону росту

7. У хворого на цукровий діабет 1-го типу спостерігається негативний азотистий баланс. Який механізм
зумовив це явище?
@Гіпопродукція анаболічних гормонів

8. У хворого на опікову хворобу спостерігається негативний азотистий баланс. Який механізм зумовив це
явище?
@Втрата білків

9. У пацієнта, хворого на гломерулонефрит, виявлена протеїнурія. Вміст білків у крові становить 30 г/л.
Який азотистий баланс у хворого?
@Негативний
10. У хворого на спонгіоформну губчасту енцефалопатію (хворобу Крейтцфельдта – Якоба) у клітинах
центральної нервової системи накопичуються убіквітинізовані протеїни. Який процес порушений у
нейронах при цій хворобі?
@Протеасомний протеоліз

593.Піддослідна тварина знаходилась в термостаті при +36 оС. Протягом тривалого часу у неї спостерігалось
значне збільшення частоти і глибини дихання. Яке порушення водно-електролітного обміну виникло при
цьому?
@Дегідратація гіперосмолярна

1. У хворого на гострий ентерит з тяжкою діареєю розвинулося порушення водноелектролітного обміну


(ВЕО). Яке порушення ВЕО спостерігається в даному випадку?
@Дегідратація гіперосмолярна

2. У хворого з травматичною комою, який знаходиться у стані непритомності 4-у добу, спостерігаються
зниження тургору тканин, сухість слизових оболонок, гіпотензія. Який тип розладу водно-
електролітного балансу характеризує поява зазначених ознак?
@Дегідратація гіперосмолярна

3. Хворому з опіками II ступеню тяжкості 40% поверхні тіла вводять знеболювальні препарати та
протишокові розчини. Яке порушення водно-електролітного обміну найбільш імовірно виникає при
опіковій хворобі?
@Ізоосмолярна дегідратація

4. Робітник гарячого цеху, який протягом робочого дня пив водопровідну воду, відчув запаморочення.
Спрага відсутня, АТ 90/55 мм рт. ст. Яке порушення водноелектролітного обміну спостерігається в
даному випадку?
@Дегідратація гіпоосмолярна
5. У хворої на гіпофізарну форму нецукрового діабету розвинулося порушення водно-електролітного
обміну (ВЕО). Яке порушення ВЕО спостерігається в даному випадку?
@Дегідратація гіперосмолярна

6. У адреналектомованої тварини значно знизився артеріальний тиск. Яке порушення ВЕО спостерігається
в даному випадку?
@Дегідратація гіпоосмолярна

7. Гіперосмолярна дегідратація розвивається при нецукровому діабеті та при посиленій втраті поту. Зміни
якого з перерахованих показників будуть різноспрямованими при цих видах патології?
@Осмолярності сечі

8. У хворого на цукровий діабет виникла значна спрага, поліурія, порушення психічної діяльності. Який
тип розладу водно-електролітного балансу характеризує поява зазначених ознак?
@Дегідратація гіперосмолярна

9. У постраждалого виявлено внутрішню кровотечу, що супроводжується гіпотензією, зниженням тургору


тканин, сухістю слизових оболонок, спрагою. Який тип розладу водно-електролітного балансу
характеризує поява зазначених ознак?
@Дегідратація ізоосмолярна

603.У хворого на синдром Пархона (гіперпродукція вазопресину) розвинулося порушення водно-


електролітного обміну (ВЕО). Яке порушення ВЕО спостерігається в даному випадку?
@Гіпергідратація гіпоосмолярна

1. У хворого на гломерулонефрит виникла гостра ниркова недостатність з анурією, головним болем,


нудотою, блювотою та судомами. Яке порушення водноелектролітного балансу найбільш імовірно
виникло у хворого?
@Гіпоосмолярна гіпергідратація

2. У хворого на вторинний гіперальдостеронізм розвинулося порушення водноелектролітного обміну


(ВЕО). Яке порушення ВЕО спостерігається в даному випадку? @Гіпергідратація гіперосмолярна
3. При лікуванні зневоднення безсольовими розчинами на фоні різко зниженої видільної функції нирок
з'явилося погіршення загального стану, судомна готовність, явища набухання мозку. Яке порушення
водного обміну наявне?
@Гіпоосмолярна гіпергідратація

4. У працівника гарячого цеху внаслідок посиленого потовиділення виникла спрага, яку він тамує питтям
води без солі. Яке порушення водно-електролітного обміну може при цьому виникнути?
@Гіпоосмолярна гіпергідратація
608.У хворого з травмою уретри виникла рефлекторна анурія з головним болем, нудотою, блювотою та
судомами. Яке порушення водно-електролітного балансу найбільш імовірно виникло у хворого?
@Гіпоосмолярна гіпергідратація

1. Хворому разом з лікарськими препаратами внутрішньовенно введено 500,0 мл 5% розчину глюкози.


Яке порушення водно-електролітного балансу найбільш імовірно виникне у хворого?
@Гіпоосмолярна гіпергідратація

2. У одного з моряків, які через аварію на судні в океані змушені були пити морську воду, виникли
нестерпне почуття спраги, гарячка, непритомність. Яке порушення водно-електролітного балансу
викликало ці прояви?
@Гіперосмолярна гіпергідратація

3. Водне навантаження у адреналектомованої тварини призвело до блювання, судом та розвитку коми. Яке
порушення водно-електролітного балансу найбільш імовірно виникло у хворого?
@Гіпоосмолярна гіпергідратація

4. У тварини після введення сулеми виникла гостра ниркова недостатність з анурією, головним болем,
нудотою, блювотою та судомами. Яке порушення водноелектролітного балансу найбільш імовірно
виникло у хворого?
@Гіпоосмолярна гіпергідратація

613.При огляді хворого виявлена виражена припухлість в ділянці лівого передпліччя, що виникла після укусу
бджоли. Назвіть провідний патогенетичний механізм цього набряку:
@Мембраногенний

1. У хворого тяжка нефропатія. Лабораторні дослідження визначають значну протеїнурію, циліндрурію,


гіпопротеїнемію, гіперліпемію, гіперкаліємію. Що є найбільш істотною ланкою в патогенезі набряків у
цього хворого?
@Зниження онкотичного тиску крові

2. У хворого з серцевою недостатністю за лівошлуночковим типом з'явилися клінічні ознаки набряку


легенів. Який з перерахованих патогенетичних механізмів є первинним при цій патології?
@Застійний

3. Чоловік 64 років скаржиться на задишку, часте серцебиття, швидку стомлюваність. Увечері з'являються
набряки на нижніх кінцівках. Що з переліченого нижче є ведучим патогенетичним фактором цих
набряків?
@Підвищення гідростатичного тиску крові

4. Значна частина випадків аліментарного голодування супроводжується вираженими набряками. Який з


патогенетичних факторів набряку є провідним у цьому випадку?
@Зниження онкотичного тиску плазми крові
618.У хворого 35 років, що переніс гепатит і продовжував вживати алкоголь, розвинулися ознаки цирозу
печінки з асцитом і набряками на нижніх кінцівках. Які зміни складу крові стали вирішальними в розвитку
набряку?
@Гіпоальбумінемія

1. У жінки спостерігається пастозність повік, губ, шиї, язика, які виникли після того, як вона з'їла
апельсин. Який патогенетичний механізм є провідним у розвитку набряку в жінки?
@Підвищення проникності капілярів

2. При запаленні в судинах мікроциркуляторного русла відзначається підвищення їх проникності,


збільшення гідродинамічного тиску. У тканинній рідині наявні зрушення рН у кислий бік, підвищення
осмотичної концентрації й дисперсності білкових структур. Який вид набряку буде спостерігатися в
цьому випадку?
@Змішаний
3. У хворого, що страждає на серцеву недостатність, спостерігається збільшення печінки, набряки нижніх
кінцівок, асцит. Який механізм є провідним в утворенні цього набряку?
@Застійний

4. Хворій 34 років 3 роки тому було поставлено діагноз хронічного гломерулонефриту. За останні 6
місяців з'явилися набряки. Що лежить в основі їх розвитку?
@Протеїнурія
623.У хворого на первинний гіперальдостеронізм (синдром Конна) виникли м’язова слабкість,
зниження моторики шлунка і кишок, гіпотензія. Яке порушення викликало ці зміни?
@Гіпокаліемія

1. У дитини 10 місяців, хворої на рахіт, виникли дратівливість, спазмофілія, порушення


мінералізації кісток та їх деформації. Яке порушення викликало ці зміни? @Гіпокальціемія

2. У хворого на хворобу Аддісона вміст Na+ у плазмі крові 100 ммоль/л. Чим може проявитися
такий стан?
@Набуханням клітин

3. При експериментальному гіпервітамінозі Д у тварини розвинулись артеріосклероз Менкеберга


в аорті, гіперсекреція в шлунку та камені в нирках. Яке порушення викликало ці зміни?
@Гіперкальціемія

4. У хворого у плазмі крові: вміст Na + 170 ммоль/л, осмолярність – 330 мосмоль/л. Чим може
проявитися такий стан?
@Зневодненням клітин

5. У дитини 5 місяців, хворої на рахіт, виникли дратівливість, спазмофілія, порушення


мінералізації кісток. Яке порушення викликало ці зміни?
@Гіповітаміноз Д

6. Хворий на первинний гіперальдостеронізм (синдром Конна) скаржиться на артеріальну


гіпертензію. Яке порушення викликало ці зміни?
@Гіпернатріемія

7. Після адреналектомії у тварини виникла зупинка серця в діастолі і вона загинула. Яке
порушення викликало зупинку серця?
@Гіперкаліемія

8. У дітей найбільші прояви рахіту припадають на ранню весну. Що зумовлює загострення


хвороби в зимовий час?
@Зниження інсоляції

9. Який механізм компенсує надмірну втрату солей натрію в організмі при тривалому потінні в
умовах жаркого мікроклімату?
@Підвищення вироблення альдостерону

10. При профілактичному огляді дітей закарпатського селища у багатьох виявляється поширений
карієс. Дефіцит якої речовини в їжі має відношення до розвитку карієсу?
@Фтору
11. У тварини, яка тривалий час знаходилась на дієті з недостатністю одного з катіонів, значно
знизилась збудливість нервових и м’язових клітин, зменшився тонус судин. Якого іону не
вистачає в організмі?
@Калію

12. У щурів, які тривалий час отримували лише вуглеводну їжу, спостерігалось накопичення води в
тканинах. Який патогенетичний механізм є головним у розвитку набряку в даному випадку?
@Гіпоонкотичний

13. У хворого з серцевою декомпенсацією при пункції черевної порожнини отримано прозору
лимонно-жовтого кольору рідину, відносна щільність - 1012, альбумінів - 1%, глобулінів - 0,2%,
фібриногену немає, одиничні еритроцити, 1-3 лейкоцити в полі зору. Назвіть провідний
патогенетичний механізм утворення цієї рідини:
@Гідростатичний

14. У хворого з гіпертонічним кризом виник набряк легенів. Що є головним чинником у розвитку
цього стану?
@Підвищення гідростатичного тиску в капілярах легенів

15. Внаслідок захворювання нирок у пацієнта відзначаються набряки. В аналізах сечі значна
кількість білка. Яким механізмом можна пояснити виникнення набряків у такого пацієнта?
@Зниженням онкотичного тиску плазми крові

16. У працівника гарячого цеху внаслідок посиленого потовиділення виникла спрага. Яке
порушення водно-електролітного обміну може при цьому виникнути?
@Гіперосмолярна гіпогідрія
17. Хворий 40 років скаржиться на загальну слабкість, задишку, серцебиття, набряки на ногах. При
огляді: частота дихання - 32 за хвилину, обличчя ціанотичне, асцит, печінка збільшена, пульс -
124 уд./хв., АТ - 170/90 мм рт.ст. Який провідний фактор патогенезу набряку у цього хворого?
@Підвищення гідростатичного тиску крові в капілярах

18. У хворого на алергію виник набряк Квінке (набряк м'яких тканин тіла). Який з патогенетичних
факторів набряку є пусковим у цьому випадку?
@Підвищення проникності стінок капілярів

19. До клініки потрапив чоловік 30 років із тяжкою діареєю протягом 12 годин. Блювання не було.
Які зміни водно-електролітного балансу й кислотно-лужної рівноваги спостерігаються у
хворого?
@Негазовий ацидоз із дегідратацією

20. Чоловік 32 років протягом 4 років страждає на хронічний гломерулонефрит. Відзначаються


набряки на обличчі, останнім часом з'явилися набряки на ногах і тулубі, тобто
гломерулонефрит перебігає з нефротичним синдромом. Який з перерахованих механізмів
найбільш характерний для розвитку набряків у цього хворого?
@Зниження онкотичного тиску крові
21. Жінку 32 років вжалила оса. На шкірі лівої щоки (у місці укусу) - набряк і гіперемія. Який
механізм набряку є первинним у цьому випадку?
@Підвищення проникності капілярів
A.
1. У хворого з дихальною недостатністю рН крові 7,36. Визначення рСО2 показала наявність
гіперкапнії.Сеча має кислу реакцію. Яка форма порушення кислотноосновного стану в даному
випадку?
@Ацидоз газовий компенсований

2. У хворого виник декомпенсований негазовий ацидоз. Який з показників вказує на


декомпенсацію ацидозу?
@Зниження рН нижче 7,36

3. У хворого виник компенсований газовий алкалоз. Який з показників вказує на компенсацію


алкалозу?
@Нормальна величина рН

4. У хворого виник декомпенсований газовий ацидоз. Який з показників вказує на декомпенсацію


ацидозу?
@Зниження рН нижче 7,36

5. У хворого виник компенсований негазовий алкалоз. Який з показників вказує на компенсацію


алкалозу?
@Нормальна величина рН

6. У хворого виник декомпенсований газовий алкалоз. Який з показників указує на декомпенсацію


алкалозу?
@Підвищення рН вище 7,44

7. У хворого виник компенсований газовий ацидоз. Який з показників указує на компенсацію


ацидозу?
@Нормальна величина рН

8. У хворого виник декомпенсований негазовий алкалоз. Який з показників указує на


декомпенсацію алкалозу?
@Підвищення рН вище 7,44

9. У хворого виник компенсований негазовий ацидоз. Який з показників указує на компенсацію


ацидозу?
@Нормальна величина рН

10. У вагітної жінки розвинувся токсикоз з важкою повторною блювотою протягом доби. До кінця
доби почали з’являтися тетанічні судоми і зневоднення організму. Яке порушення КОС
викликало описані зміни?
@Негазовий видільний алкалоз

11. У хворого діагностовано первинний гіперальдостеронізм (хвороба Конна) Яке порушення КОС
може мати місце при цьому?
@Негазовий видільний алкалоз
656.При підйомі в гори у альпініста розвинулися ейфорія, головний біль, запаморочення, задишка,
яка чергувалася з апное. Про яке порушення кислотно-основного стану слід думати?
@Газовий алкалоз

1. У новонародженої дитини з пілоростенозом спостерігається багаторазова блювота, апатія,


слабкість, підвищення тонусу м'язів, іноді виникають судоми. Яка форма порушення кислотно-
основного стану розвинулася у дитини?
@Видільний алкалоз

2. У хворого під час операції з надмірною штучною вентиляцією легень виникли судоми. Їх
викликало наступне порушення КОС:
@Газовий алкалоз

3. У хворого діагностовано вторинний гіперальдостеронізм. Яке порушення КОС може мати місце
при цьому?
@Негазовий видільний алкалоз

4. У хворої з гіперацидним гастритом, яка постійно вживала великі кількості лужних мінеральних
вод, раптово виникли тетанія та судоми. Їх викликало наступне порушення КОС:
@Негазовий екзогенний алкалоз

661.У хворого з закритою черепно-мозковою травмою спостерігається виражене гіперпное. Яка


форма порушення кислотно-основного стану розвинулася у постраждалого?
@Газовий алкалоз

1. У стоматологічного хворого значно виражений синдром гіперсалівації (птіалізм), унаслідок


чого він вимушений випльовувати слину. Яка з форм порушення кислотний-основного стану
може розвинутися з часом у даного хворого?
@Видільний ацидоз

2. У хворого на цукровий діабет виявлені високий рівень гіперглікемії, кетонурія, глюкозурія,


гіперстенурія і поліурія. Яка форма порушення кислотно-основної рівноваги має місце?
@Метаболічний ацидоз

3. Хвора на цукровий діабет доставлена в лікарню у непритомному стані. При дослідженні


кислотно-основного стану (КОС) виявлений негазовий ацидоз. Який первинний механізм
зумовив виявлені зміни КОС?
@Утворення недоокислених продуктів

4. На 10 добу лікувального голодування хворий відчув збудження, з'явилося глибоке, часте


дихання, артеріальний тиск знизився до 90/60 мм рт.ст., зменшилося виділення сечі, сеча із
запахом ацетону, рН 7,3. Яке порушення КОС у хворого? @Метаболічний ацидоз
666.У хворого на холеру в результаті тривалого проносу відбувається значна втрата кишкового
вмісту, що стало причиною порушення кислотно-основного стану в організмі. Яка з перерахованих
форм порушення КОС має місце ?
@Негазовий видільний ацидоз

1. Циркуляторна гіпоксія, що виникла у хворого внаслідок хронічної серцевої недостатності,


призвела до порушення КОС. Про яку зміну КОС слід думати?
@Метаболічний лактатацидоз

2. У людини з гострою нирковою недостатністю (ГНН) спостерігаються артеріальна гіпертензія,


серцева аритмія, дихання Куссмауля, судоми. Яке зрушення КОС характерне для ГНН?
@Негазовий видільний ацидоз

3. У хворого діагностовано хронічну недостатність надниркових залоз (хворобу Аддісона). Яке


порушення КОС може мати місце в даному випадку?
@Негазовий видільний ацидоз

4. Внаслідок отруєння морфіном у хворого виникло значне пригнічення дихального центру і


порушення легеневої вентиляції. Які зрушення кислотно-основного стану можна виявити у
цього хворого?
@Газовий ацидоз
5. У хворого виявлено порушення прохідності дихальних шляхів на рівні дрібних і середніх
бронхів. Які порушення кислотно-основного стану в цього хворого?
@Респіраторний ацидоз

6. У хворого на цукровий діабет розвинулася діабетична кома внаслідок порушення кислотно-


основного стану. Який вид порушення виник при цьому?
@Метаболічний ацидоз

7. У людини з хронічним гломерулонефритом наростає загальна слабкість, різка тахікардія з


періодичною аритмією, загальмованість і сонливість. Яке зрушення КОС супроводжує
наближення уремічної коми?
@Негазовий видільний ацидоз

8. У хворого на цукровий діабет унаслідок накопичення β-оксимасляної і ацетооцтової кислот має


місце порушення кислотно-основного стану, яке називається:
@Негазовий метаболічний кетоацидоз

9. При обстеженні хворого визначається наявність гіперглікемії, кетонурії, поліурії і глюкозурії.


Яка форма порушення кислотно-основної рівноваги має місце в даній ситуації?
@Метаболічний ацидоз

10. У тварини, яка тривалий час провела в умовах надлишку СО 2, виникло наступне порушення
КОС:
@Газовий екзогенний ацидоз
A.
1. У хворого на хронічний алкоголізм спостерігаються слабкість та болі в ногах, атрофія м’язів
кінцівок, балакучість та ретроградна амнезія. Діагностована хвороба бері-бері. Дефіцит якого
вітаміну зумовлює цю патологію?
@В1

2. У хворого на бері-бері вживання вуглеводів значно погіршило стан хворого і спровокувало


психотичні прояви. Який біохімічний процес порушений в організмі при гіповітамінозі В1?
@Окислення пірувату до ацетилКоА

3. У хворого, що тривалий час знаходився на кукурудзяно-злаковій дієті, спостерігаються


дерматит, діарея, деменція. Діагностовано пелагру Дефіцит якого вітаміну зумовлює цю
патологію?
@РР

4. У хворих на арибофлавіноз один з головних механізмів захворювання пов'язаний з порушенням


ферментів дихального лагцюга. Дефіцит якого вітаміну зумовив цю патологію?
@В2

5. У хворих на гіповітаміноз В2 спостерігаються хейлоз, ангулярний стоматит, себорейний


дерматит. Функція яких ферментів порушується при цій патології? A. @Флавінових
дегідрогеназ

682.Лікування гіпохромної анемії в пацієнта з хронічним алкоголізмом препаратами заліза була


неефективним. При подальшому обстеженні діагностована сидеробластна анемія. Дефіцит якої
речовини може бути її причиною?
@Вітаміну В6

683. У хворого на туберкульоз та хронічний алкоголізм після лікування фтивазидом виник дефіцит
вітаміну В6 та діагностована залізорефрактерна анемія. Порушення синтезу яких речовин є причиною
анемії у хворого?
@Порфіринів

684. У дитини, хвороі на туберкульоз, після лікування фтивазидом виник дефіцит вітаміну В 6, що
призвело до значного сповільнення росту. Порушення обміну яких речовин є причиною цієї зміни?
@Амінокислот
685. Через деякий час після резекції пухлини тонкої кишки в хворого виявлено фолієводефіцитну
анемію. Який процес найбільше порушується при цьому?
@Мітоз

686. У жінки, яку лікували від лейкозу аміноптерином, виявлено мегалобластну анемію і лейкопенію.
Порушення синтезу якої речовини виникає при дії антагоністів фолієвої кислоти? @ДНК

687.Після завершення курсу хіміотерапії лейкозу в хворої розвинулася пневмонія. Застосування яких
препаратів стало причиною недостатності імунітету? A. @Антагоністів фолієвої кислоти

688. У жінки, яка хворіє на атрофічний гастрит, виявлено мегалобластну анемію і лейкопенію.
Порушення синтезу якої речовини виникає при дефіциті ціанкобаламіну? A. @ДНК
1. У чоловіка, якому 5 років тому видалили шлунок, стоматолог виявив атрофічнозапальні
процеси слизової ротової порожнини: яскраво-червоний, гладенький («лакований») язик,
гінгівіт. Чим зумовлені ці зміни? A. @Дефіцитом вітаміну В12.

2. Хворий з гіповітамінозом В12 скаржиться на відчуття „повзання мурашок” по тілу. Чим


зумовлене це порушення чутливості?
@Демієлінізацією чутливих нервів

3. Жінку, яка хворіє на атрофічний гастрит, турбує її швидка втомлюваність, задишка, відчуття
оніміння в кінцівках. При обстеженні виявлено гіперхромну макроцитарну анемію, лейкопенію.
Що є причиною зменшення кількості еритроцитів і лейкоцитів у крові хворої?
@Дефіцит вітаміну В12

692.У хворого, якому 5 років тому зробили резекцію шлунка, діагностовано гіперхромну анемію,
зумовлену дефіцитом ціанкобаламіну. Що спричинило розвиток анемії у хворого?
@Дефіцит внутрішнього фактора Касла

1. Після тривалого застосування сульфаніламідів у хворого з’явились крововиливи на шкірі та


слизових оболонках. Нестача якого вітаміну виникла у хворого? A. @К
2. На 2 день життя у немовляти виникли кровотеча з пуповини, мелена (чорний дьогтеподібний
кал, з червоним обідком на пелюшці), крововиливи на шкірі та слизових оболонках. Нестача
якого вітаміну викликала ці прояви? @К

3. У жінки, що страждає на жовчно-кам’яну хворобу виявлено геморрагічний синдром,


зумовлений дефіцитом вітаміну К. Які з перелічених факторів є неповноцінним при
гіповітамінозі К? @ Фактори II, VII, IX, X

696. У хворого з гіповітамінозом К виявлено зниження вмісту протромбіну (ф. II) в крові. Це
призведе до порушення перш за все: @Другої фази коагуляційного гемостазу

697. У хворого 45 років діагностована механічна жовтяниця, що супроводжується стеатореєю та


геморагіями на шкірі. У крові - зменшення вмісту протромбіну та інших факторів зсідання крові. Яка
причина порушення коагуляційного гемостазу в даного хворого? @Порушення всмоктування
вітаміну К

1. У хворого через наявність каменю у загальній жовчній протоці припинилося надходження


жовчі в кишечник. Порушення якого з процесів спостерігається при цьому?
@Всмоктування жиророзчинних вітамінів

2. Після тривалого введення великих доз ергокальциферолу у тварини виникли склерозування та


кальциноз середньої оболонки артерій. Який патологічний процес моделювали в експерименті?
@Артеріосклероз Менкеберга

3. При експериментальному гіпервітамінозі Д у тварини розвинулись артеріосклероз Менкеберга


в аорті, гіперсекреція в шлунку та камені в нирках. Яке порушення викликало ці зміни?
@Гіперкальціемія
4. У дитини 10 місяців, хворої на рахіт, виникли дратівливість, спазмофілія, порушення
мінералізації кісток та їх деформації. Яке порушення викликало ці зміни? @Гіпокальціемія

5. У дитини 5 місяців, хворої на рахіт, виникли дратівливість, спазмофілія, порушення


мінералізації кісток. Яке порушення викликало ці зміни?
@Гіповітаміноз Д
6. У дітей найбільші прояви рахіту припадають на ранню весну. Що зумовлює загострення
хвороби в зимовий час?
@Зниження інсоляції

7. Дитина 9 місяців збуджена, вередлива. У неї порушена мінералізація та ріст кісток. Нестача
якого вітаміну виникла у хворого?

8. Внаслідок порушення всмоктування вітаміну у травному каналі у хворого спостерігаєсться


ксерофтальмія та кератомаляція. Дефіцит якого вітаміну зумовив цю патологію?

9. Внаслідок тривалого дотримання суворого посту у хворого спостерігаєсться порушення зору у


вечірні та нічні години («куряча сліпота»). Дефіцит якого вітаміну зумовив цю патологію?

10. Внаслідок тривалого одноманітного харчування на початку весни у хворого спостерігаюсться


кровоточивість ясен та петехії на шкірі. Діагностована цинга. Дефіцит якого вітаміну зумовив
цю патологію? @С

11. Внаслідок тривалого одноманітного харчування на початку весни у хворого спостерігаюсться


кровоточивість ясен та петехії на шкірі. Діагностована цинга. Порушення якого процесу є
причиною геморагічної вазопатії?
@Синтез колагену

#У лікарню до кінця робочого дня доставлений працівник ”гарячого” цеху, який скаржиться на
головний біль, запаморочення, нудоту, загальну слабкість. Свідомість збережена, шкірні покриви
гіперемовані, сухі, гарячі на дотик. ЧСС- 130/хв. Дихання часте, поверхневе. Яке порушення
процесів регуляції тепла найімовірніше виникло у людини у даній ситуації?
+ Зниження тепловіддачі

#Відомо, що типові патологічні процеси розвиваються за однаковими закономірностями в різних


органах і тканинах та у різних видів тварин. Яке з перерахованих явищ можна віднести до типового
патологічного процесу?
+ Пухлина
#Людина, яка проживала в ендемічному вогнищі, перехворіла триденною малярією. Через півтора
року після переїзду в іншу місцевість захворіла малярією знову. Яка найбільш вірогідна форма цього
захворювання?
+Рецидив

#У хворого лікар діагностував гостру гонорею. З анамнезу стало відомо, що раніше він переніс
гонорею і вилікування було повним. До якої категорії інфекцій можна віднести це нове
захворювання?
+Реінфекція

#У хворого відзначається атрофія альвеолярних відростків щелепи після видалення зубів. Це є


прикладом:
+Патологічного стану

#При термометрії встановлено, що температура відкритих ділянок шкіри на 1-1,5o нижче за


температуру поруч розташованих ділянок, закритих одягом з натуральних тканин. Причиною цього є
те, що одяг, перш за все, зменшує тепловіддачу таким шляхом:
+Конвекція

#Віддачу тепла яким шляхом збільшують люди, які приймають прохолодний душ у спекотні дні?
+Теплопроведення

#У хворого виявлені такі зміни в крові:еритроцити - 2,8 Т/л, гемоглобін - 60 г/л, КП - 0,64,
ретикулоцити - 0,1%, лейкоцити - 8,7 Г/л; в мазку: мікроцитоз і пойкілоцитоз. Залізо сироватки крові
4,5 мкмоль/л. Яка анемія у хворого?
+Залізодефіцитна

#У робітника, який працював літом у щільному костюмі, різко підвищилась температура тіла,
з’явились задишка, тахікардія, нудота, судоми, втрата свідомості. Що явилось причиною тяжкого
стану робітника?
+ Зниження тепловіддачі

#Охолодження тіла людини у воді виникає значно швидше, ніж на повітрі, тому, що у воді значно
ефективнішою є віддача тепла шляхом:
+Теплопроведення

#Людина стоїть у кімнаті в легкому одязі; температура повітря +14°C. Вікна і двері зачинені. Яким
шляхом організм людини віддає найбільше тепла?
+Теплорадіація

#У холодну погоду з вітром люди мерзнуть швидше, ніж за відсутності вітру. Причиною цього є те,
що вітер збільшує, насамперед, віддачу тепла таким шляхом:
+Конвекція

#Ураження хворого одноразовою дозою іонізуючого випромінювання спричинило розвиток


кістковомозкової форми променевої хвороби. Які патологічні прояви з боку крові будуть
характерними в період удаваного благополуччя?
+Наростаюча лімфопенія, лейкопенія

#У результаті радіаційного випромінювання ушкоджені стовбурові гемопоетичні клітини. Утворення


яких клітин сполучної тканини буде порушено?
+Макрофаги

#Під час роботи щодо ліквідації наслідків аварії на АЕС, робітник одержав дозу опромінення 500
рентген. Скаржиться на головний біль, нудоту, запаморочення. Які зміни кількості лейкоцитів можна
очікувати в хворого через 10 годин після опромінення?
+Нейтрофільний лейкоцитоз
#У ліквідатора наслідків аварії на АЕС під час перебігу гострої променевої хвороби виник
геморагічний синдром. Що має найбільше значення в патогенезі цього синдрому?
+Тромбоцитопенія
#Для людини існує суворе обмеження в часі перебування на висоті понад 800 метрів над рівнем моря
без кисневих балонів. Що є лімітуючим фактором для життя в даному випадку?
+Парціальний тиск кисню в повітрі

#У періоді розпалу гострої променевої хвороби у хворого спостерігалися лейкопенія,


тромбоцитопенія, аутоінфекція, аутоинтоксикация, кровоточивість, підвищення температури тіла
Для якої форми променевої хвороби характерна дана картина?
+Кістково-мозкової
#Робітника АЕС доставили у клініку після одноразового опромінення зі скаргами на слабкість,
головний біль, підвищення температури, діарею. В аналізі крові – лейкоцитоз з лімфопеніею. Яка
стадія променевої хвороб найбільш ймовірна у паціента?
+Період первинних реакцій

#У альпініста під час сходження на висоті 6 тис. м над рівнем моря виникла ейфорія, неадекватна
оцінка обстановки, спостерігались галюцинації. Яка головна причина у розвитку цих ознак гірської
хвороби?
+Пониження парціального тиску кисню у повітрі.

#У кроля після опромінення спостерігається III період кістково-мозкової форми гострої променевої
хвороби. Ураження якої тканини є провідним у патогенезі розладів при цьому?
+Кровотворної.

#Під час ліквідації аварії на ЧАЕС робітник отримав дозу опромінення. При його обстеженні
виявлені такі дані: еритроцитів - 2*1012/л, ретикулоцитів - немає, гемоглобін -50 г/л, кількість
лейкоцитів - 3*109/л, в лейкоцитарній формулі виявлена лімофпенія число тромбоцитів 85 *109/л. Для
якої стадії променевої хвороби характерні такі зміни?
+Період розпалу.

#Альпініст на протязі кількох діб підіймався в гору. На висоті 5 000 метрів його стали непокоїти
тахіпное, тахікардія, головний біль розпираючого характеру. Вкажіть можливі причини вказаних
симптомів?
+Зниження парціального тиску кисню в повітрі

#При роботі з радіоактивними речовинами співробітник внаслідок аварії отримав дозу загального
опромінення 4 Гр. Скаржиться на головний біль, нудоту, запаморочення. Які зміни в складі крові
можна очікувати у хворого через 10 годин після опромінення?
+Нейтрофільний лейкоцитоз

#Після занурення водолаза на глибину 60 м у нього з’явилися симптоми порушення функцій


центральної нервової системи – збудження, ейфорія, ослаблення уваги, професійні помилки. Ці
симптоми пов’язані з токсичною дією на нейрони
+Азоту
#У водолаза, який проводив роботи на великій глибині, при швидкому поверненні його в умови
нормального атмосферного тиску з'явився біль в суглобах, свербіж шкіри, порушення зору, втрата
свідомості. Як називається описане явище?
+Хвороба декомпресії

#При пошкодженні клітини іонізуючим випромінюванням вмикаються механізми захисту і адаптації.


Який механізм відновлення порушеного внутрішньоклітинного гомеостазу реалізується при цьому?
+Активація антиоксидантної системи
#Ціаністий калій є отрутою, від якої смерть організму наступає миттєво. На які ферменти в
мітохондріях діє ціанистий калій?
+Цитохромоксидаза (аа3)

#Виділяють декілька груп молекулярних механізмів, які мають важливе значення в патогенезі
ушкодження клітин, що сприяє розвитку патології. Які процеси забезпечують протеїнові механізми
ушкодження?
+Пригнічення ферментів

#Внаслідок активації іонних каналів зовнішньої мембрани збудливої кліти-ни значно збільшився її
потенціал спокою. Які канали були активовані?
+Калієві

#При пошкодженні клітини іонізуючим випромінюванням вмикаються механізми захисту і адаптації.


Який механізм відновлення порушеного внутрішньоклітинного гомеостазу реалізується при цьому?
+Активація антиоксидантної системи

#Піддослідній тварині ввели блокатор цитохромоксидази, що призвело до її миттєвої загибелі. Яка із


сполук калію може викликати вказані зміни:
+Цианід

#По приїзді групи експертів на місце злочину виявлено тіло без ознак життя. В ході дослідження
крові загиблого виявлена велика концентрація іонів ціанової кислоти. Інгібування якого комплексу
дихального ланцюга мітохондрій стало причиною смерті?
+IV
#При патологічних процесах, які супроводжуються гіпоксією, відбувається неповне відновлення
молекули кисню в дихальному ланцюзі і накопичення пероксиду водню. Вкажіть фермент, який
забезпечує його руйнування:
+Каталаза

#При пошкодженні клітини іонізуючим випромінюванням вмикаються механізми захисту і адаптації.


Який механізм відновлення порушеного внутрішньоклітинного гомеостазу реалізується при цьому?
+Активація антиоксидантної системи

#У жінки 37-ми років протягом року періодично виникали інфекційні захворювання бактеріального
генезу,їх перебіг був вкрай тривалим, ремісії - короткочасними. При обстеженні виявлена
гіпогамаглобулінемія. Порушення функції яких клітин може бути прямою її причиною?
+Плазматичні клітини

#Хлопчик на другому році життя став часто хворіти на респіраторні захворювання, стоматити,
гнійничкові ураження шкіри. Навіть невеликі пошкодження ясен і слизової оболонки ускладнюються
запаленням, що протікає тривало. Встановлено, що у крові дитини практично відсутні
імуноглобуліни всіх класів. Зниження функціональної активності якої клітинної популяції лежить в
основі описаного синдрому?
+В-лімфоцити

#До приймального відділення інфекційної лікарні надійшов чоловік 25-ти років. Діагноз: СНІД.
Ураження яких клітин зумовлює стан імунодефіциту?
+Т-хелпери

#Хворий 47-ми років впродовж останніх 3-х років хворіє на туберкульоз легень, скаржиться на
задишку, важкість в області правого боку грудної стінки, температуру тіла 37,7oC. Виявлено
правобічний ексудативний плеврит. Який тип клітин передбачається у плевральному пунктаті?
+Лімфоцити

#У хворих із синдромом набутого імунодефіциту (СНІД) різко знижується імунологічна


реактивність, що проявляється розвитком хронічних запальних процесів, інфекційних захворювань,
пухлинного росту. Клітини якого типу ушкоджує ВІЛ- інфекція, внаслідок чого знижується імунний
захист?
+Т4-хелпери
#У пацієнта з бронхіальною астмою за допомогою шкірних алергічних проб встановлено
сенсибілізацію алергеном тополиного пуху. Який фактор імунної системи відіграє вирішальну роль в
розвитку цього імунопатологічного стану?
+IgE

#При відборі для ревакцинації вакциною БЦЖ у школяра поставлено пробу Манту, яка виявилася
негативною. Результат проби свідчить про такі особливості імунітету до туберкульозу:
+Відсутність клітинного імунітету

#У хворого, що надійшов до хірургічного відділення з ознаками гострого апендициту, виявлені


наступні зміни білої крові: загальна кількість лейкоцитів – 16*10 9/л. Лейкоцитарна формула: б.- 0, е.-
2%, ю.- 2%, п.- 8%, с.- 59%, л.- 25%, м.- 4%. Як класифікуються зазначені зміни?
+Нейтрофілія з регенеративним зсувом вліво

#Дитині після аналізу імунограми встановили діагноз первинний імунодефіцит гуморальної ланки
імунітету. Яка з причин може призвести до розвитку первинного імунодефіциту в організмі дитини?
+Спадкові порушення в імунній системі

#У дитини 5-ти років діагностовано хворобу Брутона, яка проявляється у важкому перебігу
бактеріальних інфекцій, відсутності В-лімфоцитів та плазматичних клітин. Які зміни вмісту
імуноглобулінів будуть спостерігатися в сироватці крові цієї дитини?
+Зменшення IgA, IgM

#У хворого з клінічними ознаками імунодефіциту проведено імунологічні дослідження. Виявлено


значне зниження кількості клітин, що утворюють розетки з еритроцитами барана. Який висновок
слід зробити на основі даних аналізу?
+Зниження рівня T -лімфоцитів

#Досить часто причиною набутих імунодефіцитів є інфекційне ураження організму, при якому
збудники безпосередньо розмножуються в клітинах імунної системи і руйнують їх. Виберіть серед
перерахованих ті захворювання, при яких має місце вищезгадане:
+Інфекційний мононуклеоз, СНІД
#У плазмі крові здорової людини знаходиться декілька десятків білків При захворюванні організму
з’являються нові білки, зокрема "білок гострої фази". Таким білком є
+С-реактивний білок

#Хлопчик на другому році життя став часто хворіти на респіраторні захворювання, стоматити,
гнійничкові ураження шкіри. Навіть невеликі пошкодження ясен і слизової ускладнюються
запаленням, що протікає три-вало. Встановлено, що у крові дитини практично відсутні
імуноглобуліни усіх класів. Зниження функціональної активності якої клітинної популяції лежить в
основі описаного синдрому?
+В-лімфоцити
#Недостатність в організмі лінолевої та ліноленової кислот призводить до ушкоджень шкіри,
випадіння волосся, сповільненого загоювання ран, тромбоцитопенії, зниження опірності до
інфекційних захворювань. Порушення синтезу яких речовин найімовірніше зумовлює вказані
симптоми?
+Ейкозаноїди

#При повторному введенні алергену починається виділення гістаміну тучними клітинами крові. До
якого рівня реактивності відноситься така відповідь організму?
+Клітинний

#У хворого діагностовано ГРВІ. У сироватці крові знайдено імуноглобуліни класу М. Який період
інфекційного процесу в даному випадку?
+Гострий
#У хворого спостерігається синдром Ді джорджі, в основі якого лежить гіпоплазія вилочкової залози
До якої форми імунної патології належить це захворювання?
+Вроджений B дефіцит Т-лімфоцитів

#Чоловік 25 років скаржиться на часто виникаючі запальні захворювання різної локалізації.


Встановлено, що він - ін'єкційний наркоман. Проба на ВІЛ-інфекцію виявилась позитивною. Який з
перелічених типів клітин імунної системи найбільш суттєво вражається ВІЛ?
+хелпери

#У хлопчика 5 міс., при дослідженні імунного статусу виявлено зменшення імуноглобулінів,


особливо IgA та IgM. В крові та лімфатичних вузлах відсутні В-лімоцити та плазматичні клітини.
Реакції Т-лімфоцитів збережені. Захворювання передається по спадковості як зчеплене із статтю. Яка
патологія спостерігається у цієї дитини?
+Хвороба Бутона
#У ВІЧ-інфікованого хворого спостерігається пригнічення активності імунної системи. Ураження
яких клітин найбільшою мірою обумовлює стан імунодефіциту у цього хворого?
+Т-хелперів

#При дослідженні стану імунної системи хворого із хронічними грибковими ураженнями шкіри
виявлено порушення клітинного імунітету. Зниження яких показників найбільш характерні при
цьому?
+Т-лімфоцитів

#Хлопчик 1,5 років постійно хворіє на піодермію та тричі хворів на пневмонію. В крові знижена
кількість имуноглобулінів G та A. Який вид імунодефіциту виник у дитини?
+Гіпогаммаглобулінемія Брутона

З метою пригнічення аутоімунних реакцій після пересадки органів обов'язковим є проведення курсу
гормонотерапії. Які гормони застосовують з цієї мети?
+Глюкокортикоїди.
#У дитини двох років встановлено діагноз гіпоплазії тимуса. Який показник стану імунної системи є
найбільш характерним для цього імунодефіцита?
+Зниження кількості Т-лімфоцитів
#У новонароджених щурят в експерименті була видалена вилочкова залоза. При цьому розвинулася
хвороба, яка характеризується різким зниженням в крові лімфоцитів, розвитком інфекцій,
спленамегаліей, зупинкою росту і летальним результатом. Яке порушення функції імунної системи
при цьому спостерігається?
+Недостатність системи Т - лімфоцитів

#У хворого 34-х років після перенесеної кишкової інфекції, викликаної сальмонелами, стали згасати
симптоми захворювання. Імуноглобуліни якого класу будуть виявлені в крові хворого в період
реконвалесценції?
+IgG
#У сироватці крові новонародженого виявлено антитіла до вірусу кору. Про наявність якого
імунітету це може свідчити?
+Природний пасивний
#У людини під дією мутагенного фактору з’явилась велика кількість мутантних клітин. Але
більшість з них були розпізнані і знищені клітинами:
+Т-лімфоцитами кілерами

#В дитячому садку проведені планові щеплення вакциною проти кору. Яким методом можна
перевірити формування післявакцинального імунітету?
+Серологічний

#Дитина 6-ти років під час гри порізала ногу осколком скла і була направлена у поліклініку для
введення протиправцевої сироватки. З метою попередження розвитку анафілактичного шоку
лікувальну сироватку вводили за методом Безредка. Який механізм лежить в основі подібного
способу гіпосенсибілізації організму?
+Зв’язування фіксованих на тучних клітинах IgE

#У пацієнтки 23-х років після використання нової губної помади з’явилися набряк і свербіння губ, а
через 2 дні - кірочки на червоній облямівці губ. Який тип алергічної реакції найбільш імовірний?
+Анафілактичний
#У хворого переливання крові ускладнилося розвитком гемотрансфузійного шоку. Назвіть тип
алергічної реакції, що лежить в основі даної патології:
+Цитотоксичний

#У чоловіка 36-ти років після перенесеної стрептококової інфекці діагностовано гострий


гломерулонефрит. Найбільш імовірно, що ураження базальної мембрани ниркових тілець виникає
внаслідок алергічної реакції такого типу:
+Імунокомплексна

#У дівчинки 18-ти років через 5 годин після вживання морепродуктів на шкірі тулуба та дистальних
відділів кінцівок з’явились маленькі сверблячі папули, які частиною зливаються між собою. Через
добу висипка самовільно зникла. Назвіть механізм гіперчутливості, що полягає в основі даних змін:
+Атопія (місцева анафілаксія)

#При підозрі на туберкульоз хворій дитині зробили пробу Манту. Через 24 години у місці введення
алергену з’явились припухлість, гіперемія і болісність. Які основні компоненти визначають цю
реакцію організму?
+Мононуклеари, Т-лімфоцити і лімфокіни

#Який стан може розвинутися через 15- 30 хвилин після повторного введення антигену внаслідок
підвищеного рівня антитіл, переважно IgE, які адсорбуються на поверхні клітин-мішеней -
тканинних базофілів (тучних клітин) та базофілів крові?
+Анафілаксія

#Медсестра зі стажем роботи 10 років захворіла на контактний дерматит верхніх кінцівок. До якого
типу імунної патології відноситься це захворювання?
+Алергічна реакція сповільненого типу

#Дитина 6-ти років знаходиться на стаціонарному лікуванні з діагнозом алергічного риніту. В крові:
зміни в лейкоцитарній формулі. Кількість яких клітин лейкоцитарного ряду може бути збільшена?
+Еозинофіли

#Під час проведення хірургічних маніпуляцій було використано новокаїн з метою знеболення. Через
10 хвилин у хворого з’явилася блідість шкірних покривів, задишка, гіпотензія. Алергічну реакцію
якого типу можна запідозрити?
+Анафілактична
#У дитини 10-ти років поставлено пробу Манту (з туберкуліном). Через 48 годин на місці введення
туберкуліну з'явилася папула розміром до 8 мм у діаметрі. Який тип реакції гіперчутливості
розвинувся після введення туберкуліну?
+Реакція гіперчутливості IV типу

#У хворого виявлена аутоімунна гемолітична анемія, що розвивається за цитотоксичним типом. Які


речовини є антигенами при алергічних реакціях ІІ типу?
+Модифіковані рецептори клітинних мембран

#Після проведення туберкулінової проби (проба Манту) у дитини через 48 годин на місці ведення
туберкуліну утворилася папула до 10 мм у діаметрі. Який механізм гіперчутливості лежить в основі
розвитку вказаних змін?
+Клітинна цитотоксичність
#На 8-й день після введення протиправцевої сироватки з приводу брудної рани стопи у пацієнта
підвищилася температура тіла до 380 , з’явилися біль у суглобах, висипка, свербіж. У крові -
лейкопенія і тромбоцитопенія. Який тип алергічної реакції розвинувся?
+Імунокомплексна

#До лікаря звернулася хвора зі скаргами на нежить, який посилюється навесні в період цвітіння
рослин. Було встановлено діагноз алергійного риніту. Які зміни лейкоцитарної формули можна
очікувати в аналізі крові цієї хворої?
+ Еозинофілія
#Хвора 27-ми років закрапала в очі краплі, до складу яких входить пеніцилін. Через декілька хвилин
з’явився свербіж та печіння тіла, набряк губ та повік, свистячий кашель; став падати артеріальний
тиск. Які імуноглобуліни беруть участь в розвитку даної алергічної реакції?
+I gE та I gG

#Після тижневого застосування нового косметичного засобу у жінки розвинулося запалення повік з
гіперемією, інфільтрацією та болючістю. Алергічна реакція якого типу розвинулася у пацієнтки?
+IV

#Експериментальній тварині, після попередньої сенсибілізації, підшкірно введено дозу антигену. У


місці ін’єкції розвинулось фібринозне запалення з альтерацією стінок судин, основної речовини та
волокнистих структур сполучної тканини у вигляді мукоїдного та фібриноїдного набухання і
некрозу. Яка імунологічна реакція має місце?
+Гіперчутливість негайного типу

#У хворої на дифтерію дитини через10 днів після введення антитоксичної протидифтерійної


сироватки з’явилася висипка на шкірі, яка супроводжува-лася сильним свербежем, підвищенням
температура тіла до 38oC, появою бо-лю у суглобах. Яку причину цих явищ можна припустити?
+Сироваткова хвороба

#У пацієнта з бронхіальною астмою за допомогою шкірних алергічних проб встановлено


сенсибілізацію алергеном тополиного пуху. Який фактор імунної системи відіграє вирішальну роль в
розвитку цього імунопатологічного стану?
+IgE
#У чоловіка 36-ти років після перенесеної стрептококової інфекції діагностовано гострий
гломерулонефрит. Найбільш імовірно, що ураження базальної мембрани ниркових тілець виникає
внаслідок алергічної реакції такого типу:
+Імунокомплексна

#При розтині хворої 28-ми років, що померла від уремії, виявлені збільшені строкаті нирки з
осередками крововиливів. Патогістологічно в судинних клубочках виявлені гематоксилінові тільця,
капілярні мембрани клубочків у вигляді дротяних петель, гіалінові тромби та осередки
фібриноїдного некрозу. За патогенезом гіперчутливість якого типу лежить в основі описаної
хвороби?
+Гіперчутливість III типу (імунокомплексна)
#Хвора 37-ми років померла під час нападу експіраторної задухи, що був спричинений контактом з
екзогенним алергеном (пилок амброзії). При гістологічному дослідженні в просвіті бронхів
спостерігаються скупчення слизу, в стінці бронхів багато тучних клітин (лаброцитів), більшість з
яких у стані дегрануляції, багато еозинофілів. До патогенезу якого типу реакцій гіперчутливості
можна віднести описані зміни?
+I типу (анафілактична)

#У хворого діагностовано тиротоксикоз. У крові знайдено антитиреоїдні антитіла. Який тип


алергічної реакції за Кумбсом і Джелом спостерігається при розвитку цього захворювання?
+Стимулюючий

#У хворого через 9 діб після введення лікувальної сироватки зўявилася кропивўянка, зуд шкіри,
набряк її та слизових оболонок, припухання лімфатичних вузлів. Яке захворювання розвинулось?
+Сироваткова хвороба.

#Жінка 27 років звернулась зі скаргами на свербіння та печію в очах, сльозотечію, чхання, виділення
з носу. Симптоми з'явились після поїздки за місто влітку. Діагностовано поліноз. Якого типу
алергічна реакція розвилась при наявному захворюванні?
+анафілактична

#У жінки 45 років, в період цвітіння трав з'явилось гостре запальне захворювання верхніх дихальних
шляхів та очей: гіперемія, набряк, слизове виділення. Який вид лейкоцитозу буде найбільш
характерним при цьому?
+Еозінофілія.
#У студентки, 20 років, третю весну підряд з початком цвітіння тополі починається свербіння і
гіперемія очей, носа, ринорея, покашлювання, дрібне уртикарне висипання відкритих ділянок тіла.
При обстеженні виявлено різке збільшення рівня Ig E. Алергологом призначена специфічна
гіпосенсибілізуюча терапія. Вкажіть, яким чином проводять дане лікування?
+Багаторазовим введенням малих доз алергену

#У хворого А. в серпні після праці на дачі развився стан, що характеризувався лікарем, як стан
підвищенної і якісно зміненої реакції на надходження до організму сполук антигенної або гаптенної
природи. Який з перерахованих станів найбільш підходить під описану лікарем характеристику?
+алергія

#Хворому М. з метою введення лікувальної дози протиправцевої сироватки було зроблено пробу на
чутливість, яка виявилась позитивною. Як найбільш правильно провести специфічну
гіпосенсибілізацію хворому? Введенням:
+малих доз протиправцевої сироватки
#Через 5-8 днів після застосування значних кількостей лікувальної сироватки у хворого було
виявлено шкірні висипання, свербіж, припухлість, біль в суглобах, підвищилася температура тіла, в
сечі з'явився білок. Був поставлений діагноз сироваткова хвороба Що є важливим фактором у
патогенезі цього синдрому?
+накопичення в крові циркулюючих імунних комплексів.

#Після прийому амідопірину у хворого виникла лейкопенія. В крові знайдені антилейкоцитарні


антитіла. Який тип алергічної реакції за Кумбсом і Джеллом виник у даному випадку?
+Цитотоксичний.

#У мишей з відсутнім волосяним покривом (тобто nude - голі) не було клітинних реакцій
уповільненого типу. Для цієї патології найбільш вірогідним є:
+Відсутність вилочкової залози

#Масугі викликав розвиток гломерулонефриту у щурів таким чином: гомогенат нирок щура вводив
кролю. Через декілька тижнів сироватку сенсибілізованого кроля вводив щурам. Який тип алергічної
реакциї за Джеллом та Кумбсом лежить в основі розвитку гломерулонефриту у щурів?
+Цитотоксичний
#Чоловіку 30 років було травмовано праве яєчко. Через 6 тижнів, коли запальний процес яєчка
закінчився, з’явився біль у лівому яєчку. Об’єктивно: шкіра над яєчком червоного кольору,
припухла, яєчко болісне на дотик, щільне. Який процес лежить в основі цього пошкодження?
+Аутоалергія.
#Жінку 44-х років втяла оса, внаслідок чого розвинувся шок. В анамнезі - тяжка алергічна реакція на
укус оси. Об’єктивно: РS- 179/хв, слабкий, АТ- 80/40 мм рт.ст., ЧД-26/хв. Яка провідна ланка
патогенезу анафілактичного шоку?
+Зниження периферійного опору судин

#В основі розвитку імунних і алергічних реакцій організмом застосовуються однакові механізми


відповіді імунної системи на антиген. Визначте основну відмінність алергічних реакцій від імунних:
+Розвиток пошкодження тканин

#При розтині хворої 28-ми років, що померла від уремії, виявлені збільшені строкаті нирки з
осередками крововиливів. Патогістологічно в судинних клубочках виявлені гематоксилінові тільця,
капілярні мембрани клубочків у вигляді дротяних петель, гіалінові тромби та осередки
фібриноїдного некрозу. За патогенезом гіперчутливість якого типу лежить в основі описаної
хвороби?
+Гіперчутливість III типу (імунокомплексна)

#Хвора 37-ми років померла під час нападу експіраторної задухи, що був спричинений контактом з
екзогенним алергеном (пилок амброзії). При гістологічному дослідженні в просвіті бронхів
спостерігаються скупчення слизу, в стінці бронхів багато тучних клітин (ла-броцитів), більшість з
яких у стані де-грануляції, багато еозинофілів. До патогенезу якого типу реакцій гіперчу-тливості
можна віднести описані зміни?
+I типу (анафілактична)

#Ліквідатору наслідків аварії на Чорнобильській АЕС, що отримав велику дозу опромінення,


проведено трансплантацію кісткового мозку. Через деякий час після проведеної операції у пацієнта
діагностовано розвиток реакції "трансплантат проти хазяїна’. Які антигени стали пусковим
механізмом виникнення цієї реакції?
+Антигени системи HLA-клітин організму ліквідатора

#В підводному човні під час занурення порушилася система подачі кисню. У підводників
збільшилися частота дихання і серцевих скорочень. Який вид гіпоксії розвинувся у підводників?
+Гіпоксична
#Вагітна жінка під час пологів втратила близько 800 мл крові. Спостерігається тахікардія,
артеріальний тиск 100/70 мм рт.ст., тахіпное до 28/хв. Який тип гіпоксії розвивається первинно в
такій клінічній ситуації?
+Кров’яна

#Робочий комунальної служби спустився в каналізаційний колодязь без засобів захисту і через
деякий час знепритомнів. Лікарями швидкої допомоги діагностовано отруєння сірководнем. Який
вид гіпоксії при цьому розвинувся?
+Гемічний
#Після ремонту автомобіля в закритому приміщенні при працюючому двигуні у чоловіка з'явилися
задишка, запаморочення, акроціаноз, частота дихання 24-26/хв. Газовий склад крові: pO2- 60 мм
рт.ст., pCO2- 30 мм рт.ст.; у крові наявний карбоксигемоглобін. Про який вид гіпоксії можна думати?
+Гемічна

#Для людини існує суворе обмеження в часі перебування на висоті понад 800 метрів над рівнем моря
без кисневих балонів. Що є лімітуючим фактором для життя в даному випадку?
+Парціальний тиск кисню в повітрі

#Чоловік 65 років страждає хронічною серцевою недостатністю по лівошлуночковому типу.


Об'єктивно: ціаноз, задишка, кашель з харкотинням, періодичні напади задухи. Який тип гіпоксії
первинно виник у хворого ?
+циркуляторна застійна

#Чоловік 36 років скаржиться на кашель з виділенням харкотиння, задишку, головний біль, загальну
слабкість. Захворів після сильного переохолодження. При огляді: шкіра бліда, температура тіла 38 оС.
Пульс- 91/хв., АТ-125/6 мм рт.ст. В аналізі крові - нейтрофільний лейкоцитоз. Поставлений -
діагноз: вогнищева пневмонія. Який тип гіпоксії має місце у хворого?
+дихальна

#Чоловік приблизно 50 років, винесений в непритомному стані з закритого помешкання, повного


диму від пожежі. Який вид гіпоксії виник у постраждавшого?
+гемічна

#У лікарню був доставлений водій, який після роботи заснув в машині з


працюючим двигуном. Прокинувшись він відчув головний біль, почалася
блювота. Що стало причиною симптомів?
+карбоксигемоглобин

#При підйомі на "висоту" у барокамері у щура з'явилося часте дихання, тахікардія, зниження напруги
рО2 в крові. Яка форма гіпоксії має місце в даному випадку?
+Гіпоксична

#Після аварії на хімічному виробництві відбулося забруднення навколишнього середовища


нітросполуками. У частини людей, які проживають в цій місцевості, з'явилась різка слабкість,
головний біль, задишка, запаморочення. Який механізм розвитку даної форми гіпоксії?
+Збільшення утворення метгемоглобіну

#У хворого внаслідок отруєння бертолетовою сіллю розвинулася гемічна гіпоксія. Утворення якої
речовини грае роль в патогенезі цієї гіпоксиї?
+Метгемоглобіну

#Хворий В. 38 років, доставлений в приймальне відділення з ознаками гіпоксії розвилася після


отруєння чадним газом. Стан середньої тяжкості, тахікардія, задишка, АТ 160/100. Який механізм
токсичної дії окису вуглецю на організм?
+Утворення карбоксигемоглобіну

#У хворого під час нападу бронхіальної астми при визначенні рСО2 в крові
виявлено наявність гіперкапнії, при визначенні РО2 гіпоксемії Який вид гіпоксії спостерігається в
даному випадку?
+Дихальна

#У хворого з варикозним розширенням вен під час огляду нижніх кінцівок відзначається: ціаноз,
пастозність, зниження температури шкіри, поодинокі петехії. Який розлад гемодинаміки має місце у
хворого?
+Венозна гіперемія

#У чоловіка 48-ми років виявлено порушення периферичного кровообігу з обмеженням припливу


артеріальної крові, при цьому має місце зблідніння даної ділянки, зниження місцевої температури.
Це порушення називається:
+Ішемія

#У жінки 60-ти років після емоційної реакції, яка була викликана гнівом, виник напад загрудинного
болю. На ЕКГ були встановлені ознаки порушення коронарного кровообігу. Який вид порушень міг
спричинити це явище?
+Ангіоспастична ішемія

#Студент на екзамені не зміг вірно відповісти на питання екзаменаційного білету, що


супроводжувалося почервонінням шкіри обличчя, відчуттям жару і невпевненістю поведінки. Який
вид артеріальної гіперемії розвинувся у даному випадку?
+Нейротонічна
#Під час гри у волейбол спортсмен після стрибка приземлився на зовнішній край стопи. Виник
гострий біль у гомілковостопному суглобі, активні рухи в ньому обмежені, пасивні - в повному
обсязі, але болісні. Потім розвинулася припухлість у ділянці зовнішньої щиколотки, шкіра
почервоніла, стала теплішою на дотик. Який вид розладу периферичного кровообігу розвинувся в
даному випадку?
+Артеріальна гіперемія

#У кроля перерізалі нерв, что іннервує праві вухо, і Відаль правий верхній шийно симпатичний
вузол. Одразу після операції провели вимірювання температури шкіри вух. Виявило, что температура
шкіри вуха кролика на боці денервації на 1,5 C вища, ніж на протилежних інтактному боці. Що з
наведеного є найбільш вірогідною причиною вказаних явищ?
+Артеріальна гіперемія нейропаралітічного типу

#У тварини через 2 тіжні після експериментально звуження ниркової артерії підвищився


артеріальний тиск. Зі збільшення дії на судину якого фактора гуморальної регуляції це пов'язано?
+ангіотензин II

#У пілота на висоті 14000 м трапилася аварійна розгерметизація кабіни. Який із видів емболій у
нього розвинувся?
+Газова.

#Після вимушеного швидкого підняття водолаза з глибини на поверхню у нього з’явилися ознаки
кесонної хвороби – біль у суглобах, свербіння шкіри, мерехтіння в очах, затьмарення свідомості.
Яким видом ємболії вони були зумовлені?
+Газовою

#У хворого з переломом гомілковостопного суглоба після зняття гіпсової пов'язки спостерігається


набряк стопи, ціаноз, місцеве зниження температури, збільшення органу в обсязі. Який вид
порушення кровообігу спостерігається при цьому?
+ Венозна гіперемія

#Під час роботи лікарю - стоматологу доводиться довго стояти на ногах, що може призвести до
застою крові у венах нижніх кінцівок та їх варикозного розширення. З порушенням якого механізму
венозного припливу крові до серця це пов’язано?
+Відсутність скорочення скелетних м’язів

#У дитини 5-ти років розвинулось гостре респіраторне захворювання, яке супроводжувалось кашлем,
виділенням значної кількості слизу із носа. Який тип запалення у хворої дитини?
+Катаральне

#Запалення характеризується розширенням кровоносних судин на ділянці пошкодження,


зменшенням кровообігу, підвищенням проникливості стінки судин. Яким з нижче наведених клітин
належить головна роль в цьому?
+Тканинні базофіли

#У плазмі крові здорової людини знаходиться декілька десятків білків. При захворюванні організму
з’являються нові білки, зокрема "білок гострої фази". Таким білком є:
+С-реактивний білок

#При моделюванні запалення нижньої кінцівки у тварини підвищилася температура тіла, збільшився
вміст антитіл та лейкоцитів у крові. Які речовини обумовили розвиток цих загальних реакцій
організму при запаленні?
+Інтерлейкіни

#Юнак 17-ти років захворів гостро, температура тіла підвищилася до 38, 50 , з’явилися кашель,
нежить, сльозотеча, виділення з носу. Яке запалення розвинулося у юнака?
+Катаральне
#При запаленні ока у хворого відмічалося накопичення мутної рідини з високим вмістом білку на дні
передньої камери, яке отримало назву - гіпопіон. Який процес лежить в основі зазначених змін?
+Порушення мікроциркуляції

#У відпочиваючого в санаторії у результаті сонячного опіку на шкірі спини утворилися міхурці,


заповнені світлою рідиною, оточені зоною гіпермії, болісні. Який з перерахованих механізмів лежить
в основі формування ексудації у вогнищі запалення?
+Збільшення колоїдно-осмотичного тиску в тканині

#Чоловік 38 років поступив у терапевтичне відділення з діагнозом: правосторонній ексудативний


плевріт. Відкачана з плевральної порожнини грудної клітки рідина прозора, має відносну щільність
1.020; містить 55 г/л, білка альбуміно-глобуліновий коефіцієнт-1.6; загальна кількість клітин - 2.8. в
1 мкл; pH - 6.5. Який тип ексудату має місце у хворого?
+серозний

#Відповідно до фізико-хімічної теорії Шаде в зоні запалення має місце: гіперосмія, гіперонкія,
ацидоз. Розвиток гіперосмії, в деякій мірі, пов'язаний із зростанням концентрації К + в зоні запалення.
Вказати причини гіперкалійіонії в запальному ексудаті
+Інтенсивна деструкція пошкоджених клітин

#У хворого через добу після травми розпух колінний суглоб. При його пункції отримано 30 мл
рідини рожевого кольору з питомою щільністю 1020. Загальний вміст білка в ній - 3%, альбуміну -
0,3%, глобулінів 2%, фібриногену - 0,7%, лейкоцитів - 1-3, еритроцитів - 15-20, місцями до 50 в полі
зору. Якого характеру ексудат отриманий при пункції колінного суглоба у хворого?
+Геморагічний

#У хворого плевритом в плевральній порожнині виявлена смердюча рідина, яка містить в собі
біогенні аміни, гази. Який різновид запалення в даному випадку?
+Гнилісне
#При дослідженні запалення піддослідній тварині ввели смертельну дозу правцевого токсину в
порожнину абсцесу, індукованого скипидаром. Але піддослідна тварина не загинула. Вкажіть
найбільш ймовірну причину такого результату досліду?
+Формування бар'єру навколо запалення
#У дитини 6 років розвинулася гіперергічна форма запалення верхніх дихальних шляхів. З’явилася
загорза серйозного порушення дихання, а тому виникла необхідність застосувати протизапальні
гормони. Серед гормонів протизапальний ефект проявляє
+Кортизол

#При моделюванні запалення на брижі жаби спостерігали крайове стояння лейкоцитів та їх еміграцію
крізь судинну стінку. Який із перелічених факторів обумовлює цєй процес?
+Вплив хемотаксичних речовин

#Під час огляду шкіри лікар помітив у хворого нагнійний процес у вигляді круглих підвищень
червонуватого коліру, оточених зоною гіперемії. Які медиатори запалення зумовили явища судинної
гіперемії?
+Гістамін

#При мікроскопії препарату брижі жаби виявлено, що в деяких капілярах відзначається


маятникоподібний рух крові, формені елементи при цьому (зокрема, лейкоцити) з осьового шару
виходять в пристінковий, а деякі навіть випускають псевдоподии в стінку капілярів. Який стадії
судинної реакції при запаленні відповідає описане явище?
+Престаз

#Кухар в результаті необачності обпік руку парою. Підвищення концентрації якої речовини
викликало почервоніння, набряклість та болючість ураженої ділянки шкіри?
+Гістамін
#У хворого з флегмоною передпліччя при мікробіологічному аналізі ексудату в зоні запалення
визначена присутність стрептококів. Які клітини будуть переважати в ексудаті?
+Нейтрофільні гранулоцити

#Autopsy of the body shows that the soft meninges of the deceased individual are plethoric, thickened,
opaque, and yellow-green colored. What type of exudative inflammation can be characterized by such
changes in the soft meninges?
+Suppurative

#У людини виявлена пухлина одного з відділів головного мозку, внаслідок чого в неї порушена
здатність підтримувати нормальну температуру тіла. Яка структура головного мозку пошкоджена?
+Гіпоталамус
#У хворого діагностовано септичний ендокардит. Температура тіла протягом 5-ти днів коливалася в
межах 39,5oC - 40,2oC. На 6-й день на тлі різкого зниження температури до 35,2oC розвинувся
колапс. Який головний механізм колапсу?
+Вазодилатація

#У хворого 26-ти років вдень раптово підвищилася температура до 39,5°С і через 6 годин
повернулася до норми. На другу добу напад повторився і температура досягла 41,5°С. Період
апірексії настав через 8 годин. Який тип температурної кривої?
+Переміжний

#В експерименті на кролику введення пірогеналу призвело до підвищення у тварини температури


тіла. Яка з перерахованих речовин відіграє роль вторинного пірогену, що бере участь у механізмі
виникнення лихоманкової реакції?
+Інтерлейкін-1

#У хворих на поворотний тиф виникає лихоманка, яка характеризується кількаденними періодами


високої гарячки, що чергується з періодами нормальної температури. Така температурна крива
називається:
+Febris recurrns

#У пацієнта після переливання 200 мл крові підвищилася температура тіла до 37,9 oC. Яка з наведених
речовин найбільш вірогідно призвела до підвищення температури?
+Інтерлейкін-1

#У хворого вдень піднялася температура тіла до 390 і через 6 годин повернулася до норми. На другу
добу напад повторився: в період пароксизму температура досягла 410 , період апірексії настав через
8 годин. Як називається такий тип температурної кривої?
+Інтермітуючий

#У хворого із запаленням легень спостерігається підвищення температури тіла. Яка біологічно


активна речовина відіграє провідну роль у виникненні цього прояву?
+Інтерлейкін-I
#При обстеженні хворого виявлені наступні клінічні прояви: шкірні покриви рожеві, теплі на дотик,
сухі, ЧСС-92/хв.,ЧДР- 22/хв., температура тіла - 39, 2oC. Яке співвідношення процесів утворення і
віддачі тепла в описаному періоді лихоманки?
+Теплопродукція дорівнює тепловіддачі

#У людини у якої лихоманка спостерігається збліднення шкірних покривів, "гусяча шкіра",озноб,


тахікардія. Який стадії лихоманки відповідає даний стан?
+Стадії підйому температури

#У хворого який довго лихоманить ранкова температура тіла була в межах 36,4-36,9оС. До вечора
вона піднялася до 37,0-38,0оС, в деякі дні до 38,8оС. Хворий температурить більше 2-х місяців. Який
тип лихоманки у хворого?
+постійний

#Чоловік 25 років скаржиться на загальну слабкість, озноб, біль у горлі. Об'єктивно: почервоніння в
області мигдаликів. Температура тіла 38.6оС. Які з перелічених клітин є головним джерелом
ендогенних пірогенів, що викликають гарячку у хворого?
+нейтрофіли

#При обстеженні температурить хворого виявлені наступні об'єктивні дані: шкірні покриви
гіперемійовані, вологі на дотик, спостерігається поліурія, полидипсия, температура тіла-37,2oС. Який
стадії лихоманки відповідає даний стан?
+Падіння температури

#У довго лихоманить хворого температура вранці була в межах 36,4-36,9oС, до вечора вона
піднімалася до 37,0-38,0oC. Який тип лихоманки за ступенем підйому температури спостерігається у
хворого?
+субфебрильна

#У чоловіка 35-ти років розвинулася гарячка, яка супроводжувалася зміщенням установчої точки
терморегуляційного центру на більш високий рівень, з послідовним чергуванням наступних стадій:
incrementi, fastigii, dеcrеmenti. При якому захворюванні можуть спостерігатися подібні зміни?
+Гостра пневмонія

#У хворого Н. приступи гарячки виникають через день. Під час приступу температура різко
підвищується і утримується на високому рівні до 2 год, а потім знижується до вихідного рівня. Цей
тип гарячки характерний для
+Малярії

#У хворого на пневмонію виникла гарячка. Що безпосередньо спричинює зміни установочної


точки температури в нейронах гіпоталамуса цього хворого?
+Простагландини Е1, Е2

#У жінки встановлено діагноз - рак шийки матки. З яким вірусом може бути асоційована ця
патологія?
+Вірус простого герпеса тип 2

#У чоловіка 63-х років рак стравоходу, метастази у лімфатичні вузли середостіння, ракова кахексія.
Яка патогенетична стадія пухлинного процесу має місце?
+Прогресії

#Чоловік 58 років, страждає раком сечового міхура. У процесі трудової діяльності мав контакт з
канцерогенними речовинами. Дія якого з нижче перелічених канцерогенів найбільш вірогідна у
данному випадку?
+b – нафтіламін

#Проводиться медичний огляд робітників цеху по виробництву анілінових барвників. Наявність


пухлини якої локалізації може бути оцінене як професійне захворювання, внаслідок контакту з бета-
нафтіламіном?
+сечового міхура

#Жінці поставлено діагноз ерозія шійки матки, яка є передпухлинною патологією. Який захисний
механізм може попередити розвиток пухлини?
+Збільшення природних кілерів (NK-клітин)

#Жінка 67 років, страждає раком шлунку з метастазами у печінку. Яка властивість пухлинних клітин
обумовлює їх здатність до метастазування?
+інфільтративний ріст
#Клінічне обстеження хворого дозволило встановити попередній діагноз рак печінки. Наявність
якого білка в сироватці крові дозволить підтвердити діагноз?
+альфа-фетопротеїн

#Встановлено, що при розвитку гепатоми в ній часто припиняється синтез жовчних кислот. Про який
вид анаплазии це свідчить?
+Функціональної

#Хворому з ревматоїдним артритом тривалий час вводили гідрокортизон. У нього з’явилися


гіперглікемія, поліурія, глюкозурія, спрага. Ці ускладнення лікування є наслідком активації процесу:
+Глюконеогенез

#При диспансерному обстеженні у хворого знайдено цукор в сечі. Який найбільш імовірний механізм
виявлених змін, якщо вміст цукру в крові нормальний?
+Порушення реабсорбції глюкози в канальцях нефрона

#У хворого 15-ти років концентрація глюкози натще - 4,8 ммоль/л, через годину після цукрового
навантаження - 9,0 ммоль/л, через 2 години - 7,0 ммоль/л, через 3 години - 4,8 ммоль/л. Ці показники
характерні для такого захворювання:
+Прихований цукровий діабет

#У хлопчика 2-х років спостерігається збільшення в розмірах печінки та селезінки, катаракта. В крові
підвищена концентрація цукру, але тест толерантності до глюкози в нормі. Спадкове порушення
обміну якої речовини є причиною цього стану?
+Галактоза

#У студента, який складає іспит, вміст глюкози у плазмі крові складає 8 ммоль/л. Збільшена секреція
якого з наведених гормонів сприяє розвитку гіперглікемії у студента?
+Глюкагон

#Хвора 46-ти років скаржиться на сухість в роті, спрагу, почащений сечопуск, загальну слабкість. У
крові: гіперглікемія, гіперкетонемія. У сечі: глюкоза, кетонові тіла. На ЕКГ: дифузні зміни в міокарді.
Який найбільш імовірний діагноз?
+Цукровий діабет

#Хвора 38-ми років надійшла до реанімаційного відділення в несвідомому стані. Рефлекси відсутні.
Цукор крові - 2,1 ммоль/л. В анамнезі - цукровий діабет з 18-ти років. Яка кома має місце у хворої?
+Гіпоглікемічна
#У новонародженого спостерігається диспепсія після годування молоком. При заміні молока
розчином глюкози симптоми диспепсії зникають. Недостатня активність якого ферменту
спостерігається у новонародженого?
+Лактаза

#У жінки 62-х років розвинулася катаракта (помутніння кришталика) на фоні цукрового діабету.
Який тип модифікації білків має місце при діабетичній катаракті?
+Глікозилювання

#Після тривалого фізичного навантаження під час заняття з фізичної культури у студентів
розвинулась м’язова крепатура. Причиною її виникнення стало накопичення у скелетних м’язах
молочної кислоти. Вона утворилась після активації в організмі студентів:
+Гліколізу

#В експерименті на кролі встановлено, що об’єм кисню, який спожива-ється головним мозком за 1


хвилину, дорівнює об’єму C O2, який виділяється клітинами мозку в кров. Це свідчить, що у
клітинах головного мозку має місце:
+Окислення вуглеводів

#При обстеженні жінки 56-ти років, що хвора на цукровий діабет 1-го типу, виявлене порушення
білкового обміну, що при лабораторному дослідженні крові проявляється аміноацидемією, а клінічно
- уповільненням загоєння ран і зменшенням синтезу антитіл. Який з перерахованих механізмів
викликає розвиток аміноацидемії?
+Підвищення протеолізу

#Молода людина 25-ти років споживає надмірну кількість вуглеводів (600 г на добу), що перевищує
її енергетичні потреби. Який процес буде активуватися в організмі людини у даному випадку?
+Ліпогенез

#Чоловік 48 років, страждає на цукровий діабет з 6 років, госпіталізований у непритомному стані,


який наступив після підвищеного фізичного навантаження. Об'єктивно: шкіра волога, тонус м'язів
кінцівок підвищений сухожильні рефлекси підвищені, дихання поверхневе, пульс 78 уд/хв., АТ-95/60
мм. рт. ст., рівень глюкози у крові 1.88 ммоль/л. Для якого із перелічених станів найбільш характерні
симптоми у чоловіка?
+гіпоглікемічної коми

#У хворого визначається наявність гіперглікемії, поліурії, гіперстенурія і глюкозурія. Для якої


нозологічної форми патології обміну характерно таке поєднання цих показників?
+Цукрового діабету

#Хворий А., 18 років, після перенесеної краснухи почав худнути, постійно відчував сухість в роті,
спрагу, у нього підвищився апетит, почалось часте сечовиділення. Обўєктивно: добова кількість сечі
6 л, глюкоза крові 17 ммоль/л, в сечі виявлена глюкоза та ацетон.
Яке захворювання виникло у хворого?
+ІЗЦД (інсулінзалежний цукровий діабет).

#У хворого на цукровий діабет знижені процеси регенерації, довго не загоюються рани. З якими
змінами в обміні речовин це пов язане?
+Пригніченням протеосинтезу.
#У хворого на цукровий діабет виникла діабетична кома, яка характеризувалася повною втратою
свідомості, арефлексія, гіпотензією, падінням температури тіла, великим гучним диханням. Як
називається такий тип дихання?
+Куссмауля

#У хворого А., 18 років після перенесеної краснухи виникла втрата маси тіла, постійне відчуття
спраги, підвищення апетиту. Об'єктивно: добова кількість сечі 6 л, глюкоза крові 17,8 ммоль/л, в сечі
виявлено глюкозу та ацетон. Яку патологію можливо запідозрити у хворого?
+Інсулінзалежний цукровий діабет

#У відділення реанімації доставлено непритомного пацієнта із запахом ацетону з ротової порожнини.


Методи експрес-аналізу виявили у крові 17,3 ммоль/л глюкози. Підвищення вмісту яких речовин
призвело до втрати свідомості?
+Кетонові тіла

#У хворий довго хворіє на цукровий діабет 2-го типу, після порушення дієти (вживання в їжу
легкозасвоюваних вуглеводів) поступово наростала загальна слабкість, знизився артеріальний тиск,
з'явилися галюцинації, судоми. Шкіра суха, чіткі прояви дегідратації організму. Рівень глюкози крові
40 ммоль / л.
Який вид коми є причиною погіршення стану хворої?
+Гіперосмолярна
#У хворої А., 18 років, після перенесеної краснухи початку відзначатися втрата маси тіла, постійне
відчуття сухості в роті, спрагу, підвищення апетиту, часте сечовиділення. Об'єктивно: добова
кількість сечі 6 л, глюкоза крові 17,8 ммоль / л, в сечі виявлено глюкоза і ацетон. Який найбільш
ймовірний патогенетичний механізм викликав підвищення рівня глюкози у хворої?
+Зменшення вироблення інсуліну

#У хлопчика М., 2 років була діагностована хвороба Гірке, що супроводжується надмірним


відкладанням глікогену в печінці і нирках, гіпоглікемією. При біохімічному дослідженні крові
виявлено:
+Дефіцит глюкозо-6-фосфатази
#У піддослідної тварини (щури) шляхом внутрішньовенного введення аллоксона був викликаний
експериментальний цукровий діабет. Який механізм дії даної речовини?
+Пошкодження beta - клітин панкреатиччних острівців
#У хворого на ранній стадії цукрового діабету спостерігається поліурія. Чим вона викликана?
+Гіперглікемією

#Внаслідок тривалого голодування в організмі людини швидко зникають резерви вуглеводів. Який з
процесів метаболізму за цих умов поновлює вміст глюкози в крові?
+Глюконеогенез

#Аналіз крові хворого на цукровий діабет показав наявність молочної кислоти у концентрації 2,5
ммоль/л. Яка кома розвинулася у хворого?
+Лактацидемічна

#У хворого струс головного мозку,що супроводжується повторним блюванням і задишкою. При


обстеженні відзначено: рН - 7,62; pCO2 - 40 мм рт.ст. Яке порушення кислотно-основного стану є у
хворого?
+Негазовий алкалоз

#У чоловіка 32-х років, хворого на пневмонію, спостерігається закупорка харкотинням дихальних


шляхів. В організмі хворого при цьому буде розвиватися така зміна кислотно-лужної рівноваги:
+Респіраторний ацидоз
#У альпініста, що піднявся на висоту 5200м, розвинувся газовий алкалоз.Що є причиною його
розвитку?
+Гіпервентиляція легенів
#У хворого з дихальною недостатністю рН крові 7,35. Визначення pCO2 показало наявність
гіперкапнії. При дослідженні рН сечі відзначається підвищення її кислотності. Яка форма порушення
кислотно-основного стану в даному випадку?
+Ацидоз газовий, компенсований
#У пацієнта у результаті тривалого блювання відбувається значна втрата шлункового соку, що є
причиною порушення кислотно-лужного стану в організмі. Яка з перерахованих форм порушення
кислотно-лужного стану має місце?
+Негазовий алкалоз

#У хворого 40-ка років ознаки гірської хвороби: запаморочення, задишка, тахікардія, рН крові - 7,50,
pCO2- 30 мм рт.ст., зсув буферних основ +4 ммоль/л. Яке порушення кислотно-основногостану має
місце?
+Газовий алкалоз

#Чоловік 53-х років доставлений у стаціонар у непритомному стані. Об’єктивно: шкіра суха, дихання
часте поверхневе, запах ацетону відсутній, Ps- 126/хв., АТ- 70/40 мм рт.ст. Вміст глюкози у крові 48
ммоль/л, реакція сечі на ацетон негативна. Для якого із перелічених станів найбільш характерні
симптоми у хворого?
+Гіперосмолярна кома

#У хворого з дихальною недостатністю рН крові 7,35. Визначення рС02 показало наявність


гіперкапнїї. При дослідженні рН сечі відзначається підвищення її кислотності. Яка форма порушення
кислотно- основного стану в даному випадку?
+Ацидоз газовий, компенсований

#У хворого після вживання недоброякісної їжі розвинулася діарея. На наступний день у нього
знизився артеріальний тиск, з'явились тахікардія, екстрасистолія. pH крові складає 7,18. Ці
порушення є наслідком розвитку:
+Негазового ацидозу

#У немовля з пілороспазмом внаслідок блювання, що часто повторювалося, з’явилися слабкість,


гіподинамія, іноді судоми. Яка форма порушення кислотно-основного стану в нього спостерігається?
+Видільний алкалоз

#У немовляти внаслідок неправильного годування виникла виражена діарея. Одним з основних


наслідків діареї є екскреція великої кількості бікарбонату натрію. Яка форма порушення кислотно-
лужного балансу має місце в цьому випадку?
+Метаболічний ацидоз
#У клініку доставлений чоловік 30 років, з профузним поносом тривалістю 12 годин. Блювоти не
було. Які зміни водно-електролітного балансу та кислотно-основної рівноваги спостерігаються у
чоловіка?
+негазовий ацидоз з дегідратацією

#У людини з хронічним гломерулонефритом наростає загальна слабкість, різка тахікардія з


періодичної аритмією, загальмованість і сонливість. Який зрушення КОС супроводжує наближення
уремічний коми?
+Негазовий видільної ацидоз

#При обстеженні хворого визначається наявність гіперглікемії, кетонурія, поліурії, гіперстенурія і


глюкозурії. Яка форма порушення кислотно-лужної рівноваги
має місце в даній ситуації?
+Метаболічний ацидоз

#При підйомі в гори у альпініста розвинулась ейфорія, яка замінилася головним болем,
запамороченням, серцебиттям, задишкою, що перейшла в апное. Яке порушення кислотно-лужної
рівноваги розвинулось в даному випадку?
+Газовий алкалоз.

#У хворого на цукровий діабет розвинулася діабетична кома внаслідок порушення кислотно-


основного стану. Який вид порушення виник при цьому?
+Метаболічний ацидоз

#У вагітної жінки розвинувся токсикоз з тяжким повторним блюванням шлунковим вмістом


протягом доби. Наприкінці доби почали проявлятися тетанічні судоми та зневоднення організму.
Який розлад кислотно-лужної рівноваги викликав дані зміни?
+Негазовий видільний алкалоз
#У вагітної жінки 24-х років після тривалого блювання було зареєстровано зниження об’єму
циркулюючої крові. Про яку зміну загальної кількості крові може йти мова?
+Поліцитемічна гіповолемія

#У хворого 41-го року відзначається гіпонатріємія, гіперкаліємія, дегідратація, зниження


артеріального тиску, м’язова слабкість, брадикардія, аритмія. З порушенням функцій яких гормонів
це пов’язано?
+Кортикостероїди
#Хворий на гіпертонічну хворобу разом з безсольовою дієтою та з антигіпертензивними засобами,
довгий час приймав гідрохлортіазид, що зумовило порушення електролітного балансу. Яке
порушення внутрішнього середовища виникло у хворого?
+Гіпохлоремічний алкалоз

#При лабораторному дослідженні крові пацієнта виявлено, що вміст білків у плазмі становить 40г/л.
Як це впливає на транс капілярний обмін води в мікроциркуляторному руслі?
+Збільшується фільтрація, зменшується реабсорбція
#У людини з масою 80 кг після тривалого фізичного навантаження об’єм циркулюючої крові
зменшився, гематокрит - 50%, загальний білок крові - 80 г/л. Такі показники крові є наслідком, перш
за все:
+Втрати води з потом

#У дорослої людини за добу виділяється 20 л сечі з низькою відносною щільністю. Найбільш


імовірною причиною цього є дефіцит в організмі:
+Вазопресину
#У тварини через 2 тижні після експериментального звуження ниркової артерії підвищився
артеріальний тиск. Зі збільшенням дії на судини якого фактора гуморальної регуляції це пов'язано?
+Ангіотензин ІІ

#У людини зменшений діурез, гіпернатріємія, гіпокаліємія. Гіперсекреція якого гормону може бути
причиною таких змін?
+Альдостерон
#При токсичному ушкодженні клітин печінки з порушенням її функцій у хворого з'явилися набряки.
Які зміни складу плазми крові є провідною причиною розвитку набряків?
+Зниження вмісту альбумінів
#У людини збільшений вміст іонів кальцію в плазмі крові, зменшений - у кістках. Надмірна секреція
якого гормону може спричинити такі зміни?
+Паратгормон

#Тварині внутрішньовенно ввели концентрований розчин хлориду натрію, що зумовило зниження


реабсорбції іонів натрію у канальцях нирок. Внаслідок яких змін секреції гормонів це відбувається?
+Зменшення альдостерону
#Внаслідок вираженого зниження концентрації кальцію в плазмі крові у дитини 2-х років виникли
тетанічні скорочення дихальних і глоткових м'язів. Зниження секреції якого гормону може бути
причиною цього?
+Паратгормон

#У хворого на ентерит, що супроводжувався значною діареєю, спостерігається зменшення кількості


води в позаклітинному просторі, збільшення її всередині клітин та зниження осмолярності крові. Як
називають таке порушення водно- електролітного обміну?
+Гіпоосмолярна гіпогідратація
#Хворий 50-ти років звернувся до клініки зі скаргами на загальну слабкість, втрату апетиту, аритмію
серця. Спостерігається гіпотонія м'язів, мляві паралічі, послаблення перистальтики кишечнику.
Причиною такого стану може бути:
+Гіпокаліємія

#Жінці 36-ти років після хірургічного втручання внутрішньовенно ввели концентрований розчин
альбуміну. Це спричинило посилений рух води у такому напрямку:
+З міжклітинної рідини до капілярів
#У хворого з масивними опіками розвинулась гостра недостатність нирок, що характеризується
значним і швидким зменшенням швидкості клубочкової фільтрації. Який механізм її розвитку?
+Зменшення ниркового кровотоку

#Під час огляду дитини 11-ти місяців педіатр виявив викривлення кісток нижніх кінцівок і затримку
мінералізації кісток черепа. Нестача якого вітаміну призводить до даної патології?
+Холекальциферол
#Чоловік 32-х років чотири роки страждає на хронічний гломерулонефрит з нефротичним
синдромом. Відзначаються явні набряки на обличчі, в останній час з’явилися набряки на ногах та
тулубі. Який із механізмів розвитку набряків найбільш вірогідний у цього хворого?
+Зниження онкотичного тиску крові
#В експерименті збільшили проникність мембрани збудливої клітини для іонів калію. До яких змін
мембранного потенціалу це призведе?
+Гіперполяризація
#У хворого 35-ти років, який часто вживає алкоголь, на фоні лікування сечогінними засобами,
виникли сильна м’язова і серцева слабкість, блювання, діарея, АТ- 100/60 мм рт.ст., депресія.
Причиною такого стану є посилене виділення з сечею:
+Калію

#У хворого, що страждає на важку форму порушення водно-сольового обміну, настала зупинка серця
в діастолі. Який найбільш вірогідний механізм зупинки серця в діастолі
+Гіперкаліємія
#У хворого, що страждає на серцеву недостатність, спостерігаються збільшення печінки, набряки
нижніх кінцівок, асцит. Який механізм є провідним в утворенні даного набряку?
+Гідродинамічний

#У хворого з нефротичним синдромом спостерігаються масивні набряки обличчя та кінцівок. Який


патогенетичний механізм є провідним в розвитку набряків?
+Зниження онкотичного тиску крові

#У людини осмотичний тиск плазми крові 350 мосмоль/л (норма -300 мосмоль/л). Це спричинить,
перш за все, посилену секрецію такого гормону:
+Вазопресин

#Тварині внутрішньовенно ввели концентрований розчин хлориду натрію, що зумовило зниження


реабсорбції іонів натрію у канальцях нирок. Внаслідок яких змін секреції гормонів це відбувається?
+Зменшення альдостерону

#У хворої людини посилений рух води з кровоносних капілярів до тканин, що викликало їх


позаклітинний набряк (збільшені розміри м’яких тканин кін-цівок, печінки тощо). Зменшення якого
параметру гомеостазу є найбільш імо-вірною причиною розвитку набряку?
+Онкотичний тиск плазми крові
#При токсичному ушкодженні клітин печінки з порушенням її функцій у хворого з’явилися набряки.
Які зміни складу плазми крові є провідною причиною розвитку набряків?
+Зниження вмісту альбумінів
#Юнака 15-ти років вжалила бджола. На місці укусу визначається набряк, гіперемія, підвищення
температури. Назвіть ініціальний патогенетичний фактор запального набряку:
+Підвищення проникності мікросудин

#У туриста під час тривалого перебування на спекоті відбулася значна втрата води, що
супроводжувалося різким зниженням діурезу. Посилення секреції яких гормонів відбувається при
цьому?
+Вазопресин й альдостерон
#Пацієнт 16-ти років, що страждає на хворобу Іценко-Кушінга, консультований з приводу надмірної
ваги тіла. При опитуванні з’ясувалося, що енергетична цінність спожитої їжі складає 1700-1900
ккал/добу. Яка провідна причина ожиріння у даному випадку?
+Надлишок глюкокортикоїдів
#Чоловік 64 років, скаржиться на задишку, часте серцебиття, швидку втомлюваність. Ввечері
з'являються набряки на нижчих кінцівках.Що із нижче переліченого являється патогенетичним
фактором цих набряків?
+підвищення гідростатичного тиску крові у венозних частинах капілярів

#Жінка 55 років, яка проживає в гірській місцевості, діагностований ендемічний зоб. Об'єктивно:
трохи підвищеної вгодованості, загальмована, апатична, збільшення щитовидної залози. Дефіцит
якого з перелічених нижче елементів викликає цей стан?
+йод

#Внаслідок захворювання нирок у паціінта відмічаються набряки. В аналізах сечі асивна


протеїнурія. Який механізм є основним у виникненні набряків у такого пацієнта?
+Зниження онкотичного тиску плазми крові

#У хворого на цукровий діабет виникла значна спрага, дисфагія та порушення психічної діяльності.
Який тип розладів водно-електролітного балансу характеризує поява вказаних ознак?
+Дегідратація гіперосмотична.

#Внаслідок травмування у хворого видалили прищитовидні залози, що супроводжувалося: млявістю,


спрагою, різким підвищенням нервово-м’язової збудливості. З порушенням обміну якої речовини це
пов’язано:
+Кальцію

#При повному (з водою) аліментарному голодуванні розвинулись генералізовані набряки. Який із


патогенетичних факторів у цьому випадку є ведучим?
+Зниження онкотичного тиску плазми крові.
#У хворого важка нефропатія з масивним набряклим синдромом, яка ускладнила хронічну
бронхоектатичну хворобу. Лабораторні дослідження виявляють рясну протеинурию, циліндрурію,
значне зниження вміст білка в сироватці крові, Гіперліпемія, гіпокаліємію та ін. відхилення. Що є
первинним і найбільш суттєвою ланкою в патогенезі набряків у даного хворого?
+Зниження онкотичного тиску крові
#У хворого 40 років після перенесеної травми головного мозку стало спостерігатися підвищене
виділення сечі (до 8 - 10 л / добу) і спрага. Аналіз сечі показав відсутність глюкозурії і низьку
відносну щільність. Яка форма порушення водно-сольового обміну можлива в некомпенсовані період
описаного захворювання?
+Гіперосмолярна гіпогідрадація
#Людині внутрішньовенно ввели 0,5 л ізотонічного розчину лікарської речовини. Які з рецепторів
насамперед відреагують на зміну водно-сольового балансу організму?
+Волюморецептори порожнистих вен і передсердь
#У хворого з патологією серцево-судинної системи розвинулись набряки на нижніх кінцівках. Який
механізм розвитку серцевого набряку?
+Підвищення гідростатичного тиску в венулах

#У людини після гострої крововтрати виникло відчуття спраги. Зміна якого гомеостатичного
параметру викликала це відчуття?
+Зменшення об’єму позаклітинної рідини

#В ході експерименту у білого щура моделювався набряк легені шляхом введення адреналіну. Який
патогенетичний механізм розвитку набряку є провідним в даному випадку?
+Гідродинамічний

#У зв’язку з крововтратою пацієнту введено 1 л розчину хлориду натрію з кон- центрацією 150
ммоль/л. Внаслідок цього, насамперед, зменшиться:
+Онкотичний тиск крові
#Після споживання солоної їжі у людини значно зменшилася кількість сечі. Підвищена секреція
якого гормону призвела до зменшення діурезу?
+Вазопресин

#Після видалення у пацієнта 2/3 шлунка у крові зменшився вміст гемоглобіну, кількість еритроцитів,
збільшилися розміри цих клітин крові. Дефіцит якого вітаміну призводить до таких змін у крові?
+B12
#Дитина 9-ми місяців харчується штучними сумішами, які не збалансовані за вмістом вітаміну B 6. У
дитини спостерігається пелагроподібний дерматит, судоми, анемія. Розвиток судом може бути
пов'язаний з порушенням утворення:
+ГАМК
#У хворого спостерігається погіршення сутінкового зору. Який з вітамінних препаратів слід
призначити пацієнту?
+Ретинолу ацетат

#У хворого з частими кровотечами з внутрішніх органів і слизових оболонок виявлені пролін і лізин
у складі колагенових волокон. Через відсутність якого вітаміну порушено їх гідроксилювання?
+Вітамін C
#У хворого відмічені такі зміни: порушення зору в сутінках, підсихання кон'юнктиви та рогової
оболонки. Такі порушення можуть бути при недостачі вітаміну :

#У хворого спостерігається гемералопія (куряча сліпота). Яка з перерахованих речовин матиме
лікувальну дію?
+Каротин

#При голодуванні (у другому його періоді) у хворого може підвищуватися кількістьліпідів в крові і
спостерігатися гипопротеинемия. Яка форма гіперліпемії має місце в даному випадку?
+ Транспортна

#Після хімічного опіку в хворого розвинувся стеноз стравоходу. Виникло різке схуднення від
затрудненого прийому їжі. У крові: ер.- 3, 0 • 1012/л, Hb-106 г/л, загальний білок - 57 г/л. Який вид
голодування в хворого?
+Неповне

#При визначенні енерговитрат організму людини методом непрямої калориметрії встановлено, що за


одну хвилину споживається 1000 мл кисню і виділяється 800 мл вуглекислого газу. Який дихальний
коефіцієнт у досліджуваної людини?
+0,8

#У людини вимірюють енерговитрати натщесерце, лежачи, в умовах фізичного і психічного спокою,


при температурі комфорту. В який час енерговитрати будуть найбільшими?
+17-18 годин вечора
#Мати зауважила занадто темну сечу у її 5-річної дитини. Дитина скарг не висловлює. Жовчних
пігментів у сечі не виявлено. Поставлено діагноз алкаптонурія. Дефіцит якого ферменту має місце у
дитини?
+Оксидаза гомогентизинової кислоти

# У людини порушено всмоктування продуктів гідролізу жирів. Причиною цього може бути дефіцит
у порожнині тонкої кишки:
+ Жовчних кислот
# У дитини виявлена схильність до ожиріння, яка є результатом діатезу. Назвіть вид діатезу, при
якому частіше може розвинутись ожиріння:
+ Нервово-артритичний
#Пацієнт46-тироків звернувся до лікаря зі скаргами на болі в дрібних суглобах ніг та рук. Суглоби
збільшені, мають вигляд потовщених вузлів. У сироватці встановлено підвищений вміст уратів. Це
може бути спричинене:
+ Порушенням обміну пуринів

#У хворого нормально забарвлений кал, у складі якого є велика кількість вільних жирних кислот.
Причиною цього є порушення:
+ Всмоктуванняжирів
#Хворий 55-ти років хворіє на хронічний гломерулонефрит протягом 15-ти років. Які зміни складу
крові або сечі найбільш характерно свідчать про обмеження секреторної функції нирок?
+ Гіперазотемія
#При обстеженні чоловіка 45-ти років, що тривалий час перебував на рослинній дієті, виявлено
негативний азотистий баланс. Яка особливість раціону стала причиноюцього явища?
+ Недостатня кількість білків
#Пацієнт 16-ти років, що страждає на хворобу Іценко-Кушінга, консультований з приводу надмірної
ваги тіла. При опитуванні з’ясувалося, що енергетична цінність спожитої їжі складає 1700-1900
ккал/добу. Яка провідна причина ожиріння у даному випадку?
+ Надлишок глюкокортикоїдів
#При алкаптонурії відбувається надмірне виділення гомогентизинової кислоти із сечею. С
порушенням метаболізму якої амінокислоти пов’язано виникнення цього захворювання?
+ Тирозин
#Порушення процесів мієлінізації нервових волокон призводить до неврологічних розладів і
розумової відсталості. Такі симптоми характерні для спадкових і набутих порушень обміну:
+Вищих жирних кислот
#У хворого діагностовано алкаптонурію. Вкажіть фермент, дефект якого є причиною цієї патології:
+Оксидаза гомогентизинової кислоти
#Батьки дитини 3-х років звернули увагу на потемніння кольору його сечі при відстоюванні.
Об'єктивно: температура у нормі, шкірні покриви чисті, рожеві, печінка не збільшена. Назвіть
імовірну причину даного стану:
+Алкаптонурія
#Причиною захворювання на пелагру може бути переважне харчування кукурудзою і зниження у
раціоні продуктів тваринного походження. Відсутність у раціоні якої амінокислоти призводить до
даної патології?
+Триптофан
#У фізично здорових молодих курсантів після важкого фізичного навантаження при одноденному
пішому переході на 50 км в сечі виявлено білок, рівень якого в середньому не перевищував 1 г/л.
Який різновид протеїнурії мав місце?
+Маршова
#При лабораторному дослідженні у хворого виявили стеаторею. Вкажіть фермент, недостатність дії
якого призвела до виникнення цього симптому:
+Ліпаза
#У хворого на хронічний гепатит виявлено значне зниження синтезу і секреції жовчних кислот. Який
процес у найбільшій мірі буде порушений у кишечнику цього хворого?
+Емульгування жирів
#У дитини із розумовою відсталістю встановлено зелене забарвлення сечі після додавання 5%
розчину FeCl3. Про порушення обміну якої амінокислоти свідчить позитивний результат цієї
діагностичної проби?
+ Фенілаланіну
#У клінічно здорових батьків народилася дитина, хвора на фенілкетонурію (аутосомно-рецесивне
спадкове захворювання). Які генотипи батьків?
+Аа х Аа
#У чоловіка 35-ти років феохромоцитома. В крові виявляється підвищений рівень адреналіну та
норадреналіну, концентрація вільних жирних кислот зросла в 11 разів. Активація якого ферменту під
впливом адреналіну підвищує ліполіз?
+ТАГ-ліпаза
#У 19-місячної дитини із затримкою розвитку та проявами самоагресії, вміст сечової кислоти в крові
- 1,96 ммоль/л. При якому метаболічному порушенні це спостерігається?
+|Синдром Леша-Ніхана

#При обстеженні хворого виявлена характерна клініка колагенозу. Вкажіть, збільшення якого
показника сечі характерне для цієї патології:
+Гідроксипролін

#Через кілька тижнів після народження у дитини почали відмічатися прояви ураження ЦНС, шкіра та
волосся посвітлішали. При доливанні до свіжої сечі 5% розчину трихлороцтового заліза з'являється
оливково-зелене забарвлення. Який найбільш вірогідний діагноз?
+Фенілкетонурія
#При обстеженні дитини з олігофренією виявлено в крові і спинномозковій рідині підвищений вміст
фенілаланіну і фенілпіровиноградної кислоти. Реакція сечі з трихлороцтовим залізом позитивна.
Виберіть – із порушення синтезу якого ферменту пов'язаний розвиток захворювання.
+Фенілаланінгідроксилаза.
#У дитини 3 років, яка страждає на квашіоркор, спостерігається порушення ороговіння епідермісу та
збільшення його злущення, є жирова інфільтрація печінки. Який тип голодування спостерігається пр
цьому?
+Білкове.
#До лікаря звернувся чоловік 65 років з скаргами на гострий біль в великих пальцях ніг. Він любить
та часто вживає пиво. Виникло підозріння на подагру. Для підтвердження діагнозу вміст якої із
перелічених речовин необхідно визначити у крові?
+сечової кислоти

#Чоловік 42 років, що страждає ожирінням за верхнім типом (плечовий пояс, лице місяцеподібне),
АТ - 160/95 мм рт.ст., глюкоза крові 8.0 ммоль/л. Вміст кортизолу в крові підвищений, а
адренокортикотропін знижений. Яка найбільш вірогідна причина розвитку гіперкортицизму?
+гормонопродуктуюча пухлина кори наднирників

#Відомо, що фенілкетонурія виникає внаслідок мутації гена, що відповідає за


перетворення амінокислоти фенілаланіну і розпаду її до кінцевих продуктів обміну – СО 2 і Н2О.
Вкажіть, який шлях обміну фенілаланіну приведе до розвитку фенілкетонурії.
+Фенілаланін - фенілпіруват - кетокислоти
#Дитина, хвора на фенілкетонурію, страждає на розумову відсталість. Який механізм буде головним
у розвитку пошкодження центральної нервової системи?
+Накопичення в крові фенілаланіну і фенілкетонів
#Чоловік 67 років страждає на атеросклероз судин головного мозку. При обстеженні знайдена
гіперліпідемія. Вміст якого класу ліпопротеїдів плазми крові найбільш вірогідно буде значно
підвищений при біохімічному дослідженні?
+Ліпопротеїди низької щільності

#Хворий скаржиться на періодичне ослаблення стула, яке пов’язує з прийомом багатої на жири їжі.
При цьому він відмічає зменшення забарвленості калу. При лабораторному обстеженні встановлено
нормальний вміст ліпідів в сироватці крові. Порушення якого із станів ліпідного обміну має місце у
даного хворого?
+Всмоктування.
#У хворого діагностована пелагра. Порушення обміну якої амінокислоти лежить в основі цього
захворювання?
+ Триптофану

#Пацієнт 46-ти років звернувся до лікаря зі скаргами на болі в дрібних суглобах ніг та рук. Суглоби
збільшені, мають вигляд потовщених вузлів. У сироватці встановлено підвищений вміст уратів. Це
може бути спричинене:
+Порушенням обміну пуринів

#У жінки 37-ми років протягом року періодично виникали інфекційні захворювання бактеріального
генезу, їх перебіг був вкрай тривалим, ремісії - короткочасними. При обстеженні виявлена
гіпогамаглобулінемія. Порушення функції яких клітин може бути прямою її причиною?
+Плазматичні клітини
#Електрофоретичне дослідження сироватки крові хворого пневмонією показало збільшення одної з
білкових фракцій. Вкажіть її:
+γ-глобуліни
#Спадкова гіперліпопротеїнемія І типу обумовлена недостатністю ліпо-протеїнліпази. Підвищення
рівня яких транспортних форм ліпідів в плазмі навіть натщесерце є характерним?
+Хіломікрони

# У крові чоловіка 26-ти років виявлено 18% еритроцитів сферичної, сплощеної, кулястої та остистої
форм. Інші еритроцити були у формі двоввігнутих дисків. Як називається таке явище?
+Фізіологічний пойкілоцитоз

#Крива дисоціації оксигемоглобіну зміщена вправо. Які зміни в організмі людини можуть бути
причиною цього?
+Гіпертермія
#У хворого в крові: ер.- 3,0*1012/л; Hb- 90г/л; ретикул.- 0,5%. В мазку: пойкілоцити, гіпохромні
еритроцити. Залізо сироватки крові - 80 мкмоль/л. Для якої патології це характерно?
+Залізорефрактерна анемія

#При аналізі крові у спортсмена виявлено: ер,- 5,5 • 1012/л, НЬ- 180 г/л, лейк,- 7 • 109/л, н,- 64%, б,-
0,5%, е,- 0,5%, м,- 8%, л,- 27%. Такі показники свідчать про стимуляцію, перш за все:
+Еритропоезу

#Мати звернулася до лікаря з приводу того, що у дитини 5-ти років під дією сонячних променів на
шкірі з'являються еритеми, везикулярний висип, свербіж шкіри. Лабораторні дослідження виявили
зменшення вмісту заліза у сироватці крові, збільшення виділення з сечею уропорфіриногену І.
Найбільш вірогідною спадковою патологією у дитини є:
+Еритропоетична порфірія

#До приймального відділення доставлено жінку 38-ми років з матковою кровотечею, що триває другу
добу. Що з наведеного буде виявлено при аналізі крові хворої?
+Зменшення гематокритного показника

#Пацієнтка 58-ми років скаржиться на підвищену втомлюваність, зниження працездатності,


сонливість, задишку під час швидкої ходи. У крові: ер.-4, 6 • 1012/л, Hb- 92 г/л, КП- 0,6. У мазку
крові - велика кількість анулоцитів та мікроцитів. Для якої анемії це характерно?
+Залізодефіцитна

#У чоловіка 43-х років з видаленою ниркою були виявлені симптоми анемії. Що зумовило появу цих
симптомів?
+Зниження синтезу еритропоетинів
#Жінка 55-ти років звернулася зі скаргами на тривалі циклічні маткові кровотечі протягом року,
слабкість, запаморочення. Об’єктивно: блідість шкіри. У крові: Hb- 70 г/л, ер.- 3, 2 • 1012/л, КП- 0,6,
лейк.- 6, 0 • 109/л, ретикулоцити - 1%; гіпохромія еритроцитів. Яка анемія у хворої?
+Хронічна постгеморагічна
#У людини до травми гематокритний показник 40%. Яким він буде через добу після втрати 750 мл
крові?
+30%
#При аналізі крові у спортсмена виявлено: ер.- 5, 5 • 1012/л, Hb- 180 г/л, лейк.- 7•109/л, н.- 64%, б.-
0,5%, е.- 0,5%, м.- 8%, л.- 27%. Такі показники свідчать про стимуляцію, перш за все:
+Еритропоезу

#В пробірку, що містить розчин NaCl 0,9%, додали краплю крові. Що відбудеться з еритроцитами?
+Залишаться без змін

#Чоловік 47 років скаржиться на слабкість, запаморочення . Півроку назад переніс операцію з


приводу резекції шлунку Аналіз крові: Hb-80г/л, еритроцити-3.5 Т/л, кольоровий показник-0.69,
ШОЕ-15 мм/год. Сироваткове залізо-5.4 мкмоль/л. Гіпохромія еритроцитів. Яка анемія найбільш
вірогідно має місце у хворого?
+залізодефіцитна анемія

#У хворого з анацидним гастритом при дослідженні крові отримані наступні


дані. Ер.- 3,0.Т / л; Нв-62 г / л; Ц.П-0,6; ретікулоц.-0,1%; тромбоц.-260 Г / л. Лейкоц.-5,6 Г / л. Б-0, Е-
З, Мц-О, Ю-О, П-З, С-66, Л-25, Мо-З. Анізоцитоз-микроцитоз виражений, ШОЕ-10 мм / год. Про яку
форму патології крові можна думати на підставі даних цього аналізу?
+Залізодефіцитна анемія

#Жінка 37 років скаржиться на загальну слабкість, часті запаморочення, утруднення ковтання їжі,
бажання їсти крейду. Шкіра та видимі слизові оболонки бліді. В крові: eр.- 3,4 *1012/л, Hb- 70 г/л, КП-
0,7, ретик.- 0,1%, лейк.- 4,7*109/л, е.- 2%, п.- 3%, с.- 64%, л.- 26%, м.- 5%. ШОЕ- 15 мм/г. Сироваткове
залізо - 7,3 мкмоль/л. Дефіцит якої речовини обумовив виникнення захворювання?
+Заліза.

#У дитини, що знаходиться на штучному вигодовуванні коров'ячим молоком,


розвинулася важка анемія: еритроцити - 3,4 * 1012 / л, Нb - 68 г / л, ретикулоцити - 0%. Яка анемія
розвинулася у дитини?
+Залізодефіцитна
#Хворий переніс операцію з приводу резекції пілоричного відділу шлунку. Через рік скаржиться на
слабкість, періодичну появу темних кіл під очима, задишку. В крові: Hb - 70г/л, ер. - 3,0 *1012/л. Які
зміни еритроцитів в мазках периферичної крові характерні для даного захворювання?
+Гіпохромні еритроцити

#На п"яту добу післі гострої крововтрати у хворого діагностована гіпохромна анемія. Який головний
механізм у розвитку гіпохромії?
+Надходження з кісткового мозку незрілих еритроцитів

#На шостому місяці вагітності в жінки з’явилася виражена залізодефіцитна анемія. Діагностичною
ознакою її була поява в крові
+Анулоцитів

#У дитини, що отримав в результаті необережного поводження з вогнем термічні опіки до 40%


поверхні тіла, показник гематокриту виявляє порушення співвідношення плазми і формених
елементів. Яка форма порушення загального обсягу крові спостерігається при цьому?
A * Поліцитемічна гіповолемія
#На останньому місяці вагітності вміст фібриногену в плазмі крові в 2 рази вище за норму. Яку
швидкість осідання еритроцитів слід при цьому очікувати?
+40-50 мм/годину

#У чоловіка 25-ти років на 4-й день після крововтрати в крові зросла кількість поліхроматофільних
еритроцитів, ретикулоцитів і навіть з’явились поодинокі нормобласти. Про що свідчить такі
гематологічні зміни?
+Процеси регенерації випереджають дозрівання клітин

#У хворої 36-ти років, яка лікувалася сульфаніламідами з приводу респіраторної вірусної інфекції, в
крові гіпорегенераторна нормохромна анемія, лейкопенія, тромбоцитопенія. В кістковому мозку -
зменшення кількості мієлокаріоцитів. Яка це анемія?
+Гіпопластична

#У хворого, що переніс 5 років тому субтотальну резекцію шлунка, розвинулась В12-


фолієводефіцитна анемія. Який механізм є провідним у розвитку такої анемії?
+Відсутність внутрішнього фактора Касла

#У чоловіка 50-ти років при обстеженнібуло виявлено зниження кількості еритроцитів у крові та
підвищення рівня вільного гемоглобіну в плазмі крові (гемоглобінемія). КП становив 0,85. Який вид
анемії спостерігається у хворого?
+Набута гемолітична
#До клініки поступив чоловік 40-ка років, якого укусила гадюка. Де переважно буде проходити
гемоліз еритроцитів у цьому випадку?
+У кровоносному руслі

#У хворого в анамнезі: з дитинства відмічався знижений рівень гемоглобіну. Лікування препаратами


заліза не дає ефекту. У крові: ер.- 3,1*1012/л, ретик.- 16%, Hb- 85 г/л, КП- 0,75; в мазку крові
анізоцити, пойкілоцити, мішенеподібні еритроцити, еритроцити з базофільною зернистістю, рівень
заліза у сироватці 30 мкмоль/л. Для якої патології системи крові характерні такі дані?
+Таласемія

#У хворого спадкова гемолітична анемія Мінковського-Шофара. Виявлення у крові яких характерних


клітин надало можливість лікарю встановити діагноз?
+Мікросфероцити

#Після видалення у пацієнта 2/3 шлунка у крові зменшився вміст гемоглобіну, кількість еритроцитів,
збільшилися розміри цих клітин крові. Дефіцит якого вітаміну призводить до таких змін у крові?
+B12

#У хворої 19-ти років з дитинства відмічалося зниження гемоглобіну до 90-95 г/л. У крові під час
госпіталізації: ер,- 3,2 • 1012/л, НЬ- 85 г/л, КП- 0,78; лейк,-5,6• 10 9/л, тромб.- 210 • 109/л. В мазку:
анізоцитоз, пойкілоцитоз, мішенеподібні еритроцити. Ретикулоцити - 6%. Лікування препаратами
заліза було неефективне. Яку патологію системи крові можна запідозрити в даному випадку?
+Таласемія

#Хворий 20-ти років скаржиться на загальну слабкість, запаморочення, швидку втомлюваність. У


крові: Hb- 80 г/л. Мікроскопічно: еритроцити зміненої форми. Причиною цього стану може бути:
+Серпоподібноклітинна анемія

#Чоловік 56-ти років потрапив до клініки зі скаргами на загальну слабкість, біль і печіння в язиці,
відчуття оніміння в кінцівках. У минулому переніс резекцію кардіального відділу шлунка. У крові:
Hb- 80 г/л; ер.- 2, 0 • 1012/л; КП-1,2, лейк.- 3, 5 • 109/л. Який вид анемії у цього хворого?
+B12-фолієводефіцитна
|#У чоловіка 30-ти років перед операцією визначили групову належність крові. Кров резус-
позитивна. Реакцію аглютинації еритроцитів не викликали стандартні сироватки груп 0 ?? (I), А? (II),
В? (III). Досліджувана кров належить до групи:
+0 (αβ) (I)

#Через рік після субтотальної резекції шлунка з приводу виразки малої кривизни виявлені зміни в
аналізі крові - анемія, лейко- і тромбоцитопенія, КП-1,3, наявність мегалобластів та мегалоцитів.
Дефіцит якого фактору обумовив розвиток цієї анемії?
+Фактор Касла

#Хворий (28 років) прибув у стаціонар зі скаргами на біль у животі, нестійкий стул, слабкість, втому,
віддишку. В анамнезі: 2 роки тому операція з приводу гострої кишкової непрохідності з резекцією 60
см тонкої кишки. При вступі в аналізі крові: кількість еритроцитів 2,4 *1012/л, ретикулоцитів 0,4%,
гемоглобіну 80 г/л, КП 1,25; в мазку крові макроанізоцити, пойкілоцити, шизоцити поодинокі
мегалоцити, мегалобласти. Для якої патології системи крові характерні ці дані?
+В12-дефіцитна анемія

#У жінки на 6-му місяці вагітності при обстеженні виявлено знижену кількість еритроцитів і
гемоглобіну, кольоровий показник – 1,4. В мазку з'явилися мегалоцити, поодинокі оксифільні
мегалобласти. Який вид анемії за патогенезом найбільш імовірний в даному випадку?
+В12 і фолієводефіцитна анемія.

#Хворий 57 років скаржиться на слабкість, серцебиття, задишку при виконанні нетяжкої роботи. В
крові: еритроцити - 0,79 Т/л, гемоглобін - 40 г/л, КП - 1,45, лейкоцити - 3,4 Г/л. В мазку: анізоцитоз і
пойкілоцитоз еритроцитів, мегалобласти і мегалоцити. Яка анемія найбільш ймовірна у цього
хворого?
+В12- фолієводефіцитна

#У пацієнта, носія спадкової серповидної аномалії еритроцитів, захворювання пневмонією


супроводжувалося гемолітичним кризом і розвитком анемії. Що є безпосередньо причиною
гемолітичного кризу в даному випадку?
+Гіпоксія, викликана пневмонією
#У жінки на 7-му місяці вагітності стала швидко наростати анемія: еритроцити -2,7 * 1012 / л, Нb -90
г / л, анізоцитоз, кількості, поодинокі мегалобластні мегалоцити, ретикулоцити - 0%. Який вид анемії
розвинувся в даному випадку?
+B12-дефіцитна

#Хвора звернулась в клініку зі скаргами на слабкість, задишку, швидку стомлюваність,


запаморочення. В крові: ер. - 1,8*1012/л; Hb - 80 г/л; к.п. - 1,5; лейк. - 3,2 *109/л. У мазку: анізоцитоз,
пойкілоцитоз, мегалобласти мегалоцити. Який найбільш вірогідний діагноз?
+В12-дефіцитна анемія

#Хвора 3 р., поступила в дитячу клініку у важкому стані з гемоглобінопатією (серпоподібноклітинна


анемія). Заміна якою аминокислотою глутамінової кислоти в бета–ланцюгу глобіну лежить в основї
утворення патологічного гемоглобіну в даному випадку?
+валіном.

#Хвора поступила в клініку на обстеження. З дитинства відмічалось зниження гемоглобіну до 90-95


г/л. Лікування препаратами заліза було неефективне. Аналіз крові при поступленні: Е–3,2х10 12/л, Hb–
85 г/л, к.п.–0,78. В мазку анізоцитоз, пойкілоцитоз, мішеневидні еритроцити, ретикулоцити –16%.
Поставлений діагноз – таласемія. До якого виду гемолітичних анемій можна віднести дане
захворювання?
+Спадкова гемоглобінопатія.

#У хворого, що прибув із Тунісу, виявлена альфа-таласемія з гемолізом еритроцитів і жовтяницею.


Хвороба була діагностована на основі наявності в крові
+Мішенеподібних еритроцитів

#У хворого з гемолітичною жовтяницею в мазку крові присутні еритроцити у вигляді


мікросфероцітов 1 - 6 в полі зору. Яка можлива причина гемолізу еритроцитів, що викликає
виникнення такої форми жовтяниці?
+Спадковий дефект розвитку їх мембран

#В аналізі крові 35-річного хворого: Нв – 58 г/л, еритроцити – 1,3х10 12/л, колірний показник – 1,3,
лейкоцити – 2,8х109/л, тромбоцити – 1,1х109/л, ретикулоцити – 2%0, ШОЕ – 35 мм/час. Визначаються
полісегментіровані нейтрофіли, а також тільця Жоллі, кільця Кебота, мегалоцити. Яка це анемія?
+В12 – фолієводефіцитна.
#У хворого після резекції шлунка розвинулася В- 12 фолієводефіцитна анемія. Який із перерахованих
кольорових показниівк характерний для цієї патології?
+1,4

#У хворого 35 років розвинулася імунна гемолітична анемія. Який показник сироватки крові зросте в
найбільшій мірі?
+Непрямий білірубін
#У чоловіка 40-ка років було встановлено діагноз: серпоподібноклітинна анемія. Який механізм
приводить до зменшення кількості еритроцитів в крові у цього хворого?
+Позасудинний гемоліз

#У хворого з гемолітичною анемією виявлено дефіцит піруваткінази в еритроцитах. За цих умов


причиною розвитку гемолізу еритроцитів є:
+Зменшення активності Na+, К+ -АТФ-ази

#При обстеженні в аналізі крові пацієнтавиявленолейкоцитоз,лімфоцитоз, клітини Боткіна-


Гумпрехта на тлі анемії. Про яку хворобу слід думати лікарю?
+Хронічний лімфолейкоз

#У хворого скарги на загальну слабкість, підвищену втому, зниження апетиту і маси тіла. В анамнезі
часті пневмонії. На підставі клінічних даних та результатів дослідження периферійної крові у нього
діагностовано хронічний лімфолейкоз. Які дегенеративні зміни лейкоцитів характерні для даного
захворювання?
+Тіні Боткіна-Гумпрехта

#Після прийому сульфаніламідів у хворого виникли лихоманка, блювання і стул з кров’ю. У крові:
лейк.- 0, 9 · 109/л (гранул.- 0, 7 · 109/л), лейкоаглютиніни. Який з термінів найбільш точно
характеризує виявлені зміни у крові?
+ Агранулоцитоз
#У хворого в лейкограмі: лейкоцити – 14-10 9/л; мієлобласти - 71%, промієлоцити, мієлоцити,
метамієлоцити - 0%, паличкоядерні нейтрофіли - 6%, сегментоядерні - 13%; лімфоцити - 7%,
моноцити - 3%. Яка патологія у хворого?
+Мієлобластний лейкоз
#У хворого через добу після апендектомії при аналізі крові виявили нейтрофільний лейкоцитоз з
регенеративним зсувом вліво. Який найбільш імовірний механізм розвитку абсолютного лейкоцитозу
у периферичній крові хворого?
+ Посилення лейкопоезу

#Чоловік 26-ти років перебуває в торпідній стадії шоку внаслідок автомобільної аварії. В крові:
лейк,- 3,2 • 109/л. Який головний механізм в розвитку лейкопенії?
+Перерозподіл лейкоцитів у судинному руслі
#У хворого 70-ти років атеросклероз ускладнився тромбозом судин нижніх кінцівок, виникла
гангрена пальців лівої стопи. Початок тромбоутворення, найбільш вірогідно, пов'язаний з:
+Адгезією тромбоцитів

#Пацієнт звернувся до лікаря зі скаргами на задишку, що виникала після фізичного навантаження.


Клінічне обстеження виявило анемію та наявність парапротеїну в зоні гамма-глобулінів. Який
показник у сечі необхідно визначити для підтвердження діагнозу мієломи?
+ Білок Бенс-Джонса

#У студента через 2 години після іспиту в аналізі крові виявлено лейкоцитоз без істотних змін у
лейкоцитарній формулі. Який найбільш вірогідний механізм розвитку лейкоцитозу?
+Перерозподіл лейкоцитів в організмі
#Хворий 23-х років скаржиться на слабкість, підвищення температури до 38 - 400C. Об’єктивно:
печінка і селезінка збільшені. У крові: Hb- 100 г/л, ер.- 2, 9 • 1012/л, лейк.- 4, 4 • 109/л, тромб.-48 •
109/л, нейтрофіли сегментоядерні - 17%, лімфоцити - 15%, бластні клітини - 68%. Всі цитохімічні
реакції негативні. Дайте гематологічний висновок:
+Недиференційований лейкоз

#У хворого в лейкограмі: лейкоци-ти - 14 • 109/л; мієлобласти - 71%, про-мієлоцити, мієлоцити,


метамієлоцити - 0%, паличкоядерні нейтрофіли - 6%, сегментоядерні - 13%; лімфоцити - 7%,
моноцити - 3%. Яка патологія у хворого?
+Мієлобластний лейкоз

#При обстеженні в аналізі крові пацієнта виявлено лейкоцитоз, лімфоцитоз, клітини Боткіна-
Гумпрехта на тлі анемії. Про яку хворобу слід думати лікарю?
+Хронічний лімфолейкоз

#У хворого 42 років при дослідженні периферичної крові виявлено: гемоглобін 80 г/л, еритроцитів
3,2 Т/л, лейкоцитів 25 Г/л; лейкоцитарна формула: базофілів - 5%, еозинофілів - 9%, міелобластів -
3%, проміелоцитів - 8%; нейтрофілів: міелоцитів - 11%, метаміелоцитів - 22%, паличкоядерних -
17%, сегментоядерних - 19%, лімфоцитів - 3%, моноцитів - 3%. Яка патологія крові найбільш
вірогідна у хворого:
+Хронічний міелолейкоз

#Ер.-3,1 Т / л Нв-90 г / л; К.П.-0,88; Тромбоцит. -110 Г / л; лейкоцит. -51 Г / л. Б-О, Е-I, Мц-О, Ю-О,
П-I, С-24, Л-70, Мо-2, лімфобластів-2% .Тені Боткіна-Гумпрехта. ШОЕ 27 мм / год. Який вид
патології супроводжується такими змінами в крові?
+Хронічний лімфолейкоз
#Ер .-3,5 Т / л Нв-110 г / л; К.П.-0,9; Тромбоцит. -100 Г / л; лейкоцит. -80 Г / л. Б-О, Е-О, Мц-О, Ю-0,
П-0, С-30, Л-2, Мо-0, мієлобласти-68%. ШОЕ 25 мм / год. Для якого виду лейкозу характерний
представлений аналіз?
+ Мієлобластний

#У хворого на атрофічний гастрит виник дефіцит вітаміу В12. Яка зміна лейкоцитарної формули є
найбільш типовою для гіповітамінозу В12?
+Ядерний зсув вправо

#У хворого С., виявлено такі зміни в периферичної крові: Ер. 3,2x1012 / л, Гем.80 г / л, Лейк. 25x109 /
л. Лейкоцитарна формула: базофіли - 5%, еозинофіли - 9%, мієлобласти - 3%, проміелоціти - 8%;
міелоціти - 11%, метамієлоцити - 22%, паличкоядерні - 17%, сегментоядерні - 19%, лімфоцити - 3%,
моноцити - 3%. Визначте найбільш ймовірну патологію яка відповідає певному опису картини крові:
+хронічний мієлолейкоз
#У хворого П., виявлені такі зміни в периферичної крові: Ер. 3,0x1012 / л, Гем 80 г / л, Лейк. 21x109 /
л. Лейкоцитарна формула: базофіли - 0%, еозинофіли - 0%/ мієлобласти - 54%, проміелоціти - 1%;
міелоціти - 0%, метамієлоцити - 0%, паличкоядерні - 1%, сегментоядерні - 28%, лімфоцити - 13%,
моноцити - 3%. Визначте найбільш ймовірну патологію яка відповідає певному опису картини крові:
+ гострий мієлобластний лейкоз

#Хворий на протязі останнього року став відмічати під_вищену втомлюваність, загальну слабість.
Аналіз крові: Е– 4.1х1012/л, Hb–119 г/л, к.п.–0.87, лейкоцити – 57х109/л, лейкоформула: Ю–0, П–0, С–
9%, Е–0, Б–0, лімфобласти–2%, пролімфоцити–5%, лімфоцити–81%, М–3%, тромбоцити – 160х10 9/л.
В мазку: нормохромія, велика кількість тіней Боткіна–Гумпрехта. Про яку патологію системи крові
свідчить дана гемограма?
+ Хронічний лімфолейкоз.

#У хворого в обох щелепах рентгенологічно виявлено численні дефекти у вигляді гладкостінних


округлих отворів. При гістологічному дослідженні - явища остеолізису і остеопорозу при явищах
слабкого кісткоутворення. В сечі хворого знайдено білок Бенс-Джонса. Назвіть захворювання:
+Мієломна хвороба

#Хворий 62-х років блідий, всі групи лімфовузлів збільшені. В крові: Hb- 60 г/л, еритроцити - 1,9 Т/л,
лейкоцити - 29 Г/л, тромбоцити - 110 Г/л. Лейкоцитарна формула: сегментоядерні лейкоцити - 10%,
лімфоцити - 8%, моноцити - 2%, бластних клітин - 80%. Цитохімічні дослідження бластних клітин:
позитивна реакція на глікоген, негативна - на ліпіди і пероксидазу. Дайте заключення про патологію:
+Гострий лімфобластний лейкоз

#Autopsy of the body of a 48-year-old man shows that the bone marrow in the flat bones, as well as in the
cylindrical bone diaphyses and epiphyses, is moist, colored gray-red or gray-yellow, and puri-form (pyoid
bone marrow). The spleen weight is 7 kg; it is dark red on section, with signs of ischemic infarctions. All the
lymph nodes are enlarged, soft, and gray-red in color. In the liver there are signs of fatty degeneration and
leukemic infiltrates. What is the most likely diagnosis?
+Chronic myeloid leukemia

#У хворого із захворюванням печінки виявлено зниження вмісту протромбіну вкрові. Це призведе,


першзавсе, допорушення:
+ Другої фази коагуляційного гемостазу

#У хворого 37-ми років на фоні тривалого застосування антибіотиків спостерігається підвищена


кровоточивість при невеликих пошкодженнях. У крові зниження активності факторів згортання крові
II, VII, IX, X, подовження часу згортання крові. Недостатністю якого вітаміну обумовлені зазначені
зміни?
+ Вітамін К
# У хлопчика 3-х років з вираженим геморагічним синдромом відсутній антигемофільний глобулін А
(факторVIII) у плазмі крові. Яка фаза гемостазу первинно порушена у цього хворого?
+ Внутрішній механізм активації протромбінази
#При обтураційній жовтяниці і жовчних норицях часто спостерігається протромбінова недостатність.
З дефіцитом в організмі якого вітаміну це пов’язано?
+K

#У результаті побутової травми у пацієнта виникла значна крововтрата, що супроводжувалося


зниженням артеріального тиску. Дія яких гормонів забезпечує швидке відновлення кров’яного тиску,
викликаного крововтратою?
+Адреналін, вазопресин

#Видалення зуба у пацієнта з хронічним персистуючим гепатитом ускладнилось тривалою


кровотечею. Яка причина геморагічного синдрому?
+Зменшення утворення тромбіну

#Пацієнт страждає на геморагічний синдром, що проявляється частими носовими кровотечами,


посттравматичними та спонтанними внутрішньошкірними та внутрішньосуглобовими
крововиливами. Після лабораторного обстеження було діагностовано гемофілію В. Дефіцит якого
фактора згортання крові обумовлює дане захворювання?
+ IX

#У хворого після оперативного видалення кісти підшлункової залози виник геморагічний синдром з
вираженим порушенням зсідання крові. Розвиток цього ускладнення пояснюється:
+Активацією фібринолітичної системи

#У хворої 43-х років на фоні септичного шоку відзначається тромбоцитопенія, зниження


фібриногену, поява в крові продуктів дегенерації фібрину, поява петехіальних крововиливів. Вкажіть
причину виникнення даних змін:
+ДВЗ-синдром

#У хворого опікова хвороба ускладнилася ДВЗ-синдромом. Яку стадію ДВЗ-синдрому можна


запідозрити, якщо відомо, що кров хворого згортається менше ніж за 3 хвилини?
+Гіперкоагуляції

#При обстеженні хворого з гемофілією виявлено зміна деяких показників крові. Який з
перерахованих ознак відповідає цьому захворюванню?
+Час згортання крові загальмовано
#У хворого виявлені множинні синяки на тілі, тривалість кровотечі за Дуке 25 хвилин, число
тромбоцитів крові 25*109/л. Для якого захворювання характерні такі ознаки?
+Спадковий дефект утворення тромбоцитів.

#Хворий 12-ти років поступив в клініку з гемартрозом колінного суглоба, з раннього дитинства
страждає кровоточивістю. Яка хвороба у хлопчика?
+Гемофілія

#У спортсмена легкоатлета (бігуна на довгі дистанції) під час змагань розвинулась гостра серцева
недостатність. В результаті чого виникла ця патологія?
+ Перевантаження серця об’ємом

#При дослідженні крові хворого виявлено значне збільшення активності МВ-форм КФК
(креатинфосфокінази) та ЛДГ-1. Яку патологію можна припустити?
+ Інфаркт міокарда

#У чоловіка 35-ти років під час тривалого бігу виникла гостра серцева недостатність. Які зміни
іонного складу спостерігаються у серцевому м'язі при цьому стані?
+ Накопичення в клітинах міокарда іонів Na+ і Ca2+
# Жінка 25-ти років скаржиться на постійний біль у ділянці серця, задишку під час рухів, загальну
слабкість. Об’єктивно: шкіра бліда та холодна, акроціаноз. Рв- 96/хв., АТ-105/70 мм рт.ст. Межа
серця зміщена на 2 см вліво. Перший тон над верхівкою серця послаблений, систолічний шум над
верхівкою. Діагностовано недостатність мітрального клапана серця. Чим обумовлене порушення
кровообігу?
+Перевантаження міокарда збільшеним об’ємом крові

#У хворого 44-х років на ЕКГ виявлені ознаки гіпертрофії обох шлуночків та правого передсердя.
Діагностовано недостатність тристулкового клапана. Який патогенетичний варіант порушення
функції серця має місце при цій недостатності?
+ Перевантаження серця об'ємом
#До відділення реанімації надійшов чоловік 47-ми років з діагнозом інфаркт міокарду. Яка з фракцій
лактат-дегідрогенази (ЛДГ) буде переважати в сироватці крові протягом перших двох діб?
+1

#Хворий 50-ти років страждає на гіпертонічну хворобу. Під час фізичного навантаження у нього
з’явилися відчуття м’язової слабкості, нестачі повітря, синюшність губ, шкіри, обличчя; дихання
супроводжувалося відчутними на відстані вологими хрипами. Який механізм лежить в основі
виникнення такого синдрому?
+Гостра правошлуночкова недостатність

#З метою відтворення серцевої недостатності серце жаби перфузували розчином бромістого кадмію -
блокатору сульфгідрильних груп. Який варіант серцевої недостатності при цьому виникає?
+Від токсичного пошкодження міокарду

#Пацієнт 64 років з гострою серцевою недостатністю, артеріальним тиском 80/60мм рт ст, добовим
діурезом 530 мл, істотно збільшена концентрація сечовини і креатиніну в крові. Назвіть
патогенетичний механізм розвитку азотемії і олігурії:
+ Зменшення фільтраційного тиску.

#В експерименті подразнюють гілочки симпатичного нерва, які іннервують серце. Це призвело до


збільшення сили серцевих скорочень, тому що через мембрану типових кардіоміоцитів збільшився:
+Вхід іонів кальцію
-#У хворого спостерігається атонія м’язів. Назвіть фермент м’язової тканини, активність якого може
бути знижена при такому стані:
+Креатинфосфокіназа

#Дитина 10-ти років перенесла кілька атак ревматізма. При його клінічному
обстеженні було встановлено, що мали місце запальні явища в суглобах
і виявилися ознаки недостатності мітрального клапана. Яке з патологічних явищ у даного хворого
можна віднести до поняття "хвороба"?
+Ревматизм

#У хворого з ознаками недостатності мітрального клапана в намнезе відзначалися атаки ревматизму,


що супроводжувалися запальними явищами в суглобах. Яке з патологічних явищ у даного больного
відноситься до категорії "патологічного стану "?
+ Недостатність мітрального клапана

#У хворого з недостатністю мітрального клапану виникла гіпертрофія лівого шлуночка серця. Який
механізм є пусковим у розвитку гіпертрофії?
+Активація генетичного апарату.

|#Через 1 годину після накладання кільця, що звужує аорту, в собаки різко зросла сила та частота
серцевих скорочень, а об’єм циркулюючої крові та товщина стінки лівого шлуночка не відрізнялися
від вихідних показників. Яка стадія гіпертрофії міокарда спостерігається у тварини?
+Аварійна

#У п’ятимісячної дівчинки виявлено застійні явища у легенях. При обстеженні виявлено зв’язок між
висхідною аортою та легеневою артерією, що в нормі спостерігається у деяких земноводних і
плазунів. Назвіть цю природжену ваду розвитку:
+Незрощення боталової протоки

#В експерименті на ссавці зруйнували певну структуру серця, що призвело до припинення


проведення збудження від передсердь до шлуночків. Що саме зруйнували?
+ Атріовентрикулярний вузол

#В експерименті подразнюють гілочки симпатичного нерва, які іннервують серце. Це призвело до


збільшення сили серцевих скорочень, тому що через мембрану типових кардіоміоцитів збільшився:
+ Вхід іонів кальцію

# При аналізі ЕКГ виявлено випадіння деяких серцевих циклів PQRST. Наявні зубці та комплекси не
змінені. Назвіть вид аритмії:
+ Синоатріальна блокада
#Під час об’єктивного обстеження хворого з діагнозом: атеросклеротичний міокардіосклероз, лікар
встановив феномен дефіциту пульсу. При якій формі порушення серцевого ритму спостерігається
такий феномен?
+ Миготлива аритмія

#У хворої 45-ти років при електрокардіографічному обстеженні виявлено такі зміни: інтервал P-Q
подовжений, при цьому випадає кожен другий або третій комплекс QRST. Яке саме порушення
провідності серця спостерігається?
+Атріовентрикулярна блокада ІІІ ступеня

#У хворого 45-ти років при аналізі ЕКГ встановлено: ритм синусовий, число передсердних
комплексів більше числа шлуночкових комплексів; прогресуюче подовження інтервалу P-Q від
комплексу до комплексу; випадіння окремих шлуночкових комплексів; зубці P та комплекси QRST
без змін. Назвіть тип порушення серцевого ритму:
+ Атріовентрикулярна блокада II ступеня

#Під час об'єктивного обстеження хворого з діагнозом: атеросклеротичний міокардіосклероз, лікар


встановив феномен дефіциту пульсу. При якій формі порушення серцевого ритму спостерігається
такий феномен?
+Миготлива аритмія

#На ЕКГ пацієнта мають місце такі зміни: зубець P - нормальний, інтервал P-Q - вкорочений,
шлуночковий комплекс QRST - розширений, зубець R - двогорбий або двофазний. Яка із форм
аритмії має місце у даного пацієнта?
+Синдром WPW (Вольфа-Паркінсона-Уайта)
#Хворий 21-го року надійшов до стаціонару з загостренням хронічного тонзиліту. Скаржиться на
слабкість, задуху при помірному фізичному навантаженні. Температура 37,5°С. ЧСС- 110/хв. ЕКГ:
ритм синусовий, інтервал РQ подовжений. Яка аритмія у хворого?
+Передсердно-шлуночкова блокада І ст.

#Жінка 49-ти років звернулася до лікаря зі скаргами на підвищену втомлюваність та появу задишки
під час фізичного навантаження. На ЕКГ: ЧСС- 50/хв, інтервал PQ- подовжений, комплекс QRS- не
змінений, кількість зубців P перевищує кількість комплексів QRS. Який вид аритмії у пацієнтки?
+ Атріовентрикулярна блокада

#У чоловіка 50-ти років раптово виникли сильне серцебиття, біль у серці, різка слабкість, підвищення
артеріального тиску; пульс аритмічний, з дефіцитом. На ЕКГ виявлено відсутність зубців Р та різна
тривалість інтервалів R-R. Яке порушення серцевого ритму в хворого?
+ Миготлива аритмія
#Хворий 65-ти років, що страждає на атеросклероз, госпіталізований до хірургічного відділення з
приводу розлитого гнійного перитоніту. Під час операції діагностовано тромбоз брижових артерій.
Яка найбільш імовірна причина перитоніту?
+ Геморагічний інфаркт

#Підліток 15 років, скаржиться на недостачу повітря, загальну слабкість, серцебиття. ЧСС 130
уд/хв, АТ-100/60 мм рт. ст.на ЕКГ, комплекс QRS нормальної форми та тривалості. Число зубців Р
та шлункових комплексів однакове, зубець Т злитий з зубцем Р.Яка аритмія серця спостерігається у
підлітка ?
+синусова тахікардія

#Людина отримала електротравму. При цьому струм перейшов через серцевий м'яз. Які
небезпечні порушення в роботі серця можуть виникнути у цііі ситуаціі, що вимагають
термінового втручання?
+Фібриляція шлуночків

#При аналізі ЕКГ встановлено: ритм синусовий, число передсердних комплексів більше числа
шлуночкових комплексів; прогресуюче подовження інтервалу P-Q від комплексу до комплексу;
випадання окремих шлуночкових комплексів, тому після зубця Р йде довга пауза; зубці Р та
комплекси QRST без змін. Назвіть тип порушення серцевого ритму.
+Неповна атріо-вентрикулярна блокада П ступеня

#У хворого на ЕКГ виявлено зсув сегмента S-T вище ізоелектричної лінії


на 1 мм і збільшення тривалості зубця T до 0,25 с. З порушенням якого
процесу пов'язані зазначені зміни на ЕКГ?
+реполяризації шлуночків

#У чоловіка 63 років зі слів родичів тричі відзначалась втрата свідомості. ЧД 18 за 1 хв., ЧСС 45 за 1
хв., АТ 100/70 мм рт. ст. На ЕКГ: частота Р 80 за хвилину, частота R 42 за хвилину, ритм правильний.
Яка найбільш вірогідна аритмія?
+ Повна AV блокада

#Через 3 тижні після гострого інфаркту міокарда у хворого з'явилися болі в серці та суглобах,
запалення легень. Який механізм є основним у розвитку постінфарктного синдрому Дресслера у
цього хворого?
+Аутоімунне запалення
#При обстеженні у юнака 16 років було виявлено прискорення серцебиття під час вдиху, сповільнене
– під час видиху. На ЕКГ відмічалося: вкорочення інтервалу RR під час вдоху та подовження його
під час видоху. Назвіть вид аритмії
+ Синусова аритмія

#Електрик, працюючи з порушенням правил техніки безпеки, випадково торкнувся оголеного


електропровода обома руками і загинув. Смерть настала внаслідок
+ Фібриляції передсердь і шлуночків

#Під час об’єктивного обстеження хворого з діагнозом: атеросклеротичний міокардіосклероз, лікар


встановив феномен дефіциту пульсу. При якій формі порушення серцевого ритму спостерігається
такий феномен?
+Миготлива аритмія

#При аналізі ЕКГ виявлено випадіння деяких серцевих циклів PQRST. Наявні зубці та комплекси не
змінені. Назвіть вид аритмії:
+Синоатріальна блокада

#У хворого на ЕКГ виявлено збільшення тривалості комплексу QRS. Наслідком чого це може бути?
+Збільшення часу охоплення збудженням шлуночків
#Аналіз ЕКГ хворого виявив відсутність зубця P. Тривалість та амплітуда QRS комплексу та зубця Т
відповідають нормі. Що є водієм ритму серця даного пацієнта?
+Передсердно-шлуночковий вузол
#У хворого напад тахікардії. Які мембранні циторецептори кардіоміоцитів доцільно заблокувати,
щоб припинити напад?
+Бета-адренорецептори

#У хворого на ЕКГ виявлено, що тривалість інтервалу RR дорівнює 1,5 с, частота серцевих скорочень
- 40/хв. Що є водієм ритму серця?
+Атріовентрикулярний вузол

#Під час розтину померлого 43-х років, що страждав на іХС з розвитком інфаркту міокарда,
патологоанатом виявив набряк легень. Які патологічні зміни могли зумовити набряк легень?
+Гостра лівошлуночкова недостатність
#Хворого 45-ти років при аналізі ЕКГ встановлено: ритм синусовий, число передсердних комплексів
більше числа шлуночкових комплексів; прогресуюче подовження інтервалу P-Q від комплексу до
комплексу; випадіння окремих шлуно-чкових комплексів; зубці P та комплекси QRST без змін.
Назвіть тип порушення серцевого ритму:
+Атріовентрикулярна блокада II ступеня

#У студента перед екзаменом виникла тахікардія. Які зміни на ЕКГ будуть свідчити про її наявність?
+Укорочення інтервалу R-R

#У хворого інфаркт міокарда. Активність якого ферменту буде значно підвищена в сироватці крові
хворого в перші години?
+Креатинфосфокіназа МВ

#На ізольованому серці шляхом охолодження припиняють функціонування окремих структур. Яку
структуру охолодили, якщо серце внаслідок цього спочатку припинило скорочення, а далі відновило
їх із частотою у 2 рази меншою за вихідну?
+Синоатріальний вузол

#Підвищення внутрішньочерепного тиску у хворого з церебральною гематомою обумовило надмірну


активність блукаючого нерва (ваготонію) та зміну частоти серцевих скорочень. Який вид аритмії
серця виникає при цьому?
+Синусова брадикардія
#У хворого з серцевою недостатністю виникла аритмія у вигляді генерації позачергових імпульсів в
пучку Гіса. Порушення якої функції серцевого м’язу спостерігається в даному випадку?
+Збудливість

#Під час серцевого нападу чоловік в автобусі втратив свідомість, з’явилися судоми. Лікар швидкої
допомоги виявив на ЕКГ, що частота скорочення передсердь перевищує частоту скорочення
шлуночків. Що може бути причиною даного стану?
+Повна поперечна блокада проведення збудження
#Хворий на трансмуральний інфаркт міокарда лівого шлуночка переведений до відділення реанімації
у важкому стані. АТ- 70/50 мм рт.ст., ЧСС- 56/хв., ЧД- 32/хв. Зазначте головну ланку в патогенезі
кардіогенного шоку:
+ Падіння серцевого викиду

#У хворого, який скаржився на біль у ділянці лівої лопатки, був діагностований інфаркт міокарду.
Назвіть вид болю у хворого?
+Іррадіюючий (відбитий)
#У пацієнта, який півтора місяця тому переніс інфаркт міокарда, діагностовано синдром Дреслера з
характерною тріадою: перикардит, плеврит, пневмонія. Який головний механізм цього ускладнення?
+Сенсибілізація організму антигенами міокарда

#У хворого на ішемічну хворобу серця відзначається гіпертрофія міокарда, тахікардія, зниження


ХОК. Який з механізмів є провідним в ушкодженні кардіоміоцитів у даному випадку?
+Пошкодження специфічних мембранних насосів

#У чоловіка 45-ти років після значного психоемоційного навантаження раптово з'явився стискаючий
біль в ділянці серця з іррадіацією в ліву руку, шию, під ліву лопатку. Обличчя стало блідим,
вкрилося холодним потом. Нітрогліцерин усунув напад болю. Який процес розвинувся у хворого?
+Стенокардія

#Хворий 49-ти років, водій за професією, скаржиться на нестерпний стискаючий біль за грудниною,
що "віддає" у ділянку шиї. Біль виник 2 години тому. Об'єктивно: стан важкий, блідість, тони серця
послаблені. Лабораторне обстеження показало високу активність креатинкінази та ЛДГ1. Для якого
захворювання характерні такі симптоми?
+Гострий інфаркт міокарда

#Одним із самих небезпечних моментів в патогенезі некрозу міокарда є подальше наростання зон
некрозу, дистрофії та ішемії. Важлива роль в цьому належить підвищенню споживання міокардом
кисню. Які речовини сприяють даному процесу?
+Катехоламіни

#Хворий 59 років госпіталізований у кардіологічне відділення в тяжкому стані з діагнозом: гострий


інфаркт міокарда в області задньої стінки лівого шлуночка та перегородки, початковий набряк
легень. Який первинний механізм, що викликає розвиток набряку легень у пацієнта?
+Лівошлуночкова недостатність

#Хворий А, 59 років, директор приватного підприємства. Після перевірки податковоїінспекцією


ввечері з'явилися інтенсивні пекучі болі, локалізовані за грудиною, иррадиирующие в ліву руку.
Через 15 хв стан хворого нормалізувався. Який з механізмів розвитку ішемії серцевого м'яза є
провідним у даного хворого?
+ Підвищення в крові рівня катехоламінів

#У жінки обмежений кровотік у нирках, підвищений артеріальний тиск. Гіперсекреція якого гормону
зумовила підвищений тиск?
+ Ренін

# У хворого з тромбофлебітом нижніх кінцівок раптово після навантаження виникли задишка, різкий
біль у грудях, ціаноз, набухання шийних вен. Яке найбільш імовірне порушення кровообігу виникло
у хворого?
+Тромбоемболія легеневої артерії

#До серцево-судинного відділення надійшов хворий зі скаргами на постійний головний біль у


потиличній ділянці, шум у вухах, запаморочення. При обстеженні: АТ 180/110мм рт.ст., ЧСС-95/хв.
Рентгенологічно визначено звуження однієї з ниркових артерій. Активація якої з перерахованих
систем викликала гіпертензивний стан хворого?
+ Ренін-ангіотензинова

#У чоловіка 65-ти років впродовж 15-ти років була виражена артеріальна гіпертензія. Останнім часом
систолічний тиск почав знижуватися, а діастолічний залишився підвищеним. Який гемодинамічний
тип артеріальної гіпертензії у хворого?
+ Гіпокінетичний

#У чоловіка 72-х років довготривала хронічна патологія легень призвела до недостатності клапанів
легеневої артерії і трикуспідального клапану, недостатності кровообігу за правошлуночковим типом.
Який тип артеріальної гіпертензії є причиноюпе- ревантаження серця об’ємом?
+ Легенева гіпертензія
#Обстеження пацієнта з високим артеріальним тиском показало в нього вторинну артеріальну
гіпертензію. Причиною такого стану є ренінпродукуюча пухлина нирки. Що є головною ланкою в
патогенезі вторинної артеріальної гіпертензії в хворого?
+ Гіперпродукція ангіотензину 2, альдостерону

#У хворого 48-ми років на хронічний гломерулонефрит наявні набряки, АТ- 210/100 мм рт.ст., ЧСС-
85/хв., межі серця розширені. Який механізм розвитку артеріальної гіпертензії є головним?
+Активація ренин-ангіотензин- альдостеронової системи

#Хворий 39-ти років з алкогольним цирозом печінки скаржиться на задишку, загальну слабкість.
Встановлено зниження артеріального тиску, розширення поверхневих вен передньої стінки живота,
спленомегалію. Яке порушення гемодинаміки спостерігається у хворого?
+Синдром портальної гіпертензії

#У хворого 70-ти років атеросклероз ускладнився тромбозом судин нижніх кінцівок, виникла
гангрена пальців лівої стопи. Початок тромбоутворення, найбільш імовірно, пов'язаний з:
+Адгезією тромбоцитів

#Чоловік 58-ми років хворіє на атеросклероз судин головного мозку. При обстеженні виявлена
гіперліпідемія. Вміст якого класу ліпопротеїдів у сироватці крові даного чоловіка найбільш
вірогідно буде підвищений?
+Ліпопротеїди низької щільності

#На перехід із горизонтального положення у вертикальне система кровообігу відповідає розвитком


рефлекторної пресорної реакції. Що з наведеного є її обов’язковим компонентом?
+Системне звуження венозних судин

#Визначте пульсовий і середньо-динамічний артеріальний тиск (мм рт.ст.) у обстежуваного, якщо


виміряний у нього артеріальний тиск становить 130/70 мм рт.ст.:
+60, 90

#У дорослої людини системний артеріальний тиск знизився з 120/70 до 90/50 мм рт.ст., що викликало
рефлекторне звуження судин. У якому з за-значених органів звуження судин буде найменшим?
+Серце
#Під час роботи лікарю–стоматологу доводиться довго стояти на ногах, що може призвести до
застою крові у венах нижніх кінцівок та їх варикозного розширення. З порушенням якого механізму
венозного припливу крові до серця це пов’язано?
+ Відсутність скорочення скелетних м’язів

#У хворого після тривалого психоемоційного напруження спостерігається підвищення артеріального


тиску, що супроводжується серцебиттям, кар-діалгіями, головним болем, запамороченням.
Домінуючим у формуванні артеріальної гіпертензії у даному випадку є збільшення:
+Тонусу артеріол

#Щуру в плевральну порожнину введено 0,5 мл повітря. Який тип недостатності дихання виникає в
даному випадку?
+ Рестриктивне порушення альвеолярної вентиляції

# У неврологічне відділення з приводу мозкового крововиливу поступив хворий 62-х років.


Об’єктивно: стан важкий. Спостерігається наростання глибини і частоти дихання, а потім його
зменшення до апное, після чого цикл дихальних рухів відновлюється. Який тип дихання у хворого?
+ Чейна-Стокса

# Чоловік 50-ти років хворіє на хронічний бронхіт, скаржиться на задишку під час фізичного
навантаження, постійний кашель з відходженням харкотиння. При обстеженні діагностовано
ускладнення - емфізема легень. Чим вона зумовлена?
+ Зниження еластичних властивостей легень
#У хворого внаслідок хронічного обструктивного бронхіту на тлі задишки, тахікардії та ціанозу під
час дослідження газового складу крові виявлено розвиток гіпоксемії та гіперкапнії. Яке порушення
зовнішнього дихання спостерігається у хворого?
+Гіповентиляція

#Хворий 47-ми років впродовж останніх 3-х років хворіє на туберкульоз легень, скаржиться на
задишку, важкість в області правого боку грудної стінки, температуру тіла 37,7oC. Виявлено
правобічний ексудативний плеврит. Який тип клітин передбачається у плевральному пунктаті?
+ Лімфоцити
#Чоловік 27-ми років після ДТП надійшов до лікарні у важкому стані із черепно-мозковою травмою.
Дихання характеризується судомним тривалим вдихом, який переривається коротким видихом. Для
якого типу дихання це характерно?
+Апнейстичне

# У хворого із вираженим пневмосклерозом після перенесеного інфільтративного туберкульозу


легень розвинулась дихальна недостатність. До якого патогенетичного типу вона відноситься?
+ Рестриктивний

#В експерименті на тварині здійснили перерізку блукаючих нервів з обох боків. Як при цьому
зміниться характер дихання?
+Стане глибоким і рідким
#У підлітка 12-ти років, який хворіє на бронхіальну астму, виник тяжкий напад астми: виражена
експіраторна задишка, блідість шкірних покривів. Який вид порушення альвеолярної вентиляції має
місце?
+Обструктивний

#Під час обіду дитина поперхнулася і аспірувала їжу. Почався сильний кашель, шкіра і слизові
ціанотичні, пульс частий, дихання рідке, видих подовжений. Яке порушення зовнішнього дихання
розвинулося у дитини? +Експіраторна задишка при асфіксії

#У шахтаря виявлено фіброз легень, що супроводжувався порушенням альвеолярної вентиляції. Який


механізм виникнення цього порушення є провідним?
+Обмеження дихальної поверхні легень

#Людина зробила максимально глибокий видих. Як називається об’єм повітря, що знаходиться в її


легенях після цього?
+Залишковий об’єм

#У людини внаслідок тривалого перебування у горах на висоті 3000 м над рівнем моря збільшилась
киснева ємкість крові. Безпосередньою причиною цього є посилене утворення в організмі:
+Еритропоетинів
#Лікар записав в історії хвороби, що у хворого дихання поверхневе (знижена глибина дихання). Це
означає, що зменшеним є такий показник зовнішнього дихання:
+Дихальний об’єм

#В результаті виробничої травми у хворого виявлено перелом кількох ребер. Який характер дихання
зазвичай спостерігається в таких випадках?
+ Поліпное

#У дівчинки 14 років, експіраторна задишка. Стан розвився після гри з собакою. В анамнезі хворіє
бронхіальною астмою. Якою із перелічених нижче біологічно активною речовиною найбільш
вірогідно викликаний спазм гладеньких м'язів бронхіол дівчинки?
+лейкотрієн Д4

#В отоларингологічне відділення госпіталізована жінка зі скаргами на попадання стороннього


предмету [вишневої кістки] у дихальні шляхи. Які зміни зовнішнього дихання слід очікувати?
+ Глибоке рідке

#У хворого на правець виникла гостра недостатність дихання. Який тип недостатності дихання
виникає в даному випадку?
+ Дисрегуляторне порушення альолярної вентиляції
#У хворого на дифтерію розвинувся набряк гортані. При цьому спостерігається рідке та глибоке
дихання з затрудненням вдоху. Який тип дихання спостерігається при цьому?
+ Стенотичне

#У чоловіка 27 років діагностований: правосторонній ексудативний плеврит з ознаками


недостатності дихання. Назвіть механізм розвитку цієї недостатності .
+порушення рухливості грудної клітини
#Хворий І., 38 років, поступив зі скаргами на біль в боці, що підсилюється при вдиху і кашлі. Больові
відчуття зменшуються в положенні лежачи на ураженому боці. При обстеженні дихання часте
поверхневе, помітно обмеження дихальної рухливості відповідної половини грудної клітки. Який
механізм зміни характеру дихання у хворого?
+Обмеження корою мозку рефлексу Герінга-Брейера
#Під час обіду дитина поперхнувся і аспірувала їжу. Почався сильний кашель, шкіра і слизові
ціанотичні, пульс прискорений, дихання рідке, Видих подовжений. Яке порушення зовнішнього
дихання розвинулося у дитини?
+ Стадія експіраторної задишки при асфіксії

#Чоловік 30 років скаржиться на задуху, важкість в правій половині грудної клітки, загальну
слабкість. Температура тіла 38,9оС. Об’єктивно: права половина грудної клітки відстає від лівої.
Плевральна пункція дала ексудат. Що являється провідним чинником ексудації у хворого?
+Підвищення проникливості стінки судин.
#У людини в результаті патологічного процесу збільшена товщина гематоальвеолярного бар’єру.
Безпосереднім наслідком цього буде зменшення:
+Дифузійної здатності легень

#Робота шахтарів у забої часто спричинює антракоз. Який вид дихальної недостатності може
розвинутися при цьому?
+Рестриктивний
#В ході обстеження людини необхідно визначити, яка частка альвеолярного повітря оновлюється під
час кожного вдиху. Який з наведених показників необхідно розрахувати для цього?
+Коефіцієнт легеневої вентиляції

#Чоловік 40-ка років скаржиться на загальну слабкість, головний біль, кашель з виділенням
мокротиння, задишку. Після клінічного огляду й обстеження поставлено діагноз: пневмонія. Який
тип гіпоксії має місце у хворого?
+Респіраторна
#У людини після довільної тривалої затримки дихання збільшилися частота й глибина дихання. Які
зміни в крові, насамперед, стали причиною цього?
+Підвищення рСО2

#Через рік після субтотальної резекції шлунка з приводу виразки малої кривизни виявлені зміни в
аналізі крові - анемія, лейко- і тромбоцитопенія, КП-1,3, наявність мегалобластів та мегалоцитів.
Дефіцит якого фактору обумовив розвиток цієї анемії?
+Фактор Касла
#Жінка 45-ти років декілька років хворіє на системний червоний вовчак у легкій формі. При
прогресуванні захворювання (з'явився міокардит) їй призначили преднізолон як імуносупресор.
Через 2 місяці прийому у хворої виникла шлункова кровотеча. Яка найбільш імовірна її причина?
+Ульцерогенна дія

#У приймально-діагностичне відділення доставили жінку 38-ми років з шлунковою кровотечею. Які


зміни найбільш імовірні з боку крові через добу?
+Зменшення гематокритного числа
#У хворого 30-ти років із гострим запаленням підшлункової залози (панкреатитом) виявлено
порушення порожнинного травлення білків. Це може бути пов’язано із недостатнім синтезом та
виділенням залозою такого ферменту:
+Трипсин

#При копрологічному дослідженні встановлено, що кал знебарвлений, у ньому знайдено краплі


нейтрального жиру. Найбільш вірогідною причиною цього є порушення:
+Надходження жовчі до кишечнику

#До гастроентерологічного відділення потрапив хворий 57 років з підозрою на синдром Золінгера-


Еллісона, про що свідчило різке збільшення рівню гастрину у сироватці крові. Яке порушення
секреторної функції шлунку найбільш вірогідне?
+ Гіперсекреція гіперацидна

#Чоловік 32 років , звернувся з приводу диспептичних розладів, при обстеженні знайдено виразковий
дефект слизової оболонки шлунка та діагностується синдром Золінгера-Еллісона. Що являється
основним патогенетични механізмом виникнення виразки у даному випадку ?
+підвищення продукції гастрину

#Хворий скаржиться на відрижку, печію, часті запори. При титруванні шлункового соку одержали
такі дані: загальна кислотність - 88 т.о., загальна HCl - 83 т.о., вільна HCl - 50 т.о., зв'язана HCl - 33
т.о., кислі фосфати та органічні кислоти - 5 т.о. Оцініть стан кислотності шлункового соку.
+Гіперацидний.

#У хворої зі скаргами на болі в епігастральній ділянці оперізуючого характеру при лабораторному


обстеженні виявлено підвищений вміст діастази в сечі, а також вміст у калі великої кількості
неперетравленого жиру. Для якої форми патології шлунково-кишкового тракту найбільш характерні
описані явища?
+ Гострий панкреатит
#Для моделювання виразки шлунка тварині ввели в гастральні артерії атофан, який спричинює їх
склерозування. Який механізм пошкодження слизової оболонки шлунку є провідним в даному
експерименті?
+ Гіпоксичний

#Хворому поставили діагноз - синдром подразненої кишки. Віділення якого інкрету найбільш
вірогідно збільшується при цьом?
+ Мотіліну

#У хворого з синдромом Zollinger-Ellison [ пухлина підшлункової залози] відзначається збільшення


секреції, перистальтики шлунково-кишкового тракту, а також діарея та пептичні виразки. Яка з
перерахованих речовин, що секретується вказаною пухлиною, викликає цей комплекс симптомів?
+Гастрин

#До складу харчових раціонів обов’язково входять продукти, в яких є клітковина. Відомо, що вона не
перетравлюється ферментами травного тракту й не засвоюється організмом. Яку роль відіграє ця
речовина?
+Стимулює моторну функцію травного каналу
#У людини порушено всмоктування продуктів гідролізу жирів. Причиною цього може бути дефіцит у
порожнині тонкої кишки:
+Жовчних кислот

#У молодого чоловіка внаслідок подразнення сонячного сплетення запальним процесом (солярит)


підвищена функціональна активність залоз шлунка, що виражається, зокрема, у збільшенні продукції
хлоридної кислоти. Яка з вказаних нижче речовин викликає гіперхлор-гідрію у даному випадку?
+Гастрин

# Пацієнт звернувся зі скаргами на гострий біль у правому підребер’ї. При огляді лікар звернув увагу
на пожовтіння склер хворого. Лабораторно: підвищена активність АлАТ та негативна реакція на
стеркобілін в калі. Для якого захворювання характерні такі симптоми?
+ Гепатит
# У хворого з жовтяницею встановлено: підвищення у плазмі крові вмісту загального білірубіну за
рахунок непрямого (вільного), в калі та сечі - високий вміст стеркобіліну, рівень прямого (зв’язаного)
білірубіну в плазмі крові в межах норми. Про який вид жовтяниці можна думати?
+ Гемолітична

#Чоловік 53-х років звернувся зі скаргами на гострий біль у правому підребер’ї. При огляді лікар
звернув увагу на пожовтілі склери хворого. Лабораторні аналізи показали підвищену активність АЛТ
та негативну реакцію на стеркобілін у калі. Для якого захворювання характерні такі симптоми?
+ Жовчнокам’яна хвороба

#У хворого 20-ти років з жовтяницею встановлено: підвищення у плазмі крові вмісту загального
білірубіну за рахунок непрямого (вільного), в калі та сечі - високий вміст стеркобіліну, рівень
прямого (зв’язаного) білірубіну в плазмі крові в межах норми. Про який вид жовтяниці можна ду-
мати?
+Гемолітична

#У хворого на підгострий септичний ендокардит при огляді лікар відзначив загальну слабкість і
іктеричність шкіри, склер і видимих слизових оболонок. У крові виявлена збільшена кількість
непрямого білірубіну. Що зумовлює жовтяничність шкіри і слизових?
+ Надпечінкова жовтяниця

#Хвора 28-ми років потрапила до інфекційної лікарні з приводу пожовтіння шкіри, склер, слизових
оболонок. Лабораторно встановлене підвищення рівня прямого білірубіну у крові. В сечі виявлений
уробіліноген і білірубін. Для якого з перелічених захворювань характерні такі зміни?
+Паренхіматозна жовтяниця

#У хворого на жовтяницю у крові підвищений вміст прямого білірубіну та жовчних кислот; у сечі
відсутній стеркобіліноген. При якій жовтяниці можлива наявність цих ознак?
+Механічна

#Хвора 48-ми років надійшла до клініки із скаргами на слабкість, дратівливість, порушення сну.
Об'єктивно: шкіра та склери жовтого кольору. У крові: підвищення рівня загального білірубіну з
переважанням прямого. Кал - ахолічний. Сеча - темного кольору (жовчні пігменти). Яка жовтяниця
має місце в хворої?
+Механічна
#Чоловік 28-ми років надійшов зі скаргами на нудоту, блювання, біль у правому підребер’ї.
Об’єктивно: жовтяничність шкіри, склер; температура тіла підвищена, печінка збільшена, сеча темна,
кал гіпохолічний. Гіпербілірубінемія (білірубін прямий та непрямий), білірубінурія, уробілінурія,
гіпопротеїнемія, зниження зсідання крові. Для якого з перелічених нижче станів найбільш характерні
ці зміни?
+Клітиннопаренхіматозна жовтяниця

#У хворого з алкогольним цирозом печінки скарги на загальну слабкість, задишку. Встановлено


зниження артеріального тиску, асцит, розширення по-верхневих вен передньої стінки живота,
спленомегалію. Яке порушення гемодинаміки спостерігається у хворого?
+Синдром портальної гіпертензії

#Жінка 55 років, скаржиться на швидку втомлюваність, роздратованість, безсоння та сонливий стан в


день, свербіння шкіри. Пульс 58 уд/хв., АТ-110/65 мм рт.ст. Кал безколірний, містить багато жиру.
Підозрюється жовчнокам'яна хвороба з закупорюванням каменем жовчної протоки.Що з
перелікованого нижче найбільш вірогідно обумовлює стан хворої?
+холемія

#У хворого жовтяницею при лабораторному дослідженні отримано такі дані,


характеризують порушення пігментного обміну: загальний вміст в сироватці
крові білірубіну - 80 мкмоль / л; вміст прямого білірубіну в сироватці крові -63 мкмоль / л; в сечі
присутній білірубін; уробіліноген і стеркобилин в сечі
відсутній. Яка причина відсутності в сечі уробіліногену при даному виді жовтяниці?
+Відсутність надходження в кишечник білірубіну
#Хворий скаржиться на біль в правому підребір ї , зуд шкіри, головний біль, дратливість, швидку
стомлюваність. При обстеженні встановлено: жовтяничний колір шкіри та слизових оболонок,
печінка збільшена, болісна при пальпації. АТ - 80/40 мм рт.ст., ЧСС - 46/хв. В крові виявлено:
вільний білірубін - 34,15 мкмоль/л, звўязаний - 35,2 мкмоль/л, жовчні кислоти; в сечі - жовчні
кислоти, звўязаний білірубін, уробіліноген; вміст стеркобіліногену в калі зменшений. Який вид
жовтяниці у хворого?
+Печінково-клітинна.
#У хворого з пухлинної обтурацією жовчовивідних шляхів у складі калу міститься велика кількість
жиру (стеаторрея). Недолік якого компонента жовчі обумовлює це явище?
+Жовчних кислот

#У хворого С. з вираженим жовтушним синдромом виявлено: в крові: рівень непрямого білірубіну -


34,5 мкмоль/л, прямого - 35,2 мкмоль/л; в сечі: жовчні кислоти, уробіліноген; в калі – кількість
стеркобіліногену зменшено. Який найбільш ймовірний вид жовтяниці розвинувся у хворого?
+Печінкова

#У хворого після отруєння грибами з’явилося жовте забарвлення шкіри та склер, темний колір сечі.
Який пігмент спричинює забарвлення сечі у хворого на гемолітичну жовтяницю?
+ Стеркобілін

#Хвора на хронічний гепатит скаржиться на підвищення чутливості до барбітуратів, які раніше вона
переносила без симптомів інтоксикації. З порушенням якої функції печінки це пов’язане у
найбільшій мірі?
+ Метаболічної
#У жінки 57 років після тривалого больового нападу у правому підребір’ї з’явилась жовтяниця, після
чого хвора звернулась до лікаря. Виникла підозра на появність у хворої гострого калькульозного
холециститу. Дослідження якого показника крові свідчить про непрохідність жовчних протоків?
+Вільного та зв’язаного білірубіну

#Хлопчику 15 років встановлено діагноз гострого вірусного гепатиту. Дослідження якого показника
крові необхідно провести для підтвердження гострого ураження печінкових клітин?
+Активність амінотрансфераз [ЛЛТ і АСТ]

#Хворому 25 років встановлено діагноз хронічного гепатиту. Хворий скаржиться на втрату маси тіла
на 10 кг протягом 2 місяців. Об'єктивно: шкіра суха, лущиться, бліда з жовтуватим відтінком,
мелкоточечние крововиливи на шкірі, кровоточивість ясен. Порушення якої функції печінки
відображають мелкоточечние крововиливи і кровоточивість ясен:
+ білоксинтетичної

#У хворого 38 років, який переніс гепатит і продовжував вживати алкоголь, розвинулися ознаки
цирозу печінки з асцитом і набряками на нижніх кінцівках. Які зміни складу крові стали
вирішальними в розвитку набряків?
+ Гіпоальбумінемія

#У повної жінки 52 років встановлено цироз печінки. Лабораторно: гіпоальбумінемія,


гіперглобулінемія. Візуально: набряк рук, повік, ніг. Найбільш імовірною причиною набряків є зміна:
+Онкотичного тиску крові

#У пацієнта, що звернувся до лікаря, спостерігається жовте забарвлення шкіри, сеча - темна, кал
темно-жовтого кольору. Підвищення концентрації якої речовини буде спостерігатися в сироватці
крові?
+Вільний білірубін

#У хворого на жовчно-кам’яну хворобу спостерігаються ознаки холемічного синдрому. Який


симптом з перерахованих зумовлений відсутністю надходження жовчі у кишечник?
+Стеаторея

# Введення знеболюючого пацієнту перед екстракцією зуба призвело до розвитку анафілактичного


шоку, який супроводжувався розвитком олігурії. Який патогенетичний механізм зумовив зменшення
діурезу в даній клінічній ситуації?
+ Зниження гідростатичного тиску в капілярах клубочків
# У чоловіка 25-ти років діагностований гострий дифузний гломерулонефрит. З анамнезу: за 18 днів
до прояву хвороби переніс ангіну. Який механізм ураження ниркових клубочків буде спостерігатися
у цьому випадку?
+ Імунний
#У дитини 10-ти років через 2 тижні після перенесеної ангіни розвинувся нефритичний синдром
(протеїнурія, гематурія, циліндрурія), що свідчить про ураження базальної мембрани клубочків
нирок. Який найбільш імовірний механізм лежить в основі ушкодження базальної мембрани?
+Імунокомплексний

#У пацієнта з хронічним захворюванням нирок розвинулась ниркова недостатність. Який з


показників найбільш імовірно свідчить про порушення реабсорбції в канальцях в даному випадку?
+Гіпо- та ізостенурія

#У хворого 38-ми років на 3-му році захворювання на системний червоний вівчак виявлене дифузне
ураження нирок, що супроводжується масивними набряками і вираженою протеїнурією. Що є
найбільш вірогідною причиною розвитку протеїнурії у пацієнта?
+Аутоімунне ушкодження нирок

#У хворого з масивними опіками розвинулась гостра недостатність нирок, що характеризується


значним і швидким зменшенням швидкості клубочкової фільтрації. Який механізм її розвитку?
+Зменшення ниркового кровотоку

#У жінки 30-ти років виникли набряки обличчя. При обстеженні виявлені протеїнурія (5,87 г/л),
гіпопротеїнемія, диспротеїнемія, гіперліпідемія. Для якого стану характерно таке поєднання
симптомів?
+Нефротичний синдром

#У результаті порушення техніки безпеки відбулося отруєння сулемою (хлористою ртуттю). Через 2
дні добовий діурез склав 620 мл. У хворого з’явилися головний біль, блювання,судоми, задишка, у
легенях - вологі хрипи. Яка патологія має місце?
+Гостра ниркова недостатність

#У хворого з гострою нирковою недостатністю на 6-й день проведення терапевтичних заходів


виникла поліурія. Чим зумовлене зростання діурезу на початку поліуричної стадії гострої ниркової
недостатності?
+Відновлення фільтрації в нефронах

#У людини внаслідок тривалого голодування швидкість клубочкової фільтрації зросла на 20%.


Найбільш вірогідною причиною змін фільтрації в зазначених умовах є:
+Зменшення онкотичного тиску плазми крові

#Чоловік 65-ти років, який страждає на подагру, скаржиться на біль в ділянці нирок. При
ультразвуковому обстеженні встановлена наявність ниркових каменів. Підвищення концентрації якої
речовини є найбільш вірогідною причиною утворення каменів у даному випадку?
+Сечова кислота
#На препараті нирки розрізняються нефрони, які лежать на межі між кір-ковою та мозковою
речовиною, мають однаковий діаметр приносних і виносних артеріол. Назвіть, яка функція буде
порушена при їхньому пошкодженні?
+ Шунтування крові при інтенсивному кровообігу
#Хворий 55-ти років хворіє на хронічний гломерулонефрит протягом 15-ти років. Які зміни складу
крові або сечі найбільш характерно свідчать про обмеження секреторної функції нирок?
+ Гіперазотемія

#У жінки з первинним гіперпаратиреоідизмом періодично повторюються напади ниркової коліки.


Ультразвукове обстеження показало наявність дрібних каменів в нирках, найбільш імовірною
причиною утворення яких є:
+Гіперкальциемія

#У хворого після автомобільної травми артеріальний тиск 70/40 мм рт.ст.


Хворий в несвідомому стані. В добу виділяє близько 550 мл сечі. Періодично виникають судоми,
дихання за типом Куссмауля. Як називається таке порушення функції нирок? Хворий в несвідомому
стані. В добу виділяє
близько 550 мл сечі. Періодично виникають судоми, дихання за типом Куссмауля. Як називається
таке порушення функції нирок?
+ Гостра ниркова недостатність

#У хворого А. після травматичного шоку развинулись ознаки ниркової недостатності. Якими


патогенетичними механізмами зумовлено цей стан?
+Зниженням объёму клубочкової фільтрації

#Чоловік 30 років, скаржиться на слабкість, жагу, головний біль та біль у попереку. Місяць тому
перехворів ангіною. На обличчі набряки. Пульс – 84 уд/хв, АТ - 175/100 мм рт.ст. В сечі - еритроцити
40-52 в полі зору, лейкоцити - 1- 2 в полі зору, білок - 4 г/л. Встановлено діагноз гострого дифузного
гломерулонефриту. Який основний механізм ураження нирок у данного хворого?
+імунне пошкодження клубочків

#У хворого 24 років через півтора тижні після важкої стрептококової ангіни проявилася набряклість
обличчя, підвищився артеріальний тиск. Гематурія і протеїнурія 1,2 г / л. У крові виявлені
антистрептококові антитіла і зниження компонентів комплементу. В мікросудинах яких структур
найбільш ймовірно локалізація скупчень імунних комплексів, що зумовили розвиток нефропатії?
+Клубочки
#Чоловік 32 років впродовж 4 років хворіє на хронічний гломерулонефрит.
Госпіталізований з ознаками анасарки: АТ- 185/105 мм рт.ст. У крові: Hb- 110 г/л, ер.- 2,6 *1012/л,
лейк.- 9,5*109/л, залишковий азот - 32 ммоль/л, загальний білок - 50 г/л. Яка зміна з найбільшою
вірогідністю вказує на гломерулонефрит з нефротичним синдромом?
+Гіпопротеїнемія

#В експерименті кролю ввели нефроцитотоксичну сироватку морськоі свинки. Яке захворювання


нирок моделювалося в цьому досліді?
+ Гострий дифузний гломерулонефрит
#У хворого із хронічною нирковою недостатністю встановлення зменшення кліренсу за інуліном до
60 мл/хв. З порушенням якої функції нирок це пов’язано?
+ Клубочкової фільтрації
#У хворого на хронічну ниркову недостатність з’явилися анорексія, диспепсія, порушення ритму
серця, свербіння шкіри. Який механізм розвитку цих порушень є головним?
+ Накопичення продуктів азотистого обміну в крові

#Хворий чоловік на протязі 17 років страждає хронічним гломерулонефритом. Пульс 82 за хвилину.


АТ 190/120 мм рт.ст. Що являється первинним механізмом підвищення артеріального тиску у
хворого?
+Підвищення загального периферичного опору.
#У жінки обмежений кровотік у нирках, підвищений артеріальний тиск. Гіперсекреція якого гормону
зумовила підвищений тиск?
+Ренін

#При розтині хворої 28-ми років, що померла від уремії, виявлені збільшені строкаті нирки з
осередками крововиливів. Патогістологічно в судинних клубочках виявлені гематоксилінові тільця,
капілярні мембрани клубочків у вигляді дротяних петель, гіалінові тромби та осередки
фібриноїдного некрозу. За патогенезом гіперчутливість якого типу лежить в основі описаної
хвороби?
+Гіперчутливість III типу (імунокомплексна)

#У хворого на хронічний дифузний гломерулонефрит розвинулася хронічна недостатність нирок. В


термінальній стадії ХНН розвивається оліго- та анурія, що спричиняється:
+Зниженням маси діючих нефронів
#До лікаря звернувся чоловік 27-ми років. При огляді було виявлено збільшення кистей, стоп та
нижньої щелепи. Крім того спостерігалися деформація суглобів (kiphosis), гормональні порушення
(імпотенція, атрофія яєчок). Функція якої залози порушена?
+ Передня частка гіпофізу

#У чоловіка 41-го року відзначаються періодичні напади серцебиття (пароксизми), сильне


потовиділення, напади головного болю. При обстеженні виявлена гіпертензія, гіперглікемія,
підвищення основного обміну, тахікардія. При якій патології наднирників спостерігається подібна
картина?
+ Гіперфункція мозкового шару

#Жінка 37-ми років звернулася до ендокринологічної клініки з виразним тремором кінцівок.


Гіперпродукція якого гормону здатна викликати такі порушення?
+Тироксин

#У пацієнта з підвищеним артеріальним тиском, тремором, тахікардією, була діагностовано


доброякісна пухлина мозкової речовини наднирників. Гіперсекреція якого гормону викликає таку
симптоматику?
+Адреналін
#Після перенесеного сепсису у хворої 27-ми років з'явився бронзовий колір шкіри, характерний для
аддісонової хвороби. Механізм гіперпігментації полягає в підвищенні секреції такого гормону:
+ Меланоцитстимулюючий

#Пацієнта турбують поліурія (7 л на добу) і полідипсія. При обстеженні не виявлено ніяких розладів
вуглеводного обміну. Дисфункція якої ендокринної залози може бути причиною даних порушень?
+Нейрогіпофіз

# Хлопчик 5-ти місяців госпіталізований з приводу тонічних судом. Хворіє з народження.


Об’єктивно: волосся жорстке, нігті витончені та ламкі, шкірні покриви бліді та сухі. В біохімічному
аналізі крові: кальцій - 0,5 ммоль/л (норма - 0,75-2,5 ммоль/л), фосфор - 1,9 ммоль/л (норма - 0,646-
1,292 ммоль/л). З чим пов’язані ці зміни?
+Гіпопаратиреоз

#До лікаря звернулася жінка 32-х років зі скаргами на відсутність лактації після народження дитини.
Дефіцитом якого гормону можна пояснити дане порушення?
+Пролактин
#Чоловік середнього віку виїхав до іншої країни на обіцяну йому роботу, але працевлаштуватися
тривалий час йому не вдавалося. Які з ендокринних залоз були виснажені у цієї людини найбільше?
+Наднирники

#У дівчинки діагностований адреногенітальний синдром (псевдогермафродитизм). Надмірна секреція


яких гормонів наднирників обумовила дану патологію?
+Андрогени

#При огляді пацієнта виявлене надмірне розростання кісток і м'яких тканин обличчя, збільшені
розміри язика, розширені міжзубні проміжки в збільшеній зубній дузі. Які зміни секреції гормонів у
нього найбільш вірогідні?
+ Збільшена секреція соматотропного гормону
#Чоловіку 46-ти років, що хворіє на дифузний токсичний зоб, була проведена операція резекції
щитоподібної залози. Після операції відмічаються відсутність апетиту, диспепсія, підвищена
нервово-м'язова збудливість. Маса тіла не збільшилася. Температура тіла у нормі. Чим, із нижче
переліченого, обумовлений стан хворого?
+Зниженням продукції паратгормону

#У підлітка внаслідок радіоактивного опромінення значно постраждала лімфоїдна система, відбувся


розпад великої кількості лімфоцитів. Відновлення нормальної формули крові можливо завдяки
діяльності залози:
+Тимус
#Жінка 44-х років скаржиться на загальну слабкість, біль у ділянці серця, значне збільшення маси
тіла. Об’єктивно: обличчя місяцеподібне, гірсутизм, АТ- 165/100 мм рт.ст., зріст - 164 см, вага - 103
кг; переважно накопичення жиру на шиї, верхньому плечовому поясі, животі. Що є основним
патогенетичним механізмом ожиріння у жінки?
+Підвищення продукції глюкокортикоїдів

#Недбалий студент раптово зустрівся з деканом. Концентрація якого гормону найшвидше


збільшиться в крові студента?
+ Адреналін
-#Жінка 26 років через рік після важких пологів з кровотечею скаржиться на загальну слабкість,
втрату маси тіла на 18 кг, відсутність менструацій. Об'єктивно: гіпоплазія молочних залоз.
Діагностовано хвороба Сіммондса. Що являється основним механізмом втрати ваги у жінки?
+зниження продукції гормонів аденогіпофізу

#У хворого при обстеженні виявлено гірсутизм, "місяцеподібне обличчя", смуги розтягування на


шкірі живота. Артеріальний тиск 190/100 мм рт.ст., глюкоза крові - 17,6 ммоль / л. При якому із
зазначених видів патології зустрічається подібна картина?
+гіперфункції кори надниркових залоз

#Після двостороннього видалення надниркових залоз у собаки з'явилася м'язова слабкість, адинамія,
зниження температури тіла, гіпоглікемія. Який з перерахованих ознак ще може спостерігатись при
надниркової недостатності?
+ Артеріальна гіпотонія

#У хворого з гіперфункцією щитовидної залози підвищена температура тіла. Яке порушення


енергетичного обміну є головним у підвищенні температури при цьому?
+Роз єднання окиснення та окисного фосфорування.

#Жінка 55 років, яка проживає в гірській місцевості, діагностований ендемічний зоб. Об'єктивно:
трохи підвищеної вгодованості, загальмована, апатична, збільшення щитовидної залози. Дефіцит
якого з перелічених нижче елементів викликає цей стан?
+йод

#У хлопчика діагностовано ендемічний зоб. Який основний механізм розвитку гіпотиреозу у хлопця?
+зниження продукції тироксину та трийодтиранину

#Методом непрямої калориметрії встановлено, що основний обмін досліджуваного на 40% нижче


повинного. Порушення діяльності якої ендокринної залози можна припустити?
+Щитовидної залози.
#У 50-річної хворої після перенесеного інфекційного захворювання головного мозку значно
збільшився діурез до 12 л за добу. При аналізі крові кількість глюкози становила 4,1 ммоль/л. Якого
гормону найвірогідніше недостатньо?
+Антидіуретичного
#У клініці встановлено, що при вагітності тяжкість симптомів ревматоїдного артриту різко
знижується. Прискорення секреції яких гормонів, що володіють протизапальну дію, спостерігається
при цьому?
+Глюкокортикоїдів
#Жінка 29 років скаржиться на загальну слабкість, втрату маси тіла на 22 кг, аменорею. Хворіє після
пологів. Об’єктивно: ріст 162 см, маса 46 кг, гіпоплазія молочних залоз. Діагностована гіпофізарна
кахексія. Зменшення продукції якого гормону явилось найбільш суттєвим у схудненні жінки?
+Соматотропіну.

#У хворої після пологів развинулись такі ознаки: атрофія скелетних м(язів, дістрофія шкіри,
випадання волосся, гіпотрофія внутрішніх органів, зниження температури тіла, артеріального
тиску, рівня глюкози в крові, атрофія щитовидної, надниркових та статевих залоз. Для якої патології
характерні ці ознаки?
+Атрофії гіпофіза

#У хворого встановлено стабільне підвищення температури тіла, тахікардія, емоційна лабільність,


тремор. Зі зміною продукціі якого гормона пов’язане виникнення цього стану?
+ Тироксину.

#У ліквідатора аварії на Чорнобильській АЕС через деякий час з'явилися скарги на підвищену
збудливість, нервозність, серцебиття, зниження маса тіла, постійну слабкість, тремтіння тіла,
відчуття жару, погану переносимість тепла. Гіперфункція якої залози може бути причиною
зазначених змін:
+щитовидної залози
#У собаки з ендокринною патологією було виявлено: зменшення споживання кисню у стані спокою,
зниження толерантності до глюкози, зниження температури тіла. Недостатністю якого гормону
можна пояснити виявлені зміни?
+Тироксину
#Хворий жаліється на підвищену дратливість, періодичний субфібрілітет. Частота пульсу -120/хв. В
крові збільшена кількість гормонів Т3 і Т4. Яку ендокринну патологію найбільш логічно
запідозрити?
+ Гіпертіреоз.
#У хворого спостерігаються напади артеріальної гіпертензії, що супроводжуються тахікардією,
рясним потовиділенням, різким болем в надчеревній ділянці. Для якого з перечисленних пухлинних
захворювань залоз внутрішньої секреції найбільш характерні дані симптоми?
+ Феохромоцитома

#Жінка 53 років, ріст 163 см., вага тіла 92 кг, рівномірне відткладання жиру, лице одутле,
малорухома, апатична. При натискуванні шкіри ноги залишається ямка. Порушенням функції якої
залози обумовлений стан хвороби?
+Щитовидної.
#У хворого 48 років спостерігається артеріальна гіпертензія, головний біль, м’язова слабкість,
судоми. В крові знижена концентрація К+ і підвищена концентрація Na+, що є наслідком
гіперсекреції:
+Альдостерону

#У чоловіка 35-ти років феохромоцитома. В крові спостерігається підвищений рівень адреналіну та


норадреналіну, концентрація вільних жирних кислот зросла в 11 разів. Активація якого ферменту під
впливом адреналіну підвищує ліполіз?
+ТАГ -ліпаза

#При гістологічному дослідженні щитоподібної залози визначається значна інфільтрація


лімфоцитами з утворенням лімфоїдних фолікулів, руйнування паренхіматозних елементів,
розростання волокон сполучної тканини. Для якого захворювання характерна така картина?
+Зоб Хашимото

#Пацієнт 16-ти років, що страждає на хворобу Іценко-Кушінга, консультований з приводу надмірної


ваги тіла. При опитуванні з’ясувалося, що енергетична цінність спожитої їжі складає 1700-1900
ккал/добу. Яка провідна причина ожиріння у даному випадку?
+Надлишок глюкокортикоїдів

#У чоловіка 25-ти років з переломом основи черепа виділяється великий об’єм сечі з низькою
відносною щільністю. Причиною змін сечоутворення є порушення синтезу такого гормону:
+Вазопресин
#An unconscious patient was brought into the hospital. The smell of acetone can be detected from the
patient’s mouth. Blood glucose - 25 mmol/L, ketone bodies -0.57 mmol/L. What hormone deficiency can
result in the development of this condition?
+Insulin

#У людини виявлена пухлина одного з відділів головного мозку, внаслідок чого в неї порушена
здатність підтримувати нормальну температуру тіла. Яка структура головного мозку пошкоджена?
+ Гіпоталамус

# В клініку доставлено чоловіка з травмою спини. Під час обстеження виявлено перелом хребців
грудного відділу. Під час об’єктивного огляду нейрохірургом виявлено: нижче рівня перелому з
правого боку відсутня глибока чутливість, з лівого боку - порушена температурна та тактильна
чутливість. Яке ураження з боку спинного мозку є у хворого?
+ Синдром Броун Секара

# У хворої 49-ти років відзначається обмеження довільних рухів у лівих кінцівках. Тонус м’язів у
лівих руці та нозі підвищений за спастичним типом, посилені місцеві сухожилкові рефлекси,
виявляються патологічні рефлекси. Який найбільш імовірний механізм призвівдо розвитку м’язової
гіпертонії та гіперрефлексії?
+ Зниження гальмівних низхідних впливів
#В експерименті у тварини в результа¬ті проведеного перетинання депресорного нерва та
руйнування каротидних клубочків розвинулась стійка гіпертензія. З порушен¬ням якої функції
нервової системи пов’яза¬не це явище?
+Трофічна

#У людини звужені зіниці. Чим це зумовлено?


+Зростання тонусу парасимпатичних центрів
#У стоматолога на прийомі дуже неспокійний пацієнт, який ніяк не може зручно влаштуватися в
кріслі, норовить схопити доктора за руку, заглядає на маніпуляційний стіл, цікавиться в медсестри,
чи стерильні інструменти. Який темперамент у цього пацієнта?
+Холерик

#У жінки 68 років після інсульту відсутні рухи в верхній та нижній правій кінцівках. Тонус м’язів
цих кінцівок і рефлекси в них підвищені. Є патологічні рефлекси. Яка це форма параліча?
+Геміплегія
#У жінки, що хворіє на міастенію, виникли розлади дихання, що вимагали застосування штучної
вентиляції легень. Який вид недостатності розвинувся у даної хворої?
+Нервово-м'язовий.

#Після крововиливу в мозок у хворого стали неможливими активні рухи лівої руки і ноги. Тонус
м'язів цих кінцівок збільшений, їх спинальні рефлекси різко посилені, розширені зони рефлексів.
Позитивний рефлекс Бабінського Назвіть вид розладу нервової системи у хворого.
+Центральний параліч.

#У хворого через 7 років після резекції шлунка виявлена гіперхромна В12


дефіцитна анемія і фунікулярний міелоз. Який патогенез уражень спинного мозку?
+ Накопичення метилмалонової кислоти.

#У хворого в результаті вогнепального поранення стегна пошкоджений сідничний нерв. Будь-який


вплив на хвору кінцівку спричиняє жорсткий, нестерпний біль. Який механізм формування болісних
відчуттів найбільш ймовірний у цьому випадку?
+Каузалгічний

#У тварини в експерименті проведена перерізка обох блукаючих нервів. Як зміниться характер


дихання у тварини?
+ Рідке і глибоке

#У хворого після відкритої травми хребта виявлено розрив правої половини спинного мозку.
Зникнення якого виду чутливості слід очікувати тількі з боку розриву?
+ Пропріоцептиної

#Під час бойових дій у госпіталь доставили солдата з тяжким осколочним пошкодженням хребта. У
пораненого встановлено наявність перерізки правої половини спинного мозку (синдром Броун-
Секара). Зникненням якого виду чутливости проявляеться цей синдром?
+ пропріоцептивної - справа

#У хворої Л., 40 років, через місяць після поєднаної автомобільної травми відсутні активні рухи у
всіх суглобах правої ноги. Обсяг м'язів в області гомілки правої ноги на 2 см менше, ніж на лівій.
Ахилов і колінний рефлекси справа відсутні. Визначається термгіпестезія і гіпальгезія на зовнішній
поверхні правої ноги, втрата пропріоцептивної чутливості в ділянці стопи. Порушенням якого
відділу рухового аналізатора обумовлені наявні розлади?
+Периферичних нервів

#Внаслідок ДТП у потерпілої 37-ми років виникло неутримання сечі. Які сегменти спинного мозку
пошкоджені?
+S2-S4

#У хворого, який на тлі атеросклерозу переніс ішемічний інсульт, спостерігається порушення рухової
функції у вигляді геміплегії. Яка з перерахованих ознак є характерною для уражених кінцівок при
даній патології?
+Гіпертонус м’язів

#Жінку 44-х років вжалила оса внаслідок чого розвинувся шок. В анамнезі вже була важка алергічна
реакція на жалення оси. Об’єктивно: пульс 179/хв., слабкий,АТ-80/40 ммрт.ст., ЧД26/хв. Яка
провідна ланка патогенезу анафілактичного шоку?
+ Зниження периферійного опору судин

# У хворого діагностовано септичний ендокардит. Температура тіла протягом 5-ти днів коливалася в
межах 39,5oC 40,2oC. На 6-й день на тлі різкого зниженнятемпературидо35,2oC розвинувся колапс.
Який головний механізм колапсу?
+ Вазодилатація
#В результаті травми пошкоджений спинний мозок (з повним розривом) на рівні першого шийного
хребця. Що відбудеться з диханням?
+ Припиняється
#Чоловік 53-х років доставлений у стаціонар у непритомному стані. Об’єктивно: шкіра суха, дихання
часте поверхневе, запах ацетону відсутній, Ps- 126/хв., АТ- 70/40 мм рт.ст. Вміст глюкози у крові 48
ммоль/л, реакція сечі на ацетон негативна. Для якого із перелічених станів найбільш характерні
симптоми у хворого?
+ Гіперосмолярна кома

#У реанімаційному відділенні знаходиться хворий у коматозному стані. При дослідженні крові


відзначено збільшення концентрації іонів K+ і зменшення - Ca++, ацидоз, збільшення рівнів сечовини,
сечової кислоти. Який вид коми за етіологією найбільш імовірний?
+Ниркова
#У хворого 23-х років в результаті черепно-мозкової травми виник набряк мозку. Який механізм
пошкодження клітин безпосередньо призвів до набряку мозку?
+Електролітно-осмотичний

#Внаслідок дії на організм електричного струму міської електромережі впродовж 0,1 сек у напрямку
"права рука-голова" у постраждалого спостерігалась зупинка дихання. Вкажіть найбільш імовірний
механізм цього ускладнення:
+ Тотальний параліч дихального центру
#Хворого доставлено у медичний заклад в коматозному стані. Зі слів супроводжуючих вдалося
з'ясувати, що він знепритомнів під час тренування на завершальному етапі марафонської дистанції.
Який вид коми найімовірніше можна запідозрити у даного пацієнта?
+Гіпоглікемічна

#У хворого діагностовано грип. Після прийому антипіретиків стан його різко погіршився: свідомість
потьмарена, АТ- 80/50 мм рт.ст., РS- 140/хв., температура тіла різко знизилась до 35,8°С. Яке
ускладнення виникло у даного хворого?
+Колапс

#У підлітка 12-ти років, який впродовж 3-х місяців різко схуд, вміст глюкози у крові 50 ммоль/л. У
нього розвинулася кома. Який головний механізм її розвитку?
+Гіперосмолярний
#Внаслідок землетрусу чоловік 50-ти років дві доби перебував під завалом. Після звільнення з-під
завалу рятівниками у нього був встановлений синдром тривалого розчавлення. Виникнення якого
ускладнення в подальшому найбільш вірогідне?
+Гостра ниркова недостатність

#У хворого внаслідок травми розвинувся травматичний шок, у перебігу якого мали місце наступні
порушення: АТ- 140/90 мм рт.ст., Ps- 120/хв. Хворий метушливий, багатослівний, блідий. Якій стадії
шоку відповідає цей стан?
+ Еректильна

#У водія, який потрапив у ДТП, отримав травму та знаходиться у стані шоку, спостерігається
зменшення добової кількості сечі до 300 мл. Який основний патогенетичний фактор цієї зміни
діурезу?
+ Падіння артеріального тиску

#Хвора доставлена бригадою швидкої допомоги. Об’єктивно: стан важкий, свідомість відсутня,
адинамія. Шкірні покриви сухі, запалі очі, ціаноз обличчя, тахікардія, запах ацетону з рота.
Результати аналізів: глюкоза крові - 20,1 ммоль/л (у нормі - 3,3-5,5 ммоль/л), у сечі - 3,5% (у нормі -
0). Який найбільш вірогідний діагноз?
+Гіперглікемічна кома

#До клініки поступив чоловік 40-ка років, якого укусила гадюка. Де переважно буде проходити
гемоліз еритроцитів у цьому випадку?
+У кровоносному руслі
#Хворого на трансмуральний інфаркт міокарда лівого шлуночка переведено до відділення реанімації
у важкому стані. АТ-70/50 мм рт.ст., ЧСС- 56/хв., ЧД- 32/хв. Зазначте головну ланку в патогенезі
кардіогенного шоку:
+Падіння серцевого викиду
#Жінка 22 років, поступила у стаціонар в коматозном стані. Об'єктивно: шкіра суха, бліда, запах
ацетону з рота, дихання Куссмауля, пульс 130 уд/хв., АТ- 95/60 мм.рт.ст., вміст глюкози у крові - 37.7
ммоль/л. Для якого із перелічених станів найбільш характерні симптоми, що спостерігаються?
+гіперкетонемічної коми

#Хворого доставили в клініку в коматозному стані. Дихання шумне, глибоке. Чути запах ацетону.
Вміст глюкози в крові 15,2 ммоль/л, кетонові тіла - 100 мкмоль/л. Для якого виду коми характерні
такі розлади?
+Кетоацидотичної

#У хворого після важкої травми грудної клитки розвинувся шок та з’явилися ознаки гострої
ниркової недостатності [ГНН]. Що є провідним механізмом розвитку ГНН в даному випадку?
+ Падіння артеріального тиску

#В реанімаційне відділення доставили водія, який попав в автомобільну аварію. Він не зразу реагує
на питання, байдужий до всього, блідий, дихання поверхневе, рідке, АТ дорівнює 75/50 мм рт. Ст.
Назвіть головну ланку патогенезу вказаної патології.
+Гальмування центральної нервової системи.

#Хворий Ж., 48 років, знаходиться в реанімації після отруєнням чотирьоххлористим вуглеводнем.


Стан важкий, хворий без свідомості, пульс ниткоподібний, слабкого наповнення, ЧСС 40 уд / хв АТ
75/40 мм.рт.ст., дихання періодичне типу Біота. В крові прямий білірубін - 155 мкмоль / л. У сечі
високий вміст аміаку і білірубіну. Що стало основною причиною описаного стану у хворого?
+ Печінкова недостатність,

#У хворого в третьому періоді лихоманки настав критичний падіння температури тіла. При цьому
спостерігалася тахікардія і зниження артеріального тиску до 80/60 мм рт. ст. Який вид колапсу
розвинувся при цьому?
+ Інфекційно-токсичний
#У чоловіка 35 років через 30 хвилин після автомобільної аварії виявлена масивна травма нижніх
кінцівок без значної зовнішньої крововтрати. Постраждалий знаходиться у збудженному стані . Який
компонент патогенезу травматичного шоку є у пацієнта ведучим і потребує негайного корегування?
+Біль.

#Введення знеболюючого пацієнту перед екстракцією зуба призвело до розвитку анафілактичного


шоку, який супроводжувався розвитком олігурії. Який патогенетичний механізм зумовив зменшення
діурезу в даній клінічній ситуації?
+Зниження гідростатичного тиску в капілярах клубочків

#Після операції на кишечнику у хворого з’явились симптоми отруєння аміаком за типом печінкової
коми. Який механізм дії аміаку на енергозабезпечення ЦНС?
+Гальмування ЦТК в результаті зв’язування альфа-кетоглутарату
#Під час автомобільної аварії людина отримала сильний удар в епігастральну ділянку, внаслідок чого
виникла зупинка серця. Що могло стати причиною таких змін серцевої діяльності?
+Підвищення тонусу блукаючого нерва
#У хворого на гострий міокардит з’явилися клінічні ознаки кардіогенного шоку. Який із вказаних
нижче патогенетичних механізмів є провідним в розвитку шоку?
+Зниження насосної функції серця
#Хворий 50-ти років страждає на гіпертонічну хворобу. Під час фізичного навантаження у нього
з’явилося відчут- тя слабкості, нестачі повітря, синюшність слизової оболонки губ, шкіри обличчя.
Дихання супроводжувалося чутними на відстані вологими хрипами. Який механізм лежить в основі
виникнення такого синдрому?
+Гостра лівошлуночкова недостатність

#Чоловіку 44 років з гострою пневмонією призначили пеніцилін внутрішньом’язово. Після


проведення ін’єкції стан хворого різко погіршився: з’явилась задишка, хворий покрився холодним
потом. Пульс 140/хв. слабкого наповнення. АТ 90/40 мм.рт.ст. Яке ускладнення найбільш ймовірно
виникло у хворого?
+Анафілактичний шок.
#Жінка 29-ти років поступила у пологове відділення в терміні вагітності 40 тижнів. Шийка матки
розкрита, але скорочення матки відсутнє. Лікар дав засіб гормональної природи для посилення
пологової діяльності. Назвіть засіб:
+Окситоцин

#У хворої внаслідок запалення порушена ендокринна функція фолікулярних клітин фолікулів


яєчника. Синтез яких гормонів буде пригнічений?
+ Естрогени
#Секреція яких гормонів гіпофізу гальмується після прийому оральних контрацептивів, які містять
статеві гормони?
+Гонадотропні

#У хворої внаслідок запалення порушена ендокринна функція фолікулярних клітин фолікулів


яєчника. Синтез яких гормонів буде пригнічений?
+Фолікулостимулюючий гормон
#Чоловіку 46-ти років, що хворіє на дифузний токсичний зоб, була проведена операція резекції
щитоподібної залози. Після операції відмічаються відсутність апетиту, диспепсія, підвищена
нервово- м’язова збудливість. Маса тіла не збільшилася. Температура тіла у нормі. Чим, із нижче
переліченого, обумовлений стан хворого?
+Зниженням продукції паратгормону

#У дитини 2-х років виникли судоми внаслідок зниження концентрації іонів кальцію в плазмі крові.
Функція якого ендокринного органу знижена?
+Прищитоподібні залози

#Після перенесеного сепсису у хворої 27-ми років з’явився бронзовий колір шкіри, характерний для
аддісонової хвороби. Механізм гіперпігментації полягає в підвищенні секреції такого гормону:
+Меланоцитстимулюючий
#Хворому 45-ти років при оперативному втручанні на щитовидній залозі випадково видалили
прищитовидні залози. Це призвело до:
+ Тетанії
#У мишей з відсутнім волосяним покривом (тобто nude - голі) не було клітинних реакцій
уповільненого типу. Для цієї патології найбільш вірогідним є:
+Відсутність вилочкової залози

#Внаслідок травмування у хворого видалили прищитовидні залози, що супроводжувалося: млявістю,


спрагою, різким підвищенням нервово-м’язової збудливості. З порушенням обміну якої речовини це
пов’язано:
+Кальцію

# Стресовий стан і больове відчуття у пацієнта перед візитом до стоматолога супроводжуються


анурією (відсутністю сечовиділення). Це явище зумовлене збільшенням:
+ Секреції вазопресину та адреналіну
#З метою оцінки адаптації до фізичного навантаження лікар провів обстеження робітників після
виконання важкої праці. Які зміни в загальному аналізі крові можна виявити?
+Перерозподільчий лейкоцитоз

#Недбалий студент раптово зустрівся з деканом. Концентрація якого гормону найшвидше


збільшиться в крові студента?
+ Адреналін

#Під час футбольного матчу між вболівальниками виникла сутичка. На фоні негативних емоцій в
одного учасника сутички були розширені зіниці й підвищене серцебиття. Активація якої системи
організму забезпечує такі вегетативні зміни при негативних емоціях?
+Симпато-адреналова

#Чоловік 25 років, потупив у лікарню через одну годину після автомобільної катастрофи. Картина
гострого стресу без розповсюджених пошкоджень. Яка із перелічених змін формених елементів
крові буде найбільш характерною для цього стану?
+еозинопенія
#Жінка 30-ти років народила хлопчика з розщепленням верхньої губи ("заяча губа"і "вовча паща").
При додатковому обстеженні виявлені значні порушення нервової, серцево-судинної систем та зору.
При дослідженні каріотипу діагностована трисомія за 13-ю хромосомою. Який синдром наявнийу
хлопчика?
+ Патау
#До медико-генетичної консультації звернулася жінка 30-ти років у якої в ядрах більшості клітин
епітелію слизової оболонки щоки було виявлено по два тільця Барра. Який попередній діагноз можна
встановити?
+ Трисомія за X-хромосомою

#У подружжя народився син, хворий на гемофілію. Батьки здорові, а дідусь за материнською лінією
також хворий на гемофілію. Визначте тип успадкування ознаки.
+ Рецесивний, зчеплений зі статтю

#Для діагностування деяких хромосомних хвороб використовують визначення статевого хроматину.


Назвіть хворобу, при якій потрібне це визначення:
+ Синдром Шерешевського-Тернера

#При проведенні амніоцентезу в клітинах плоду виявлено по 2 тільця статевого хроматину (тільця
Барра). Для якого захворювання характерна дана ознака?
+ Трисомія X
#Виникнення нижчеперерахованих за¬хворювань пов’язане із генетичними факторами. Назвіть
патологію із спадковою схильністю:
+Цукровий діабет

#Юнак 18-ти років звернувся до медико- генетичної консультації з приводу відхилень у фізичному і
статевому розвитку. При мікроскопії клітин слизової оболонки рота виявлене одне тільце Барра.
Вкажіть найбільш імовірний каріотип юнака:
+47, XXY
#У чоловіка 22-х років високого росту та астенічної будови тіла з ознаками гіпогонадизму,
гінекомастією та зменшеною продукцією сперми (азооспермія) виявлено каріотип 47 ХХУ. Який
спадковий синдром супроводжується такою хромосомною аномалією?
+Клайнфельтера

#Жінка 27-ми років звернулася зі скаргами на дисменорею та безпліддя. При обстеженні виявлено:
зріст жінки 145 см, недорозвинені вторинні статеві ознаки, на шиї крилоподібні складки. При
цитологічному дослідженні в соматичних клітинах не виявлено тілець Барра. Який діагноз встановив
лікар?
+Синдром Шерешевського-Тернера
#У новонародженої дитини спостерігаються: судоми, блювання, жовтяниця, специфічний запах сечі.
Лікар-генетик висловив підозру про спадкову хворобу обміну речовин. Який метод дослідження
необхідно використати для постановки точного діагнозу?
+Біохімічний

#В родині зростає дочка 14-ти років, у якої спостерігаються деякі відхилення від норми: зріст нижче,
ніж у однолітків, відсутні ознаки статевого дозрівання, шия дуже коротка, плечі широкі. Інтелект в
нормі. Яке захворювання можна припустити?
+Синдром Шерешевського-Тернера

#Чоловік звернувся до лікаря з приводу безпліддя. Має високий зріст, зниження інтелекту,
недорозвинення статевих залоз. У епітелії слизової оболонки порожнини рота виявлений статевий
хроматин (1 тільце Барра). Про яку патологію можна думати?
+ Синдром Клайнфельтера

#У здорових батьків, спадковість яких не обтяжена, народилась дитина з чисельними вадами


розвитку. Цитогенетичний аналіз виявив в соматичних клітинах дитини трисомію за 13-ю
хромосомою (синдром Патау). З яким явищем пов'язане народження такої дитини?
+ Порушення гаметогенезу
#До гінеколога звернулася жінка 28-ми років з приводу безпліддя. При обстеженні знайдено:
недорозвинені яєчники та матка, нерегулярний менструальний цикл. При дослідженні статевого
хроматину в більшості соматичних клітин виявлено два тільця Бара. Яка хромосомна хвороба най-
більш вірогідна у жінки?
+Синдром трипло-Х

#У новонародженого хлопчика спостерігається деформація мозкового та лицьового черепа,


мікрофтальмія, деформація вушної раковини, вовча паща, і т.ін. Каріотип дитини - 47, ХУ, 13+. Про
яку хворобу йде мова?
+Синдром Патау
#До генетичної консультації звернулася сімейна пара, в якій чоловік хворіє на інсулінозалежний
цукровий діабет, а жінка здорова. Яка вірогідність появи інсулінозалежного діабету у дитини цього
подружжя?
+Більше, ніж в популяції
#При диспансерному обстеженні хлопчику 7-ми років встановлено діагноз - дальтонізм. Батьки
здорові, кольоровий зір у них у нормі, але у дідуся по материнській лінії така ж аномалія. Який тип
успадкування цієї аномалії?
+ Рецесивний, зчеплений зі статтю

#При медичному огляді у військкоматі був виявлений хлопчик 15-ти років, високого зросту, з
євнухоїдними пропорціями тіла, гінекомастією, з ростом волосся на лобку за жіночим типом.
Відмічається відкладання жиру на стегнах, відсутність росту волосся на обличчі, високий голос;
коефіцієнт інтелекту знижений. Виберіть каріотип, що відповідає даному
захворюванню:
+ 47, XXY

#Внаслідок впливу альфа-випромінювання ділянка ланцюга ДНК повернулася на 180 градусів. Яка з
перелічених видів мутацій відбулася в ланцюзі ДНК?
+Інверсія

#У клінічно здорових батьків народилася дитина, хвора на фенілкетонурію (аутосомно-рецесивне


спадкове захворювання). Які генотипи батьків?
+Аа х Аа

#В медико-генетичній консультації при обстеженні хворого хлопчика в крові були виявлені


нейтрофільні лейкоцити з однією "барабанною паличкою". Наявність якого синдрому можна
запідозрити у хлопчика?
+Синдром Клайнфельтера

#У жінки під час гаметогенезу (в мейозі) статеві хромосоми не розійшлися до протилежних полюсів
клітини. Яйцеклітина була запліднена нормальним сперматозоїдом. Яке хромосомне захворювання
може бути у дитини?
+ Синдром Шерешевського-Тернера

#При обстеженні 2-місячного хлопчика педіатр звернув увагу, що плач дитини схожий на нявкання
кішки, відзначаються мікроцефалія і вада серця. За допомогою цитогенетичного методу був
встановлений каріотип - 46 XY, 5р-. На якій стадії мітозу досліджували каріотип хворого?
+Метафаза
#Чоловік, що страждає на спадкову хворобу, одружився із здоровою жінкою. У них було 5 дітей, три
дівчинки і два хлопчика. Усі дівчатка успадкували хворобу батька. Який тип спадкування цього
захворювання?
+Домінантний, зчеплений з X - хромосомою

#При вивченні родоводу сім’ї, в якій спостерігається гіпертрихоз (надмірне оволосіння вушних
раковин), виявлена ознака трапляється в усіх поколіннях тільки у чоловіків і успадковується від
батька до сина. Визначте тип успадкування гіпертрихозу:
+Зчеплений з Y-хромосомою

#У хлопчика зі спадково обумовленими вадами зразу ж після народжен-ня спостерігався характерний


синдром, який називають "крик кішки". У ран-ньому дитинстві малюк мав "нявкаючий" тембр
голосу. Під час дослідження каріотипу цієї дитини було виявлено:
+Делецію короткого плеча 5-ї хромосоми

#У новонародженої дитини виявлено наступну патологію: аномалія розвитку нижньої щелепи та


гортані, що супроводжується характерними змінами голосу, а також мікроцефалія, вада серця,
чотирьохпалість. Найбільш імовірною причиною таких аномалій є делеція:
+Короткого плеча 5-ої хромосоми

#У фібробластах шкіри дитини із хворобою Дауна виявлено 47 хромосом. Визначте тип аномалії:
+Трисомія 21
#У юнака 18-ти років діагностовано хворобу Марфана. При дослідженні встановлено: порушення
розвитку сполучної тканини, будови кришталика ока, аномалії серцево-судинної системи,
арахнодактилія. Яке генетичне явище зумовило розвиток цієї хвороби?
+Плейотропія

#A victim of a traffic accident was received by the intensive care unit. The patient is in a grave condition
that can be characterized as a severe pathologic process that leads to exhaustion of vital functions and puts
the patients into the marginal state between life and death due to critical reduction of capillary circulation in
the affected organs. The patient is in the state of:
+Agony
#Insufficiency of the mitral valve was founded at man who 12 years ago suffered from rheumatic
myocarditis. Investigations have shown that the inflammatory process is absent, minute volume of
circulation is sufficient. Name the notion of general nosology.
+Pathological state

#Atrophy of bone marked in the patient after the removal of teeth. This is an example of:
+Pathological condition

#A 50-year –old man has a stomach ulcer. After the treatment digestion become to normal, pain disappeared,
and his mood improved. However, in a couple of weeks the patient developed pain in epigastric region,
heartburn and belching. How this situation might be interpreted?
+Disease relapse
#A 12-year-old boy after coming home from the school began to complaint of headache, nausea, chill,
periodic muscle pain, loss of appetite, and fatigue. What disease period can show such signs?
+Prodromal

#A student fell ill with acute respiratory disease at the end of winter following long period of mental
overloading. What is the cause of the disease?
+Pathogenic microorganism
#A 39-years-old patient has been suffering from gastric ulcer for last 4 years. Pain in epigastric region,
heartburn, nausea, and constipation appear mainly in autumn and spring. Name this condition.
+Relapse

#Gasping respiration appears in a patient with severe lung pathology. What terminal condition is this
characteristic for?
+Agony

#A 10-yeaes-old child endured several rheumatic attacks. At examination of him it was established that he
had inflammatory process in his joints and signs of mitral valve insufficiency. What pathological appearance
in this patient may be attributed?
to "disease"?
+Rheumatism
#A 28-year-old patient with symptoms of acute appendicitis was admitted to the surgical department. Acute
pains in his right epigastric area and near the umbilicus were registered during palpation on examination.
Schetkin-Blumberg symptom was positive. Which period of disease was observed in that patient?
+Period of manifestation

#Increase in pulse rate, respiratory rate, and increase of BP were noticed on the height of 1000 m above the
sea in a 25-year-old woman, coming for holyday in the mountainous resort. In some time, all those
symptoms disappeared. Which process was noticed in a woman?
+Adaptation

#A man with the complex of deviation of his health is considered sick. What is the more typically for
disease?
+Disorder of physiological regulation of functions

#A 25-year-old man has spent a long time in the sun under high air humidity. As a result of it his body
temperature rose up to 39oC. What pathological process is it?
+Hyperthermia
#The temperature of the ambient environment is 38oC and relative air humidity is 50%. What ways of heat
emission provide maintaining a constant temperature of the human body?
+Evaporation

#At the end of the working day a worker of a hot work shop has been delivered to a hospital. The patient
complains of headache, dizziness, nausea, general weakness. Objectively: the patient is conscious, his skin is
hyperemic, dry, hot to the touch. Heart rate is of 130/min. Respiration is rapid, superficial. What disorder of
thermoregulation is most likely to have occurred in this patient?
+Reduced heat transfer

#A patient with heatstroke was delivered to the admission room. What compensatory reactions develop in
the patient’s body in such case?
+Peripheral vasodilatation

#Acute general disorders are observed at long-term action of environmental high temperature on the
organism. What is the base of this phenomenon?
+Dehydration
#A patient due to long-term work in the apartment in the condition of hot microclimate the body temperature
rose considerably. The state became worse till evening: temperature of body 40°C, there was stupor, pallor
of the skin, bleeding from the nose. Arterial pressure was 80/50 mmHg. What extremal state is observed in a
patient?
+Thermal shock
#Having helped to eliminate consequences of a failure at a nuclear power plant, a worker got an irradiation
doze of 500 roentgens. He complains of headache, nausea, dizziness. What changes in leukocytes quantity
can be expected 10 hours after irradiation?
+Neutrophilic leukocytosis

A woman who was infected with toxoplasmosis during the pregnancy has a child with multiple congenital
defects. This is a result of:
+Teratogenesis

Medical examination at the military registration and enlistment office revealed that a 15-year-old boy was
high, with eunuchoid body proportions gynecomastia, female pattern of pubic hair distribution. The boy had
also fat deposits on the thighs, no facial hair, high voice, subnormal intelligence quotient. Which karyotype
corresponds with this disease?
+47, XXY

One of the parents is suspected of having phenylketonuria recessive gene. What is the risk of giving birth to
a child with inborn phenylketonuria?
+0%

A 28-year-old female patient consulted a gynecologist about sterility. Examination revealed underdeveloped
ovaries and uterus, irregular menstrual cycle. Analysis of the sex chromatin revealed 2 Barr’s bodies in most
somatic cells. What chromosome disease is most likely?
+Triple X syndrome
Sex chromosomes of a woman didn’t separate and move to the opposite poles of a cell during gametogenesis
(meiosis). The ovum was impregnated with a normal spermatozoon. Which chromosomal disease can be
found in her child?
+Turner syndrome

A couple has a son with hemophilia. The parents are healthy but the maternal grandfather also has
hemophilia. Specify the type of inheritance:
+Recessive sex-linked
During a prophylactic medical examination, a 7-year-old boy was diagnosed with daltonism. His parents are
healthy and have normal color vision, but his grandfather on his mother’s side has the same abnormality.
What is the type of the abnormality inheritance?
+Recessive, sex-linked
According to the phenotypic diagnosis a female patient has been provisionally diagnosed with X-
chromosome polysomia. This diagnosis can be confirmed by a cytogenetic method. What karyotype will
allow to confirm the diagnosis?
+47(ХХХ)

Examination of an 18-year-old girl revealed the following features: hypoplasia of the ovaries, broad
shoulders, narrow pelvis, shortening of the lower extremities, "sphinx neck". Mental development is normal.
The girl was diagnosed with Turner’s syndrome. What kind of chromosome abnormality is it?
+Monosomy X
A 35-year-old male patient has been referred by an andrologist for the genetic counselling for the deviations
of physical and mental development. Objectively: the patient is tall, has asthenic constitution, gynecomastia,
mental retardation. Microscopy of the oral mucosa cells revealed sex chromatin (single Barr body) in 30% of
cells. What is the most likely diagnosis?
+Klinefelter syndrome

A 25-year-old patient consulted a doctor about dysmenorrhea and infertility. Examination revealed that the
patient was 145 cm high and had underdeveloped secondary sex characteristics, alar folds on the neck.
Cytological study didn’t reveal any Barr bodies in the somatic cells. What diagnosis was made?
+Turner syndrome
Amniocentesis revealed two sex chromatin bodies (Barr bodies) in each cell of the sample. What disease is
this character typical for?
+Trisomy X

A boy referred to a genetics clinic was found to have 1 drumstick in blood neutrophils. The boy is likely to
have the following syndrome:
+Klinefelter
A female patient has medical-genetic consultation. Physical examination revealed pterygiumcolli deformity
(webbed neck), broad chest, underdeveloped breasts. Study of buccal epithelium cells revealed no X-
chromatin in the nuclei. This indicates that the patient has the following syndrome:
+Turner
A 2-year-old boy is diagnosed with Down syndrome. What chromosomal changes may be the cause of this
disease?
+Trisomy 21

Sex chromatin was detected during examination of a man’s buccal epithelium. It is characteristic of the
following chromosome disease:
+Klinefelter syndrome

A 45-year-old woman gave birth to a boy with cleft maxilla (cleft lip and palate). On additional examination
there are significant disturbances of the boy’s nervous, cardiovascular and visual systems. Karyotype
investigation allowed diagnosing the patient with trisomy 13. What syndrome is present in boy?
+Klinefelter
Clinical presentation of a woman allowed provisionally diagnosing her with X polysomy. Cytogenetic
method is applied to clarify the diagnosis. The diagnosis will be confirmed if the patient’s karyotype is?
+47,ХХХ

A 40-year-old pregnant woman underwent amniocentesis. Examination determined the fetal karyotype to be
47, XY+21. What fetal pathology was detected?
+Turner syndrome

A 20-year-old young man with tall stature, asthenic body type, signs of hypogonadism and gynecomastia,
and low sperm count (azoospermia) has karyotype 47XXY. What hereditary syndrome can be characterized
by this chromosomal anomaly?
+Turner syndrome

A child with a history of frequent tonsillitis and pharyngitis has been diagnosed with lymphadenopathy and
splenomegaly. His appearance is characterized by pastosity and paleness, muscular tissue is poorly
developed. Lymphocytosis is present. What kind of diathesis is it?
+Lymphohypoplastic diathesis

A child is pale, pastose, muscular tissue is bad developed, lymph nodes are enlarged. He often suffers from
tonsillitis and pharyngitis; blood has signs of lymphocytosis. The child is also predisposed to autoallergy
diseases. What type of diathesis can be presumed in this case?
+Lymphohypoplastic
On examination the patient was determined to have strong, balanced, inert type of higher nervous activity
according to Pavlov’s classification. What temperament according to Hippocrates is it?
+Choleric

After an immunoassay a child was diagnosed with immunodeficiency of humoral immunity. What is the
reason for the primary immunodeficiency development in the child?
+Hereditary abnormality of immune system

The immunoblot detected gp120 protein in the blood serum. This protein is typical for the following disease:
+HIV-infection

HIV has gp41 and gp120 on its surface interacts with target cells of an organism. Which of the following
human lymphocyte antigens is gp120 complementary bound with?
+CD 4

Examination of a child who frequently suffers from infectious diseases revealed that IgG concentration in
blood serum was 10 times less than normal, IgA and IgM concentration was also significantly reduced.
Analysis showed also lack of B-lymphocytes and plasmocytes. What disease are these symptoms typical for?
+Bruton’s disease
A 13-year-old boy presents with eczematous rashes on his shins and torso. Anamnesis states cases of otitis,
pneumonia and furuncles in the patient. Blood test: platelets- 70×109/l, low activity of T helper and T
suppressor cells, low IgM, with normal IgA and IgG. What immunodeficiency disease does this boy have?
+Louis-Barr syndrome

Parents of a 5-year-old child report him to have frequent colds that develop into pneumonias, presence of
purulent rashes on the skin. Laboratory tests have revealed the following: absence of immunoglobulins of
any type; naked cells are absent from the lymph nodes punctate. What kind of immune disorder is it?
+X-linked hypogammaglobulinemia (Bruton type agammaglobulinemia)

A 10-year-old child had the tuberculin test administered. 48 hours later a papule up to 8 mm in diameter
appeared on the site of the injection. What type of hypersensitivity reaction developed after the tuberculin
injection?
+Type IV hypersensitivity reaction
During surgical manipulations a patient has been given Novocain injection for anesthesia. 10 minutes later
the patient developed paleness, dyspnea, hypotension. What type of allergic reaction is it?
+Anaphylactic immune reaction

A man with a long-term history of bronchial asthma died from asphyxia. Histological examination of his
lungs revealed that the lumens of bronchioles and minor bronchi contained a lot of mucus with some
eosinophils. There was also sclerosis of intra-alveolar septa, dilatation of alveolar lumens. What mechanism
accounts for the development of hypersensitivity reaction?
+Reagine reaction
A patient has been diagnosed with acute glomerulonephritis that developed after he had had streptococcal
infection. It is most likely that the affection of basal glomerular membrane is caused by an allergic reaction
of the following type:
+Immune complex

A child with suspected tuberculosis was given Mantoux test. After 24 hours the site of the allergen injection
got swollen, hyperemic and painful. What are the main components that determine such response of the
body?
+Mononuclear cells, T-lymphocytes and lymphokines

A child entering the school for the first time was given Mantoux test in order to determine if there was a
need for revaccination. The reaction was negative. What is the meaning of this test result?
+No cell-mediated immunity to tuberculosis

A child cut his leg with a piece of glass while playing and was brought to the clinic for the injection of
tetanus toxoid. In order to prevent the development of anaphylactic shock the serum was administered by
Bezredka method. What mechanism underlies this method of desensitization of the body?
+Binding of IgE fixed to the mast cells

10 days after having quinsy caused by beta-hemolytic streptococcus a 6- year-old child exhibited symptoms
of glomerulonephritis. What mechanism of glomerular lesion is most likely in this case?
+Immunocomplex
A 22-year-old woman ate some seafood. 5 hours later the trunk and the distal parts of limbs got covered with
small itchy papules which were partially fused together. After one day, the rash disappeared spontaneously.
Specify the hypersensitivity mechanism underlying these changes:
+Atopy (local anaphylaxis)
During blood transfusion a patient has developed intravascular erythrocyte hemolysis. What kind of
hypersensitivity does the patient have?
+II type (antibody-dependent)

A 30-year-old patient has dyspnea fits, mostly at night. He has been diagnosed with bronchial asthma.
What type of allergic
reaction according to the Gell-Coombs classification is most likely in this case?
+Anaphylactic

Several minutes after a dentist administered Novocain for local anesthesia of a patient’s tooth, the following
symptoms sharply developed in the patient: fatigue, skin itching. Objectively the following can be observed:
skin hyperemia, tachycardia, BP dropped down to 70/40 mm Hg. What kind of allergic reaction is this
pathology?
+Anaphylactic

After sensitization a test animal received subcutaneously a dose of antigen. At the site of injection, a
fibrinous inflammation developed with alteration of vessels wall, basal substance, and fibrous structures of
connective tissue. The inflammation took form of mucoid and fibrinoid necrosis. What immune response
occurred in the test animal?
+Delayed-type hypersensitivity
A 5-year-old child is diagnosed with Bruton syndrome (X-linked agammaglobulinemia) that manifests itself
as severe clinical course of bacterial infection and absence of B-lymphocytes and plasma cells. What
changes of immunoglobulin content can be observed in blood serum of the child with immunodeficiency?
+No changes

During ascent into mountains a person develops increased respiration rate and rapid heart rate. What is the
cause of these changes?
+Decrease of O2 partial pressure
When ascending to the top of Elbrus, a mountain climber experiences oxygen starvation, dyspnea,
palpitations, and numbness of the extremities. What kind of hypoxia has developed in the mountain climber?
+Hypoxic

Measurements of the arterial pCO2 and pO2 during an attack of bronchial asthma revealed hypercapnia and
hypoxemia respectively. What kind of hypoxia occurred in this case?
+Respiratory

A man has suffered multiple bone fractures of his lower extremities during a traffic accident. During
transportation to a hospital his condition was further aggravated: blood pressure decreased, there were signs
of pulmonary artery embolism. What kind of embolism is the most likely in the given case?
+Air embolism

During the intravenous transfusion of the saline the patient’s condition deteriorated dramatically, and the
patient died from asphyxiation. Autopsy revealed acute venous congestion of internal organs with the
dramatic right heart dilatation. When the right ventricle was punctured underwater, the bubbles escaped.
What pathological process occurred in the patient?
+Air embolism

A student failed to answer all the questions of examination paper correctly. As a result, he blushed, felt hot
and lost confidence. What type of arterial hyperemia has developed in this case?
+Neurotonic hyperemia

A patient with obliterating endarteritis underwent ganglionic sympathectomy. What type of arterial
hyperemia should have developed as a result of the surgery?
+Neuroparalytic

A 54-year-old female was brought to the casualty department after a car accident. A traumatologist
diagnosed her with multiple fractures of the lower extremities. What kind of embolism is most likely to
develop in this case?
+Fat

A 25-year-old patient complains on increasing pain in his leg muscles occurring during walking and forcing
him to make frequent stops. Objectively: skin of legs is pale, no hair-covering, toenails are with trophic
changes, no pulsation of pedal arteries. The most probable cause of these changes is:
+Ischemia

A patient with chronic heart failure presents with increased blood viscosity. Capillaroscopy detected damage
to the vessel walls of the microcirculation system. What disorder is possible in the given case?
+Blood “sludge” phenomenon

During the exam a student was unable to correctly answer all the questions in his question card, which was
accompanied by the reddening of his face and hot sensation. What type of arterial hyperemia did the student
develop in this case?
+Pathologic

Microscopy of the puncture sample obtained from the inflammation focus of the patient with cutaneous
abscess revealed numerous blood cells of different types. What cells are the first to transfer from vessels to
tissues during inflammation?
+Neutrophils

A 30-year-old man complains of suffocation, heaviness in the chest on the right, general weakness. Body
temperature is 38,9oC. Objectively the right side of the chest lags behind the left side during respiration.
Pleurocentesis yielded exudate. What is the leading factor of exudation in the patient?
+Increased permeability of vessel wall
A 7-year-old child has acute onset of disease: temperature rise up to 38 C, rhinitis, cough, lacrimation, and
large-spot rash on the skin. Pharyngeal mucosa is edematous, hyperemic, with whitish spots in the buccal
area. What kind of inflammation caused the changes in the buccal mucosa?
+Catarrhal inflammation

Blood plasma of a healthy man contains several dozens of proteins. During an illness new proteins can
originate, namely the protein of "acute phase". Select such protein from the listed below:
+C-reactive protein

The cellular composition of exudate largely depends on the etiological factor of inflammation. What
leukocytes are the first to get into the focus of inflammation caused by pyogenic bacteria?
+Neutrophil granulocytes
As a result of careless handling of an iron, a 34-year-old female patient has got acute pain, redness, swelling
of her right index finger. A few minutes later, there appeared a blister filled with a transparent liquid of
yellow color. The described changes are a manifestation of the following pathological process:
+Exudative inflammation

Lymphocytes and other cells of our body synthesize universal antiviral agents as a response to viral invasion.
Name these protein factors:
+Interferon
A patient, having suffered a thermal burn, developed painful boils filled with turbid liquid in the skin. What
morphological type of inflammation has developed in the patient?
+Serous

Necrotic focus has appeared on burn, swallowing and red skin. What is the main mechanism of necrobiosis
improvement in inflammatory area?
+Secondary alteration

A cook burnt his arm with steam. What substance increased and led to development of redness, edema and
painfulness of affected area of skin?
+Histamine

Enlargement and deformation of joints were revealed in a patient with rheumatism. What type of
inflammation underlies these changes?
+Alterative

Indicate inflammatory mediators which have to be inhibited for decrease in exudation:


+Histamine

A patient has high body temperature, redness, edema, painfulness on her right forearm. What biological
active substances intensify inflammatory reaction?
+Kinins

What inflammatory mediator is formed due to limited proteolysis of plasma globulins?


+Bradykinin
Fever and increase of antibodies and leukocytes have appeared in animal under experimental modeling of
inflammation. What substances conditioned to all these common reactions in inflammation?
+Interleukins

Which of following inflammatory mediators are formed under the influence of lypooxygenase?
+Leukotrienes

A tumor is detected in one of the regions of the patient’s brain, resulting in the patient’s inability to maintain
normal body temperature. What brain structure is damaged?
+Hypothalamus
After transfusion of 200 ml of blood a patient presented with body temperature rise up to 37, 9oC. Which of
the following substances is the most likely cause of temperature rise?
+Interleukin-1

A patient has been diagnosed with influenza. His condition became drastically worse after taking antipyretic
drugs. His consciousness is confused, AP is 80/50 mm Hg, Ps is 140/m, body temperature dropped down to
35, 8oC. What complication developed in this patient?
+Collapse
This year influenza epidemic is characterized by patients’ body temperature varying from 36,9to 37,9o C.
Such fever is called:
+Subfebrile

A patient with lobar pneumonia has had body temperature of 39oC with daily temperature fluctuation of no
more than 1oC for 9 days. This fever can be characterized by the following temperature curve:
+Persistent

Body temperature at the patient with an infection disease was increased to a day 39,5-40,50C and has been
kept at this altitude about an hour and then returned to baseline. At which disease does this type of
temperature curve occur?
+Malaria

In a patient elevation of body temperature takes turns with drops down to normal levels during the day. The
rise in temperature is observed periodically once in four days. Specify the type of temperature curve:
+Febrisintermittens

A patient has acute bronchitis. The fever up to 38, 5oC had lasted for a week, presently there is a decrease in
temperature down to 37, 0oC. Specify the leading mechanism in the 3rd stage of fever:
+Peripheral vasodilation
A patient suffers from intermittent fevers and normalizations of body temperature that occur during the day.
The temperature rise is observed regularly every fourth day. Specify the type of temperature curve:
+Febrisinternuttens

At simulation of inflammation of the lower extremity the animal experienced raise of the temperature,
increase of amount of antibodies and leucocytes in the blood. What substances caused this general reaction
of the organism on inflammation?
+Interleukin

A patient has relapsing fever that is characterized by periods of several days of high temperatures,
alternating with periods of normal temperature. This temperature curve is called:
+Febrisrecurrens

Body temperature at the patient with an infection disease was increased to a day 39,5-40,50C and has been
kept at this altitude about an hour and then returned to baseline. What type of temperature curve described in
this case?
+Intermittens

A female patient has been diagnosed with cervical erosion, which is a precancerous pathology. What defense
mechanism can prevent the development of a tumor?
+Increase in natural killer level (NK cells)

A patient died of cancerous cachexia with primary localization of cancer in the stomach. Autopsy revealed
acutely enlarged liver with uneven surface and numerous protruding nodes; the nodes had clear margins in
the section, rounded shape, gray-pink color, varying density, sometimes contained necrotic foci.
Histologically: there are atypical cells in the nodes. What pathologic process occurred in the liver?
+Cancer metastases

Lung cancer developed at the patient who smoked tobacco for a long time. What of the carcinogens
contained in tobacco smoke and concern to PAH?
+Benzpiren

There is high stage of interaction between lung cancer and tobacco smoking. What chemical carcinogen is
contained in tobacco smog?
+3,4-benspyren

Patient, 62 years old, a fireman, complains of general weakness, sudden weight loss, husky voice, shortness
of breathing, dry cough. Laryngeal tumor that germinates vocal cords and epiglottis was discovered at
laryngoscopy. Name the most likely cause of the tumor in this patient:
+Polycyclic aromatic hydrocarbons

In a patient with metastases of lung carcinoma introduction of cytostatics led to suspension of metastases
growth at first but later metastases resumed spread. What is the most possible mechanism of secondary
growth of metastases?
+Absence of contact braking

A 56-years-old patient, who had contact with diethylnitrozamine at his work place, complains onpain in right
subcostal area, weakness, loss of appetite, and decreased workability. At examination of this patient: surface
of his liver is rough, splenomegaly and ascites are present in him; his body temperature is 37.2oC; in his
blood analysis ESR is 25 mm/hour, besides neutrophilic leukocytosis, and hypochromic anemia were found.
What disease developed in the patient’s organism?
+Cancer of liver

Malignant tumor of lung was diagnosed in a patient. What feature of tumour growth testifies its malignancy?
+Infiltrative growth

What biological process augmentation is typical for tumor cells?


+Anaerobic glycolysis
In 1910 Raus managed to cause sarcoma in chickens by cell-free infiltrate inserting. What was the method of
experimental modeling?
+Induction
A patient with urinary bladder cancer was working in coke factory. What substance was the most probable
reason of this pathological condition?
+Naphtylamine

They got nitrogenous nitrite to experimental animals. A tumor was developed in 80% of animals. What was
the group of cancerogens?
+Nitrosamines

A 40-year-old woman with Cushing disease presents with steroid diabetes. On biochemical examination she
has hyperglycemia and hypochloremia. What process activates in the first place in such patients?
+Gluconeogenesis

A woman complains of visual impairment. Examination revealed obesity in the patient and her fasting
plasma glucose level is hyperglycemic. What diabetes complication can cause visual impairment/blindness?
+Microangiopathy

A patient with insulin-dependent diabetes mellitus has been administered insulin. After a certain period of
time the patient developed fatigue, irritability, excessive sweating. What is the main mechanism of such
presentations developing?
+Decreased glyconeogenesis
A patient with diabetes mellitus suffer from persistently nonhealing surgical wound, which is a sign of
disrupted tissue trophism. What is the cause of such disorder?
+Disruption of protein metabolism regulation
A 12-year-old teenager has significantly put off weight within 3 months; glucose concentration rose up to 50
millimole/l. He fell into a coma. What is the main mechanism of its development?
+Hyperosmolar

The patient with complaints of permanent thirst applied to the doctor. Hyperglycemia, polyuria and
increased concentration of 17-ketosteroids in the urine were revealed. What disease is the most likely?
+Steroid diabetes

Before the cells can utilize the glucose, it is first transported from the extracellular space through the
plasmatic membrane inside them. This process is stimulated by the following hormone:
+Insulin
According to the results of glucose tolerance test, the patient has no disorder of carbohydrate tolerance.
Despite that, glucose is detected in the patients’s urine (5 mmol/l). The patient has been diagnosed with renal
diabetes. What renal changes cause glucosuria in this case?
+Decreased activity of glucose reabsorption enzymes

Examination of a 56-year-old female patient with a history of type 1 diabetes revealed a disorder of protein
metabolism that is manifested by aminoacidemia in the laboratory blood test values, and clinically by the
delayed wound healing and decreased synthesis of antibodies. Which of the following mechanisms causes
the development of aminoacidemia?
+Increased proteolysis
Prior to glucose utilization in cells it is transported inside cells from extracellular space through plasmatic
membrane. This process is stimulated by the following hormone:
+Insulin

A 15-year-old patient has fasting plasma glucose level 4,8 mmol/l, one hour after glucose challenge it
becomes 9,0 mmol/l, in 2 hours it is 7,0 mmol/l, in 3 hours it is 4,8 mmol/l. Such parameters are
characteristic of:
+Subclinical diabetes mellitus
A 50-year-old inpatient during examination presents with glycosuria and blood glucose of 3,0 mmol/l, which
are the most likely to be caused by:
+Renal disorder

Ketosis develops in the patients with diabetes mellitus as the result of activation of fatty acids oxidation
processes. What acid-base imbalance can result from accumulation of excessive ketone bodies in the blood?
+Metabolic acidosis

A patient suffers from disrupted patency of the airways at the level of small and medium-sized bronchial
tubes. What changes of acid-base balance can occur in the patient?
+Respiratory acidosis

A newborn child with pylorostenosis has often repeating vomiting accompanied by apathy, weakness,
hypertonicity, sometimes convulsions. What disorder form of acid-base balance is it?
+Nongaseous alkalosis
An infant has pylorospasm, weakness, hypodynamia, convulsions as a result of frequent vomiting. What
kind of acid-base misbalance is it?
+Excretory alkalosis

After taking poor-quality food a patient developed repeated episodes of diarrhea. On the next day he
presented with decreased arterial pressure, tachycardia, extrasystole. Blood pH is 7,18. These abnormalities
were caused by the development of:
+Nongaseous acidosis
A patient with diabetes developed a diabetic coma due to the acid-base imbalance. Specify the kind of this
imbalance:
+Metabolic acidosis

A patient with respiratory failure has blood pH of 7,35. pCO2 test revealed hypercapnia. Urine pH test
revealed an increase in the urine acidity. What form of acid-base imbalance is the case?
+Compensated respiratory acidosis

A hypertensive patient had been keeping to a salt-free diet and taking antihypertensive drugs together with
hydrochlorothiazide for a long time. This resulted in electrolyte imbalance. What disorder of the internal
environment occurred in the patient?
+Hypochloremic alkalosis
A 30-year-old man with diabetes mellitus type I was hospitalized. The patient is comatose. Laboratory tests
revealed hyperglycemia and ketonemia. What metabolic disorder can be detected in this patient?
+Metabolic acidosis

A woman with enteritis accompanied by severe diarrhea presents with loss water in the extracellular space,
high water content in the cells and decreasing blood osmolality. Name this type of water-electrolyte
imbalance?
+Hypoosmolarhypohydration
A woman with hypophyseal diabetes insipidus developed a water-mineral imbalance. What type is of water-
mineral imbalance develops in such case?
+Hyperosmolar hypohydration
A concentrated solution of sodium chloride was intravenously injected to an animal. This caused decreased
reabsorption of sodium ions in the renal tubules. It is the result of the following changes of hormonal
secretion:
+Aldosterone reduction

A patient is 44 years old. Laboratory examination of his blood revealed that content of proteins in plasma
was 40 g/l. What influence will be exerted on the transcapillary water metabolism?
+Filtration will be increased, reabsorption- decreased

A patient with enteritis accompanied by massive diarrhea has low water rate in the extracellular space, high
water rate inside the cells and low blood osmolality. What is such disturbance of water-electrolytic
metabolism called?
+Hypo-osmolar hypohydration
A 20 -year old patient complains of morbid thirst and increased diuresis (up to 10 l daily). Glucose
concentration in blood is normal but it is absent in urine. The patient has been diagnosed with diabetes
insipidus. What hormonal drug is the most appropriate for management of this disorder?
+Vasopressin

Atria of an experimental animal were overextended by blood that resulted in decreased reabsorption of Na+
and water in renal tubules. This can be explained by the influence of the following factor upon kidneys:
+Natriuretic hormone

A 56 -year old patient suffering from cardiac insufficiency has edema of feet and shins, edematous skin is
pale and cold. What is the leading mechanism of edema pathogenesis?
+Rise of hydrostatic pressure in venules

A 49-year old woman spent a lot of time standing. As a result of it she got leg edema. What is the most
likely cause of the edema?
+Increase in hydrostatic pressure of blood in veins
A patient who had been continuously taking drugs blocking the production of angiotensin II developed
bradycardia and arrhythmia. A likely cause of these disorders is:
+Hyperkalemia
A patient with a pathology of the cardiovascular system developed edema of the lower extremities. What is
the mechanism of cardiac edema development?
+Increased hydrostatic pressure at the venous end of the capillary
30 minutes after drinking mango juice a child suddenly developed a local swelling in the area of the soft
palate, which impeded swallowing and, eventually, respiration. Mucosa of the swollen area was hyperemic
and painless. Blood test revealed moderate eosinophilia. Body temperature was normal. Anamnesis states
that the elder sister of the child has been suffering from bronchial asthma attacks. What kind of edema has
developed in the child?
+Allergic

A patient has severe blood loss caused by an injury. What kind of dehydration will be observed in this
particular case?
+Iso-osmolar

A 50-year-old man, who has been suffering from chronic hepatic failure for several years, has developed
ascites. What is the main mechanism of this disorder development?
+Increased pressure in portal vein system

Upon toxic damage of hepatic cells resulting in disruption of liver function the patient developed edemas.
What changes of blood plasma are the main cause of edema development?
+Decrease of albumin content

Patient with bile duct obstruction typically presents with inhibited blood clotting and develop hemorrhages
due to insufficient assimilation of vitamin:
+K

An 8-year old girl presents with signs disturbed twilight vision. This condition is caused by the deficiency of
vitamin:
+A

A sick child presents with high content of phenyl pyruvate in urine (normally it is practically absent). Blood
phenylalanine level is 350mg/L (norm 15mg/l). what disease are these symptoms characteristic of?
+Phenylketonuria

A woman has been limiting the amount of products in her diet to lose some weight. 3 months later she
developed edemas and her diuresis increased. What dietary component deficiency is the case of this?
+Proteins
A 52-year-old man presents with fever and pain in the joints. Both of his first metatarsophalangeal
articulations are deformed, swollen and reddened. Blood urea is high. The patient is diagnosed with gout.
What is the main developmental factor in the pathogenesis of this disease?
+Hyperuricemy
A 2-year-old child with mental and physical retardation has been delivered to a hospital. He presents with
frequent vomiting after having meals. There is phenylpyruvic acid in urine. Which metabolism abnormality
is the reason for this pathology?
+Amino-acid metabolism
A patient has been diagnosed with alkaptonuria. Choose an enzyme whose deficiency can be the reason for
this pathology:
+Homogentisic acid oxidase

A patient complains about dyspnea provoked by the physical activity. Clinical examination revealed anemia
and presence of the paraprotein in the zone of gamma-globulins. To confirm the myeloma diagnosis, it is
necessary to determine the following index in the patient’s urine:
+Bence Jones protein

A 2 -year old child with mental and physical retardation has been delivered to a hospital. He presents with
frequent vomiting after having meals. There is phenylpyruvic acid in urine. Which metabolism abnormality
is the reason for this pathology?
+Amino-acid metabolism
A 62- year old woman complains of frequent pain attacks in the area of her chest and backbone, rib
fractures. Her doctor suspected myeloma (plasmocytoma). What of the following laboratory characteristics
will be of the greatest diagnostic importance?
+Paraproteinemia

Toxic affection of liver results in dysfunction of protein synthesis. It is usually accompanied by the
following kind of dysproteinemia:
+Absolute hypoproteinemia
A 46-year-old female patient consulted a doctor about pain in the small joints of the upper and lower limbs.
The joints are enlarged and shaped like thickened nodes. Serum test revealed an increase in urate
concentration. This might be caused by a disorder in metabolism of:
+Purines
In case of alkaptonuria, homogentisic acid is excreted in urine in large amounts. The development of this
disease is associated with a disorder of metabolism of the following aminoacid:
+Tyrosine

A 49-year-old man complains of pain in his metatarsophalangeal joints and joint deformation. In blood
hyperuricemy can be observed. X-ray has revealed metatarsophalangeal joint space narrowing, erosion,
periarticular calcification of the both joints, osteoporosis. Microscopy has revealed inflammatory
granulomatous reaction surrounding necrotizing masses in the area of the first metatarsophalangeal joint.
Choose the most likely diagnosis:
+Gout (podagra)

During examination of a teenager with xanthomatosis the family history of hypercholesterolemia is revealed.
What transportable lipids are increased in concentration in case of such a disease?
+Low-density lipoproteins

During regular check-up a child is determined to have interrupted mineralization of the bones. What vitamin
deficiency can be the cause?
+Calciferol

#A 12-year-old patient was found to have blood serum cholesterol at the rate of 25 mmol/l. The boy has a
history of hereditary familial hypercholesterolemia, which is caused by the impaired synthesis of the
following protein receptors:
+Low density lipoproteins

A patient with exacerbated burn has developed esophagus stenosis. The patient presents with acute weight
loss due to problematic food intake. Blood test: erythrocyte -3,0х1012/l, Hb-106 g/l, total protein - 57 g/l.
What type of starvation does patient suffer from?
+Incomplete starvation

When measuring power inputs of a man by the method of indirect calorimetry the following results were
obtained:1000 ml oxygen consumption and 800 ml carbon dioxide liberation per minute. The man under
examination has the following respiratory coefficient:
+0,8
There is only one source of water for the body at absolute starvation - a process of organic compounds
oxidation. Which of the following substances in these conditions is the main source of endogenous water?
+Fats

Negative nitrogen balance, hypoproteinemia, violation of water-salt metabolism combined with normal
function of digestive system were founded at the vegetarians. Name the reason of this state.
+Monotonous protein diet
At 10th day of medical starvation the patient suffers from excitation, deep, noisy breathing, blood pressure
dropped to 90/60 mmHg, oliguria, urine with a smell of acetone. Name the reason of this state.
+Ketosis
Esophagus stenosis developed at a patient after a chemical burn. Sharp weight loss raised. In the blood:
erythrocyte -3,0х1012/l, Hb-106 g/l, total [crude] protein - 57 g/l. What type of starvation developed at a
patient?
+Incomplete

One of the causes of pernicious anemia is disturbed synthesis of transferrin - Castle’s intrinsic factor - by the
parietal cells of the stomach. What substance is called Castle’s extrinsic factor?
+Cobalamin

A woman has come to the hospital with complaints of general weakness, dizziness and dyspnea. Recently
she has been taking levomycetin (chloramphenicol) for prevention of enteric infection. Blood test:
erythrocytes - 1.9 ・1012/L, hemoglobin - 58 g/L, color index - 0.9, leukocytes - 2.2 g/L, reticulocytes -
+0.3%. What type of anemia is it indicative of?
Hypoplastic

A 30-year-old patient’s blood test has revealed the following: erythrocyte count is 6x1012/l, hemoglobin is
10,55 mmol/l. Vaquez’s disease was diagnosed. Name the leading part of pathogenesis:
+Neoplastic erythroid hyperplasia

A 42-year-old patient complains of pain in the epigastral area, vomiting; vomit masses have the color of
coffee-grounds; the patient suffers from melena. Anamnesis records gastric ulcer disease. Blood formula:
erythrocytes - 2, 8 · 10 12 /l, leukocytes – 8x109 /l, Hb- 90 g/l. What complication is it?
+Hemorrhage
A 27-year-old patient with injury to the neck has lost approximately 30% of the blood volume. The patient’s
condition is severe: blood pressure is 60/40 mm Hg, heart rate is 140/min., respiratory rate is 30/min.,
conscious. Characterize the condition of the patient’s circulatory system:
+Hypovolemic shock

On the fifth day after the acute blood loss a patient has been diagnosed with hypochromic anemia. What is
the main mechanism of hypochromia?
+Release of immature red blood cells from the bone marrow

In a car accident a man got injured and lost a lot of blood. What changes in peripheral blood are most likely
to occur on the 2nd day after the injury?
+Erythropenia

A 26 -year old pregnant woman is under treatment at an in-patient hospital. After a continuous attack of
vomiting she was found to have reduced volume of circulating blood. What kind of change in general blood
volume is the case?
+Polycythemic hypovolemia

A 38-year-old patient with uterine hemorrhage lasting for 2 days was delivered to the admission ward.
Which of the following will be revealed in the patient’s blood?
+Decrease in the hematocrit

A 32-year-old patient was admitted to the hospital with gross blood loss due to auto accident trauma. Ps –
110 Bpm, RR- 22 pm, BP- 100/60mm Hg. What changes in the blood will occur in an hour after the blood
loss?
+Hypovolemia

Examination of a 43 y.o. anephric patient revealed anemia symptoms. What is the cause of these symptoms?
+Reduced synthesis of erythropoietin

In the blood of a 26-year-old man 18%of erythrocytes of the spherical, ball-shaped, flat and spinous shape
have been revealed. Other erythrocytes were in form of the concavo-concave disks. How is this phenomenon
called?
+Physiological poikilocytosis
A patient’s blood was analyzed and the decreased erythrocyte’s sedimentation rate (ESR) was discovered.
What disease from the listed below is accompanied with decreased ESR?
+Polycytemia

Patient 54 year-old, 5th day after surgical operation. Blood count: Erythrocytes 3,6x1012/l, Hemoglobin 95
g/l, Erythrocyte’s hemoglobin content (color index) 0,78; Leukocytes 16x109/l, Platelets 450x109/l Blood
picture: anizocytosis, poikilocytosis, reticulocytes-3,8%. What anemia does this patient have?
+Acute posthemorragic anemia

Examination of a 52-year-old woman has revealed a decrease in the amount of red blood cells and an
increase in free hemoglobin in the blood plasma (hemoglobinemia). Color index is 0,85. What type of
anemia is being observed in the patient?
+Acquired hemolytic

Along with normal hemoglobin types there can be pathological ones in the organism of an adult. Name one
of them:
+HbS

A patient is diagnosed with chronic atrophic gastritis attended by deficiency of Castle’s intrinsic factor. What
type of anemia does the patient have?
+B12 -deficiency anemia

Biochemical analysis of an infant’s erythrocytes revealed evident glutathione peroxidase deficiency and low
concentration of reduced glutathione. What pathological condition can develop in this infant?
+Hemolytic anemia

Examination of a 52-year-old female patient has revealed a decrease in the amount of red blood cells and an
increase in free hemoglobin in the blood plasma (hemoglobinemia). Color index is 0,85. What type of
anemia is being observed in the patient?
+Acquired hemolytic

A 37-year-old female patient complains of headache, vertigo, troubled sleep, numbness of limbs. For the last
6 years she has been working at the gas-discharge lamp-producing factory in the lead-processing shop.
Blood test findings: low haemoglobin and RBC level, serum iron concentration exceeds the norm by several
times. Specify the type of anemia:
+Iron refractory anemia

A 19-year-old female patient has had low haemoglobin rate of 90-95 g/l since childhood. Blood count results
obtained after hospitalization are as follows: erythrocytes - 3,2 x 1012/l, Hb- 85 g/l, color index - 0,78;
leukocytes - 5,6 x 109/l, platelets – 210x 109/l. Smear examination revealed anisocytosis, poikilocytosis and
target cells. Reticulocyte rate is 6%. Iron therapy was ineffective. What blood pathology corresponds with
the described clinical presentations?
+Thalassemia

A 34-year old woman was diagnosed with hereditary microspherocytic hemolytic anemia (Minkowsky-
Shauffard disease). What mechanism caused hemolysis of erythrocytes?
+Membranopathy

A 15- year old girl has pale skin, glossitis, gingivitis. Blood count: erythrocytes - 3,3 cot 1012/l, hemoglobin
- 70 g/l, color index - 0,5. Examination of blood smear revealed hypochromia, microcytosis, poikilocytosis.
What type of anemia is it?
+Iron-deficient

As a result of increased permeability of the erythrocyte membrane in a patient with microspherocytic


anaemia (Minkowsky-Shauffard disease) cells receive sodium ions and water. Erythrocytes take form of
spherocytes and can be easily broken down. What is the leading mechanism of erythrocyte damage in this
case?
+Electrolytic osmotic

A 20- year old patient complains of general weakness, dizziness, quick fatigability. Blood analysis results:
Hb- 80g/l. Microscopical examination results: erythrocytes are of modified form. This condition might be
caused by:
+Sickle-cell anemia

A 56 -year old patient came to a hospital with complaints about general weakness, tongue pain and burning,
sensation of limb numbness. In the past he underwent resection of forestomach. In blood: Hb- 80 g/l;
erythrocytes - 2, 0·1012/l; colour index - 1,2, leukocytes - 3, 5 109/l. What anemia type is it?
+B12-folate deficient
Surgical removal of a part of stomach resulted in disturbed absorption of vitamin В12, it is excreted with
feces. The patient was diagnosed with anemia. What factor is necessary for absorption of this vitamin?
+Gastromucoprotein

A patient with hypochromic anemia has splitting hair and loss of hair, increased nail brittling and taste
alteration. What is the mechanism of the development of these symptoms?
+Deficiency of iron-containing enzymes
Examination of initial molecular structure revealed substitution of the glutamic acid by valine. What
inherited pathology is it typical for?
+Sickle-cell anemia

24 hours after an appendectomy the patient’s blood test shows neutrophilic leukocytosis with a regenerative
shift. What is the most likely mechanism of absolute leukocytosis development in the patient’s peripheral
blood?
+Intensification of leukopoiesis
After a severe stress the patient presents with eosinopenia in the blood test. In this case the decreased
number of eosinophils can explain changes in the level of the following hormones:
+Glucocorticoids

A 59-year-old woman has been hospitalized in a surgical ward due to exacerbation of chronic osteomyelitis
of the left shin. Blood test: leukocytes - 15,0x109 /l. Leukogram: myelocytes - 0%, metamyelocytes - 8%,
stab neutrophils - 28%, segmented neutrophils - 32%, lymphocytes - 29%, monocytes - 3%. Such blood
count would be called:
+Regenerative left shift

A 3-year-old child has eaten some strawberries. Soon he developed a rash and itching. What was found in
the child’s leukogram?
+Eosinophilia

A 23-year-old patient with acute pulpitis has elevated body temperature and an increase in the WBC count
up to 14x109 /L. The leucogram is as follows: basophils - 0, eosinophils - 2, monocytes - 0, immature
neutrophils - 4, stab neutrophils - 8, segmented neutrophils - 56, lymphocytes - 26, monocytes - 4. How can
we interpret these changes in the white blood cells?
+Neutrophilia with a regenerative left shift

A 49-year-old male patient with myocardial infarction has been admitted to the cardiology department. What
changes in the peripheral blood cells are induced by the necrotic changes in the myocardium?
+Neutrophilic leukocytosis
Having helped to eliminate consequences of a failure at a nuclear power plant, a worker got radiation dose of
500 Roentgen. He complains of headache, nausea, dizziness. What changes in leukocytes quantity can be
expected 10 hours after irradiation?
+Neutrophilic leukocytosis

After an attack of bronchial asthma, a patient had his peripheral blood tested. What changes can be
expected?
+Eosinophilia
As a result of a road accident a 26- year-old man is in the torpid phase of shock. Blood count: leukocytes - 3,
2 109/l. What is the leading mechanism of leukopenia development?
+Leukocyte redistribution in the bloodstream

Two hours after an exam a student had a blood count done and it was revealed that he had leukocytosis
without significant leukogram modifications. What is the most probable mechanism of leukocytosis
development?
+Redistribution of leukocytes in the organism

A 47- year old man with myocardium infarction was admitted to the cardiological department. What changes
of cellular composition of peripheral blood are induced by necrotic changes in the myocardium?
+Neutrophilic leukocytosis

A 5 -year old child is ill with measles. Blood analysis revealed increase of total number of leukocytes up to
13 · 109/l. Leukogram: basophils - 0, eosinophils - 1, myelocytes - 0, juvenile neutrophils - 0, band
neutrophils - 2, segmented neutrophils - 41, lymphocytes - 28, monocytes - 28. Name this phenomenon:
+Monocytosis
Examination of a patient admitted to the surgical department with symptoms of acute appendicitis revealed
the following changes in the white blood cells: the total count of leukocytes is 16 109/l. Leukocyte formula:
basophils - 0, eosinophils - 2%, juvenile forms - 2%, stab - 8%, segmentonuclear - 59%, lymphocytes - 25%,
monocytes- 4%. The described changes can be classified as:
+Neutrophilia with regenerative left shift

A 26- year old man is in the torpid shock phase as a result of a car accident. In blood: 3, 2 109/l. What is the
leading mechanism of leukopenia development?
+Redistribution of leukocytes in bloodstream

A 16-year-old boy was performed an appendectomy. He has been hospitalized for right lower quadrant
abdominal pain within 18 hours. The surgical specimen is edematous and erythematous. Infiltration by what
of the following cells is the most typical for the process occuring here?
+Neutrophils

As a result of the damage of one of the Atomic Power Plant reactors the runout of radioelements took place.
People in the high-radiation area were radiated with approximately 250-300 r. They were immediately
hospitalized. What changes in the blood count would be typical for the victims?
+Lymphopenia
Blood sampling for the haematology is recommended to carry out on an empty stomach and in the morning.
What changes in blood formula are possible if blood sampling was carried out after food intake?
+Increase of leukocyte number

14. Blood sampling for bulk analysis is recommended to be performed on an empty stomack and in the
morning. What changes in blood composition can occur if to perform blood sampling after food intake?
+Increased contents of leukocytes

A 54-year-old man complains of general weakness, frequent colds and bruises constantly appearing on his
body. Blood test: erythrocytes - 2.5x1012/L; Hb - 80 g/l; Colour index - 0.9; reticulocytes - absent; platelets
– 50x109/L; leukocytes – 58x109/L; leukogram: basocytes - 5%, eosinophils - 15%, myeloblasts - 6%,
myelocytes - 10%, juvenile - 18%, stab neutrophils - 26%, segmented neutrophils - 10%, lymphocytes - 8%,
monocytes - 2%, ESR - 40 mm/hour. What hematologic conclusion can be made?
+Chronic myelogenous leukemia

A patient is 20 years old, an athlete. He addressed a doctor with complaints of fatigue, fever up to 38 o C -
40 o C. Objectively: the liver and spleen are enlarged, lymph nodes on palpation are slightly enlarged, dense,
painless. Blood test: Нb- 100 g/l; erythrocytes - 2, 9x1012/l; leukocytes -4, 4x109/l. Leukogram: 68% of
blast cells. Cytochemical investigation of blast cells revealed negative reactions to glycogen, peroxidase,
non-specific esterase, lipids. Name this disease:
+Acute undifferentiated leukemia

A 39-year-old patient underwent hematologic tests. The following results were obtained: RBC-2,8·1012/L,
Нb-80 g/L, color index - 0,85, reticulocytesm - 0,1%, platelets – 16х109/L, WBC – 60х109/L. Basophils - 2,
eosinophils - 8, promyelocytes - 5, myelocytes - 5, immature neutrophils - 16, stab neutrophils - 20,
segmented neutrophils - 34, lymphocytes - 5, monocytes -5. What form of blood pathology are these results
indicative of?
+Chronic myeloid leukemia

Microscopical examination of an enlarged cervical lymph node revealed blurring of its structure, absence of
lymphoid follicles; all the microscopic fields showed cells with roundish nuclei and thin limbus of basophil
cytoplasm. It is known from the clinical data that other groups of lymph nodes are also enlarged as well as
spleen and liver. What disease might be suspected?
+Lymphoid leukosis

A 23 y.o. patient complains of weakness, temperature rise up to 38 - 400C. Objectively: liver and spleen are
enlarged. Hemogram: Hb- 100 g/l, erythrocytes - 2, 9 1012/l, leukocytes - 4, 4 109/l, thrombocytes - 48
109/l, segmentonuclear neutrophils - 17%, lymphocytes - 15%, blast cells - 68%. All cytochemical reactions
are negative. Make a hematological conclusion:
+Undifferentiated leukosis

After pancreatic surgery the patient developed hemorrhagic syndrome with disturbed 3rd stage of blood
clotting. What will be the most likely mechanism of the hemostatic disorder?
+Fibrinolysis activation
A 3-year-old boy with pronounced hemorrhagic syndrome has no antihemophilic globulin A (factor VIII) in
the blood plasma. Hemostasis has been impaired at the following stage:
+Conversion of prothrombin to thrombin
A patient visited a dentist to extract a tooth. After the tooth had been extracted, bleeding from the tooth
socket continued for 15 minutes. Anamnesis states that the patient suffers from active hepatitis. What
phenomenon can extend the time of hemorrhage?
+Decrease of fibrinogen content in bloods

A 60-year-old man suffering from chronic hepatitis frequently observes nasal and gingival hemorrhages,
spontaneous hemorrhagic rashes on the skin and mucosa. Such presentations result from:
+Decreased synthesis of prothrombin and fibrinogen

After implantation of a cardiac valve a young man systematically takes indirect anticoagulants. His state was
complicated by hemorrhage. What substance content has decreased in blood?
+Prothrombin

A patient is diagnosed with hereditary coagulopathy that is characterised by factor VIII deficiency. Specify
the phase of blood clotting during which coagulation will be disrupted in the given case:
+Thromboplastin formation

A 28-year-old patient complains of frequent gingival haemorrhages. Blood test revealed the clotting factor II
(prothrombin) deficiency. What phase of blood coagulation is impaired in this patient?
+Thrombin generation

A patient, who has been suffering for a long time from intestine disbacteriosis, has increased hemorrhaging
caused by disruption of posttranslational modification of blood-coagulation factors II, VII, IХ, and Х in the
liver. What vitamin deficiency is the cause of this condition?

A 12-year-old patient has been admitted to a hospital for hemarthrosis of the knee joint. From early
childhood he suffers from frequent bleedings. Diagnose the boy’s disease:
+Hemophilia
After a tourniquet application a patient was found to have petechial hemorrhages. The reason for it is the
dysfunction of the following cells:
+Platelets

A disaster fighter at a nuclear power plant developed hemorrhagic syndrome on the background of acute
radiation disease. What is the most important factor of syndrome pathogenesis?
+Thrombocytopenia

A tooth extraction in a patient with chronic persistent hepatitis was complicated with prolonged hemorrhage.
What is the reason for the haemorrhagic syndrome?
+Decrease in thrombin production

After a disease a 16-year-old boy is presenting with decreased function of protein synthesis in the liver as a
result of vitamin K deficiency. This may cause disorder of:
+Blood coagulation

A patient suffers from the hemorrhagic syndrome that shows itself in frequent nasal bleedings, posttraumatic
and spontaneous intra-cutaneous and intra-articular hemorrhages. After a laboratory study a patient was
diagnosed with the type B hemophilia. This disease is provoked by the deficit of the following factor of
blood coagulation:
+IX

As a result of post-translation modifications some proteins taking part in blood coagulation, particularly
prothrombin, become capable of calcium binding. The following vitamin takes part in this process:
+K

Patients with bile ducts obstruction suffer from inhibition of blood coagulation, bleedings as a result of low
level of vitamin assimilation. What vitamin is in deficiency?

A 2-year-old child has got intestinal dysbacteriosis, which resulted in hemorrhagic syndrome. What is the
most likely cause of hemorrhage of the child?
+Vitamin K deficiency

Punctate hemorrhage was found out in the patient after application of a tourniquet. With dysfunction of what
blood cells is it connected?
+Platelets
A patient underwent a surgery for excision of a cyst on pancreas. After this he developed hemorrhagic
syndrome with apparent disorder of blood coagulation. Development of this complication can be explained
by:
+Activation of fibrinolytic system

An athlete (long-distance runner) during a contest developed a case of acute cardiac insufficiency. This
pathology resulted from:
+Cardiac volume overload
After a severe psycho-emotional stress a 48-year old patient suddenly developed acute heart ache irradiating
to the left arm. Nitroglycerine relieved pain after 10 minutes. What is the leading pathogenetic mechanism of
this process development?
+Spasm of coronary arteries
The patient with acute myocardial infarction was given intravenously different solutions during 8 hours with
medical dropper 1500 ml and oxygen intranasally. He died because of pulmonary edema. What caused the
pulmonary edema?
+Volume overload of the left ventricular
ECG of a 44-year-old patient shows signs of hypertrophy of both ventricles and the right atrium. The patient
was diagnosed with the tricuspid valve insufficiency. What pathogenetic variant of cardiac dysfunction is
usually observed in case of such insufficiency?
+Heart overload by volume

12 hours after an acute attack of retrosternal pain a patient presented an increase of aspartate
aminotransferase activity in blood serum. What pathology is this deviation typical for?
+Myocardium infarction

A 48-year-old is unconscious. He has a history of several syncopal episodes with convulsions. ECG shows
deformed QRS complexes unconnected with P waves, atrial contraction are approximately 70/min,
ventricular contractions - 25-30/min. Name the type of arrhythmia in this case:
+Complete atrioventricular block

ECG of the patient shows increased duration of the QRS complex. What is the most likely cause?
*Disturbed conduction in the atrioventricular node
A 15-year-old teenager complains of lack of air, general weakness, palpitations. Heart rate is 130/min.. BP is
100/60 mm Hg. ECG: QRS complex has normal shape and duration. The number of waves and ventricular
complexes is equal, T wave merges with P wave. What type of cardiac arrhythmia observed in the teenager?
*Sinus tachycardia

A 67-year-old patient complains of periodic heartache, dyspnea during light physical activities. ECG reveals
extraordinary contractions of heart ventricles. Such arrhythmia is called:
+Extrasystole

A patient complains of palpitations after stress. Pulse is 104/min., P-Q=0,12 seconds, there are no changes in
QRS complex. What type of arrhythmia does the patient have?
+Sinus tachycardia

Since a patient has had myocardial infarction, atria and ventricles contract independently from each other
with a frequency of 60-70 and 35-40 per minute. Specify the type of heart block in this case:
+Complete atrioventricular

A patient who had been continuously taking drugs blocking the production of angiotensin II developed
bradycardia and arrhythmia. A likely cause of these disorders is:
+Hyperkalemia

#A 45-year-old patient was admitted to the cardiological department. ECG data: negative P wave overlaps
QRS complex, diastolic interval is prolonged after extrasystole. What type of extrasystole is it?
+Atrioventricular

#A patient who had been continuously taking drugs blocking the production of angiotensin II developed
bradycardia and arrhythmia. A likely cause of these disorders is:
+Hyperkalemia

After a physic trauma a woman developed periodical increases in her blood pressure accompanied by
headache, palpitations and general weakness. What mechanism of hypertension development does this
woman have?
+Increased arteriolar tone
A 59-year-old man, a business manager, developed intense burning retrosternal pain that irradiates to the left
arm. The pain occurred in the evening after tax audit. 15 minutes later the patient’s condition normalized.
What mechanism of angina pectoris development is leading in this patient?
+Increased level of blood catecholamines

A 16-year-old girl fainted when she tried to quickly change her position from horizontal to vertical. What
caused the loss of consciousness in the girl?
+Decreased venous return

A patient has insufficient blood supply to the kidneys, which caused the development of pressure effect due
to the constriction of arterial resistance vessels. This is the result of the vessels being greatly affected by the
following substance:
+Angiotensin II

A 41-year-old man has a history of recurrent attacks of heartbeats (paroxysms), profuse sweating, headaches.
Examination revealed hypertension, hyperglycemia, increased basal metabolic rate, and tachycardia. These
clinical presentations are typical for the following adrenal pathology:
+Hyperfunction of the medulla

A 43-year-old-patient has arterial hypertension caused by increase in cardiac output and general peripheral
resistance. Specify the variant of hemodynamic development of arterial hypertension in the given case:
+Eukinetic

A patient has been diagnosed with influenza. His condition drastically worsened after taking antipyretic
drugs. He is unconscious, AP is 80/50 mm Hg, Ps is 140/m, body temperature dropped down to 35, 8oC.
What complication developed in this patient?
+Collapse

Autopsy has revealed shrunken kidneys weighing 50 mg, with fine grained surface and uniformly thinned
substance. Microscopic investigation has shown the thickening of arteriole walls due to accumulation of
homogeneous pink-colored masses in them. Glomeruli were undersized, sclerotic, with atrophied tubules.
What disease are these changes characteristic of?
+Essential hypertension
A patient with constant headaches, pain in the occipital region, tinnitus, dizziness has been admitted to the
cardiology department. Objectively: AP- 180/110 mm Hg, heart rate - 95/min. Radiographically, there is a
stenosis of one of the renal arteries. Hypertensive condition in this patient has been caused by the activation
of the following system:
+Renin-angiotensin

A month after surgical constriction of rabbit’s renal artery the considerable increase of systematic arterial
pressure was observed. What of the following regulation mechanisms caused the animal’s pressure change?
+Angiotensin-II
An aged man had raise of arterial pressure under a stress. It was caused by activation of:
+Sympathoadrenal system

Arterial hypertension is caused by the stenosis of the renal arteries in the patient. Activation of what system
is the main link in the pathogenesis of this form of hypertension?
+Renin-angiotensin
Prophylactic medical examination of a 36- year old driver revealed that his AP was 150/90 mm Hg. At the
end of working day he usually hears ear noise, feels slight indisposition that passes after some rest. He was
diagnosed with essential hypertension. What is the leading pathogenetic mechanism in this case?
+Neurogenic

Arterial pressure of a surgeon who performed a long operation raised up to 140/110 mm Hg. What changes
of humoral regulation could have caused the rise of arterial pressure in this case?
+Activation of sympathoadrenal system

A 70 -year old man is ill with vascular atherosclerosis of lower extremities and coronary heart disease.
Examination revealed disturbance of lipid blood composition. The main factor of atherosclerosis
pathogenesis is the excess of the following lipoproteins:
+Low-density lipoproteins

When studying the pulmonary ventilation values, the reduction of forced expiration volume has been
detected. What is the likely cause of this phenomenon?
+Obstructive pulmonary disease
A patient, who has been suffering from bronchial asthma for a long time, developed acute respiratory failure.
What is the main mechanism of pathology development in this case?
+Obstructive disorders of pulmonary ventilation
An unconscious young man in the state of morphine intoxication has been delivered into an admission room.
The patient’s respiration is slow and shallow due to suppression of the respiratory center. What kind of
respiratory failure occurred in this case?
+Ventilatory disregulation

A 28-year-old patient undergoing treatment in the pulmonological department has been diagnosed with
pulmonary emphysema caused by splitting of alveolar septum by tissue trypsin. The disease is caused by the
congenital deficiency of the following protein:
+α1-proteinase inhibitor

A patient with bronchial asthma has developed acute respiratory failure. What kind of respiratory failure
occurs in this case?
+Obstructive disturbance of alveolar ventilation

A patient with marked pneumofibrosis that developed after infiltrating pulmonary tuberculosis has been
diagnosed with respiratory failure. What is its pathogenetic type?
+Restrictive

A 23-year-old patient has been admitted to a hospital with a cranio-cerebral injury. The patient is in a grave
condition. Respiration is characterized by prolonged convulsive inspiration followed by a short expiration.
What kind of respiration is it typical for?
+Apneustic

Protective function of saliva is based on several mechanisms, including the presence of enzyme that has
bactericidal action and causes lysis of complex capsular polysaccharides of staphylococci and streptococci.
Name this enzyme:
+Lysozyme

A 35-year-old man with peptic ulcer disease has undergone antrectomy. After the surgery secretion of the
following gastrointestinal hormone will be disrupted the most:
+Gastrin
Administration of doxycycline hydrochloride has caused an imbalance of the symbiotic intestinal microflora.
Specify the kind of imbalance caused by the antibiotic therapy:
+Dysbacteriosis
A hospital has admitted a patient complaining of abdominal bloating, diarrhea, flatulence after eating protein
foods. These signs are indicative of the impaired digestion of proteins and their increased degradation.
Which of the following compounds is the product of this process?
+Indole

A 30-year-old male patient with acute pancreatitis has been found to have a disorder of cavitary protein
digestion. The reason for such condition can be the hyposynthesis and hyposecretion of the following
enzyme:
+Tripsin

A male patient has been diagnosed with gastric ulcer. Bacteriological examination of biopsy material from
the affected part of stomach revealed small colonies of gram-negative, oxide reductase-positive flexibacteria
that grew on the chocolate agar on the fifth day. Which of the following microorganisms is the most likely
causative agent?
+Helicobacter pylori

A patient present with acute attack of cholelithiasis. Laboratory examination of the patient’s faces will show
the following in this case:
+Negative reaction to stercobilin

Patients with bile duct obstruction typically present with inhibited blood clotting and develop hemorrhages
due to insufficient assimilation of vitamin:
+K

Encephalopathy has developed in a child with hemolytic disease of the newborn. What substance had
increased in the child’s blood, resulting in damage to the CNS?
+Unconjugated bilirubin

A 46-year-old woman suffering from cholelithiasis developed jaundice. Her urine became dark yellow,
while feces are light-colored. What substance will be the most increased in concentration in the blood serum
in this case?
+Conjugated bilirubin
A patient with jaundice has high total bilirubin that is mainly indirect (unconjugated), high concentration of
stercobilin in the feces and urine. The level of direct (conjugated) bilirubin in the blood plasma is normal.
What type of jaundice can be suspected?
+Hemolytic

Upon toxic damage of hepatic cells resulting in disruption of liver function the patient developed edemas.
What changes of blood plasma are the main cause of edema development?
+Decrease of albumin content

A 16-year-old adolescent is diagnosed with hereditary UDP (uridine diphosphate) glucuronyl transferase
deficiency. Laboratory tests revealed hyperbilirubinemia caused mostly by increased blood content of the
following substance:
+Unconjugated bilirubin

A 50-year-old man, who has been suffering from chronic hepatic failure for several years, has developed
ascites. What is the main mechanism of this disorder development?
+Increased pressure in portal vein system

A 43-year-old patient suffers from acute pancreatitis with disrupted common bile duct patency. What
condition can develop in this case?
+Mechanical jaundice

A patient has been admitted to the contagious isolation ward with signs of jaundice caused by hepatitis virus.
Which of the symptoms given below is strictly specific for hepatocellular jaundice?
+Increase of ALT, AST level

A 53-year-old male patient complains of acute pain in the right hypochondrium. Objective examination
revealed scleral icterus. Laboratory tests revealed increased ALT activity, and stercobilin was not detected in
the stool. What disease is characterized by these symptoms?
+Cholelithiasis

A patient has normally colored stool including a large amount of free fatty acids. The reason for this is a
disturbance of the following process:
+Fat absorption
Enzymatic jaundices are accompanied by abnormal activity of UDPglucuronyl transferase. What compound
is accumulated in blood serum in case of these pathologies?
+Unconjugated bilirubin

An infectious disease unit admitted a patient with signs of jaundice caused by hepatitis virus. Select an
indicator that is specific only for parenchymatous jaundice:
+Increase in ALT and AST rate

Toxic affection of liver results in dysfunction of protein synthesis. It is usually accompanied by the
following kind of dysproteinemia:
+Absolute hypoproteinemia

Blood analysis of a patient with jaundice reveals conjugated bilirubinemia, increased concentration of bile
acids. There is no stercobilinogen in urine. What type of jaundice is it?
+Obstructive jaundice
A patient presents with icterus of skin, sclera and mucous membranes. Blood plasma the total bilirubin is
increased, stercobilin is increased in feces, urobilin is increased in urine. What type of jaundice is it?
+Haemolytic

Jaundice treatment involves administration of barbiturates inducing the synthesis of UDP-glucuronyl


transferase. A medicinal effect is caused by the
production of
+Direct reacting (conjugated) bilirubin

A coprological survey revealed lightcolored feces containing drops of neutral fat. The most likely reason for
this condition is the disorder of:
+Bile inflow into the bowel

A viral infection has damaged cells that form walls of bile capillaries. This stimulated conditions for inflow
of bile into the blood of sinusoidal capillaries. What cells are damaged?
+Hepatocytes
Hepatitis has led to the development of hepatic failure. Mechanism of edema formation is activated by the
impairment of the following liver function:
+Protein-synthetic
A patient suffers from hepatocirrhosis. State of antitoxic liver function can be characterized by examination
of the following substance exreted by urine:
+Hippuric acid

A patient complains of frequent diarrheas, especially after consumption of rich food, weight loss. Laboratory
examination revealed steatorrhea; his feces were hypocholic. What might have caused such condition?
+Obturation of biliary tracts

After a disease a 16-year-old boy is presenting with decreased function of protein synthesis in the liver as a
result of vitamin K deficiency. This may cause disorder of:
+Blood coagulation
A 48-year-old patient was admitted to the hospital with complaints about weakness, irritability, sleep
disturbance. Objectively: skin and sclera are of yellow color. In blood: increased concentration of total
bilirubin with prevailing direct bilirubin. The feces are acholic. The urine is dark (contains bile pigments).
What type of jaundice is it?
+Mechanic

A 50-year-old patient during examination presents with glucosuria and blood glucose of 3.0 mmol/l, which
are the most likely to be caused by:
+Renal disorder

A 38-year-old man, who has been suffering from systemic lupus erythematosus for 3 years, developed
diffuse renal lesions accompanied by massive edemas, marked proteinuria, hyperlipidemia and
dysproteinemia. What is the most likely mechanism of proteinuria development in this case?
+Autoimmune damage to the nephrons
A lab rat has subcutaneously received mercury (II) chloride in the amount of 5 mg/kg. 24 hours later the
plasma creatinine concentration increased several times. What mechanism of retention azotemia is observed
in this case?
+Decreased glomerulus filtration

A traumatology unit received a patient with crushed muscular tissue. What biochemical indicator of urine
will be raised in this case?
+Creatinine
A 40-year-old woman was diagnosed with glomerulonephritis based on her clinical symptoms and the results
of urine analysis. Anamnesis states chronic tonsillitis. What microorganisms are the most likely cause for the
kidney damage in this case?
+Streptococci

Glomerular filtration of a person, who has been starving for a long time, has increased by 20%. The most
likely cause of filtration changes in the given conditions is:
+Decrease of blood plasma oncotic pressure

A 12-year-old child has developed nephritic syndrome (proteinuria, hematuria, cylindruria) 2 weeks after the
case of tonsillitis, which is a sign of affected glomerular basement membrane in the kidneys. What
mechanism is the most likely to cause the basement membrane damage?
+Immune complex

Urine analysis has shown high levels of protein and erythrocytes in urine. This can be caused by the
following:
+Renal filter permeability

Due to the use of poor-quality measles vaccine for preventive vaccination, a 1-year-old child developed an
autoimmune renal injury. The urine was found to contain macromolecular proteins. What process of urine
formation was disturbed?
+Filtration

Urine analysis has shown high levels of protein and erythrocytes in urine. This can be caused by the
following:
+Renal filter permeability

A patient has insufficient blood supply to the kidneys, which has caused the development of pressor effect
due to the constriction of arterial resistance vessels. This is the result of the vessels being greately affected
by the following substance:
+Angiotensin II
14 days after quinsy a 15-year-old child presented with morning facial swelling, high blood pressure, "meat
slops"urine. Immunohistological study of a renal biopsy sample revealed deposition of immune complexes
on the basement membranes of the capillaries and in the glomerular mesangium. What disease developed in
the patient?
+Acute glomerulonephritis
A month after surgical constriction of rabbit’s renal artery the considerable increase of systematic arterial
pressure was observed. What of the following regulation mechanisms caused the animal’s pressure change?
+Angiotensin-II

A child has an acute renal failure. What biochemical factor found in saliva can confirm this diagnosis?
+Increase in urea concentration
A patient with a history of chronic glomerulonephritis presents with azotemia, oliguria, hypo- and
isosthenuria, proteinuria. What is the leading factor in the pathogenesis of these symptoms development
under chronic renal failure?
+Mass decrease of active nephrons

A patient has been diagnosed with acute glomerulonephritis that developed after he had had streptococcal
infection. It is most likely that the affection of basal glomerular membrane is caused by an allergic reaction
of the following type:
+Immune complex

A patient is 44 years old. Laboratory examination of his blood revealed that content of proteins in plasma
was 40 g/l. What influence will be exerted on the transcapillary water metabolism?
+Filtration will be increased, reabsorption- decreased
A patient with massive burns developed acute renal insufficiency characterized by a significant and rapid
deceleration of glomerular filtration. What is the mechanism of its development?
+Reduction of renal blood flow

A 30- year old woman has face edema. Examination revealed proteinuria (5,87 g/l), hypoproteinemia,
dysproteinemia, hyperlipidemia. What condition is the set of these symptoms typical for?
+Nephrotic syndrome

Two weeks after lacunar tonsillitis a 20- year-old man started complaining about general weakness, lower
eyelid edemata. After examination the patient was diagnosed with acute glomerulonephritis. What are the
most likely pathological changes in the urine formula?
+Proteinuria

As a result of continuous starvation the glomerular filtration rate has increased by 20%. The most probable
cause of the glomerular filtration alteration under the mentioned conditions is:
+Decrease in the oncotic pressure of blood plasma

A patient with massive burns developed acute renal insufficiency characterized by a significant and rapid
deceleration of glomerular filtration. What is the mechanism of its development?
+Reduction of renal blood flow

An electron micrograph of a kidney fragment presents an afferent arteriole. Under its endothelium some big
cells can be seen that contain secretory granules. What type of cells is it?
+Juxtaglomerular

Examination of a 43 y.o. anechoic patient revealed anemia symptoms. What is the cause of these symptoms?
+Reduced synthesis of erythropoietins

Violation of safety rules resulted in calomel intoxication. Two days later the daily diuresis was 620 ml. A
patient experienced headache, vomiting, convulsions, dyspnea, moist rales in lungs. What pathology is it?
+Acute renal insufficiency

On the 6th day of treatment a patient with acute renal insufficiency developed polyuria. Diuresis
intensification at the beginning of polyuria stage of acute renal insufficiency is caused by:
+Renewal of filtration in nephrons

A 65- year old man suffering from gout complains of kidney pain. Ultrasound examination revealed renal
calculi. The most probable cause of calculi formation is the strengthened concentration of the following
substance:
+Uric acid

A 30 -year old woman has face edema. Examination revealed proteinuria (5,87 g/l), hypoproteinemia,
dysproteinemia, hyperlipidemia. What condition is the set of these symptom typical for?
+Nephrotic syndrome

A patient is 44 years old. Laboratory examination of his blood revealed that content of proteins in plasma
was 40 g/l. What influence will be exerted on the trans-capillary water exchange?
+Filtration will be increased, reabsorption- decreased
A driver who got a trauma in a road accident and is shocked has reduction of daily urinary output down to
300 ml. What is the main pathogenetic factor of such diuresis change?
+Drop of arterial pressure

During removal of the hyperplastic thyroid gland of a 47-year-old woman, the parathyroid gland was
damaged. One month after the surgery the patient developed signs of hypoparathyroidism: frequent
convulsions, hyperreflexia, laryngospasm. What is the most likely cause of the patient’s condition?
+Hypocalcemia

On examination the patient presents with hirsutism, moon-shaped face, stretch marks on the abdomen. BP is
190/100 mm Hg, blood glucose is 17.6 mmol/L. What pathology is such clinical presentation characteristic
of?
+Adrenocortical hyperfunction
A 40-year-old woman suffers from Cushing’s disease - steroid diabetes. On biochemical examination she
has hyperglycemia and hypochloremia. What process activates in the first place in such patients?
+Gluconeogenesis

Corticosteroid hormones regulate the adaptation processes of the body as a whole to environmental changes
and ensure the maintenance of internal homeostasis. What hormone activates the hypothalamo-pituitary-
adrenal axis?
+Corticoliberin
Autopsy of a 40-year-old woman, who died of cerebral hemorrhage during hypertension crisis, revealed:
upper body obesity, hypertrichosis, hirsutism, stretchmarks on the skin of thighs and abdomen. Pituitary
basophil adenoma is detected in the anterior lobe. What diagnosis is the most likely?
+Cushing’s disease

Examination of a 42-year-old patient revealed a tumor of adenohypophysis. Objectively: the patient’s weight
is 117 kg, he has moon-like hyperemic face, red-blue striae of skin distension on his belly. Osteoporosis and
muscle dystrophy are present. AP is 210/140 mm Hg. What is the most probable diagnosis?
+Cushing’s disease

In the course of an experiment adenohypophysis of an animal has been removed. The resulting atrophy of
thyroid gland and adrenal cortex has been caused by deficiency of the following hormone:
+Tropic hormones
A man has a considerable decrease in diuresis as a result of 1,5 l blood loss. The primary cause of such
diuresis disorder is the hypersecretion of the following hormone:
+Vasopressin
A 20- year old patient complains of morbid thirst and increased diuresis (up to 10 l daily). Glucose
concentration in blood is normal but it is absent in urine. The patient has been diagnosed with diabetes
insipidus. What hormonal drug is the most appropriate for management of this disorder?
+Vasopressin

The secretion of which hypophysial hormones will be inhibited after taking the oral contraceptives
containing sex hormones?
+Gonadotropic hormone
A 32-year-old patient consulted a doctor about the absence of lactation after parturition. Such disorder might
be explained by the deficit of the following hormone:
+Prolactin

A patient complains of polyruria (7 liters per day) and polydipsia. Examination reveals no disorders of
carbohydrate metabolism. These abnormalities might be caused by the dysfunction of the following
endocrine gland:
+Neurohypophysis
Roentgenological examination of skull base bones revealed enlargement of sellar cavity, thinning of anterior
clinoid processes, destruction of different parts, destruction of different parts of sella turcica. Such bone
destruction might be caused by a tumour of the following wndocrinous gland:
+Hypophysis
Examination of a patient revealed overgrowth of facial bones and soft tissues, tongue enlargement, wide
interdental spaces in the enlarged dental arch. What changes of the hormonal secretion are the most likely?
+Hypersecretion of the somatotropic hormone

A 32-year-old patient consulted a doctor about the absence of lactation after parturition. Such disorder might
be explained by the deficit of the following hormone:
+Prolactin

A patient has a decreased vasopressin synthesis that causes polyuria and as a result of it evident organism
dehydration. What is the mechanism of polyuria development?
+Reduced tubular reabsorption of water
Osmotic pressure of a man’s blood plasma is 350 mosmole/l (standard pressure is 300 mosmole/l). First of
all it will result in high secretion of the following hormone:
+Vasopressin

Intake of oral contraceptives containing sex hormones inhibits secretion of the hypophysiae hormones.
Secretion of which of the indicated hormones is inhibited while taking oral contraceptives with sex
hormones?
+Follicle-stimulating

A 46-year-old patient has complained of headache, fatigue, thirst, pains in the spine and joints for the last 2
years. Clinically observed disproportional enlargement of hands, feet, nose, superciliary arches. He notes
that he needed to buy bigger shoes three times. What is the main reason of such disproportional enlargement
of different parts of the body?
+Cartilaginous tissue proliferation under growth hormone influence

Prolonged treatment of hypothyroidism has caused general dystrophy, dental caries, tachycardia, tremor of
extremities. What drug is the cause of these side effects?
+L-thyroxin

A patient with signs of osteoporosis and urolithiasis has been admitted to an endocrinology department.
Blood test revealed hypercalcemia and hypophosphatemia. These changes are associated with abnormal
synthesis of the following hormone:
+Parathyroid hormone

A 4 -year old child with hereditary renal lesion has signs of rickets, vitamin D concentration in blood is
normal. What is the most probable cause of rickets development?
+Impaired synthesis of calcitriol

A 5-month-old boy was hospitalized for tonic convulsions. He has a lifetime history of this disease.
Examination revealed coarse hair, thinned and fragile nails, pale and dry skin. In blood: calcium - 1,5
mmole/l, phosphor - 1,9 mmole/l. These changes are associated with:
+Hypoparathyroidism

Parodontitis is treated with calcium preparations and a hormone that stimulates tooth mineralization and
inhibits tissue resorption. What hormone is it?
+Calcitonin

A child has abnormal formation of tooth enamel and dentin as a result of low concentration of calcium ions
in blood. Such abnormalities might be caused by deficiency of the following hormone:
+Parathormone

A 46-year-old patient suffering from the diffuse toxic goiter underwent resection of the thyroid gland. After
the surgery the patient presents with appetite loss, dyspepsia, increased neuromuscular excitement. The body
weight remained unchanged. Body temperature is normal. Which of the following has caused such a
condition in this patient?
+Reduced production of parathormone
A 2 y.o. child has convulsions as a result of lowered concentration of calcium ions in blood plasma. It is
caused by reduced function of:
+Parathyroid glands

Kidneys of a man under examination show increased resorbtion of calcium ions and decreased resorbtion of
phosphate ions. What hormone causes this phenomenon?
+Parathormone

A 41-year-old man has a history of recurrent attacks of heartbeats (paroxysms), profuse sweating, headaches.
Examination revealed hypertension, hyperglycemia, increased basal metabolic rate, and tachycardia. These
clinical presentations are typical for the following adrenal pathology:
+Hyperfunction of the medulla

There are cortical and medullary substances separated by connective tissue layer in the endocrine gland
specimen. Parenchyma cells make up three zones in cortical substance, with rounded masses in the
superficial zone, parallel chords in the middle one, reticular structure of cell chords in the deep one. What
gland is it?
+Adrenal gland

A 19-year-old male was found to have an elevated level of potassium in the secondary urine. These changes
might have been caused by the increase in the following hormone level:
+Aldosterone
A 38-year-old female patient complains of general weakness, cardiac pain, increased appetite, no
menstruation. Objectively: the height is 166 cm, weight 108 kg, the patient has moon-shaped face,
subcutaneous fat is deposited mainly in the upper body, torso and hips. There are also blood-red streaks. Ps-
62/min, AP- 160/105 mm Hg. Which of the following diseases is the described pattern of obesity most
typical for?
+Cushing pituitary basophilism

A 44- year old woman complains of general weakness, heart pain, significant increase of body weight.
Objectively: moon face, hirsutism, AP is 165/100 mm Hg, height - 164 cm, weight - 103 kg; the fat is mostly
accumulated on her neck, thoracic girdle, belly. What is the main pathogenetic mechanism of obesity?
+Increased production of glucocorticoids

A concentrated solution of sodium chloride was intravenously injected to an animal. This caused decreased
reabsorption of sodium ions in the renal tubules. It is the result of the following changes of hormonal
secretion:
+Aldosterone reduction
A patient ill with neurodermatitis has been taking prednisolone for a long time. Examination revealed high
rate of sugar in his blood. This complication is caused by the drug influence upon the following link of
carbohydrate metabolism:
+Gluconeogenesis activation

A patient suffering from pheochromocytoma complains of thirst, dry mouth, hunger. Blood test for sugar
revealed hyperglycemia. What type of hyperglycemia is it?
+Adrenal

A middle-aged man went to a foreign country because he had been offered a job there. However he had been
unemployed for quite a long time. What endocrine glands were exhausted most of all in this man?
+Adrenal glands

A patient with android-type obesity had been suffering from arterial hypertension, hyperglycemia, glycosuria
for a long time and died from the cerebral haemorrhage. Pathologic examination revealed pituitary basophil
adenoma, adrenal cortex hyperplasia. What is the most likely diagnosis?
+Itsenko-Cushing’s syndrome
To prevent the transplant rejection after organ transplantation it is required to administer hormonotherapy for
the purpose of immunosuppression. What hormones are used for this purpose?
+Glucocorticoids
A 59- year old patient is a plant manager. After the tax inspection of his plant he felt intense pain behind his
breastbone irradiating to his left arm.15 minutes later his condition came to normal. Which of the possible
mechanisms of stenocardia development is the leading in this case?
+High catecholamine concentration in blood

Examination of a patient revealed hyperkaliemia and hyponatremia. Low secretion of which hormone may
cause such changes?
+Aldosteron

A 40 y.o. patient complains of intensive heartbeats, sweating, nausea, vision impairment, arm tremor,
hypertension. From his anamnesis: 2 years ago he was diagnosed with pheochromocytoma.
+Hyperproduction of what hormones
causes the given pathology?
Catecholamines

Continious taking of a drug can result in osteoporosis, erosion of stomach mucous membrane, hypokaliemia,
retention of sodium and water, reduced content of corticotropin in blood. Name this drug:
+Prednisolone

A 30-year-old woman developed the signs of virilism (body hair growth, balding temples, disturbed
menstrual cycle). What hormone can cause this condition when hyperproduced?
+Relaxin

Parents of a 10 y.o. boy consulted a doctor about extension of hair-covering, growth of beard and moustache,
low voice. Intensified secretion of which hormone must be assumed?
+Of testosterone

A girl is diagnosed with adrenogenital syndrome (pseudohermaphroditism). This pathology was caused by
hypersecretion of the following adrenal hormone:
+Androgen
A female patient presents with endocrine dysfunction of follicular cells of the ovarian follicles resulting from
an inflammation. The synthesis of the following hormone will be inhibited:
+Estrogen
A 30-year-old female exhibits signs of virilism (growth of body hair, balding temples, menstrual disorders).
This condition can be caused by the overproduction of the following hormone:
+Testosterone

64-year-old woman presents with disturbed fine motor function of her fingers, marked muscle rigidity and
tremor. The neurologist diagnosed her with Parkinson's disease. What brain structures are damaged resulting
in this disease?
+Substantia niagra

A patient got a gunshot wound of hip which damaged the sciatic nerve. Any impact on the affected limb
causes severe, excruciating pain. What mechanism of pain is most likely in this case?
+Causalgic

After the traumatic tooth extraction, a patient is complaining of acute, dull, poorly-localized pain in gingiva,
body temperature rises up to 37, 5oC. The patient has been diagnosed with alveolitis. Specify the kind of
pain in this patient:
+Protopathic

After amputating the upper extremity, a patient had a bad pain in it. Which mechanism of the pain feeling
formation is more possible in this case?
+Phantom
After a road accident a patient was diagnosed with a trauma of the brachium with incomplete rupture of the
median nerve. Besides, disorders of the motor and sense functions, the patient complains of sharp, stinging,
intolerable pain. What kind of pain is it?
+Causalgia

A patient, complaining of pain in the region of the left scapula, was diagnosed with myocardial infarction.
What kind of pain is it?
+Irradiating (reflected)
A patient after hypertension stroke does not have voluntary movements in his right arm and leg with the
increased muscle tone in these extremites. What type of disfunction of nervous system is it?
+Central paralysis
A 68-year-old woman cannot move her upper and lower right extremities after stroke. Muscle tone of these
extremities and reflexes are increased. There are pathological reflexes. What form of the paralysis is it?
+Hemiplegia

A 28 -year old man had a gunshot wound of shin that resulted in an ulcer from the side of the injury. What is
the main factor of neurodystrophy pathogenesis in this case?
+Traumatization of peripheral nerve

An experimental rat with extremity paralysis has no tendon and cutaneous reflexes, muscle tone is
decreased, but muscles of the affected extremity maintain their ability to react with excitation to the direct
action of continious current. What type of paralysis is it?
+Flaccid peripheral

Increase of blood pressure and rapid pulse are noticed in a sportsman at the start before competitions.
Influence of which part of the CNS can above-mentioned changes be explained?
+Cortex of hemispheares
A patient had hemiplegia after insult. What disorder is observed in this case?
+Movement

In an experiment a part of the brain of an animal has been removed. As a result, asynergia, atonia, and
dysmetria developed. What part of the brain was removed?
+CerebeIlum

Patients, suffering from epilepsy, have specific centers in brain cortex. They function as a pathological
determinant. What is the underlying mechanism of the formation of these centers?
+Formation of generator of pathological exaltation

A 68-year-old woman had a stroke which resulted in the absence of voluntary movements of both upper and
lower right extremities. The tonus of muscles and reflexes is increased. Pathological reflexes are observed.
What kind of paralysis is it?
+A Tetraplegic.
After a hypertonic crisis a patient presents with lacking spontaneous movements in his right arm and leg,
muscle tone of these extremities is increased. What type of motor dysfunction has developed in this case?
+Central paresis

ЭКСТРЕМАЛЬНЫЕ СОСТОЯНИЯ (3)


As a result of a trauma a patient has developed traumatic shock that led to the following disorders: AP is
140/90 mm Hg, Ps is 120 bpm. The patient is fussy, talkative, pale. Such state relates to the following shock
phase:
Erectile

After a road accident a victim has tachycardia, arterial blood pressure 130/90 mm Hg, tachypnoe, the skin is
pale and dry, excitation of central nervous system is observed. What shock stage is the patient most likely
in?
Erectile
Terminal
Torpid
Preshock (compensation stage)
Agony

A 27-year-old patient with injury to the neck has lost approximately 30% of the blood volume. The patient’s
condition is severe: blood pressure is 60/40 mm Hg, heart rate is 140/min., respiratory rate is 30/min.,
conscious. Characterize the condition of the patient’s circulatory system:
Hypovolemic shock

Pathophysiology of blood and hemopoietic organs.

#Anemia, leuko- and thrombocytopenia, color index-1.3, presence of megaloblasts and megalocytes were
determined in the laboratory analysis of blood of a patient a year later after he was operated on for subtotal
resection of the stomach for the ulcer of lesser curvature of the stomach. What factor deficiency results in
these changes?
+Gastromucoprotein
-Gastrin
-Pepsin
-Chlorine hydrate
-Mucin
#Amino acids replacement in alpha and beta chains of hemoglobin takes place in a number of
hemoglobimopathies. Which of them is typical for HbS (sickle-cell anemia)?
-Glycine to serine
-Aspartate to lysine
-Methionine to histidine
+Glutamate to valine
-Alanine to serine

#Hereditary microspherocytic hemolytic anemia (Mincovsky-Shoffar disease) was diagnosed in a woman


aged 34. What mechanism caused hemolysis of erythrocytes in the patient?
-Enzymopathy
-Hemoglobinopathy
-Autoimmune impairment
+Membranopathy
-Hypoplasia of bone marrow

#Megaloblastic anemia was diagnosed in a patient. What substance deficiency may cause the development
of this disease?
+Cyanocobalamin
-Cholecalciferol
-Magnesium
-Glycine
-Copper

#Three years ago a man aged 45 was operated on for stomach resection. After the operation the content of
erythrocytes in the blood is 2.0x1012, Hb 85 g/l, color index-1.27. What vitamin absorption is impaired that
causes the change of erythropoiesis?
-C
-P
-A
-B6
+B12

#A patient, carrier of hereditary sickle-cell erythrocytes anomaly, had pneumonia accompanied by hemolytic
crisis and development of anemia. What is the main cause of hemolytic crisis in this case?
-Hyperoxia
-Heterozygosis HbS
-Mutation of structural gene
+Hypoxia caused by pneumonia
-Blood osmolarity change

#Examining the oral cavity of a patient, a dentist paid attention to the presence of inflammatory-dystrophy
process in the mucous membrane (Gunter’s glossitis, atrophic stomatitis). Blood analysis revealed
hyperchromic anemia. What factor is a cause of this disease?
-Hypovitaminosis B6
-Hypovitaminosis A
-Increase of stomach juice acidity
-Hypovitaminosis B1
+Hypovitaminosis B12

#A female patient complains of malaise, weakness, breathlessness, rapid fatigability, and dizziness. Her
blood test data: erythrocytes-1.8x1012/L, Hb-80 g/L, leukocytes-3.2x109/L, color index-1.5. Anisocytosis,
poikilocytosis, megaloblasts, megalocytes were found in smear. What is the possible diagnosis?
+B12-deficiency anemia
-Posthemorragic anemia
-Acute leukemia
-Iron deficiency anemia
-Immunohemolytic anemia

#A patient, aged 50, complains of a bad appetite, loss of weight, weakness, pain in the stomach area, and
eructation. At laboratory examination of him: Hb-2x10 12/L, stomach secretion 0.4 l, pH of stomach juice-7.
Pernicious anemia is diagnosed in this patient. What compound of gastric juice deficiency is the cause of the
disease?
-Pepsin
-Renin
-Secretin
-HCL
+Castle’s factor

#Funicular myelosis and hyperchromic anemia developed in a man 7 years later after the stomach resection
due to ulcer. What pathogenic mechanism of changes in spinal cord is the most possible one?
-Hypoxia impairment in anemia
+Accumulation of methylmalonic acid in cyanocobalamin deficiency
-Impairment of DNA in cyanocobalamin deficiency
-Deficiency of folic acid
-Deficiency of iron containing enzyme

#On the seventh day after hemorrhage caused by a trauma, the following was revealed in a patient’s blood:
erythrocytes - 2.8x1012/l, Hb - 3.9mmol/l, reticulocytes - 15%, acidophilic and polychromatophilic
normoblasts were found in smear. What is the mechanism of appearance of regenerative forms of blood
erythrocytes?
-Going out of blood deposition
-Increase permeability of erythrocyte sprout of bone marrow
-Intensification of regeneration of erythroid cells in bone marrow
+Inhibition of maturation of erythroid cells in bone marrow
-Inhibition of synthesis of erythroprotein inhibitor

#When treating a patient for malaria with primachine, hemolysis occurs in him. At examination deficiency
of Glucose-6-phosphate Dehydrogenase in patient’s erythrocytes was revealed. What metabolic process is
impaired in this patient?
-Aerobic carbohydrate oxidation
-Glycogenesis
-Glycolysis
-Gluconeogenesis
+Pentose-phosphate pathway

#B12-folate deficient anemia was diagnosed in a patient aged 55. What hematological index will be the most
sensitive for pathogenetic treatment?
-Increase of number of erythrocytes
-Increase of number of reticulocytes
+Reduction of anisocytosis
-Decrease of ESC
-Decrease of amount of hemoglobin

#In a pregnant woman, in the 7th month of pregnancy, anemia begins rising fast. In her blood test: number of
erythrocytes is 2.7x1012/L, amount of hemoglobin is 90 g/L, anisocytosis, poicilocytosis, megaloblasts and
megalocytes were found, reticulocytes of 0%. What sort of anemia develops in this case?
-Thalassemia
+B12-deficiency anemia
-Post-hemorrhagic anemia
-Iron-deficiency anemia
-Hemolytic anemia

#In an infant, who is under an artificial nutrition with cow milk, severe anemia has developed. At the blood
count of the infant: number of erythrocytes is 4x10 12/L, content of hemoglobin is 68 g/L, reticulocytes of
0%. What kind of anemia developed in the infant?
-Sickle-cell anemia
-Inborn hemolytic anemia
-B12-deficiency anemia
-Hypopastic anemia
+Iron-deficiency anemia

#Predomination of erythroblasts, normoblasts and megaloblasts was revealed in blood analysis of a patient
with anemia. The same cells were found in bone marrow. What type of anemia do these changes
characteristic for?
-Post-hemorrhagic
-Hemolytic
-Aplastic
+B12-folate deficiency anemia
-Iron-deficiency anemia

#Two years ago a patient was operated on for resection of pyloric portion of the stomach. There is weakness,
periodical appearance of dark circles under eyes, breathlessness in this patient. In the blood analysis: HB-
70g/L, erythrocytes-3.3x1012/L, color index-0.7. What erythrocyte changes in the blood smear are the most
typical for the given state?
+Microcytes
-Megalocytes
-Schizocytes
-Ovalocytes
-Macrocytes
#Singular oxyphilic normoblasts appeared in the blood of a patient after acute post-traumatic hemorrhage
composing 15% of blood volume. On supravital staining 25% of reticulocytes were found. What is the
patient anemia according to its ability of regeneration?
+Hyperregenerative
-Regenerative
-Hyporegenerative
-Aregenerative
-Hypo- and aregenerative

#What index of blood analysis is the most typical for beta-thalassemia?


-Considerable decrease of erythrocytes and hemoglobin
-Erythrocytes with basophilic stippling
+Increase of fetal hemoglobin
-Target-like erythrocytes
-Increase of met-hemoglobin

#A patient had anemia due to profuse blood loss. What blood changes are typical at the beginning of
development of acute post-hemorrhagic anemia?
-Presence of megalocytes in the blood
-Absence of reticulocytes
-Poikilocytosis, anisocytosis
-Hyperchromia
+Normochromia

#Erythropenia, hyperchromia, normocytes, macrocytes, megalocytes, poikilocytosis were found out in a


patient’s blood at examination. What is the cause of this pathology?
-Ascariasis
+Deficiency of gastromucoprotein
-Iron deficiency in food
-Trichocephaliasis
-Fequent loss of blood

#While studying a blood smear different forms of erythrocytes were found. Which of them are regenerative?
-Anisocytes
-Poikilocytes
+Ploychromatophils
-Ovalocytes
-Megalocytes

#The presence of agranulocytosis was determined in the blood analysis of a liquidator of accident at
Chernobyl atomic power station who had got 5 Gr dose of irradiation. What pathogenetic mechanism is the
leading one in its appearance?
+Inhibition of leucopoiesis
-Increased penetration of granulocytes into tissues
-Increase of leucocytes destruction
-Impairment of going out of mature leukocytes from bone marrow
-Development of autoimmune process
#During the development of acute pulpitis a patient complained of paroxysm of pain in the upper jaw, which
is increasing at night, fever. At examination leucocytosis was established in the blood. What kind of
leucocytosis is possible in this case?
-Basophilic leukocytosis
-Lymphocytosis
-Eosinophilic leukocytosis
-Monocytosis
+Neutrophilic leuccytosis

#Considerable increase of the number of eosinophils in a unit of blood volume was determined during the
examination of a 5 years old boy. What may cause eosinophilia in this patient?
+Helminthic invasion
-Obesity
-Hypodynamia
-Hypothermia
-Physical exertion

#In the patient’s blood analysis the number of leukocytes is 250*109/L. What syndrome does this patient
have?
+Leukemia
-Leucocytosis
-Leucopoenia
-Leucomoid reaction
-Hyperleucocytosis

#Leucocytosis was found out in a person who didn’t complain of his health. The cause of this may be that
fact that the blood was taken for analysis after:
+Physical load
-Mental work
-Rest at a health resort
-Considerable use of water
-Usage of alcohol

#Lymphocytosis was revealed in a patient. What diseases may be accompanied by lymphocytosis?


-Sepsis
-Helminthic invasion
-Somatotropin insufficiency
+Pertussis, chicken pox
-Bronchial asthma

#Neutropoenia is found out in a patient who has the manifestations of immunodeficiency. What diseases
may neutropoenia be determined in?
-After profuse haemorrhage
+Fuzarium fungus poisoning
-Myeloleukemia
-Insufficiency of sexual gland function
-Septic process

#Increase of the number of eosinophils was determined in a patient with endocrine pathology during
examination. Which of the named diseases may be accompanied by eosinophilia?
-Pheochromocytoma
-Conn’s disease
-Acromegaly
-Cushing disease
+Addison’s disease

#Functional insufficiency of monocytes is accompanied by immunodeficiency. What BAS produced by


monocytes stimulate specific response?
-Interleukin-2
-Lysocime
+Interleukin-1
-Fibronectin
-Myeloperoxidase

#Relative neutropoenia with degenerative shift was revealed in the blood of a patient with TB during
examination. What change of differential blood count corresponds to this state?
+Decrease of the number of segmented forms and increase of band forms of neutrophils
-Decrease of lymphocyte number
-Increase of monocyte number
-Increase of basophil number
-Decrease of eosinophil number

#Neutrophil leukocytosis was revealed in worker during examination. What pathologic condition may
neutrophil leukocytosis result from?
-Chronic loss of blood
+Septic condition
-Viral infection
-Radiation sickness
-Benzene poisoning

#A patient aged 32 with massive hemorrhage due to car accident trauma was admitted to the hospital. Pluse-
100 beats per min, respiratory rate-22 per 2 min, BP-100/60 mmHg. What blood change will be the most
characteristic in an hour after hemorrhage?
-Erythropoenia
-Hypoproteinemia
+Hypovolemia
-Leucopoenia
-Erythrocyte hypochromia

#During the examination of peripheral blood of a patient aged 42, it was revealed: Hb-80g/L, erythrocytes-
3.2 x 1012/L, leukocytes-25 x 109/L, leukocytic formula: basophils-5%, eosinophils- 9%, myeoblasts- 3 %,
promyelocytes-8%, neutrophils-17%, segmented- 19%, lymphocytes- 3 %, monocytes- 3%. What blood
pathology is the most possible in this patient?
-Panmyelophthisis
-Erythroleukemia
-Acute myeloblast leukemia
+Chronic myelogenous leukemia
-Promyelocytic leukemia

#The experiment was carried out on a rabbit. The increase of the number of erythrocytes and hemoglobin in
the blood due to the stimulation of erythropoiesis by erythropoietin was determined 2 weeks later after the
narrowing of renal artery. What increases the formation of erythropoietin?
-Hypoosmia
-Hypercapnia
+Hypoxemia
-Hyperosmia
-Hypovolemia

#Hemolytic anemia with decrease of osmotic erythrocyte resistance that averaged 0.6-0.5 was revealed in a
smear of venous blood during microscopic examination. What substance accumulation in the blood plasma
may also indicate the development oh hemolytic anemia?
-Creatinine
-Urea
+Indirect bilirubin
-Lactic acid
-Inorganic phosphate

#Hypochromia of erythrocytes, micro-, anisocytosis, poikilocytosis, are determined in the blood in case of
development of iron deficiency and iron refractory anemia. What index must be determined to carry out
differential diagnosis of these anemias?
-Serum chlorine
-Serum phosphorous
-Serum magnesium
-Serum calcium
+Serum iron

#B12 folic deficiency anemia developed in a patient after stomach resection. What color index is typical for
this disease?
+1.30
-1.15
-1.0
-0.85
-0.70

#A victim of a car accident has lost much blood. What impairment of general blood volume takes place?
-Simple hypovolemia
-Polycythemic normovolemia
-Olygocysthemic normovolemia
-Polycystehmic hypovolemia
+Oligocysthemis hypovolemia
#Excessive flow of estrogens into the blood due to follicle persistence (a state when follicle does not reach
complete maturation and ovulation does not take place) often cause uterine bleeding. What anemia may
develop in this case?
+Iron deficiency
-Sideroachrestic
-Sickle cell
-Hypoplastic
-Metaplastic

#Which of the below named anemias relevant to hemoglobinopathies?


-Minkovsky-Shoffar disease
-Iron deficiency anemia
-B12 deficiency
+Thalassemia
-Iron refractory anemia

#Evaluate the blood analysis: erythrocytes 3.10; Hb 90g\L reticulocytes 0.5%. In the smear there are
poikilocytes hypochromatic erythrocytes. Blood serum iron is 80 micromol/l. What pathology is this typical
for?
-Minkovsky-Shoffer disease
-Iron deficiency anemia
-B12 deficiency
-Sickle cell anemia
+Iron refractory anemia
#What conditions are accompanied by neutrophilic leukocytosis with shift of differential count to the left?
+Purulent inflammation
-Tuberculosis
-Infections mononucleiosis
-Agranulocytosis
-Alimentaray leukocytosis

#What blood pathology is the presence of Philadelphia chromosomes in the blood cells and bone marrow
cells typical for?
-Acute myelogenous leukemia
+Chronic myelogenous leukemia
-Hodgkin’s disease
-Burkitt’s lymphoma
-Chronic lymphocyte leukemia

#Patient M, aged 20 was admitted to the hospital complaining of high temperature, pain in the bones, and
hemorrhage from his gums. Blood analysis of this patient shows: erythrocytes-2.5x10 12/L; Hb-80g/L;
leucocytes-2.0x109/L; thrombocytes-6.0x109/L; differential count: eosinophils-1%; stab neutrophils-1%;
segmented neutrophils-10%; lymphocytes-10%; monocytes-3%; blast cells-75%. What pathology is this
blood analysis typical for?
-Hodgkin’s disease
-Burkitt’s lymphoma
+Acute leukemia
-Infections mononucleosis
-Chronic leukemia

#General amount of leucocytes is 90x109/l. In differential count: eosinophils-1%; basophils-0%; juvenile


neutrophils-0%; stab neutrophils-2%; segmented neutrophils-20%; prolymphocytes-2%; lymphocytes-70%;
Botkin-Gumprecht cells. Cervical, submandibular lymph nodes are enlarged. What pathology is such blood
picture typical for?
-Acute lymphoblastic leukemia
-Hodgkin’s disease
-Infectious mononucleosis
+Chronic lymphocytic leukemia
-Chronic myelogenous leukemia

#Hypochromic anemia was found out in a patient aged 54, who had a prolonged contact with lead at his
work. Treatment with iron preparations for a month didn’t give any effect. The increased amount of iron was
determined in blood serum. What is this anemia due to?
-Vitamin B12 deficiency
-Porphirin synthesis impairment
-Folic acid deficiency
+Hypoplasia of red bone marrow
-Erythrocyte hemolysis

#A patient aged 20, has periodically yellowness of sclera and skin that is accompanied by weakness.
Minkovski-Shoffar disease is diagnosed. What is the most typical for blood picture in this disease?
-Reticulositosis
-Agranulocytosis
-Macrocytosis
+Microspherocytosis
-Thrombocytosis

#Posthemorrhagic anemia has developed in a patient with periodical bleeding due to uterus fribromyoma.
What type of chronic post hemorrhagic anemia takes place in this case?
-Megaloblastic, hyperchromic, hyperegenerative
-Megaloblastic, hypochromic, hyporegenerative
-Erythroblastic, hyperchromic, hyporegenerative
-Erythroblastic, hypochromic, hyperregenerative
+Erythroblastic, hypochronic, hyporegenerative

#A female patient with impairment of menstrual cycle accompanied by prolonged bleeding the blood
analysis was made which revealed hypochromia, decrease of reticulocytes, microcytosis. What group does
this anemia belong to according to pathogenesis?
-B12 folic deficiency anemia
+Iron deficiency anemia
-Hypoplastic anemia
-Hemolytic anemia
-Metaplastic anemia
#Patient R. aged 12 was operated on for acute phlegmonous appendicitis. At the examination of his blood,
the amount of leukocytes is 12.109/L. On the blood smear there are: basophils – 0%; eosinophils – 2%;
monocytes – 2%; juvenile neutrophils – 0%; stab neutrophils – 30%; segmented neutrophils – 43%;
lymphocytes – 23%; monocytes – 0%. Stab (immature) neutrophils have pyknosis of nuclei. There is
anisocytosis of neutrophils; some of them have toxic granulation. What form of change of leukocyte blood
composition takes place in this case?
-Neutrophilia with degenerative shift to the left
-Neutrophilia with regenerative shift to the left
-Neutrophilia with hyperegenerative shift to the left
+Leukemoid reaction of neutrophil type
-Neutrophilia with the shift to the right

#Which of the below given hematological characteristics corresponds to chronic post-hemorrhagic anemia?
-Hyperregenerative, hypochromic with erythroblasts type of bleeding
-Regenerative, hypochromic with erythroblasts type of bleeding
-Hyporegenerative, hypochromic with megaloblasts type of bleeding
+Hyporegenerative, hypochromic with erythroblasts type of bleeding
-Hyporegenarative, hyperchromic with megaloblasts type of bleeding

#A female patient, aged 25, was admitted to the hematological department with complains of the appearance
of hemorrhages of different sizes on the body; during menstruation there are uterine bleedings. She has been
ill for ten years. Paleness of skin and mucous membranes were determined on examination; there are
hemorrhages of different size and color on the upper and lower extremities. Pulse = 100 beats/minute, AP
110/70 mmHg. Blood analysis shows: erythrocytes 3.3*10 12 /L, Hb 80g/L, thrombocytes 33*109/L; time of
blood coagulation: beginning is at 2nd minute, end is at 6th minute; time of bleeding (according to Duke) -
15 minutes. What is the possible diagnosis?
-Marchiafava-Michelli disease
+Thrombocytopoenic purpura
-Glanzman’s thrombasthenia
-Willeberandt-Yurgens thrombocytopathia
-Chronic myelogenous leukemia

#Neutrophil leukocytosis is determined in a patient with chronic myelogenous leukemia. Which variant of
nuclear shift of differential count to the left is the most typical for chronic myelogenous leukemia?
-Regenerative
-Degenerative
+Hyper regenerative
-Regenerative and degenerative
-Hyporegenerative

#A patient with atrophic gastritis has vitamin B 12 deficiency. What variant of nuclear shift of differential
count is most typical for B12 hypovitaminosis?
-Hyper regenerative to the left
-Degenerative to the left
-Regenerative and degenerative to the left
+To the right
-Regenerative to the left
#Hypochromic anemia was diagnosed in a patient on the 7th day after acute hemorrhage. What mechanism
is the leading one in its development?
-Increase of iron excretion from the organism;
-Impairment of globin synthesis;
-Increased erythrocyte destruction in the spleen;
-Impairment of iron absorption in the intestine;
+Increased penetration of immature erythrocytes from bone marrow

#Hemiparesis appeared in a patient with acute promyelocytic leukemia. What is the main mechanism of the
impairment of CNS in this case?
-Intoxication by leukemic cells decay products;
+Formation of leukemic infiltrates;
-Impairment of desintoxicative function of the liver;
-Cachexia;
-Increase of thrombogenesis

#Acute pain in the lower jaw, swelling of the cheek, temperature of 37.6 oC occurred in the patient who had
had dental caries for some years. What changes in the patient’s blood may be observed in this case?
+Neutrophilic leukocytosis;
-Leucopoenia;
-Moncytosis;
-Anemia;
-Eosinophilia

#A patient aged 43 has stomatitis, glossitis, the tongue is crimson colour, smooth. Blood analysis revealed:
Hb 100 g/l; erythrocytes 2.3x1019/l; color index 1.30. What is the patient’s state due to?
-Erythrocytes haemolysis
-Hypoplasia of red bone marrow
-Impairment of porphyrin synthesis
-Iron deficiency
+Vitamin B12 deficiency

#After resection of small intestine a patient complained of increased fatigability, infringement of taste,
brittleness of nails, quick decay of dental enamel, and appearance of breathlessness on physical exertion.
Which of the below given substances is the source of impairments in the patient’s organism?
-B12-folic deficiency anemia
-Hypocalcaemia
-Vitamin D deficiency
-Hyponatriaemia
+Iron deficiency anemia

#A female patient took analgin because of toothache. Dark urine, icteric sclera, weakness appeared in her
two days later. Which of the causes given below is the most possible one?
+Immune hemolytic anemia
-Hypoplastic anemia
-Thrombocytopoenia
-Thrombocytopathy
-Agranulocytosis

#A patient with acute pulpitis has an increased body temperature and a number of leukocytes up to 14.10 9/L,
differential count represents: basophils-0%; eosinophils-2%; megakaryocytes-0%; juvenile neutrophils-2%;
stab neutrophils-8%; segmented neutrophils-58%; lymphocytes-26%; monocytes-4%. How can you evaluate
such changes in blood?
+Neutrophilic lekocytosis with regenerative shift to the left
-Neutrophilic lekocytosis degenerative shift to the left
-Neutrophilic lekocytosis with hyperregenerative shift to the left
-Lymphocytosis
-Neutrophilic leukocytosis with shift to the right

#A patient aged 30 took sulfaethidole for infectious process of mucous membrane in the mouth locally (as
powder). Preparation has hemotoxic action and the treatment was complicated by the development of
agranulocytosis. At examination it was revealed:
-Increase of agranulocytes in the blood;
-Decrease of the number of granulocytes in the blood on the background of leukocytosis;
+Decrease of the number of granulocytes in the blood on the background of leucopoenia;
-Decrease of the number of neutrophylic granules with their simultaneous increase;
-Loss of their granules by granulocytes

#A women fell ill with purulent stomatitis. What index of complete blood count is characteristic for this
disease?
-Lymphocytosis
-Thrombocytosis
+Leukocytosis
-Anemia
-Monocytosis

#It is recommended to perform a clinical examination of blood in an empty stomach. What compounds of
peripheral blood may be changed after taking food?
-Decrease in number of platelets
+Elevation of number of leukocytes
-Elevation of number of erythrocytes
-Increase in content of plasma proteins
-Reduction of number of erythrocytes

#.Acute inflammatory disease of upper airways and eyes appears in 45-years-old woman at the period of
blossom of grass. Symptoms of this disease are hyperemia, edema, and mucous secretions. What kind of
leukocytosis is the most characteristic one for this disease?
-Neutrophilia
-Monocytosis
+Eosinophilia
-Basophilia
-Lymphocytosis
#Enlargement of liver and spleen, anemia, and myeloblasts in peripheral blood were revealed in a patient
with acute leukemia. What is the main trait, which allows to distinguish acute myelogenous leukemia from
chronic one?

-Pancytopoenia
-Blast cells in peripheral blood
-Anemia
+Leukemic gap
-Thrombocytopoenia

#Extraction of a tooth, in a patient with chronic lymphocytic leukemia, was complicated by prolonged
bleeding. What may cause the hemorrhagic syndrome in this patient?
-Anemia
-Lymphocytosis
-Eosinopoenia
-Neutropoenia
+Thrombocytopoenia

#In blood analysis of 37-years-old woman following data were revealed: content of hemoglobin is 60 g/L,
number of erythrocytes is 3.0x1012/L, and color index of 0.6; differential count of leukocytes without any
changes; number of platelets is 200x109/L; reticulocytes count of 0.3%; ESR of 18 mm/hour; microcytosis
and poikilocytosis of erythrocytes. Indicate the most probable type of anemia according to mechanisms of its
development
-Hypoplastic anemia
-Hemolytic anemia
-Acute post-hemorrhagic anemia
+Iron deficiency anemia
-B12-folate deficiency anemia

#In a patient aged 35 autoimmune hemolytic anemia developed. What index of blood serum is the most
increased in this case?
-Mesobilinogen
-Stercobilinogen
-Direct bilirubin
+Indirect bilirubin
-Protoporphirin

#A patient addressed a dentist with complaints of affections of mucous membrane of his mouth. During the
examination of the patient ulcerous stomatitis with necrosis in center was revealed at him in area of his
palate. In the history of disease of the patient recently endued pneumonia and taking of medicines
(sulfonilamides) were present. After administering treatment the doctor pointed the patient for blood
analysis. What pathology from below mentioned does doctor suppose?
+Immune agranulocytosis
-Iron deficiency anemia
-Thrombocytopoenia
-Infectious mononucleosis
-Infectious lymphocytosis
#A 40-years-old male patient, who was bitten by snake, was admitted to the hospital. What place does
hemolysis occur at in this case?
+In vessels
-In hepatic cells
-In the spleen
-In the bone marrow
-In renal parenchyma

#Myocardial infarction was diagnosed in 65-years-old man. Neutrophilic leukocytosis with left shift is
present in the blood of this patient. What factors underlie this phenomenon?
+Products of tissue decay
-Elevation of mass of muscular fibers
-Disorders of alveolar ventilation
-Decrease in glycogen content in the myocardium
-Increase of arterial pressure

#B12 deficiency anemia was diagnosed in a man aged 57 after examination of him. Treatment was
administered to this patient. Control blood test in this patient was performed in 3 years. What is the most
adequate criterion for enhancement of erythropoiesis?
+Increase in number of reticulocytes
-Increase in hemoglobin content
-Decrease in color index
-Normoblastic hematopoiesis
-Increase in number of leukocytes

#Patient with chronic hypoacidic gastritis has hypochromic anemia. In the blood smear of this patient
agranulocytes, micro- and anisocytosis, and poikilocytosis are revealed. Name this anemia
+Iron deficiency anemia
-Acute post-hemorrhagic anemia
-Thalassemia
-Sickle-cell anemia
-Pernitious anemia

#Increased concentration of leukopoietins in blood was found in a patient with acute appendicitis. What kind
of lekocytosis occurs in these conditions?
+Neutrophilic
-Basophilic
-Eosinophilic
-Lymphocytosis
-Monocytosis

#A patient has deficiency of cyancobalamine and folic acid that leads to disorder of leucopoiesis. What
changes takes place in these conditions?
+Leucopoenia
-Eosinophilia
-Basophilia
-Hemophilia
-Hyperemia

#A patient, who was exposed to ionizing radiation, has panmyelophtisis and secondary infections. What
changes occur in blood analysis in this case?
+Agranulocytosis
-Leucocytosis
-Eosinophilia
-Basophilia
-Hyperemia

#A patient with leukemia has general number of leukocytes of 120.0x10 9/L. What kind of leukemia does this
patient have?
+Leukemic
-Leucopenic
-Subleukemic
-Aleukemic
-Erythremia

#In a woman hypochromic anemia was diagnosed after delivery accompanied by marked bleeding. What
pathologic forms of erythrocytes are characteristic ones for this type of anemia?
+Anisocytes
-Target-like erythrocytes
-Sickle-cell anemia
-Spherocytes
-Microcytes

#Patient with chronic leukemia has sharply increased temperature, breathlessness, marked muscular
weakness at insignificant physical exertion, increased sweating, cough. What mechanism of leukemia
influence upon organism underlies complications in this patient?
+Immunodeficiency due to functional inability of leukocytes
-Internal bleeding because metastases into vessel wall
-Anemia
-Tumor progression
-Airway obstruction because of development of metastases

#When examining a blood in a patient, who endued bleeding three days ago, following data was revealed:
number of leukocytes is 12x109/L, basophils count is 0%, eosinophils count is 3%, myelocytes count 0%,
juvenile neutrophils count is 3%, stab neutrophils count is 12%, segmented neutrophils count is 62%,
lymphocytes count 16%, and monocytes count is 4%. What kind of changes of leukocyte differential count
takes place in this case?

+Neutrophilia with regenerative shift to the left


-Neutrophilia with degenerative shift to the left
-Neutrophilia with shift to the right
-Absolute lymphopoenia
-Absolute monocytopoenia
#A 40-years-old patient, who was admitted to the surgical department with diagnosis of phlegmona of thigh,
had high temperature, tachycardia, and breathlessness. On the blood test of this patient: number of
leukocytes is 25x109/L; eosinophils count is 1%, myelocytes count is 1%, juvenile neutrophils count is 15%,
band neutrophils count is 25%, segmented neutrophils count is 40%, lymphocytes count 14%, monocytes
count is 4%. What kind of shift in differential count is present in this case?
-Regenerative
+Hyperregenerative
-Degenerative
-Regenerative-degenerative
-Leukemoid

#What is the cause of intracellular hemolysis?


+Genetic infringements of erythrocytes
-Malaria
-Action of hemolytic poison
-Infection with hemolytic streptococcus
- Transfusion of incompatible blood

#What kind of disorders of total blood volume appears in case of absolute erythrocytosis?
+Polycytemic hypervolemia
-Oligocytemic hypervolemia
-Oligocytemic hypovolemia
-Simple hypervolemia
-Simple hypovolemia

#Beta-thalassemia was revealed in a patient, who came from Tunis. The disease was accompanied by
hemolysis and jaundice. The disease was diagnosed on the base of presence in blood:
+Target-like erythrocytes
-Grained erythrocytes
-Polychromatophil erythrocytes
-Normocytes
-Reticulocytes

#During the examination of adolescents that reside in mounting region increase in level of erythrocytes and
hemoglobin in peripheral blood was found out. What is the reason for indicated erythrocytosis?

+Exogenous hypoxia
-Diseases of lungs
-Congenital heart disease
-Condensation of blood due to large loss of water
-Vakes’s disease

#Content of hemoglobin and number of erythrocytes significantly decreases in a patient’s blood from time to
time. It was found out that such attacks appear after taking some horse beans. What kind of anemia takes
place in this case?
+Enzymopathy
-Membranopathy
-Iron deficiency anemia
-Hemoglobinopathy
-Acquired hemolytic anemia

Pathophysiology of hemostasis and antihemostasis

#A 32-years-old male patient has hemorrhage after injury of vessel that connected to formation of friable
thrombi. What coagulation factor deficiency has led to this disorder?
-II (prothrombin)
-III (thromboplastin)
-VII (proconvertin)
-XII (Hageman’s factor)
+XIII (fibrin stabilizing factor)

#A man, who has been to Arctic for a long time, has hemorrhage from gums, his teeth sway and pull out.
What is the initial mechanism in scurvy development?
-Infringement of elastin synthesis
-Affection of alveolar process
+Impairment of collagen synthesis
-Fragility of capillaries
-Insufficient tightness of round ligament of tooth

#Preliminary exfoliation of placenta and hemorrhagic shock develop in a pregnant woman during the
delivery. While taking woman’s blood for analysis it coagulates in syringe. The acute DIC syndrome is
diagnosed in this woman. What pathogenic mechanism appears to be initial in disorder of hemostasis in this
case?
-Activation of sympathetic-adrenal system
-Elevation of platelet aggregation
+Activation of tissue thromboplastin derived from destroyed tissues
-Cascade activation of plasma coagulation factors
-Oppression of aticoagulative system of blood

#It was found out before operating on that the patient has bleeding time increased up to 9 min. What blood
cells deficiency do these changes result from?
-Monocytes
+Platelets
-Erythrocytes
-Lymphocytes
-Leukocytes

#Breathlessness, acute pain in the chest, cyanosis, and enlargement of neck veins rapidly develop in a patient
suffered from thrombophlebitis. What is the most possible disorder of blood circulation in this patient?
+Pulmonary thromboembolism
-Cerebral thromboembolism
-Coronary thromboembolism
-Mesenterial thromboembolism
-Portal thromboembolism

#Numerous subcutaneous hemorrhages appear in a patient suffered from hepatic cirrhosis. What is a possible
reason for this?
-Excessive decay of vitamin C
-Disorder of vitamin K synthesis
-Hypocalemia
+Reduction in synthesis of factor II (prothrombin)
-Deficiency of factor III (thromboplastin) in blood plasma

#Hemorrhagic syndrome with disorders of the third stage of blood coagulation develops in a patient after she
was operated on uterus. What is the most probable mechanism of hemostasis disorder in this case?
-Qualitative abnormalities of fibrinogen
+Activation of fibrinolysis
-Deficit of fibrin stabilizing factor
-Decrease in ptothrombin synthesis
-Decrease in fibrinogen synthesis

#Hemorrhagic diathesis has developed in a patient with streptococcus infection. What is the reason for
hemorrhage development?
-Elevation of heparin content in the blood
-Vitamin A deficiency
-Increase in callicrein content in blood
+Enhanced fibrinolysis
-Vitamin C deficit

#Point hemorrhages appear on a forearm of a patient after putting a tourniquet on a patient’s arm. What
functions of blood cells do these changes connect to?
-Neutrophils
+Platelets
-Erythrocytes
-Basophils
-Macrophages

#A teeth was pulled out from the patient with chronic hepatitis. Bleeding, which developed after that, could
not be seized for 2 hours. Performed blood analysis establishes increase in content of several coagulation
factors. What kind of hemostasis is damaged in this case?
-Platelet-vascular
+Coagulation
-Vascular stasis
-Platelet reaction
-Vascular reaction
#Numerous hemorrhages and bruises were found out on the patient’s body. At examination of this patient:
his bleeding time by Duke is 25 min, his number of platelets is 25x10 9/L. What disease are these symptoms
characteristic for?
-Hemophilia B
-Von Willebrand’s disease
-Vitamin C deficiency
+Hereditary defect of platelet formation
-Hemophilia A

#A 65-years-old male patient suffered from atherosclerosis was admitted to the surgical department because
of diffuse purulent peritonitis. When the patient was operated on thrombosis of mesenterial vessels was
diagnosed in him. What is the most possible reason for peritonitis development?
-Hemorrhagic infarction
-Agiospastic ischemia
+Ischemic infarction
-Stasis
-Compressive ischemia

#Hemorrhagic syndrome connected to disorders of the third phase of coagulation developed in a patient after
hi was operated on pancreas. What is the possible mechanism of development of hemostasis disorder?

-Elevation in content of heparin in patient’s blood


-Reduction in fibrinogen synthesis
+Activation of fibrinolysis
-Deficit of fibrin stabilizing factor
-Reduction in prothrombin synthesis

#Sharp pain and edema of tissues develop in a patient with thrombosis of veins of his right calf. What is the
possible consequence of thrombosis?
-Hypertrophy of tissues
+Circulatory hypoxia
-Enhancement of drainage of tissues
-Intensification of metabolism
-Enhancement of functional activity of tissues

#Thrombosis of coronary arteries occurs in a patient with atherosclerosis of cordial vessels. What is the
reason for formation of thrombi?
+Heparin deficiency
-Thrombocytopenia
-Acceleration of blood flow
-Activation of fibrinolysis
-Vitamin K deficiency

#An elderly patient was admitted to the hospital with thrombosis of veins of his calf. What is the reason for
thrombosis development?
-Decrease in prothrombin concentration in blood
-Increase in heparin concentration in blood
+Injury of vessel wall
-Slowing-down of blood flow
-Activation of plasmin

#When examining a patient increased blood coagulation (thrombophilia) was found out in him. What
reasons favor such state development?
-Intensification of prostacyclin synthesis in vessel wall
-Low concentration of thrombin in blood
-High concentration of heparin in blood
+Deficit of inhibitors of proteolytic enzymes
-High concentration of adrenalin in blood

#Thrombocytopenia was found out in blood of a patient with Werlgoff’s disease. Why the pathology of
platelets leads to hemorrhage?
+Decrease in thromboplastin formation
-Decrease in concentration of prothrombin in blood
-Decrease in concentration of heparin in blood
-Activation of fibrinolytic system
-Decrease in concentration of fibrinogen in blood

#Hemophilia A was diagnosed in a patient with hemorrhagic syndrome. What is the reason for this disease
development?
-Absence of Stewart’s factor (factor X)
-Absence of Christmas’s factor (factor IX)
+Absence of factor VIII
-Absence of Rosenthal’s factor (factor XI)
-Low concentration of Hageman’s factor (factor XII) in blood

#A patient was admitted to the hospital with abundant hemorrhoid bleeding. This patient has been suffering
from hepatic cirrhosis for a long time. What is the reason for hemorrhage development under hepatic
cirrhosis?
-Activation of fibrinolysis
-Plasmin deficiency
+Prothrombin deficiency
-Low concentration of thrombostenin in blood
-Excess of heparin

#Shonlein-Henoch disease was diagnosed in a patient. What changes in blood cells number are characteristic
for this disease?
+Eosinophilia
-Thrombocytopenia
-Erythropoenia
-Polycytemia
-Eosinopoenia
#Patient has hemorrhage from gums, subcutaneous hemorrhages, and frequent nasal bleedings. Significant
Thrombocytopenia was revealed in blood test of this patient. What is the reason for bleeding development in
case of thrombocytopenia?
-Thrombin deficiency
+Reduction of thromboplastin formation
-Excessive heparin formation
-Activation of fibrinolysis
-Excess of prostacyclins

#While performing intravenous injections trauma of venous vessels leads to thrombosis. What favors
thrombosis under injury of vessel wall?
-Activation of plasmin
-Activation of fibrinolysis
-Activation of phospholipase D
-Enhanced production of prostacyclin by endotheloicytes
+Accumulation of thromboxan A2

#Changes of some indices of blood were revealed at examination of a patient suffered from hemophilia.
What of enumerated indices corresponds to this condition?
-Thrombocytopenia
-Bleeding time by Duke takes longer
-Eosinophilia
+Time of coagulation takes longer
-Afibrinogenemia

#A boy has congenital disorder of hemostasis: he has prolonged hemorrhage even in case of insignificant
injuries, subcutaneous bruises, and bleedings in joint cavities, which restrict the movement activity. Patient’s
blood does not coagulate for a long time if it is taken out from the organism and it does not contain
coagulation factor VIII. Blood cells count including number of platelets is within norm. What underlies the
congenital disease of this boy?
+Hereditary gene defect linked with X chromosome
-Toxicosis of pregnancy in boy’s mother
-Intrauterine infection
-Intrauterine intoxication
-Intrauterine immune conflict

#Hemophilia B was diagnosed in a child who has hemorrhagic syndrome. This type of hemophilia results
from absence of:
+Coagulation factor IX (Christmas’s factor)
-Coagulation factor II (prothrombin)
-Coagulation factor VIII (antihemophilic globulin)
-Coagulation factor XI (thromboplastin)
-Coagulation factor XII (Hageman’s factor)

#It is known that thrombus undergo same changes after its formation. What sort of finishing of thrombus
formation is the most dangerous for a patient?
-Aseptic lysis
+Septic lysis
-Organization without recanalization
-Organization with recanalization
-Calsification of thrombus

#Such appearances as petechiae and ecchymoses develop in a boy aged 7, who often fall ill with acute
respiratory diseases. Pathology of internal organs is absent in this patient. What pathology is present in this
case?
-Hypoplastic anemia
+Thrombocytopenia
-Acute leukemia
-Hemophilia
-Chronic leukemia

#Course of atherosclerosis in 70-years-old patient is complicated by thrombosis of vessels of lower


extremities and development of gangrene of left foot toes. The beginning of thrombosis is connected to:
-Activation of prothrombinase
+Adhesion, aggregation and agglutination of platelets
-Conversion of prothrombin to thrombin
-Conversion of fibrinogen to fibrin
-Decreased heparin synthesis

#Antihemophilic globulin A (factor VIII) is absent in the blood plasma of a boy with significant hemorrhagic
syndrome. What phase of hemostasis is infringed primarily in this boy?
-Retraction of blood clot
-Conversion of fibrinogen to fibrin
-Conversion of prothrombin to thrombin
-Extrinsic pathway of prothrombinase (thrombokinase) activation
+Intrinsic pathway of prothrombinase (thrombokinase) activation

#Which of essential factors of thrombus formation the most often serves as the reason for thrombosis?
-Decrease in activity of anticoagulative system
-Activation of systems of blood sedimentation
+Injury of vessel wall
-Acceleration of blood flow
-Turbulence of blood

#A patient complains of hemorrhage from gums. What vitamin deficiency may lead to this phenomenon?
-B1
-D
-B2
-A
+K

#Prolonged bleeding complicated pulling out of a tooth in a patient, who suffered from chronic leukemia.
What may serve a reason for hemorrhagic syndrome in this patient?
+Thrombocytopenia
-Anemia
-Erythropenia
-Lymphocytosis
-Neutropenia

#Bleeding developed in a 15-years-old patient after her teeth was pulled out. It was found out that after
endued hepatitis A this patient has point hemorrhages (petechiae and ecchymoses) and bruises on her skin,
and she had three accidents of nasal bleeding. Before this she did not have hemorrhages even after pulling
out of her tooth. It was established under examination that the bleeding time is increased in this patient. At
the patient’s blood test hypochromic anemia, decrease in number of platelets, and markedly reduced
adhesion of platelets are present. Prothrombin and thrombin time are normal in this patient. Her differential
blood count is normal too. What is the most possible diagnosis?
-Hereditary thrombocytopathy
-Acquired thrombocytopenia
-Hereditary thrombocytopenia
-Hemophilia
+Acquired thrombocytopathy

#A patient complains of frequent hemorrhages from gums. When carrying out blood test in this patient
deficiency of factor of hemocoagulation (prothrombin) was revealed. What phase of blood coagulation is
primarily impaired in the patient?
+Formation of thrombin
-Fibrinolysis
-Formation of fibrin
-Retraction of blood clot
-Formation of prothrombinase

#A 12-years-old patient was admitted to the hospital with hemarthrosis of knee joint (hemorrhage into joint
cavity). This patient has been suffering from hemorrhages since early childhood. What disease does this boy
suffer from?
+Hemophilia
-Hemorrhagic vasculitis
-Iron deficiency anemia
-Vitamin B12 deficiency anemia
-Thrombocytic purpura

#A 30-years-old female patient, who is suffering from megrim, often takes analgin. Hemorrhages on skin
and frequent nasal bleedings appear in her at the recent time. At her blood analysis: number of platelets is
30x109/L; bleeding time is increased. What do these changes result from?
+Autoimmune thrombocytopenia
-Hemorrhagic vasculitis
-Hemolytic anemia
-Angiohemophilia
-Thrombocytopathy

#What kind of pathologic processes underlie formation of thrombus?


+Astringent metamorphose of thrombocytes
-Increase of osmotic pressure
-Decrease in number of thrombocytes
-Increase of oncotic pressure
-Decrease in number of erythrocytes

#A worker at pharmaceutical plant addresses the doctor with complaints of general malaise, significant
hemorrhages from gums, nasal bleedings, and numerous subcutaneous hemorrhages. At blood analysis of
this patient following was revealed: number of erythrocytes is 2.2x10 12, content of hemoglobin is 48 g/L,
presence of neutropenia with relative lymphocytosis, and number of platelets is 35x10 9/L. What is the
possible pathogenesis of thrombocytopenia in this patient?
+Decrease of platelet production
-Enhanced platelet destruction
-Increase platelet utilization
-Redistribution of platelets
-Increased loss of platelets

#Intravenous injection of mercury chloride to experimental animal evokes development of thrombosis in it.
What is the basic pathogenic factor in development of this pathologic process?
+Injury of vessel wall
-Activation of coagulation system
-Activation of platelets adhesion
-Slowing-down of blood flow
-Decrease in anticoagulants activity

Pathophysiology of cardiovascular system

#Thrombosis of coronary artery caused the development of myocardial infarction. What mechanism of the
impairment will be dominating in this disease?
-Electrolyte-osmotic;
-Asidotic;
-Protein;
-Lipid;
+Calcic

#Acute cardiac insufficiency appeared in a patient with arterial hypertension due to hypertensive crisis. What
mechanism of cardiac insufficiency is the main in this case?
+Overload of heart by resistance;
-Absolute coronary insufficiency;
-Relative incompetence;
-Overload of heart by rush of blood;
-Myocardial impairment

#A patient aged 59 was hospitalised at cardiological department in a severe state with the diagnosis of acute
myocardial infarction of the posterior wall of the left ventricle and septum, and starting pulmonary edema.
What is primary mechanism which causes the development of pulmonary oedema in the patient?

-Pulmonary arterial hypertension;


+Left ventricular failure;
-Pulmonary venous hypertension;
-Hypoxemia;
-Decrease of alveolocapillary diffusion of oxygen

#A female patient aged 18, complains of general weakness, quick fatigability, depressed mood. She has
asthenic type of constitution. Pulse 68 per min., AP-90/60mm Hg. Primary neurociculatory arterial
hypertension was diagnosed. What is the main factor of decreasing of arterial pressure in a patient?
-Decrease of minute volume of the blood;
-Decrease of cardiac output;
+Decrease of the tension of resistant vessels;
-Hypovolemia;
-Deposition of the blood in the vines of systemic circulation

#A patient has stable and marked increase of arterial pressure, increased extracellular fluid volume,
increased content of Na+ and decrease of K+ in the blood, positive effect of saluretic treatment. What is the
mechanism of development of hypertension in the patient?
+Mineralocorticoid
-Renin-angiotensine
-Renovascular
-Reflexogenic
-Cento-ischemic

#While climbing upstairs on the 5th floor a patient has got an increased arterial pressure. The cause is the
increase of:
+Minute volume of the blood
-The number of functioning capillaries
-Content of ions in blood plasma
-Viscosity of the blood
-Circulating volume of the blood

#On the diagnosis of myocardial infarction the main role belongs to enzymodiagnosis. The definition of
content level in the blood of what enzyme is the most important during the first 2-4 hours after infarction?
-Aldolase
-Lipoprotein lipase
-Alanine aminotranspherase
+Creatin phosphokinase
-Acetylcholinesterase

#On analysis ECG it was determined: sinus rhythm, correct, interval RR is 0.58 sec, location and duration of
other intervals, waves and segments are not changed. Call the type of arrhythmia:
+Sinus tachycardia
-Sinus bradycardia
-Indioventricular rhythm
-Sinus arrhythmia
-Ciliary arrhythmia
#One of the most dangerous moments in pathogenesis of myocardial necrosis is the further increase of the
zones of necrosis, dystrophy and ischemia. The important role in this belongs to the increase of the use of
the oxygen by myocardium. What substance contributes to this process?
-Chlorine ion
-Cholesterol
+Catecholamine
-Acetylcholine
-Adenosine

#The functioning of certain structures is stopped on the isolated heart by means of cooling. What structure is
cooled if due to this the contractions stopped at first, but then they began with a rate 2 times slower than
initial one?
-Sinoatrial node
+Purkinje’s fibres
-Limbs of His’ bundle
-Atrioventricular node
-His’ bundle

#A patient with chronic glomerulonephritis has edema, BP is 210/100 mmHg; the rate of heartbeat is 85 per
minute; the borders of the heart are dilated. What is the leading mechanism in the development of arterial
hypertension?
-Increase of the activity of sympathetic adrenal system
-Hyperfunction of the heart
+Activation of renin-angiotensin-aldosterone system
-Increase of circulating volume of the blood
-Increase of vasopressin output

#The patient’s ECG shows that interval RR=1.5 sec, heart rate - 40 per min. What is the pacemaker of the
heart?
-Left limb of His’ bundle
-Sinus node
+His’ bundle
-Right limb of the His bundle
-Atrioventricular node

#Pulmonary edema developed in a patient with hypertonic crisis. What is the main factor in the pathogenesis
of his state?
-Increase of arterial pressure
-Permeability increase of the vessels of pulmonary circulation
-Increase of hydrostatic pressure in the capillary of the lungs
+Resistibility increase of the lung vessels
-Decrease of oncotic pressure of blood plasma

#During the examination of blood for activity of AsAT and AlAT in the patient who complained of pain in
the chest and in upper part of the abdomen, the following results were received: activity of AsAT 2 times
higher than AlAT activity. What disease does the patient have?
-Acute infectious hepatitis
+Acute pancreatitis
-Myocardial infarction
-Chronic hepatitis
-Cirrhosis of the liver

#Redistribution of organ blood supply took place in a young man, aged 20 during the load. What organ did
the blood flow increase in most of all?
-Brain
-Kidneys
-Liver
-Skeletal muscles
+Heart

#Clinical signs of developing pulmonary edema appeared in a patient with cardiac insufficiency of left
ventricular type. Which of the pointed pathogenic mechanism is the primary in such pathology?
+Hydrodynamic
-Congestive
-Colloid-osmotic
-Lymphogenous
-Membranogenous

#A patient has cyanosis, increase of the liver, oedema of the lower extremities due to the right ventricular
insufficiency. What is the cause of the development of right ventricular failure?
+Cardiogenic cirrhosis of the liver
-Functional shunting in lungs
-Hypercatecholaminemia
-Increase of venous pressure
-Hypotension of pulmonary circulation

#A woman, aged 25, complains of constant pain in the heart area, breathlessness on movement, and general
malaise. She has pale and cold skin, acrocyanosis. Her pulse is 96/min and her BP is 105/70 mmHg. Heart
border in her shifted 2 cm left. The first sound is weakened over the apex of heart; there is systolic murmur
over the apex. Diagnosis is insufficiency of the mitral valve of the heart. What is the cause of the blood
circulation failure?
+Myocardial overload by the increased blood volume
-Myocardial overload by the increased of resistance of blood outflow
-Myocardial failure
-Volume decreased of circulating blood
-Volume increased of vascular bed

#A patient with acute myocardial infarction was being given 1500ml of different solutions intravenously
during 8 hours, oxygen intranasally. Death occurs due to pulmonary edema. What was the cause of the
pulmonary edema?
+Overload of the left ventricle by the volume+
-Decrease of oncotic pressure due to hemodilution
-Allergic reaction
-Neurogenic reaction
-Oxygen inhalation

#Functional hypertrophy of the left ventricle of the heart developed in a sportsman, aged 20, due to constant
physical load. What morphofunctional process do these changes result from?
+Increase of cell size and number of contractile organelles
-Increase of fibroblast number
-Increase of the number of conductive cardiomyocytes
-Increase of the amount of connective tissue
-Increase of the amount of fat tissue

#A patient with renal disease accompanied by parenchyma ischemia has a high arterial pressure. What
leading factor is the cause of the increase of AP in this patient?
+Excess of angiotensin II
-Excess of antidiuretic hormone
-Increase of heart output
-Increase of sympathetic nervous system tonus
-Hypercatecholaminemia

#During the operation reflex increased of vagus nerve influence on the heart happened. What may occur in
this case?
+Cardiac arrest
-Increase of atrioventricular node conduction
-Increase of myocardium conduction
-Intensification of myocardium contractions
-Increase of heart rate

#Decrease of R-R interval was revealed on ECG of a man. What changes in the heart work are observed in
this case?
+Increase of heart rate
-Decrease of heart rate
-Increase of force of contractions
-Decrease of force contraction
-Decrease of force and rate of contractions

#Large amount of isoenzymes of creatine kinase of MV-form was revealed in the blood of the patient with
destructive changes in the muscular tissue. What is the most possible diagnosis?
+Myocardial infarction
-Muscular atrophy
-Muscular dystrophy
-Polymyositis
-Myopathy

#Patient’s attack of tachycardia was stopped by pressing on the eyeballs (Danini-Ashner reflex). In the
decrease of the heart rate there is intensification of the influence on the sinoatrial node of:
+Vagus nerves
-Sympathetic nerves
-Autonomic nervous system
-Sympathoadrenal system
-Catecholamines

#Considerable increase of PQ interval was found out on ECG. It means that conduction of stimulation is
delayed by:
+AV node
-Atria
-His’ bundle
-Purkinje’s fibres
-Ventricles

#Sharp marked pains in the substernal area that radiate to the left arm cannot be controled by nitro-glycerine
for 30 minutes. What changes developed in the patient’s hearts?
+Myocardial ischemia
-Pathological myocardial hypertrophy
-Sharp increase of coronary blood flow
-Mitral incompetence
-Inflammation of pericardium

#The activity of what enzymes is it necessary to determine in pathology of cardiac muscle with diagnostic
and prognostic aim?
-Arginase, peptidase, phosphatase
-Decarboxylase, decanidase, lactate dehydrogenase
+Creatin kinase, transaminase, lactate dehydrogenase
-Lysozyme, citrate synthatase, succinate dehydrogenase
-Neuroaminase, aldolase, hexakinase

#A severe stress was caused in an experimental animal. Necrotic injuries of myocardium developed in this
stage. What is the main cause in the pathogenesis of these injuries?
-Increase of calcium content in cardiomyocytes
-Decrease of adenosine triphosphoric acid synthesis in mitochondria
-Changes in the work of Na+-K+ pump
+Insufficiency of coronary circulation
-Decrease of adenosine triphosphoric acid activity of myosin

#A patient male has had a chronic disease of kidneys for 12 years. AP - 200/130mmHg. Pulse –75 beats per
min. The main factor that causes the increase of pressure in this case is the increase of:
-Minute volume of the heart
-Heart rate
-Circulating volume of the blood
+Systemic peripheral resistance
-Venous recurrence

#During the attack of heartbeat a patient with thyrotoxicosis has an irregular pulse of different filling, pulse
deficiency is observed. Waves P are absent; small in amplitude, disorderly undulations (P waves), and
irregular ventricular complexes of ordinary configuration are noted. What kind of rhythm impairment is
observed in a patient?
-Sinus tachycardia
-Sinoatrial block
+Ciliary arrhythmia
-Paroxysmal tachycardia
-Ventricular extrasystole

#At examination of a patient strengthening of a second pulmonic sound, hypertrophy of the right ventricle
wall are determined. What changes of hemodynamic take place in pulmonary circulation?
-Hyperperfusion of the lungs
-Hypoperfusion of the lungs
-Manifestation of Hering-Breuer reflex
-Development of broncho-alveolar vascular anastomosis
+Spasm of resistant vessels in the lungs

#A patient with mitral failure has an enlargement of the liver, edema of lower limbs. What is the leading
mechanism of the development of cardiac edema?
-Increase of venous recurrence
-Increase of tissue drainage
+Participation of renin-angiotensin-aldosteron system
-Decrease of oncotic pressure
-Decrease of transudation

#At examination a patient’s arterial pressure is 190/100 hg. What factors leads to increase of arterial
pressure?
+Spasm of resistance vessels
-Increase of venous recurrence
-Toxygenic dilation of cardiac muscle
-Aler-Lilestrand reflex
-Kitaevs reflex

#A patient with myocardial infarction has a mark paleness of skin, oliguria, AP 100/90 mm Hg, and pulse
100 beats/min. What compensative mechanism maintains relative high level of AP?
-Hypokalemia
-Hypoperfusion of the lungs
+Centralization of blood circulation
-Increase of the level of vasodilators in blood
-Secondary aldosteronism

#Marked frequency of the patients pulse was determined during the examination, what is sinus tachycardia
due to?
-Hypothyrosis
-Hypokalemia
-Tonus increase of vagus nerve
+Increase of speed of slow diastolic depolarisation
-Excess of acetylcholine

#Considerably slow pulse was determined in a patient at examination. What is sinus bradycardia due to?
+Decrease of speed of slow diastolic depolarization
-Hypercatecholaminemia
-Decrease of circulating blood volume
-Hyperkalemia
-Haemic hypoxia

#In cardiac pathology homeometric mechanism of compensation in the work of the left ventricle takes place
in:
-Stenosis of atrioventricular foramen
-Mitral incompetence
-Aortic incompetence
-Hypertension of pulmonary circulation
+Hypertension of systemic circulation

#In cardiac pathology heterometric mechanism of compensation connected with overloading of left ventricle
volume takes place in:
-Stenosis of atrioventricular foramen
-Stenosis of aortic osteum
+Aortic incompetence
-Hyper tension of pulmonary circulation
-Hypertension of systemic circulation

#Sharp increase of AsAT activity was determined in the blood serum of a patient 12 hrs later after an acute
attack of pain in the substernal area. Which of the pathogenesis is the most possible?
-Collagenosis
-Diabetes mellitus
+Myocardial infarction
-Virus hepatitis
-Diabetes insipidus

#Pressing pain in the heart area with irradiation to the left arm, neck and under the left shoulder blade
suddenly appeared in a male patient aged 45, after considerable psycho-emotional exertion. His face became
pale and covered with cold perspiration. Nitro-glycerin relieved the attack of pain. What process in the
patient?
+Angina pectoris
-Perforation of stomach ulcer
-Psychogenetic shock
-Myocardial infarction
-Insult

#In recreation of the arterial hypertension in a dog a thickness in the left ventricle wall increased 1.7 times in
a month, but the circulating blood volume was not changed in comparison with the initial data. What stage
of myocardial hypertrophy is observed in the animal?
-Initial
-Repair
-Complete hypertrophy
+Decompensation
-Progressive cardiosclerosis

#The rate of spread of pulse wave turned out to be considerably higher in a man aged 70, than in a man aged
25. The cause of this is decrease of:
-Arterial pressure
-Cardiac output
+Elasticity of vascular wall
-Rate of cardiac contractions
-Circulation rate

#On recording ECG of a patient with hyperfunction of the thyroid gland increase of rate of cardiac
contractions was registered. Shortening of what ECG element indicates this?
+Interval R-R
-Interval P-Q
-Interval P-T
-Segment P-Q
-Complex QRS

#A patient excretes water from the organism less than he uses it for 24 hours. What disease may lead to this
state?
+Cardiac insufficiency
-Pancreatitis
-Cystitis
-Hepatitis
-Infectious diseases

#A man has got an electro trauma. Current went through the cardiac muscle. What dangerous impairment in
the work of the heart demanding urgent measures may appear in this situation?
-Bradycardia
-Extrasystole
-Atrial fibrillation
+Ventricular fibrillation
-Tachycardia

#A patient who underwent myocardial infarction a month and a half ago had Dressler’s syndrome with
typical triad: pericarditis, pleurisy, and pneumonia. The cause of its development is:
+Sensibilization of the organism by myocardial antigens
-Decrease of resistance to infectious agents
-Activation of saprophytic micro flora
-Intoxication of the organism by necrotic products
-Injection of myocardial enzymes in the blood

#A patient with rheumatism had incompetence of the left atrioventricular foramen and decompensation of
cardiac activity. The characteristic hemodynamic index of this state is:
+Decrease of cardiac output
-Slowing down of blood flow
-Decrease of arterial pressure
-Increase of venous pressure
-Widening of microcirculatory bed

#A patient with rheumatic myocarditis began to feel intermissions in the work of heart. By means of
auscultation of the heart, feeling the pulse and ECG investigation it was determined that this was connected
with the appearance of ventricular extra systoles the characteristic feature of which is compensatory pause.
Its appearance is due to:
+Refractivity of ventricular myocardium to the next impulse
-Retention of stimulation in atrioventricular node
-Retrograde conduction of stimulation to atria
-Inhibition of function of sinoatrial node
-Impairment of contraction of ventricular myocardium

#Considerable increase of myocardial mass of left ventricle was determined in a patient with hypertension. It
was due to:
+Increase of cardiomyocyte volume
-Increase in amount of cardiomyocytes
-Enlargement of connective tissue
-Retention of water in myocardium
-Fat infiltration of myocardium

#A patient with aortic atherosclerosis has left ventricular hypertrophy as a compensatory phenomenon.
Compensatory role of hypertrophy comes to:
+Normalization of load on each cardiomyocyte
-Improvement of delivery of oxygen to myocardium
-Activation of synthesis of macroergs in myocardium
-Economical use of energy by cardiomyocytes
-Increase of stroke volume of the blood

#A patient who is treated for myocardial infarction at the in-patient department has lost suddenly his
consciousness several times a day. During the attack pulse is absent, heart sounds are not heard, face is
cyanotic, and convulsions appear, arterial pressure is not determined. The diagnosis is Morgagni-Adams-
Stokes syndrome. It appears due to:
+Development of full atrioventricular blockade
-Weakness of sinoatrial node
-Attack of ventricular paroxysmal tachycardia
-Impairment of excitability of ventricular myocardium
-Cardiosclerotic changes in the heart

#Electrocardiography examination of a patient with hypertension showed such results: right sinus rhythm,
rate of cardiac constrictions is 92 per min; duration of PQ - 0.2 sec, QRS without change, a patient has
disturbance of:
-Refractivity
-Conduction
+Automatism
-Stimulation
-Contractility
#By electrocardiogram investigation the following data were determined in a patient who had suffered the
grippe; the rate of heart is 140 beats per min, sinus rhythm, the range of R-R is not more than 0.15 c;
duration of PQ-0.2 c; QRS is not changed. These indicate of the development of:
-Sinus tachyarrhythmia
+Sinus tachycardia
-Nonparoxysmal tachycardia
-Paroxysmal tachycardia
-Ventricular fibrillation

#The signs of heart failure appeared in a man aged 56, during carrying out hard work, feelings of air
shortage, heart beating, and general weakness. Objectively heart borders are dilated, the heart rate is 92 beats
per min and arterial pressure is 180/110 mm hg. These signs are due to:
+Increase of peripheral resistance
-General hypoxia of an organism
-Insufficiency of coronary circulation
-Increase of diastolic filling
-Neurotrophic disturbances

#A patient aged 47 with mitral incompetence has the symptoms of cardiac insufficiency: breathlessness,
cyanosis, oedema of lower limbs. Objectively: the borders of the heart are dilated, the heart rate is 104 beats
per min, and arterial pressure is 125/85 mm hg. These symptoms are due to:
-Increase of peripheral resistance
-Autoallergic changes of myocardium
+Increase of diastolic filling
-Insufficiency of coronary circulation
-Neurotrophic disturbances

#Dilation of the heart, AP-155/100 mm hg, the heart rate 95 beats per min, was revealed in a patient aged 63
with hypertension during the examination. The most effective mechanism, which will contribute to the
normalization of AP, is:
-Inhibition of aldosteron synthesis
-Blocking of angiotensin synthesis
+Inhibition of catecholamine action
-Blocking of vasopressin action
-Administration of salt low diet

#After suffered rheumatism a patient had aortic stenosis. Point out what mechanism of compensation takes
place in the left ventricle:
+Homeometric
-Heterometric
-Systolic
-Diastolic
-Coronary

#After suffered rheumatism a patient had aortic incompetence. Indicate what mechanism of compensation
takes place in the left ventricle in this case:
-Homeometric
+Heterometric
-Systolic
-Diastolic
-Coronary

#Acute myocardial infarction of a patient includes three areas: zone of necrosis, dystrophy and ischemia.
The changes of what ECG wave characterises the place of necrosis in this case?
-R
-S
+Q
-T
-P

#A patient with acute myocardial infarction, which was caused by thrombosis of coronary arteries, was
treated with fibrinolytc preparation urkinase that led to the development of reperfusion syndrome. Which of
the mentioned syndromes is associated with reperfusion?
-Dressler’s
-Chediak-Higashi
-Morgagni-Adams-Stokes
+No-reflow
-Wolff-Parkinson-White

#Which of the factors is the most frequent cause of formation of acquired valvular heart disease?
-Septic endocarditis
-Syphilis
-Atherosclerosis
-Mechanical factor
+Rheumatism

#Acute pain in the heart area, marked skin paleness and loss of consciousness developed in a patient after
short intensive physical excretion. What heart failure may be suspected in this patient?
-Mitral incompetence
+Stenosis of aortic osteum
-Stenosis of mitral orifice
-Aortic incompetence
-Stenosis of the right atrioventricular opening

#A patient aged 40, complains of increased fatigue, appearance of pain in the heart area on physical exertion.
Objectively: cyanotic skin (especially on the fingers of the hands and legs, lobule of the ear), pulse rate 96
beats/min, AP-110/85 mm of hg. There are oedemas in the lower and middle third of the cruses. The edge of
the liver projects 3 cm from under the costal arch. Which form of circulatory insufficiency may be suspected
in this patient?
-Acute left ventricular failure
-Acute vascular insufficiency
-Chronic left ventricular failure
+Chronic right ventricular failure
-Acute right ventricular failure

#A patient complains of breathlessness that appears even on slight physical exertion, cough with sputum,
which recently has obtained bloody character. Objectively: the skin of the face and mucous membrane of the
lips particularly are cyanotic. Which impairment of the heart work may be suspected in this patient?
+Stenosis of mitral orifice
-Mitral incompetence
-Stenosis of aortic osteum
-Aortic incompetence
-Stenosis of the right atrioventricular opening

#A patient has constant high arterial pressure - 160/110 mm of hg. Periodically it may rise up to 220-
240/120-130 mm of hg. What type of hypertension has this patient?
-Psycho emotional
-Cerebral
-Renal
+Adrenal
-Essential

#A patient, female, aged 48 complains of breathlessness on slight physical exertion, pains of pressing
character in substernal area, appearance of syncope on exertion. According to a case history the patient
suffered rheumatism. Heart failure was diagnosed - stenosis of aortic osteum, myocardial hypertrophy. What
is the main mechanism of the development of myocardial hypertrophy?
-Hypertrophy of myocardiocytes
+Diffuse oedema of interstice
-Synthesis increase of contractile proteins in myocardium
-Hyperplasia of intermediate tissue of myocardium
-Accumulation of water and electrolytes in myocardiocytes

#A patient 45 complains of breathlessness on slight physical exertion, oedema of the legs, frequent quinsies
are pointed out in case history, she is being ill for 2 years. Rheumocarditis, combined mitral incompetence,
insufficiency of blood circulation were diagnosed. What is hemodynamic mechanism of decompensation?
-Decrease of circulating blood volume
-Decrease of venous pressure
-Increase of arterial pressure
+Decrease of minute volume of the heart
-Tachycardia

#A patient with ischemic heart disease had a sudden severe attack of angina pectoris: the face is pale, cold,
damp skin, AP-70/50 mm Hg, extrasystolia. Myocardial infarction and cardiogenic shock were diagnosed.
Name the leading link of pathogenesis?
-Hypotension
-Pain syndrome
-Toxaemia
-Extrasystolia
+Decrease of minute blood volume
#Mitral incompetence without impairment of blood circulation was revealed in a patient aged 25. What
mechanism provided the state of the heart compensation?
+Heterometric mechanism
-Homeometric mechanism
-Inotropic action of catecholamines
-Increase of the heart mass
-Strengthening of protein synthesis

#Aortic stenosis was revealed in a girl aged 15, but without impairments of blood circulation. What the
mechanism provided the condition of heart compensation?
+Homeometric mechanism
-Inotropic action of catecholamines
-Heterometric mechanism
-Decrease of heart mass
-Strengthening of protein synthesis

#ECG of a child aged 5 shows the impairment of action of cardiac rhythm. On holding breathing the cardiac
rhythm becomes normal. What kind of impairment was found out on ECG?
-Sinus extrasystole
-Ciliary arrhythmia
+Respiratory arrhythmia
-Atrial extrasystole
-Transversal heart block

#Strong palpitation, pain in the heart, sharp weakness, increase of AP, irregular pulse with deficiency
developed suddenly in a man aged 50. ECG showed absence of wave P and different R-R intervals. What
impairment of cardiac rhythm has a patient?
-Transversal heart block
-Paroxysmal tachycardia
-Respiratory arrhythmia
+Ciliary arrhythmia
-Sinus extrasystole

74. ECG show that wave P overlaps the wave T what is observed in case of atrial obstruction. At what rate
of the heart contractions will it occur?
-150 beats per minute
-160 beats per minute
+170 beats per minute
-140 beats per minute
-130 beats per minute

#During the experiment a white rat was given intraperitoneal injection of 0.1% adrenalin solution at 1
mg/100g of body mass. 30 minutes later respiration become hurried and shallow, then sharp and convulsive,
foamy fluid appeared from the nasal cavity, there was acrocyanosis. The animal died when acute pulmonary
edema developed. What pathogenetic mechanism is the leading in this case?
-Toxic
-Membranogenous
-Lymphogenous
+Hydrodynamic
-Colloid-osmotic

#Patient V., aged 67 suffers from atherosclerosis of heart vessels and brain vessels. Hyperlipidemia was
revealed on examination. What class of lipoproteins of blood serum is of greatest importance in the
pathogenesis of atherosclerosis?
+Lipoproteins of low density
-Chylomicrones
-Beta lipoproteins
-Lipoproteins of high density
-Complex of fat acids with albumins

#The planned investigation of cardiohemodinamics readings of patient M. with chronic cardiac insufficiency
was carried out. Which of the named readings is the main sign of the development of cardiac
decompensation?
-Development of tachycardia
+Decrease of stroke volume
-Tonogenic dilatation
-Increase of peripheral vascular resistance
-Increase of central venous pressure

#While analyzing the ECG of the patient with myocardial infarction attention was paid to absence of wave P,
presence of wavy isoelectric line with plenty of small waves, frequent and irregular location of QRS
complex. What impairment of rhythm does the described picture of ECG correspond to?
-Ventricular fibrillation
-Idioventricular fibrillation
+Atrial fibrillation
-Paroxysmal tachycardia
-Ventricular extrasystole

#Periodical appearance of ventricular extrasystole was revealed during ECG investigation of patient P. It
was determined that wave P was absent before extrasystole. What is the cause of its disappearance?
-Appearance if refractory period in atria
-Impulse block in sinus node
-Appearance of refractory period in ventricles
-Block of impulse conduction round the atria
+Impossibility of retrograde conduction through A-V node

#During ultra-sound investigation dilation of heart cavities was revealed in patient S. who suffered from
hypertension. Which of the below mentioned signs are evidence of development of tonogenic dilation?
+Dilation of heart cavities with increase if stroke volume
-Dilation of cavities without change if stroke volume
-Dilation of cavities with change of stroke volume
-Uniform dilations of heart borders
-Irregular dilations of heart borders
#Quick fatigue, breathlessness on physical exertion, feeling of “sink” and momentary cardiac arrest appeared
in a patient D, aged 13, 2 weeks later after secondary suffered tonsillitis. ECG showed decrease of voltage of
waves periodical prolapse of some cardiac cycles PQRST, RCC-55 beats/minute. What is the most possible
mechanism of the impairment of heart functions?
-Toxic impairment of sinus node
+Impairment of sinus node by immune complexes
-Sclerotic changes in sinus node
-Ischemic impairment of sinus node
-Parasympathetic irritation of sinus node

#During ECG investigation irregular atrioventricular extrasystoles were determined in a patient aged 38. The
impairment of what properties of myocardium composes the base of extra systole pathogenesis?
+Excitability
-Automatism
-Conduction
-Contractility
-Tonicity

#Patient Y, aged 40, is treated for pulmonary tuberculosis, acrocyanosis, breathlessness, extension of heart
borders, increase of AP and number of erythrocytes, pachyemia, neutrophilic leukocytosis are marked on
examination. Which of the below mentioned symptoms of chronic hypoxia are regarded as lasting
compensatory mechanisms of organism
+Pachyemia
-Dilation of the cavities and myocardial hypertrophy
-Leucocytosis
-Increase of AP
-Increase of respiratory rate

#Increase of blood viscosity was revealed in a patient with chronic cardiac insufficiency, the damage of the
walls of vessels of microcirculatory channel was found out on capillaroscopy. Which of the impairments of
peripheral circulation are possible in this case?
+Slage phenomenon
-Thrombosis
-Arterial hyperemia
-Embolism
-Venous hyperaemia

#Changes that testify to the impairment of cardiomyocytes were recorded on the ECG of an experimental
animal after the injection of uabain (substance which blockades K+/Na+-depending ATP). What molecular
mechanisms had decisive significance in this case?
+Electrono-osmotic
-Lipid
-Acidotic
-Calcic
-Protein
#Patient, aged 44, complains of strangulation, palpitation, pains in the right costal interspace, and edemas on
the legs. Pulsation of cervical veins, enlargement of the liver, edemas of the lower extremities were found
out on examination. ECG shows the signs of hypertrophy of both ventricles and right auricle. Tricuspid
incompetence was diagnosed. What is pathophysiological variant of this incompetence?
-Overloading of the heart by exertion
-Primary myocardial insufficiency
+Overloading of the heart by volume
-Coronary insufficiency
-Cardiac tamponade

#Acceleration of the heartbeat during breathing in and deceleration during breathing out were revealed in a
youth, aged 16, on examination. ESG showed shortening of RR interval during inspiration and its
lengthening during expiration. Name the kind of arrhythmia
-Ciliary arrhythmia
-Sinus tachycardia
-Idioventricular rhythm
+Sinus arrhythmia
-Sinus bradycardia

#Choose in what form below mentioned states that cause of cardiac insufficiency heterometric mechanism of
compensation (Frank-Starling) takes place:
-Hypertension of systemic circulation
-Hypertension of pulmonary circulation
-Stenosis of mitral orifice
-Stenosis of aortic osteum
+Aortic incompetence

#A patient, aged 58, was hospitalised with complaints of unpleasant feelings in the heart area; there was
myocardial infarction in anamnesis. ECG shows registration of fibrillation waves (F) instead of P-waves;
QRST complexes are widened and deformed. Ventricular rhythm is right; the rate of contractions of
ventricles is 48 per minute. What is the most possible impairment of the rhythm in this case?
+Frederic’s syndrome
-Morgagni-Adams-Stokes syndrome
-Wolf-Parkinson-White syndrome
-Atrioventricular block of II stage. Mobits’ type I
-Atrioventricular block of II stage. Mobits’ type II

#Name the condition, which accompanied by pressure overload of left ventricle


-Stenosis of mitral orifice
+Stenosis of aortic orifice
-Incompetence of mitral valve
-Incompetence of aortic valve
-Pulmonary hypertension

#A 24-year-old female patient was admitted to the hospital with complaints of headache, pain in kidney area,
and general weakness. She was suffered from tonsillitis one month before. At examination patient has BP of
180/110 mmHg; in patient’s blood analysis erythrocytes – 3.1x10^12/L, leucocytes – 12.6x10^9/L, ESR –
28 mm/hour; in patient’s urinalysis – marked proteinuria, hematuria, leukocyturia. What is the mechanism of
hypertension development?
-Renovascular
-Reflexogenic
-Aldosteron-induced
+Renal
-Mineralocorticoid-induced

#In a patient with hypertension the pulse rate dropped from 72 to 52 bpm during hypertonic crisis and
maintained at this level for 10 days. Intramuscular injection of 1 mg of atropine led to increasing of pulse
rate at 16 bpm. What group of arrhythmias does described disorder of cardiac rhythm belong to?
-Ventricular fibrillation
-Atrial palpitation
-Disturbance of rhythm formation
-Disturbance of rhythm conduction
+Disturbance of automatism

#Decrease in pulse rate down to 50 bpm was observed in a patient with cerebral hemorrhage. His pulse was
rhythmic. What is the mechanism of this disorder of cardiac rhythm?
+Irritation of nervus vagus
-Reduction of rate of free diastolic repolarization
-Influence of sympathetic mediator
-Extension of site of sinoatrial node
-Elevation of rate of free diastolic repolarization

#A patient died from myocardial infarction, which was proved by data of clinical and electrocardiographical
examinations. No changes of coronary vessels were found at autopsy of this patient. What evokes
myocardial infarction in this patient?
A- Increase in tonus of sympathetic nervous system
+Enhanced secretion of catecholamines
-Obstruction of vessel with embolus
-Thrombosis of coronary vessels
-Rheumatic coronaritis

#A patient with neuro-circulatory distony has heartbeat rate increased up to 130 bpm. Clinical symptoms of
organic heart injury were not found in this patient. At pressing on carotid sinus heart rate decreased, but after
returned to previous frequency. What is the origin of this disorder of heart rhythm?
-Fluctuations of parasympathetic tonus
-Inflammatory lesion of myocardium
+Enhanced influence of sympathetic nervous system
-Ischemic injury of myocardium
-Toxic injury of myocardium

#Thrombosis of anterior intraventricular coronary artery developed after coronarography in a patient with
ischemic heart disease and atherosclerosis of coronary arteries. What mechanism is the most significant for
development of this complication?
-Slowing of blood flow
+Injury of endothelium of vessel wall
-Increase in blood coagulant concentrations
-Decrease in blood anticoagulant concentrations
-Reduction of fibrinolytic system activity

#Under hypertrophy of myocardium mass of heart increases due to:


+Enlargement of each muscular fiber
-Growth of connective tissue
-Increase in number of myocardiocytes
-Growth of adipose tissue
-Enhancement of blood supply of heart muscle

#Changes in ECG was shown at first by lengthening of P-Q interval, after that by falling out of single QRS
complexes, later by increasing in number of fallen out ventricle complexes, and at last atriums constrict with
frequency of 70 bpm and ventricles constrict with frequency of 35 bpm. Described changes are characteristic
for:
-Intraatrial block
+Cross-sectional block
-Intraventricular block
-Arrhythmia due to disturbances of automatism
-Arrhythmia due to disturbances of conduction

#The first link in atherosclerosis development is:


-Slowing of blood flow
-Degenerative-proliferative changes of internal layer of arteries
-Excessive depositions of blood plasma lipoproteins in internal layer of arteries
-Formation of fibrous plaque on internal layer of arteries
+Disorders of intactness of arterial wall

#A patient suddenly lost consciousness and develop cramps. At his ECG followings were revealed: 2 to 3 P
cogs related to 1 QRST complex. What property of heart conductive system is impaired?
-Automatism
-Excitability
+Conduction
-Constriction
-Reproduction of frequency of excitement

Pathophysiology of respiratory system.

#A young man with suspicion on narcotic poisoning was admitted into neurological department. Which of
the disorders of external respiration can be expected?
-Asphyxia
-Alveolar hyperventilation
-Kussmal respiration
-Biot’s Respiration
+Alveolar hypoventilation
#In pathogenesis of which types of respiration the main link is the fall of excitability of respiratory centre to
carbon dioxide due to oxygen starvation of this centre?
+Accelerated and deep breathing
-Expiratory dyspnea
-Periodic respiration
-Inspiratory dyspnea
-Combined dyspnea

#As a result of casualty the obturation of the lung trachea occurred. Which stage of respiration will be
impaired first?
-Tissue respiration
+Lung ventilation
-Exchange of gases in the lungs
-Exchange of gases in tissues
-Transport of oxygen and carbon dioxide

#The functional state of respiratory system was examined in a patient with emphysema. What is more
characteristic for this state?
+Increase of tidal volume
-Increase of vital volume of the lungs
-Increase of inspiratory reserve volume
-Decrease of total volume of the lungs
-Decrease of inspiratory reserve volume

#The syndrome of respiratory insufficiency is often observed in premature born children. What is the main
cause of this?
-Swallowing of amniotic water
+Immaturity of lung alveoli due to deficiency of surfactant
-Intrauterine hypercapnia
-Imperfection of the nervous regulation of the respiratory act
-Intrauterine asphyxia

#Paleness of the skin, accelerated superficial respiration is observed in a newborn. Numerous diffused
atelectasis are revealed at X-ray examination. What is the most possible cause of this condition?
-Pneumothorax
-Hydrothorax
-Tuberculosis
-Bronchial asthma
+Surfactant deficiency

#A patient who was at a resuscitation department with skull injury suddenly developed convulsions on the
background of regaining consciousness, and short arrest of breathing was changed by solitary sighs with
calming down character. What types of respiration appeared in the patient?
A+ Gasping respiration
-Cheyne-Stokes respiration
-Biot’s respiration
-Kussmaul respiration
-Apnoeystic respiration

#During meal a child breathe in a seed. What respiratory changes will develop in the child?
-First expiratory dyspnoea, then inspiratory one
+At first inspiratory dyspnoea, then expiratory one
-At first arrest of breathing, then expiratory dyspnea
-Expiratory dyspnoea, then Cheyne-Stokes respiration
-Inspiratory dyspnoea, then Biot’s respiration

#A partial respiratory insufficiency developed in a patient due to a chronic impairment of the organs of
respiratory system. What is a characteristic sign of partial respiratory insufficiency?

-Hypoxemia and hypercapnia


-Hypoxemia and decrease of alveolar ventilation
-Decrease of alveolar ventilation and hypercapnia
-Hypoxemia and gas acidosis
+Hypoxemia without hyperapnia

#When developing of pulmonary emphysema in a patient the following is notified:


+Increase of functional “dead space”
-Inspiratory dyspnea
-Cardiac insufficiency (left ventricular type)
-Decrease of resistance to the airflow in the respiratory tract
-Decrease of functional “dead space”

#Decrease of the passage at the level of middle and small bronchi is observed in a patient. What process will
be the leading in the development of respiratory insufficiency?
-Disturbance of diffusion
-Hyperperfusion
+Hypoventilation
-Hypoperfusion
-Hyperventilation

#In the decrease of the middle and small bronchial passages in a patient the following is observed:
-Development of gas alkalosis
-Decrease of pressure in pulmonary circulation
-Development of inspiratory dyspnea
-Hypocapnia
+Decrease of pO2 and increase of pCO2 in the alveolar air

#In obstructive type of respiratory disturbances in a patient it will be determined


-Increase in forced vital volume of the lungs
+Expiratory dyspnea
-Increase in vital volume of the lungs
-Decrease of the tidal volume
-Decrease of the total volume of the lungs
#In restrictive type of respiratory disturbances in a patient the following is revealed
-Decrease of rate and increase of depth of respiration
-Expiratory dyspnea
-Increase of vital capacity of the lungs
-Increase of tidal volume of the lungs
+Decrease of tidal volume of the lungs

#Gas alkolosis is revealed in the patient is blood. What process impairment is connected with this
development?

-Impairment of diffusion
+Hyperventilation
-Hyperperfusion
-Hypoventilation
-Hypoperfusion

#The application of oxygen did not eliminate hypoxemia in a patient with partial respiratory insufficiency.
What is the mechanism of respiratory insufficiency?
-Disturbance of diffusion
-Hyperperfusion (functional shunt)
-Hypoventilation
+Hyperperfusion (anatomical shunt)
-Hyperventilation

#Patient aged 62 was admitted into neurological department due to cerebral haemorrhage. Grave condition.
Increase of depth and rate of respiration and then its decrease and apnoea is observed. After that the cycle of
respiratory movements start again. What type of respiration appeared in the patient?
-Apneustic respiration
-Kussmaul respiration
-Gasping respiration
+Cheyne-Stokes respiration
-Biot’s respiration

#After Typhno’s test index decreased to 30% in the patient. The development of what pathological process
in the organs of respiratory systems does it indicate?
-Lobular pneumonia
-Pneumothorax
+Obstructive bronchitis
-Tuberculous pleurisy
-Pneumosclerosis

#A patient has developed athelectasis, which was accompanied by alveolar collapse. What contributes to
this?
-Hyperventilation
-Spasm of lung vessels
-Arterial hypertension
+Surfactant deficiency
-Respiratory acidosis

#A patient has the pathology of the lung with the disturbance of external respiration and development of
hypercapnia and hypoxia. How many times is the ability of CO 2 to diffusion through alveolar-capillary
membrane higher than the same one of oxygen?
-5
-10
-15
-20
+25

#Patient’s respiratory rate is 10 per min, respiratory volume - 1.1 l whistling rals are heard over the lungs on
auscultation. Which of pathogenetic forms of external respiratory disturbance has this patient:
-Diffuse-pneumonia
-Diffuse-restrictive
-Ventilative-restrictive
-Primary-dyskinetic
+Obstructive

#Which of the pathogenetic forms of external respiratory disturbance may develop in traumatic costal
fracture?
-Primary-dyskinetic
+Ventilative-restrictive
-Obstructive
-Diffuse-restrictive
-Diffuse pneumonia

#Which of pathogenetic forms of external respiratory disturbance may appear in cerebrospinal trauma of the
level of cervico-thoracic part of the spine?
-Obstructive
+Primary-dyskinetic
-Ventilative-restrictive
-Diffuse-restrictive
-Diffuse pneumonic

#In which of pathologic processes does obstructive form of disturbance of external respiration appear?
-Pulmonary edema
-Pneumonia
-Pleurisy
+Bronchial asthma
-Open pneumothorax

#In which of pathologic processes does ventilative-restrictive form of external respiratory disturbance
develop?
+Pulmonary edema
-Bronchial asthma
-Poliomyelitis
-Syringomyelitis
-Bronchitis

#In what form of dyspnea develops in insufficiency of surfactant system of the lungs?

-Rare shallow respiration


-Fast deep respiration
+Fast shallow respiration
-Expiratory dyspnea
-Rare deep respiration

#Which of pathogenetic forms of external respiratory disturbance may develop in craniocerebral trauma?
-Diffuse-pneumonic
-Diffuse-restrictive
-Ventilative-restrictive
+Primary dyskinetic
-Obstructive

#A patient aged 23 was admitted to the hospital with a craniocerebral injury in a poor condition. Respiration
is characterised by spasmatic effort of inspiration which does not stop and sometimes is broken by
expiration. What type of respiration is this typical for?
+Apneustic respiration
-Gasping respiration
-Kussmal’s respiration
-Cheyne-Stokes’ respiration
-Biot’s respiration

#A patient aged 64 was admitted to the hospital with complaints of cough with sputum, marred dyspnea.
Objectively forced position, respiratory rate - 32 per min: additional muscles take part in the act of
respiration. X-ray examination: the lungs have increased transparency. What is the main link in the
pathogenesis of respiratory insufficiency of this patient?
+Decrease of elastic abilities of the lungs
-Accumulation of sputum in the lungs
-Thin mucus membranes of bronchi
-Impairment of surfactant system of the lung
-Pneumosclerosis

#Both vagus nerves were cut during experiment on animal. How may vagus respiration be characterized?
-Frequent and deep
+Very rare and deep
-Rare and shallow
-Frequent and shallow
-Periodic

#A child with diphtheria developed larynx edema. What type of respiration is observed?

-Gasping respiration
-Apneustic respiration
-Kussmal’s respiration
+Dyspnea
-Biot’s respiration

#A female patient was admitted to the hospital with exertion of chronic bronchitis. Antibiotic treatment was
administered. On the fourth day general condition became worse: the temperature persisted, dyspnea and
cough increased, itching eruptions appeared on the skin, diffused dry whistles were heard on auscultation.
There is eosinophilia (10%) in the blood. Aggregation of symptoms of the patient is caused by the
development of:
-Asthmatic bronchitis
-Bronchial asthma
-Bronchopneumonia
-Allergic reaction
+Drug allergic reaction

#The deficiency of alpha-1-antitrypsine was determined in a patient with pulmonary emphysema. What is
the leading mechanism of the development of emphysema in this patient?
-Excessive discharge of elastase by lymphocytes
-Increase of histamine discharge
-Dilation of bronchioles
+Activation of elastolysis
-Decrease of elastase discharge by neutrophilic granulocytes

#A patient with bronchial asthma has developed acute respiratory insufficiency. What type of respiratory
insufficiency appears in this case?
-Disregulative disorder of alveolar ventilation
-Diffuse
-Perfuse
-Restrictive disturbance of alveolar ventilation
+Obstructive disturbance of alveolar ventilation

#0.5 ml of air was introduced into a rat’s pleural cavity. What type of respiratory insufficiency appears in
this case?
+Restrictive disturbance of alveolar respiration
-Obstructive disturbance of alveolar ventilation
-Perfuse
-Diffuse
-Disregulative disturbance of alveolar ventilation

#Acute respiratory insufficiency appeared in a patient with tetanus. What type of respiratory insufficiency
develops in this case?
-Restrictive disturbance of alveolar respiration
+Disregulative disturbance of alveolar ventilation
-Obstructive disturbance of alveolar ventilation
-Perfuse
-Diffuse
#Increased content of carbon dioxide is in a room. How does respiration change (its depth and rate) in a man
entered this room?
-Depth of respiration increases
-Rate of respiration increases
+Depth and rate of respiration increase
-Depth of respiration decreases
-Rate of respiration decreases

#When eating a child aspired some food. Strong cough, cyanosis of skin and mucous membranes, increase in
pulse rate, rare respiration, and lengthening of inspiration developed in him. What kind of disorder of
external respiration developed in this child?
-Stenotic respiration
+Expiratory dyspnea stage in asphyxia
-Inpiratory dyspnea stage in asphyxia
-Biot’s respiration
-Alternative respiration

#A 56-year-old patient, who is fitter by occupation, has been suffering form fibrous-cavernous tuberculosis
of lungs for 10 years. During later 3 weeks cough and weakness intensify, amount of purulent-mucous
phlegm with blood increases in him. What is the reason for developed ventilation insufficiency in this case?
+Decrease in number of functioning alveoli
-Disturbances of functions of respiratory center
-Disturbances of functions of neuro-muscular apparatus
-Disturbances of agility of the thorax
-Disturbances of airways passage

#A 50-year-old unconscious patient was admitted to the endocrinology department. It is known that patient
has been suffering from diabetes mellitus for many years. What kind of disorders of external respiration will
be present in this patient?
+Kussmaul’s respiration
-Asphyxia
-Biot’s respiration
-Stenotic respiration
-Chain-Stocks respiration

#Attack of suffocation developed in a 45-year-old woman suffered from bronchial asthma for a long time.
What pathogenetic mechanism does this phenomenon have?
+Expiratory narrowing of small bronchi
-Loose of elasticity of lung tissue
-Decrease in sensitivity of respiratory center
-Disorders of agility of the thorax
-Impairment of perfusion of lung tissue

#Edema of larynx developed in a child on the background of allergic reaction. What kind of respiratory
failure has developed in this case?
+Obstructive
-Restrictive
-Disregulative
-Parenchimatous
-

#A patient with both-sided lung edema was in the emergency department. What kind of dyspnea is observed
in this case?
A+ Shallow frequent respiration
-Deep frequent respiration
-Shallow rare respiration
-Shallow respiration
-Asphyxic respiration

#Lung fibrosis with disorders of alveolar ventilation developed in a miner. What is the leading mechanism in
this disorder development?
+Restriction of respiratory surface of lungs
-Narrowing of upper airways
-Disorders of neural regulation of respiration
-Restriction of agility of the thorax
-Spasm of bronchi

#A 60-year-old male patient in severe condition was admitted to the hospital. The patient has been suffering
from diabetes mellitus for 10 years. At second day of his stay at the in-patient department his condition
sharply worsened: coma developed, noisy respiration appeared, which was accompanied by deep
inspirations with forced expirations and participation of expiratory muscles. What kind of disorders of
respiration is observed in the patient?
+Kussmaul’s respiration
-Stenotic respiration
-Tachipnea
-Chain-Stocks’ respiration
-Biot’s respiration

#A patient with craniocerebral trauma has respiration, which is characterized by respiratory movements
becoming deeper and then becoming shallower and after that short pause in respiration occurs. What
periodical respiration is this type characteristic for?
+Chain-Stocks’ respiration
-Biot’s respiration
-Kussmaul’s respiration
-Gasping respiration
-Apneastic respiration

#A female patient N., aged 16, with asphyxia was admitted to the emergency department. She has expiratory
dyspnea with respiration rate of 8 per minute, BP of 80/50 mmHg, heartbeat rate of 40 bpm, and narrowed
pupils. What phase of asphyxia does this patient have?
+II phase
-I phase
-II phase
-IV phase
-

#A boy was admitted to resuscitation department. He has inspiratory dyspnea, general excitation, widened
pupils, tachycardia, and BP of 160/100 mmHg. What phase of asphyxia does this patient have?
+I phase
-II phase
-III phase
-IV phase
-

#A 15-year-old girl has been suffering from bronchial asthma for 5 years. What kind of hypoxia does this
patient have?
+Respiratory
-Circulatory
-Hemic
-Tissue
-Mixed

Pathophysiology of digestive system.

#After taking a fatty food a patient feels nausea, flaccidity, later the sign of steathorrhea has appeared,
cholesterol in the blood 9,2ml/c. The cause of this state is the deficiency of:
-Chylomicrones
-Triglyceroides
+Bile acids
-Fatty acids
-Phopholipids

#A patient aged 45 had the diagnosis ulcer of the stomach. On examination of secretory function of the
stomach it was determined that the amount of basal secretion was 100mole/hr, acidity of basal secretion -
60mml/hr. What factors action contributes to the hypersecretion in the stomach?
+Gastrine
-Pancreatic polypeptide
-Somatostatin
-Glucagon
-Betaendorphin

#On laboratory examination increased amount of diastase in the urine and also a large amount of undigested
fat in stool were revealed in a patient female with complain of circular character pain in epigastric area.
What form of gastrointestinal tract pathology are described signs typical for?
-Inflammation of large intestine
-Acute appendicitis
-Infectious gastritis
-Ulcerous disease of the stomach
+Acute pancreatitis
#A part of patient pancreas was resected. What kinds of product must be limited in his diet?
-Sour milk product
-Fruits
-Not fatty boiled meat
+Fatty and fried meal
-Vegetables

#What enzyme deficiency is the cause of maldigestion of fats in the gastrointestinal tract and increase of
neutral fats in the stool?
-Gastric lipase
-Intestinal lipase
-Hepatic lipase,
-Enterokinase
+Pancreatic juice

#Decrease of synthesis and secretion of trypsin is observed in chronic pancreatitis. The splitting of what
substances is broken?
+Proteins
-Nucleic acids
-Polysaccharides
-Lipids
-Liposoluble vitamins

#In coprologic investigation it is determined that stool is discoloured, there are drops of neutral fat in it. The
most possible cause of this is the impairment of?
-Secretion of intestinal juice
-Process of absorption in the intestine
+Entering of bile into the intestine
-Acidity of gastric juice
-Secretion of pancreatic juice

#A patient had been taking antibiotics of a wide spectrum of action for a long period of time that caused
decrease of appetite, nausea, and diarrhea with saprogenic smell. What is the side effect of treatment?
-Hepatotoxic action
-Allergic reaction
+Dysbacteriosis
-Nephrotoxic action
-Direct irritative action

#On examination of a patient suffering from acute pancreatitis increased amount of chylomicrons was
determined in the blood. What enzyme activity is sharply decreased in this pathology?
+Lipoprotienelipase
-Pancreatic lipase
-Pancreatic phospholipase
-Tissue triglyceride lipase
-Tissue diglyceride lipase
#The analysis of gastric juice of an elderly man who complained of unmotivated weakness, sickness,
absence of appetite showed achylia, achlorhydrea, and presence of lactic acids and coagulated blood,
decreased of pepsin secretion. What disease causes such clinical-laboratory symptoms?
+Cancer of the stomach
-Chronic gastritis
-Chronic pancreatitis
-Cavitary Maldigestion
-Acute gastritias

#A man with chronic hepatitis has dyspeptic disorders: decrease of appetite, nausea, unstable stool, and
steatorrhea. What is the mechanism of dyspeptic disorders in hepatic pathology?
-Intoxication
-Hypoglycemi
-Cholalemia
+Hypocholea
-Hyperbilirubemia

#Spasmodic pains in the abdomen and repeated diarrhoea with mucus appeared in a healthy person 3-5 hours
later after taking meals. This was preceded by nausea and momentaneous vomiting, general weakness, loss
of appetite. What is the most possible cause of the desired symptoms?
-Chronic pancreatitis
+Food intoxication
-Enterocolitis
-Hyperacid state of the stomach
-Chronic gastritis

#A man who works at a storage battery plants complains of constant feeling of weight and periodical
spasmodic pains in the abdomen, constant retention of stool (not more often than one time per three day).
This is accompanied by frequent headaches, flaccidity, absence of appetite, and bad taste in the mouth. What
are the causes of these disorders?
-Hyper acid state of the stomach
-Hypoacid state of the stomach
+Spastic lead colie with constipation
-Chronic pancreatitis
-Parietal maldigestion

#Due to chronic gastritis a man has the impaired structure of the mucous membrane, decreased indices of
acid formation function of the stomach. The most essential negative result of this will be the impairment of:
-Excretion of secretum
-Pancreatic juice secretion
-Secretory function of small intestine
-Evacuation of chyme into duodenum
+Protien digestion
#On laboratory examination increased amount of diastase in the urine and also a large amount of undigested
fat in stool were revealed in a patient with complains of circular character pains in epigastric area. What
form of gastrointestinal tract pathology are the described signs typical for?
-Inflammation of the large intestine
-Acute appendicitis
-Infectious hepatitis
-Ulcerous disease of the stomach
+Acute pancreatitis

#A patient aged 35 with ulcerous disease had a rejection of antral portion of the stomch. What
gastrointestinal hormone secretion will be impaired due to operation?
+Gastrin
-Secritin
-Neurotensin
-Histamine
-Cholecystokinin

#A patient aged 57 was admitted to a gastrointestinal department with suspicious on Zolliger-Ellison


syndrome, which was proved by sharp increase of gastrin level in the blood serum. What impairment of
secretory gastric function is the most possible?
-Hyperacid hyposecretion
+Hyperacid hypersecretion
-Hypoacid hypersecretion
-Achylia
-Hypoacid hyposecretion

#A patient with signs of encephalopathy was hospitalised in neurological in- patients department and co
relation between increase of encephalopathy and substances passing from intestine into systemic blood
circulation was determined. What compounds formed in the intestine may cause endotoximia?
-Acetoacetat
-Biotin
+Indole
-Ornitin
-Buthirat

#150 ml of meat broth were introduced into the stomach cavity of experimental dog to a probe the content of
what substance will increase quickly in the animal’s blood?
-Insulin
-Vasointestinal peptide
-Neurotensin
-Somatostatin
+Gastrin

#A patient age 37 was admitted into a surgical department with the signs of acute pancreatitis; vomiting,
diarrhea, bradycardia, hypotension wearness, dehydration of the organism. What preparation must be used
first of all?
-Ephedrine
-No-spa
-Pephenazine
+Contrical
-Platyphyllin

#A patient has a stone in the common bile duct, which has stopped passing bile into the intestines. The
impairment of what digestive process is observed in this case?
-Digestion of carbohydrates
-Absorption of carbohydrates
+Digestion of fats
-Absorption of proteins
-Digestion of proteins

#On fibroscopy of the stomach ulcer was revealed in antrum portion, which was associated with
dissemination of mucosa by Helicobacter pylori. The role of this agent in the formation of ulcer results in:
+Damage of mucous barrier
-Inhibition of mucosa regeneration
-Impairment of microcirculation in mucous
-Stimulation of HCL secretion
-Stimulation of pepsin secretion

#A patient with ulcer of stomach has impairment of equilibrium between the factors of aggression and
defense. What factor contributes to the development of gastric ulcer?
-Mucin
+Helicobacter pylori
-Bicarbonate
-Prostaglandin E2
-Prostacyclin

#After resection of duodenum a patient has developed a syndrome of duodenal in sufficiency due to the
impairment of its endocrine function with signs of cell insufficiency of APUD system. What hormone is
produced by A-cells of this portion of intestine?
-Histamine
-Insulin
-Seratonin
-Secrtin
+Glucagon

#A patient complains of nausea, which often ends with vomiting. These dyspeptic phenomena become more
laborious with time. What stomach function is infringed in this patient?
+Evacuation
-Motility
-Reservoir
-Incretory
-Secretory
#A patient aged 42 was admitted to the hospital with gastric hemorrhage (ulcerous disease was excluded by
a patient). Ulcer and hyperplasia of gastric mucose were revealed. What investigation is it necessary to carry
out to confirm the diagnosis of Zollinger-Ellison syndrome in this patient?
+Definition of gastrin level in the blood
-Tomography of pancreas
-Definition of secretin level in the blood
-Definition of activity level of pancreatic enzymes
-Investigation of activity level of pancreatic secretion

#The amount of protein in the blood was examined in a patient after resection of sizeable part of small
intestine. What change of protein amount may be expected in this patient?
-Hyperproteinemia
+Hypoproteinemia
-Paraproteinemia
-Hypergammaglobulinemia
-Dysproteinemia

#Ulcerous disease of the stomach is revealed in a woman aged 52, who has been ill with contact dermatitis.
On examination it is determined that the patient had been taking corticosteroid preparations for a long period
of time, but acidity of gastric juice was increased. What mechanism caused the increase of gastric secretion?
+Decrease of prostaglandin content
-Increase of secretin secretion
-Gastrin hyposecretion
-Increase of histamine content
-Increase of gastrin excretion

#A patient complains of vomiting, eructation, pain in epigastric area, constipation, and emaciation. On
examination a basal secretion of HCL is 6mml/gr; maximal one is 60mml/gr. What state has acid-forming
function of this patient?
-Hypoacid
-Anacid
+Hyperacid
-Hypersecretory
-Normacid

#A patient aged 42 complains of pains in epigastric area, vomiting; vomitory masses have color of “coffee
grounds”, melena. In anamnesis there is ulcerous disease of the stomach. On examination: skin is pale, heart
rate-110 beats per minute; AP-90/50mm.hg. Blood analyses: erythrocytes-2.8 x 1012/l; leucocytes-8x109/l;
hemoglobin-90gr/l. What is the possible complication that developed in the patient?
-Penetration
-Perforation
+Hemorrhage
-Cancerous degeneration
-Pylorostenosis

#The investigation of pancreatic function was carried out in experimental animal by radioisotopic method.
The percentage of excreted with stool 1131- albumin is 53%. This is the evidence of:
-Increase of exocrine function
-Increase of incretory function
-Inhibition of incretory function
+Inhibition of exocrin function
-Normal function

#Malabsorption syndrome is:


-Syndrome of membraous maldigestion
-Syndrome of gastric malabsorption
-Syndrome of malabsorption in small intestine
+Syndrome of intestinal malabsorption
-Malabsorption of proteins

#A patient complains of dyspeptic disorders, melaena, hemorrhoidal bleedings. While examination of the
patient enlargement of veins at the anterior surface of abdomen and increased abdomen dimensions were
revealed. What pathology of GIT may manifest by such symptoms?
-Colitis
+Portal hypertension
-Intestinal autointoxication
-Peptic ulcer
-Enteritis

#A 67-year-old woman, who has cholecystitis for a long time, suddenly developed sharp pain in the upper
part of abdomen, nausea, and vomiting after food intake. Acute pancreatitis was diagnosed in this patient.
What is the main link in pathogenesis of this disease?
+Preliminary activation of pancreatic enzymes
-Decrease in enzyme levels in pancreatic juice
-Intensification of enzyme activation in duodenum
-Reduction of pancreatic polypeptide secretion
-Increased level of cholecystokinin

#A patient has increased gastric secretion in response to both mechanical and chemical stimuli. He has high
acidity of gastric juice with pH equals 2.0 on an empty stomach. After the breakfast pH restores to normal in
12 minutes and is continuously dropping during following 2 hours. What type of stomach secretion is being
observed in this patient?
+Excitable
-Brakable
-Inert
-Asthenic
-Normal

#Frequent liquid stool, signs of intoxication, and dehydration developed in a newborn at 5-6 day. After this
newborn was fed by acid-milk mixtures indicated symptoms disappeared. The conclusion of innate lactose
deficiency was made. What process was broken in this newborn?
+Membrane digestion
-Cavital digestion
-Excretory function of intestine
-Secretory function of pancreas
-Secretion of gastric juice

#Ulcers of stomach were found out in a patient, who was treated with glucocorticoids for a long time. What
is the main mechanism of ulcer development in this case?
+Increase in secretion and acidity of gastric juice
-Decrease in histamine concentration in stomach wall
-Increased tonus of sympathetic nervous system
-Increased production of prostaglandins E
-Decreased tonus of parasympathetic nervous system

Pathophysiology of the liver

#Flabby contraction of gall bladder was revealed in a woman aged 55 after introducing some of vegetable oil
into duodenum. What hormone insufficiency with such state?
-Gastrin
-Enterogastrin
-VIP
-Pancreozymin
+Cholecystokinin

#A patient aged 25 has a diagnosis of chronic hepatitis. A patient has lost 10 kg of his body weight for 2
months. Objectively: the skin is dry, desquamative, and pale with yellowish color, small punctate
hemorrhages on the skin, stomatorrhagia. The impairment of what hepatic function do petechial hemorrhage
and stomatorrhagia prove?
-Glycogen synthetic
-ekromogenic
-Detoxicative
-Depositing
+Albumin synthetic

#On examination bile congestion in the liver and cholelitiasis were revealed in a patient. Point out the main
component of cholelitiasis in this state:

-Calcium bilirubinate
-Triglycerides
-Protein
+Cholesterol
-Mineral salts

#What kind of jaundice is characterized by increased amount of direct bilirubin in the blood, appearance of
bilirubin in urine, acholic stool?
-Hemolytic
+Obstructive
-Parenchymatous
-
-
#Residual nitrogen and urea were determined in the patient’s blood analysis. The amount of urea in the
residual nitrogen is considerably reduced. The disease of what organ is characterized by this analysis?
-Intestine
-Kidneys
-Stomach
-Heart
+Liver

#A patient complains of general weakness, boring pain in the abdomen, bad appetite, suspicion on jaundice.
Blood serum contains 77.3 mcml/L of total bilirubin and 70.76 mcml/L of conjugative bilirubin. What is the
most possible type of jaundice?
+Mechanical jaundice
-Acute hepatitis
-Hepatic cirrhosis
-Parenchymatous jaundice
-Hemolytic jaundice

#In 70’s the scientists determined that the cause of severe jaundice in newborns was the impairment of
connection of bilirubin in hepatocytes. What substance is used for the formation of conjugate?
+Glucoronic acid
-Pyruvic acid
-Uric acid
-Sulphuric acid
-Lactic acid

#A man aged 38 with ecteric skin has anemia,enlarged spleen, hyperbilirubinemia, urobilinuria, hypercholic
stool. What condition are these changes typical for?
+Suprahepatogenous jaundice
-Subhepatogenous jaundice
-Cellular-hepatogenous jaundice
-Gilbert’s syndrome
-Syndrome of hepatic insufficiency

#It’s determined that a patient with jaundice has increased amount of total bilirubin instead of indirect one
(free) in blood plasma,high content of stercolbilin in stool and urine,the level of direct (connected) bilirubin
in blood plasma is normal.what kind of jaundice is it?
-Mechanical
-Icterus of new borns
-Parenchymatous (hepatic)
+Hemolytic
-Gilbert’s disease

#In inflammatory process colloidal properties of bile are impaired in gall bladder and this results in the
formation of gall stones. What substance crystallization is the main cause of the formation?
+Cholesterol
-Urate
-Chloride
-Oxalate
-Phosphate

#After an accident a completely crashed man’s liver was removed.what disorders do of hepatic absence may
cause death during the first hours after operation?
+Hypoglycemia
-Intoxication
-Fall of AP
-Sharp ascites
-Hemophilia and hemorrhage

#rked isoosmotic hyperhydration has developed in a patient with hepatocirrhosis. What is the leading
mechanism of dyshydria development?
-Cardio-vascular insufficiency
-Growth of wall capillary permeability
-Rushyer-petrovsky reflex
+Secondary aldosteronism
-Hypoproteinemia

#On the background of pain in the right hypochondrium and yellowness ‘a patient with hepatic cirrhosis has
constant dyspeptic disorders in a kind of bitter taste in the mouth, feeling of heaviness in epigatric area,
nausea, unstable stool, steatorrhea. What is the main cause of the described disorders?
-Hypoglycemia
+Hypocholia and intoxication
-Increase of stercobilin
-Hyperbilirubinemia
-Hypoproteinemia

#In a severe course of viral hepatitis, a patient has developed adynamia, sleepiness at day time and insomnia
at night, inadequate behavior (delirated ideas). There is fetor hepaticus, Kussmaul’s respiration. What kind
of metabolism impairment causes these symptoms?
-Carbohydrate metabolism and hypoglycemia
-Water and salt metabolism and hyperhydration
-Fat metabolism
-Pigmental metabolism and hyperbilirubinemia
+Nitrogenous metabolism

#There is increase of indole amount and decrease of indican 1 in the patient’s urine. This indicate the
impairment of
-Filtering function of kidneys
+Detoxication function of liver
-Reabsorptive funtion of kidney
-Albumin synthetic function of liver
-Secretory function of pancreas
#Increase of direct and indirect bilirubin is determined in blood of a patient with marked yellowness of
sclerae and skin. There is great amount of bilirubin and urobilin in the urine, traces of stercobilin, decrease
of stercobilin in stool. Define pathogenic type of jaundice in a patient
+Parenchymatous
-Hemolytic
-Mechanical
-By-pass
-Transmissible

#A patient admitted to the hospital has clearly marked widened subcutaneous veins in the area of umbilicus
(“the head of medusa”). Which of the large venous vessels has the impaired passage?
-V. renalis
+V. porta
-V. iliaca inferior
-V. mesenterica superior
-V. mesenterica inferion

#In coprologic investigation it is determined that stool is colorless; there are drops of neutral fat in it. The
most possible case of this is impairment of:
+Entering the bile into intestine
-Secretion of intestine juice
-Acidity of gastric juice
-Processes of absorption in the intestine
-Secretion of pancreatic juice

#A patient had nausea and malaise after taking fatty foodstuffs. Sings of steatorrhea developed in this patient
some time later. Content of cholesterol in patient’s blood is 9.2 mmol/L. This condition results from
deficiency of:
-Chylomicrones
-Triglycerids
+Bile acids
-Phospholipids
-Lipase

#Low level of albumins and fibrinogen were revealed in the patient’s blood. The decreased activity of what
hepatocyte organells cause this phenomenon?
-Agranular endoplasmic network
-Mitochondria
+Granular endoplasmic network
-Goldgi’s complex
-Lysosoms

#A patient was admitted into a clinic with signs of acute alcohol poisoning. What changes of carbohydrate
metabolism are typical for this condition?
-Gluconeogenesis increases in the liver
-Glycogen decomposition increases in the liver
-Aerobic decomposition of glucose increases in the muscles
-Anaerobic decomposition of glucose predominates in the muscles
+The rate of gluconeogenesis decreases in the liver
#A man aged 54 was admitted into the clinic with complaints of pains in the right hypochondrium vomiting
with blood. Objectively: enlargement of hepatic size, varicose of esophagus and stomach, bleeding from
them. The functional disorder of what vessel took place?
-Vena hepatica
-Vena cave superior
+Vena porta
-Aorta abdominalis
-Vena cava inferior

#The development of acute pancreatitis in a patient is accompanied by the impairment of permeability of


common bile duct. What pathologic process may this result in?
-Hemolytic jaundice
-Parenchymatous jaundice
-Hepatic coma
+Mechanical jaundice
-Portal hypertension

#Which of the factors plays the leading role in the development of encephalopathy in hepatic insufficiency?
+Increase of concentration of toxic substances in the blood
-Hyperaldosteronism
-Hyperbilirubinemia
-Hypofibrinogenia
-Hypoproteinemia

#A patient has a diagnosis of ascites. There are no edemas in the other parts of the body. There are large
vessels of cyanotic color on the abdomen. What pathology has this patient?
-Essential hypertension
-Hypertension of pulmonary circulation
+Portal hypertension
-Chronic circulatory insufficiency
-Hepatic hypertension

#Arterial hypertension may develop in a case of hepatic insufficiency. Which of these factors causes the
increase of arterial pressure in such cases?
+Aldosteron
-Adrenaline
-Renin
-Noradrenalin
-Angiotensin II

#A patient aged 25 is ill with jaundice, his skin has got yellow and green color, there’s skin itch, hypocholic
stool throbbing gall bladder. What is the origin of jaundice?
-Serum hepatitis
+Obturative genesis
-Hepatic cirrhosis
-Leptospirosis
-Post-transfusion

#A patient complains of general weakness, breathlessness. Decrease of AP, ascites, widening of superficial
veins of anterior abdominal wall, splenomegaly were established in this patient. What impairment of
hemodynamic has this patient?
+Portal hypertension syndrome
-Left ventricular failure
-Right ventricular failure
-Collapse
-Arterial hypotension

#A patient with jaundice complains of erythrism, headache, and insomnia. Objectively: pulse-54 beats/min.
AP- 90/60 mmHg. Coagulation of blood decreases. There are traces scratching on skin. The action of what
component causes these symptoms?

-Bilirubin
-Cholesterol
-Bile pigment
+Bile acid
-Fatty acid

#A patient was admitted to the hospital with complaints of dyspeptic disorders, melena, hemorrhoidal
bleeding. Extensions of the vessels on the anterior abdominal wall in combination with the enlargement of
size of the abdomen were revealed on examination of this patient. What pathology of gastrointestinal tract
shows these symptoms?
-Ulcerous disease
-Intestinal autointoxication
-Portal hypertension
+Colitis
-Enteritis

#Yellow color of the skin and the sclera, dark urine appeared in a patient after fungus poisoning. What
pigment causes the color of urine in the patient with hemolytic jaundice?
-Biliverdin
-Verdoglobin
-Unconjugated bilirubin
-Monoglucoronide bilirubin
+Stercobilin

#A female patient with chronic hepatitis complains of increase sensitivity to barbiturates which she used
previously without any symptoms of intoxication. The disorder of what hepatic jaundice is responsible for
this state?
-Bile formation
-Hemodynamic
+Metabolic
-Hemopoetic
-Phagocytotic

#Marked jaundice appeared in a patient 3 months later after the operation on his upper jaw. What
pathological process may this patient have?
-Suprahepatic jaundice
-Hereditary hemolytic jaundice
+Hepatic jaundice
-Subhapetic jaundice
-Cholecystitis

#A female patient aged 45 was admitted to a hospital with complains of sudden pain in the abdominal cavity,
increase of temperature, and leukocytosis. Which of factors caused these changes in the woman’s condition?
+Gall stones
-Bacteria
-Mechanical energy
-Acids
-Viruses

#In a 38-year-old patient, who endued viral hepatitis C and is abusing alcohol, symptoms of hepatic cirrhosis
with ascites and edemas on lower extremities developed. What changes in blood composition underlies
edema development?
-Hypoglycemia
+Hypoalbuminemia
-Hypoglobulinemia
-Hypokalemia
-Hypocholesterolemia

Renal pathophysiology

#A patient has severe nephropathy with a massive edematous syndrome, which complicated bronchiectatic
diseases. Laboratory investigations show massive proteinuria, cylindruria, considerable decrease of protein
content in the blood serum, hyperlipidemia, hypokalemia and other deviations. What is the main link in the
pathogenesis of the patient’s edemas?
+Decrease of oncotic blood pressure
-Increase of extracellular fluid pressure
-Increase of hydrostatic blood pressure
-Blockade of lymph drainage
-Hyperpermeability of microvessels

#A patient with renal insufficiency has a disorder of hemopoietin synthesis produced in the kidneys. What
formed blood elements development is impaired?
+Erythrocytes
-Granulocytes
-Thrombocytes
-B-lymphocytes
-T-lymphocytes

#The narrowing of afferent glomerular renal arteriole caused the decrease of urination. The cause of this is a
decrease of:
-Reabsorption of glucose
-Reabsorption of ions
-Reabsorption of urea
-Reabsorption of water
+Effective filtration pressure

#A man aged 32 has been ill with chronic glomerulonephritis for 4 years. He was hospitalized with the signs
of anasarca. AP-185/105 mm. Hg. Blood analysis shows: Hb-110gm/l; erythrocytes-2.6x10 12/l; leucocytes-
9.5x109/l; residual nitrogen-32 mmol/l; total protein-50 gm/l. What change points to glomerulonephritis with
nephrotic syndrome?
-Anemia
-Arterial hupertension
-Hyperzotemia
+Hypoproteinemia
-Leukocytosis

#Sharp decrease in sodium content in the blood serum was revealed in a patient with renal insufficiency.
There are pale crumby edemas of the face manifesting in the morning. What substance composing
intercellular matrix comes from blood bed?
-Collagen
-Elastin
-Procollegen
+Hyaluronic acid
-Fibronectin

#Edemas appeared in a patient after Streptococcus infection. The urinalysis was made and allowed to make a
diagnosis of acute glomerulonephritis. What pathologic changes in urine confirm the diagnosis?
+Fresh erythrocytes
-Protein with high molecular mass and lixiviated erythrocytes
-Protein only with low molecular weight (up to 40000
-Increased excretion of sodium
-Leukocyturia

#After poisoning by salts of heavy metals a patient has developed nongaseous acidosis, urine pH=6.0,
glucosuria without hyperglycemia, polyuria, natriuresis. Which of the structures is damaged?
-B-cells of Lanhergans islets
-Nephron glomeruli
+Nephron tubules
-Nuclei of hypothalamus
-Adrenal cortex
#A patient age 48 with fibro-cavernous tuberculosis complains of general weakness decrease of diurnal
urination, edemas of the trauma and extremities, increase of AP up to 180/90 mm. Hg Urinalysis shows
increase of protein, hyaline and granular cylinder, erythrocytes. A month later he died from renal
insufficiency. Enlargement of the heart and large “fatty kidneys” were revealed at autopsy (weight of
kidneys is 240.0 gm -260.0 gm). What disease was fibro-cavernous tuberculosis of the lungs complicated
by?

-Glomerulonephritis
-Nephrotic syndrome
+Amyloidosis
-Pyelonephritis
-Nephrosclerosis

#A patient with chronic glomerulonephritis has edemas, AP-210/100 mmHg, and the rate of heart beats-85
per minute. The borders of the heart are dilated. What is the leading mechanism in the development of
arterial hypertension?
-Increase of sympathetic-adrenal system activity
-Hyperfunction of the heart
+Activation of renin- angiotensin -aldosteron system
-Increase of circulatory blood volume
-Increase of vasopresin discharge

#The presence of glucose in the urine in its normal concentration in the blood serum was determined in a
patient aged 18 on laboratory examination. The most possible cause of this is the impairment of
+Tubular reabsorption
-Glomerular filtration
-Tubular secretion
-Secretion of the glucocorticoids
-Insulin secretion

#In a week and a half after a severe streptococcal tonsillitis a patient aged 24 developed edematous face,
increased arterial pressure, hematuria, and proteinuria of 1.2 g/L .His blood analysis shows antistreptococcal
antibodies and decrease of complement components. In the microvessels of what structures is the most
possible localization of immune complex deposits which caused the development of nephropathy
-Proximal tubules
+Gomeruli
-Descendent tubules
-Henle’s loop
-Pyramids

#Increased amount of proteins in the urine was revealed in a patient with acute glomerulonephritis. The
impaired function of what nephron structures is the presence of protein in the urine connected to?
+Basal membrane of glomerulus capillaries
-Epithelium of parietal layer of glomerulus capsule
-Epithelium of thin tubules
-Epithelium of distal tubules
-Epithelium of Henle”s loop
#Proteinuria (5 g/l) due to low molecular weight proteins and hematuria with lixiviated erythrocytes were
revealed in patient’s urinalysis. What renal function disorder do these findings point out?
+Increase in glomerular permeability
-Increase in tubular secretion
-Increase in tubular excretion
-Decrease in tubular reabsorption
-Extrarenal disorders

#A patient with chronic glomerulonephritis has symptoms of anemia. What causes these symptoms?
+Decrease in erythropoietin synthesis
-Loss of erythrocytes in urine
-Increased destruction of normal erythrocytes
-Erythrocytes hemolysis
-Iron deficiency for hemoglobin synthesis

#A patient with chronic renal disease is edematous, pale, and his BP is increased. Laboratory examination
shows protein and erythrocytes in patient’s urine. Protein content in the blood is the normal. What is the
main link in the pathogenesis of edematous syndrome?
-Arterial hypertension
-Hyperazotomia
+Secondary aldosteronism
-Hematuria
-Poteinemia

#A patient with chronic renal disease is edematous, pale, and his BP is increased. Laboratory examination
revealed protein and erythrocytes in his urine, hyperazotemia, and decrease in erythrocytes and hemoglobin
in his blood. What is the main link in pathogenesis of arterial hypertension of this patient?
+Activation of renin-angiotensin system
-Anemia
-Hyperazotemia
-Increase of circulatory blood volume
-Proteinuria

#A patient with diabetes mellitus has developed chronic renal failure with the development of uremia and the
rate of glomerular filtration of 8 ml/min. What is the mainly possible mechanism of decrease in glomerular
filtration rate and the development of chronic renal failure in this patient?
-Spasm of afferent glomerular arteriole
-Occlusion of tubular lumen of nephron by hyaline cylinders
-Increase of osmotic blood pressure
-Decrease of systemic BP
+Decrease of the number of working nephron

#Severe poisoning by mercury salts has lead to decrease and then to ceasing of patient’s urination. There are
headache, nausea, and vomiting in this patient. Laboratory investigation reveals quickly increasing azotemia.
The established diagnosis is the acute renal failure, stage of oligouria-anuria. What disorders of aqueous and
osmotic homeostatic does this patient have at this stage of the disease?
-Hypoosmolar hypohydration
-Hyperosmolar hypohydration
-Isoosmolar hypohydration
+Hypoosmolar hyperhydration
-Hyperosmolar hyperhydration

#A patient with chronic renal disease is edematous, pale, his AP is increased, and he has vomiting and
diarrhea with ammoniac smell. Laboratory investigation reveals protein and erythrocytes in his urine,
hyperazotemia, decrease of erythrocytes and hemoglobin in his blood. What is the main link pathogenesis of
anemia which complicated renal disease?
-Loss of erythrocytes with urine
-Toxic influence of urea on bone marrow
+Deficiency of erythropoietin
-Impaired of iron absorption due to diarrhea
-Impaired of regulation of erythropoiesis by endocrine glands

#A man suffering from gout complains of pains in the area of kidneys. Ultra sound examination reveals the
presence of renal calculi. What substance increased concentration causes the formation of calculi in this
case?
-Bilirubin
+Uric acid
-Urea
-Cystine
-Cholesterol

#What hormone increased secretion is caused by activation of renin-angiotensin system in renal hypoxia?
-Hydrocortisone
-Thyroxin
+Aldosteron
-Insulin
-Parathormone

#A man has decreased urination, hypernatremia, hypokalemia. What hormone hypersecretion is the cause of
such changes?
-Vasopressin
-Atrial natriuuretic factor
-Parathormone
+Aldosterone
-Adrenalin
#A patient with acute renal insufficiency developed anuria (diurnal urination - 50 ml). Which of the bellow-
mentioned mechanisms is the main one in its development?
+Decrease of glomerular filtration
-Impairment of renal blood circulation
-Increase of water reabsorption
-Increase of sodium reabsorption
-Difficulty of urine outflow
#A patient with chronic glomerulonephritis has vegetation of collagenic fibers in interstitium near tubules in
which reabsorption of sodium ions decreases. What underlies these changes in reabsorption of sodium ions
in tubules?
+Inhibition of energy metabolism
-Activation of glycolysis
-Inhibition of lipid peroxidation
-Stabilization of lysosomal membranes
-Increase of antioxidative activity

#The damage of proximal portion of nephron with decrease of sodium ion reabsorption was observed in a
patient after poisoning by corrosive sublimate. How much maximally may sodium ions be reabsorbed in this
part of nephron?
-65 %
+80 %
-50 %
-35 %
-20 %

#The damage of proximal portion of nephron was held in a patient in 24 hours later after the poisoning by
corrosive sublimate. What electrolyte reabsorption is impaired in this pathology?
-Ions of potassium
-Ions of chloride
+Ions of sodium
-Ions of calcium
-Ions of magnesium

#A patient aged 35 complains of pain in the lumbar area, edema under eyes, and increased fatigability in
usual physical loads. Protein was revealed in patient’s urine (0.99gr/l). Patient’s BP is -160/110 mmHg.
What pathology has the patient?
-Pyelitis
+Nephritic syndrome
-Cystitis
-Acute renal failure
-Nephrotic syndrome

#What pathological process is non-selective, non-massive proteinuria is characteristic for?


+Nephritic syndrome
-Urethritis
-Chronic renal failure
-Acute renal failure
-Nephrotic syndrome

#What origin of protein is the most possible in selective proteinuria with intensity 12 gr/day?
-Suprarenal
+Tubular
-Glomurular
-Urethra
-From urinary bladder

#What pathological process is characterized by combination of massive proteinuria (25gr/day) with


generalized edema?
-Urethritis
-Chronic renal failure
+Nephrotic syndrome
-Cystitis
-Nephritic syndrome

#What pathological process may be complicated by acute renal failure?


-Chronic lung abscess-
-Viral hepatitis-
-Polycystitic kidneys-
+Extensive burn+
-Hypertensive disease-

#Acute pylonephritis was diagnosed in a patient with complains of increased temperature, pain in the lumbar
area, and frequent and painful urination .Which of the infectious agents is a cause of the disease?
-Staphylococci
-Streptococci
+Escherichia coli
-Brucella
-Anaerobic flora

#A patient suddenly has developed colicky pains in the are of kidney with irradiation to grain, nausea, and
vomiting; the discharge of urine for 24 hours is 90 ml. Determine mechanism of anuria
-Impairment of filtration
-Increase of reabsorption
-Blocking of a work of a part of nephrons
+Reflex anuria
-Obturation of ureters

#A man of aged 72 is ill with chronic glomerulonephritis. On examination following was determined:
absence of appetite, vomiting, diarrhea, skin itching, anemia, the content of residual nitrogen in the blood is
45mm/l. The indicated signs are caused by:
-Increase of glomerular membrane permeability
+Disturbance of nephron function
-Autoimmune damage of nephrone function
-Renal ischemia
-Disturbance of concentrating mechanism

#A patient of age 32 with acute glomerulonephritis, who did not follow regime of NaCl and water limitation,
suddenly has lost his consciousness and convulsions appeared in him. His BP is 220/120 mmHg, he has
mydriatic pupils and bradycardia. What complication has appeared in this patient?
-Acute heart failure
-Hypertensive crisis
-Renal coma
-Epilepsy
+Eclampsy

#A woman aged 25 with frequent prolonged tonsillitis in her anamnesis came to а doctor with complaints of
periodical headaches, undue fatigability, periorbital edemas. Clinic-laboratory investigations show moderate
arterial hypertension, proteinuria, hypoproteinemia, hyperlipidemia. Bilateral disturbance with insignificant
increase of echogenity of renal parenchyma was diagnosed by ultrasound examination of kidneys. What
disease is the mast possible in this case?
-Chronic pyelonephritis
-Acute glomerulonephritis
-Lipoid nephrosis
+Chronic glomerulonephritis
-Acute renal failure

#Protein, which level didn’t exceed 1g/L, was revealed in the urine of physically healthy young military men
after a hard physical exertion during one day foot match (50 km). What kind of proteinuria takes place first
of all?
-False proteinuria
-Organic proteinuria
-Alimentary proteinuria
-Dehydrated proteinuria
+Cyclic proteinuria

#After a severe trauma a patient developed shock with signs of acute renal failure. What is the leading
mechanism in the development of acute renal failure in this case?

-Decrease of oncotic blood pressure


-Increase of pressure in renal arteries
-Increase of pressure in glomerular capsule
-Damage of outflow of urine
+Fall of arterial pressure

#Decrease in insulin clearance down to 60 ml/min was determined in a patient with chronic renal failure.
What renal function impairment is this connected with?
+Glomerular filtration
-Tubular secretion
-Reabsorption in proximal part of nephron
-Reabsorption in distal part of nephron
-Reabsorption in collecting renal tubules

#A patient with chronic renal insufficiency has developed anorexia, dyspepsia, impairment of cardiac
rhythm, skin itching. What is the main mechanism of the development of these impairments?
-Disturbance of lipid metabolism
+Accumulation of products of nitrogen metabolism in the blood
-Changes of carbohydrate metabolism
-Renal acidosis
-Disturbance of water and electrolyte metabolism

#A patient who has been suffering from osteomyelitis of the mandible for many years has extensive edemas;
marked massive proteinuria is revealed in his urine. Which form of complications in the course of
osteomyelitis has this patient?
-Pyelitis
+Nephrotic syndrome
-Nephritic syndrome
-Chronic renal failure
-Renal calculi disease

#After automobile accident a patient’s BP is 70/40 mm Hg. The patient is unconscious. His diurnal urination
is about 500 ml. There are periodical convulsions, Kussmaul’s respiration in him. What is the cause of the
disorder of urination?
+Considerable acute hypotension
-Increase of glomerular filtration
-Decrease of tubular reabsorption
-Increase of tubular reabsorption
-Intoxication by metabolites of nitric exchange

#In a man rate of glomerular filtration rises in 20% as a result of prolonged starvation. What is the most
possible reason for changes of filtration under indicated conditions?
-Increased permeability of renal filter
-Increase in filtration coefficient
-Increased systemic arterial pressure
+Decreased oncotic pressure of blood plasma
-Increase in renal blood flow

#Under experiment morphological impairment of epithelial cells of distal part of nephron was induced in
rats. What functional processes in kidneys are weakened in this case?
-Filtration
-Reabsorption of sodium and glucose
-Reabsorption of glucose
+Reabsorption of water and salts
-Reabsorption of proteins

#Glucosuria and aminoaciduria were found out in a patient with nephritis. What mechanism of reabsorption
of glucose and amino acids is impaired in this case?
-Pinocytosis
-Primary active transport
-Simple diffusion
+Secondary Na-dependent transport
-Phagocytosis

#Following changes were observed in a patient after poisoning with salts of heavy metals: increased level of
residual nitrogen, hyperphosphatemia, hypersulfatemia, hyperacidemia, and decreased alkaline reserve.
What structures impairment has led to described changes?
+Glomerulus of nephron
-Langerhgans’ islets
-Hepatocytes
-Cortex of adrenal glands
-Tubules of nephron

#A 48-year-old man was admitted to the hospital with aggravation of chronic glomerulonephritis.
Examination of this patient found out a presence of chronic renal failure in him. What does azotemia in
chronic renal failure results from?
+Decreased glomerular filtration
-Decreased tubular reabsorption
-Decreased tubular excretion
-Disorders of protein metabolism
-Disorders of water-salt balance

#A patient was admitted to the hospital with complaints of absence of urination during 24 hours and pains in
lumbar area. Catheterization of patient’s urinary bladder does not relief the patient. What pathology may be
cause of anuria in this patient?
+Bilateral renal calculi
-Heart failure
-Bleeding
-Vomiting
-Severe intoxication

#Inhibitor of phosphorilation in cells of nephron tubules – floridzine, was introduced to an animal in


experiment. What substances reabsorption impairment may develop in this case?
+Disorders of reabsorption of glucose
-Disorders of reabsorption of non-organic phosphate and calcium
-Disorders of reabsorption of amino acids
-Disorders of reabsorption of proteins
-Disorders of reabsorption of sodium ions and water

#What pathological changes are characteristic for glomerulonephritis?


+Presence of high molecular weight proteins (70,000 D) in urine
-Presence of fresh erythrocytes in urine
-Presence of low molecular weight proteins (about 40,000 D) in urine
-Increased excretion of sodium and hyaline cylinders in urine
-Leukocyturia without hematuria

Pathophysiology of endocrine system.

#The height of an adult is 100cm, in proportional constitution and normal mental development. What
hormone insufficiency do the indicated signs characterize?
+Growth hormone
-Thyroxin
-Antidiuretic
-Mineralocorticoids
-Gonadotropic hormone

#Under the influence of harmful ecological factors the normal formation of lysosomes is inhibited in
thyrocytes. What stage of hormone production of thyroid gland will be impaired?
-Synthesis of colloid
-Iodinisation of colloid
-Synthesis of thyroglobulin
-Reabsorption of colloid
+Proteolysis of phagocytosed colloid from follicles

#Diabetes insipidus appeared in a patient after cerebral infarction with impairment of hypothalamus nuclei.
What is the cause for increased urination in this patient?
-Decrease of reabsorption of sodium
-Acceleration of glomerular filtration
+Decrease of water reabsorption
-Decrease of arterial pressure
-Hyperglycemia

#What hormone stimulates inclusion of calcium in osteoblasts of bone tissue in tooth?


-Insulin
-Thyroxin
-Parathormone
-Hydrocortisone
+Calcitonin

#A 10-year-old child has a height of 178 cm and weight of 64 kg. What endocrine gland disorder does this
connected with?
-Sexual glands
-Adrenal glands
-Parathyroid glands
+Pituitary gland
-Thyroid gland

#A patient with thyrotoxicosis has hyperthermia, bulimia, and loss of weight. What kind of impairment does
this connected to?
-Reaction of fat synthesis
-Lyses of adenosintriposphoric acid
+Conjunction oxidation and phosphorylation
-Reaction of citric acid cycle
-Reaction of beta-oxidation of fatty acid

#During an experiment on animals the nervous tracts leading to the crus of hypophysis were broken. That
impaired the entrance of the fallowing hormones into the blood
-Hypophysis hormones
+Vasopressin and oxytocin
-Adenohypophysis hormone
-Thyrotropic hormone
-Adenocorticotropic hormone

#A careless student suddenly meets his dean. What hormone concentration will be increased in student’s
blood more rapidly?
-Somatotropin
-Hydrocortisone
-Corticotropin
-Thyroliberine
+Adrenalin

#A women aged 44 complains of general malaise, pain in the heart area, and considerable increase of body
weight. At examination of this patient following symptoms are revealed: lunar face, hirsutism, BP 165/100
mmHg, height is 164 cm, weight is 103kg, the adipose deposits mainly on the neck, shoulders, abdomen.
What is the main pathogenic mechanism of the woman’s obesity?
-Decrease of thyroid hormone production
-Decrease of glycogen production
+Increase of glucocorticoid production
-Increase of insulin production
-Increase of minerelocorticoid production

#A child has a time disturbance of teeth eruption, enamel anomalies, and its lips and tongue are enlarged.
What hormone deficiency causes these changes?
+Thyroxin
-Parathormone
-Thyrocalcitonin
-Insulin
-Somatotropin

#At clinical examination of a woman it was determined increase of basal metabolism rate by 40%, increased
perspiration, tachycardia, and leanness. What endocrine gland functions are impaired and what direction in?
-Sexual glands, hypofunction
-Cortical substance of adrenal glands, hyperfunction
-Medulla of adrenal glands, hyperfunction
+Thyroid gland, hyperfunction
-Pancreatic gland, hyperfunction

#A young man aged 17 has the signs of retention of sexual development; his height is 184 cm; he is asthenic;
his secondary sexual characters are weakly developed. Sex chromatin is absent in this patient’s cells. At the
age of 4 he endured epidemic parotitis. What was the cause of hypogonodism?
-Deficiency of gonadoliberine
-Deficiency of gonadotropin
-Cryptorchism
+Orchitis
-Chromosomal anomalies (XXY)
#A patient aged 20 complains of polydipsia and abundant urination (up to 10 liters for 24 hrs); concentration
of glucose in his blood is normal; in his urine glucose is absent. What hormone absence may cause such
clinical picture?
-Oxytocin
+Vasopressin
-Insulin
-Hydrocortisone
-Triiodothyronine

#A woman with primary hyperparathyroidism has periodically repeated attacks of renal colic. Ultrasound
examination shows presence of small stones in her kidneys. What is the cause of the formation of these
stones?
-Hypercholesterolemia
-Hyperuricemia
-Hyperkalemia
-Hyperphosphatemia
+Hypercalcemia

#Substances which imitate morphine effects but which are produced in CNS may be used for analgesia.
Indicate them:
-Somatotropin
-Oxytocin
+Beta-endorphin
-Vasopressin
-Calcitonin

#Dilated pupils, dryness of mucous membranes, accelerated heart rate, constipation, and tremor are observed
in some people during emotional exertion. What system activation will lead to such changes in the
organism?
-Pituitary-adrenal system
-Sympathetic nervous system
+Sympathoadrenal system
-Somatic nervous system
-Parasympathetic nervous system

#In ancient India those people who were suspected in committing crimes were given swallow a handful of
dry rice. Criminals couldn’t swallow rice due to decreased salivation (oligosalivation) caused by:
-Decrease of blood supply of salivary glands
+Activation of sympathoadrenal system
-Activation of parasympathetic nucleus of glossopharyngeal nerve
-Inhibition of sympathoadrenal system
-Activation of parasympathetic nucleus of facial nerve

#A patient aged 23 complains of headache, changes of his appearance (increase of size of feet, fingers,
features of the face), hoarseness of the voice, worsening of the memory. The disease has begun 3 years ago
without any causes. Increase of superciliarry arches, nose, and tongue were found during physical
examination of this patient; the blood sugar was normal; the urinalysis is out of changes. What is the cause
of such condition?
-Excessive production of corticosteroids
+Excessive production of somatotropin
-Insufficiency of aldosterone
-Insufficiency of glucagon
-Insufficiency of thyroxin

#Adrenalin is used to prolong Novocain action in infiltration anesthesia. What adrenalin action is this effect
connected with?
-Potentiation of Novocain action at the level of CNS-
-Inhibition of function of nerve ending and conductors-
-Dilatation of vessels-
+Constriction of vessels
-Inhibition of tissue esterases

#A patient aged 80 complains of increased appetite, thirst, excretion of large amount of urine, and worsening
of general condition after taking sweet food. What disease is it?
-Hypercorticoidism
-Hyperthyroidism
-Hypothyroidism
-Diabetes insipidus
+Diabetes mellitus

#Persistent hyperglycemia developed in a patient with Cushing’s syndrome under the influence of excessive
amount of glucocorticoids. What is the mechanism of hyperglycemia in this case?
-Activation of glycogenolysis at the liver
+Activation of gluconeogenesis
-Activation of insulinaze of the liver
-Decrease of hexokinase activation
-Increase of absorption of glucose in intestine

#A man aged 38 is in stress state due to industrial conflict. Which of below mentioned hormones participate
in starting stress reaction of the organism?
+Adrenalin
-ACTH
-Glucagon
-Hydrocortisone
-Thyroxin

#A woman aged 28 addressed to a doctor a month later after delivery with complaints of decreased breast
milk formation. What hormone insufficiency caused this condition?
-ACTH
-Glucagon
-Insulin
+Prolactin
-GH
#While examining a patient a doctor suspected Cushing’s syndrome. What substance determination in
patient’s blood will prove doctor’s supposition?
-Tocopherol
-Adrenalin
-Cholesterol
-Retinol
+Cortisol

#The attacks of convulsions appeared in a patient after resection of thyroid gland. What preparation must be
administered in this case?
-Somatotropin
-Insulin
-Thyroxin
-Prednisolon
+Parathyroidin

#A dog with endocrine pathology had decreased oxygen usage in the state of rest, decreased body
temperature, and decrease of glucose tolerance. What hormone insufficiency may explain the discovered
changes?
-Growth hormone
+Thyroxin
-Gonadotropin
-Adrenocorticotropic hormone
-Insulin

#A patient aged 50 complains of thirst, he drinks much water, has marked polyuria. His blood glucose - 4.8
mmol/L, glucose and acetone bodies are absent in the urine; urine is colourless, its specific gravity - 1.002-
1.004. What is the cause of polyuria?
+Insufficiency of ADH
-Hypothyrosis
-Insulin insufficiency
-Aldosteonism
-Thyrotoxicosis

#A patient aged 40 was hospitalized with complaints of general malaise, convulsions of upper and lower
extremities, his BP - 160/100 mm Hg. The results of laboratory examination of him: blood glucose - 6.5
mmol/L, calcium - 2 mmol/L, phosphates - 1 mmol/L, sodium-160 mmol/L. Urination-700 ml for 24 hours.
What pathology causes such state?
+Hyperaldosteronism
-Hypoaldosteronism
-Hyperparathyroidism
-Thyrotoxycosis
-Rickets

#Pilosis by male type and increase of muscular mass began appearing in a boy of 5 years old who had
developed previously without declination from the age norms. At examination of the patient marked
secondary male sexual signs were revealed, but the size of his testes corresponds to his age. What is the
cause of precocious puberty?
+Androgen producing adrenal tumor
-Hormone producing testis tumor
-Increase of gonadotropin production
-Increase of adrenocorticotropin production
-Increase of gonadoliberin production

#Why must a patient, who has taken prednisolone for rheumatoid arthritis for a long time, avoid contacts
with infectious patients?
+Because of the development of secondary immunodeficiency
-Because of risk of exacerbation of arthritis
-Because of risk of thromboembolic complications development
-Because of the development of lymphopenia
-Because of the blockade of interferon formation
#Liquidator of an accident at Chernobilskaya AES began complaining of increased excitability, nervousness,
heartbeat, decrease of body weight, constant weakness, body tremor, feeling of fever, bad heat endurance.
What gland hyperfunction may be the cause of such state?
+Thyroid gland
-Adenohypophysis
-Adrenal gland
-Medulla of adrenal gland
-Parathyroid gland

#Muscular weakness, adynamia, decrease of body temperature, hypoglycemia, developed in a dog after two-
sided resection of adrenal gland. What other manifestation of adrenal insufficiency may be noted?
-Lymphopenia
+Arterial hypotension
-Increase of glycogen synthesis
-Hypernatriemia
-Hypokalemia

#A woman complains of increased irritability, perspiration, weakness, loss of body weight, tremor of
extremities, increased heartbeat rate, and exophthalmia. What metabolic impairment in the organism
accompanies this disease?
-Increase of adenosine triphosphoric acid synthesis
-Decrease of organism sensitivity to hypoxia
-Weakening of phospholipase activation
+Increase of basal metabolism
-Decrease of cholesterol lysis

#A patient who suffered from severe thyrotoxicosis had been operated on for strumectomy after that
weakness, sensitivity to cold, increase of body weight, paleness and dryness of skin developed. What are the
other manifestations of hypothyrosis?
+Inhibition of CNS activity
-Tachycardia
-Increase of basal metabolism
-Increase of intestinal peristalsis
-Decrease of tolerance to carbohydrates

#In case of hypercortisolism – Cushing’s disease – the following changes in the organism take place:
-Development of cachexia
-Impoverishment of the liver with glycogen
+Hyperglycemia
-Hypotension
-Lymphocytosis

#A patient was admitted to the hospital with complaints of loss of weight, quick fatigability, darkening of
skin. His heart sounds are dull. What are the other manifestations of adrenal insufficiency?
-Lymphopenia
+Adynamia
-Arterial hypertension
-Increase of minute blood volume
-Increased appetite

#A patient admitted to the hospital complains of quick fatigability, loss of weight, hyperpigmentation of the
skin. Her heart sounds are dull. She has pulse rate - 96 beats per minute and BP - 90/50 Hg. What metabolic
impairments are observed in hypocortisolism?
-Hyperkalemia
-Increase of glycogen synthesis
-Hypernatremia
-Hyperhydration
+Hypoglycemia

#The manifestations of hypoparathyrosis developed in a patient after strumectomy. What changes in the
organism are observed in this case?
-Hypophosphatemia
-Resorption of bone tissue
-Acidosis
+Hypocalcaemia
-Decrease of neuromuscular excitability

#Small height, disproportional development of the body, and insufficient mental development were found in
a boy of 10 during examination. What hormone deficiency caused these changes?
+Thyroxin
-Parathormone
-Thyrocalcitonin
-Adenocorticotropic hormone
-Oxytocin

#With the help of indirect calorimetrical measurement it was determined that the basal metabolism of the
patient was 40% lower than the proper one. What endocrine gland hypofunction is the cause of described
changes?
-Adrenal glands
+Thyroid gland
-Epiphysis
-Thymus
-Pancreas

#After suffered sepsis a bronze color of the skin typical for Addison’s disease appeared in a woman aged 27.
Hyperpigmentation occurs due to the increased secretion of:
+Melanocytstimulating hormone
-Growth hormone
-Adrenocorticotropic hormone
-B-lipotropic hormone
-Thyrotropic hormone

#A patient with rheumatic arthritis was given hydrocortisone for a long period of time. He developed
hyperglycemia, polyuria, glycosuria, and thirst. These complications after treatment develop due to the
activation of the process of:
+Gluconeogenesis
-Glycogenolysis
-Glycogenesis
-Glycolysis
-Lipolysis

#Fibrillary muscular tics of arms, legs and face appeared in a woman aged 46 after the operation on thyroid
gland. These disorders may be eliminated by injection of:
+Parathyroidin
-Triiodthyronine
-Thyrotropin
-Thyroxin
-Calcitonin

#Atrophy of the testis developed in an athlete who systemically used androgen hormones. This phenomenon
is due to the inhibition of secretion of:
+Gonadotropic hormone
-Corticoliberine
-Prolactoliberin
-Gonadoliberine
-Testosterone

#During the examination of the patient a doctor found out Cushing’s disease that is characterized by obesity.
It is connected with:
-Excessive use of fats with meal
-Impairment of ventromedial nuclei of hypothalamus
-Inhibition of adrenalin synthesis by adrenal glands
+Production of excessive amount of glycocoticoids
-Hereditary tendency to hyperlipemia
#A patient with diffuse toxic goiter has marked exophthalmia. The appearance of exophthalmia is explained
in this pathology by:
-Antibody circulation against thyroglobulin
-Presence of immunoglobulins in the blood
-Production of exophthalmia factor by hypophysis
+Increase of adrenorecpetor sensitivity to catecholamines
-Pathogenic action of thyroid prostaglandins

#A patient with myxedema came to a doctor. Her face was puffy with poor facial expression; she had
thickened nose and lips. These signs can be explained by:
-Free filtration of sodium in glomeruli
+Accumulation of hydrophilic mucous substances
-Impairment of sensitivity of volumo- and osmoreceptor
-Increase of sodium reapsorption in tubules (canaliculi)
-Increased permeability of capillary walls

#After physic exertion a patient with pheochromocytoma complains of tachycardia, increased arterial
pressure, and sharp pain in epigastric area. These attacks may be explained by:
+Massive release of catecholamines by adrenal glands
-Release of norepinephrine by sympathetic nerves
-Activation of hypothalamus vegetative nuclei
-Increase of thyroid hormone secretion
-Increased synthesis of adrenocorticotrophic hormone

#Excessive secretion of a certain hormone was observed in a patient with pheochromocytoma -a tumor,
which appears from the medulla of adrenal glands. How is this hormone called?
-Glucagon
-Insulin
-Thyroxin
+Adrenalin
-Somatotropin

#Adenoma, which grows from the cells of glomerular zone of adrenal cortex and is the source of excessive
formation of aldosterone was found in a patient. This resulted in a development of primary
hyperaldosteronism or Conn’s disease. What electrolyte exchange does this hormone influence?
-Ferric ion
-Calcium ion
-Magnesium ion
-Chlorine ion
+Sodium ion

#Weakness, thirst, sharp increase of neuromuscular excitability with the development of parathyroid tetany
were observed in a dog 1-2 days later after resection of thyroid glands. What disturbance of electrolyte
exchange takes place in this condition?
-Hypercalcaemia-
+Hypocalcaemia+
-Hypomagnemia-
-Hypermagnemia-
-Hyponatremia-

#Removal of cancer tumor of the testis in a patient before the period of sexual maturity resulted in the
development of eunochoidism and was accompanied by the deficiency of the production of:
+Androgens
-Estrogens
-Kinines
-Prostaglandins
-Cytokines

#Male patient aged 55 had an increase of pituitary gland dimensions and hyperplasia of adrenal cortex. At
examination of the patient: BP-190/90 mm Hg, content of blood glucose is 20 mmol/L; there are glucosuria,
obesity, and histurism. What pathology are these changes typical for?
-Barraker-Simmond disease
-Adipose-genital dystrophy
-Cushing’s syndrome
-Addison’s disease
+Cushing’s disease

#A patient aged 29 had acute massive blood loss during delivery. Then the following changes developed:
acute loss of weight, atrophy of skeletal muscles, thinning of skin, decrease of body temperature,
hypotension, and hypoglycemia. What pathology of pituitary gland is the most possible in this case?
+Sheehan’s disease
-Parhon’s syndrome
-Diabetes insipidus
-Adipose-genital dystrophy
-Pituitary dwarfism

#A 28-years-old patient complains of flaccidity, quick mental and physical fatigue, and dyspeptic disorders.
During the examination following was found out: positive TB tests, hypoglycemia, BP - 90/60 mm Hg,
hyponatremia, skin pigmentation. What disease of adrenal glands underlies observed symptoms?
-Cushing’s syndrome
+Addison’s disease
-Acute insufficiency of adrenal cortex
-Hypofunction of medullar layer of adrenal glands
-Conn’s syndrome

#Which of the signs that develops in hyperthyrosis is the most important for making diagnosis?

-Tachycardia
-Subfebrile temperature
+Increase of basal metabolism
-Increase excitability
-Disorder of sleep
#A patient aged 41 complains of weakness, sweating, fever, tremor of hands, BP-160/90 mm Hg. Diffuse
toxic goiter was diagnosed (Basedow’s diseas). What is the main mechanism of impairment of the function
of cardio-vascular system in this disease?
-Decrease of tonus of sympathetic nervous system tonus
+Increase of tonus of sympathetic nervous system tonus
-Auto immune reactions
-Increase of catabolism
-Hyperthermia

#A girl aged 5 had the symptoms of premature sexual development, menses, growth of mammary gland,
adipose deposits in the field of pelvis and femur. The most possible cause of premature sexual development
is:
-Adrenal adenoma
-Hypopituitarism
-Polycystic ovary syndrome
-Hyperplasia of adrenal glands
+Hormone- active tumor of ovaries

#Conn’s syndrome was diagnosed in a patient who complained of muscular weakness, increased urination
(at night), and increased arterial blood pressure. What is typical for this syndrome?
-Increase of renin, increase of aldosterone, and increase of potassium
-Decrease of renin, increase of aldosterone, and increase of potassium
+Decrease of renin, increase of aldosterone, and decrease of potassium
-Decrease of renin, decrease of aldosterone, and decrease of potassium
-Increase of renin, decrease of aldosterone, and increase of potassium

#A female patient aged 44 complains of general malaise, large increase of body weight, growth of hair on
the face, arrest of menses, BP-165/100 mm Hg. What diagnostic test will help to differentiate Cushing’s
disease from Cushing’s syndrome?
+Level of ACTH in blood plasma
-Level of cortisol in plasma
-Excretion of 17-oxyketosteroids with urine
-X-ray of the skull
-Number of eosinophils in blood

#A patient aged 42 complains of sharp loss of body weight, weakness, appearance of pigmental spots on the
skin and mucus membranes. Addison’s disease was diagnosed in this patient. Which of the following tests is
the most important for diagnostics of Addison’s disease?
-Content of 17-oxyketosteroids in urine
-Level of potassium and sodium in blood plasma
+Tests with injection of ACTH
-Level of cortisol in plasma
-Content of blood glucose

#A 55-years-old man had an attack of angina pectoris which w as accompanied by a severe pain behind the
breastbone and resulted in anuria. What mechanism influenced on the appearance of anuria?
-Excessive somatotropin secretion
+Excessive vasopressin secretion
-Decrease of corticotropin secretion
-Vasopressin deficiency
-Somatotropin deficiency-

#Enlargement of thyroid gland, aloofness, increased basal metabolism, tachycardia, tremor of fingers were
found out in a women aged 52. Psychic excitability was increased in her. The leading part of pathogenesis of
this state is:
+Increase of mitochondrial membrane permeability
-Negative nitrogen balance
-Hypocholesteremia
-Decrease of protein synthesis
-Hyperglycemia

#Thyrotoxicosis was produced in an animal experimentally. For confirming this state it is necessary to
determine the level of metabolism according to food taking and physical load. What experimental method
must be used in this case?
-Determination of iodine content in the thyroid gland
-Determination of concentration of organic iodine in blood
+Direct calorimetry
-Determination of oxygen tension in the blood
-Determination of the content of the gland hormone in the blood

#Three months later after delivery female patient C., aged 30, began to complain of increase of body weight,
excessive adipose deposits, mainly in the area of the neck and face, growth of hair on the upper lip. The
increase of size of cella turcica was determined by X-ray examination. What pathogenic mechanism
underlies determined pathology?
+Hypersecretion of glucocorticoids
-Increase of food taking
-Hyposecretion of thyroxin
-Development of diabetes mellitus
-Birth of a child

#A boy of 14 years old visited endocrinologist. His mother complains of his being behind in physical
development and growth. The boy has proportional constitution, his is 104 cm tall, and his secondary sexual
characters are not marked. The cause of this pathology is hyposecretion of:
+Gonadotropic hormone
-Growth hormone
-All hormones of adenohypophysis
-ACTH
-Thyroid stimulating hormone

#Patient female aged 47 was admitted to the hospital with complaints on increased nervous excitability
disorder of sleep, heartbeat, pain in the heart area, and muscular weakness. Objectively: the patient is thin,
has exophthalmia, tremor of hands, increase of reflexes. She has body temperature - 37.5 oC, heartbeat - 150
beats per minute; at her ECG sinus tachycardia was revealed. Thyroid gland is increased. Arrhythmia in this
patient develops is due to:
+Increase of tonus of sympathetic nervous system
-Decrease of tonus of vagus nerve
-Intoxication
-Metabolic increase in myocardium
-Increase of catchecolamine level

#A 51-years-old male patient has been suffering from tuberculosis for 10 years. He is abusing alcohol. At
last time, complaints of irritability, fast fatigability, syncope, muscular weakness, and loss of weight appear
in him. At examination of the patient: hyperpigmentation of his skin; his BP is 90/60 mmHg. What is the
most possible reason for disease development in this patient?
-Chronic alcohol intoxication
+Impairment of adrenal gland due to tuberculosis
-Alcoholic injury of the liver
-Hypothalamic tumor
-Thyroid gland impairment

#A woman aged 40 complains of general malaise, pain in the heart area, increase of body weight. At
examination of the patient: she is 164 cm height and 104 kg weight; she has predominant deposits of fat at
face, neck, shoulders, and abdomen; her BP – 165/100 mmHg; content of glucose in her blood – 7.8
mmol/L. What endocrine gland function is infringed and what direction at?
+Pituitary gland, basophilic adenoma, hyperfunction
-Pituitary gland, eosinophilic adenoma, hyperfunction
-Sexual glands, hypofunction
-Cortex of adrenal glands, hypofunction
-Thyroid gland, hypofunction

#In a patient aged 23 BP rapidly drops down to 70/40 mmHg after surgical removal of left adrenal gland.
Patient has rapid, weak pulse, vomiting, and cramps; he is covered by cold sweat.What is the most possible
cause for this condition?
-Hypertrophy of the right adrenal gland
-Hypofunction of adenohypophisis
+Atrophy of the right adrenal gland
-Hyperfunction of adenohypophisis
-Hypofunction of neurohypophisis

#A 17-years-old female patient, who has been ill for 6 months, complains of appearance of mustache and
beard, roughness of voice, and absence of menses. At examination of the patient hirsutism and undeveloped
mammal glands and sexual organs were revealed. What glands impairment has lead to this pathology?
+Adrenal glands
-Thyroid gland
-Neurohypohpisis
-Adenohypophisis
-Sexual glands

#A child aged 4 lags behind at physical development, sleeps badly, and has increased irritability, thirst, and
polyuria. Glucose is absent in patient’s urine. Patient has negative reaction to vasopressin injection. What is
the mechanism of revealed disorders?
+Disturbance of realization of hormone biological activity
-Impairment of neural regulation of endocrine glands
-Disorders of biosynthesis and secretion of hormones
-Disorders of neuroendocrine regulation
-Disorders of feedback mechanisms

#Loss of body weight, fatigue, pulling teeth out, and coming hairs out began in a patient in several months
after delivery. She often became ill. At examination of the patient: subcutaneous adipose tissue is virtually
absent; BP – 90/55 mmHg; body temperature – 36.0 oC. Biochemical analysis of patient’s blood: glucose
content – 3.0 mmol/L. There is increased level of growth hormone and adrenocorticotropic hormone in her
blood. What kind of pituitary functions is impaired in this patient?
-Pituitary dwarfism
+Panhypopituitarism
-Acromegaly
-Cushing’s disease
-Diabetes insipidus

#Hans Sallie, author of doctrine of stress, showed that different stimuli (heat, cold, pain) always evoke
standard non-specific reaction: 1) involution in thymico-lymphatic system; 2) hemorrhage acute ulcers of
stomach and duodenum. Name the third compound of classical triad

+Hypertrophy of adrenal cortex


-Hypertrophy of beta cells of Langerhans’ islets
-Myocardial infarction
-Parodontitis
-Cerebral hemorrhage

#Polyuria and polydipsia developed in a 40-years-old patient accordingly to impairment of hypothalamic-


pituitary connective way. What hormone deficiency evokes this changes?
-Oxytocin
+Antidiuretic hormone
-Adrenocorticotropic hormone
-Growth hormone
-Thyroid stimulating hormone

#In a patient excessive development of bones and soft tissues of his face, enlargement of his tongue, and
widened interspaces between teeth in enlarged teeth arch were found out. What changes of hormone
secretion are the most possible in this patient?
-Elevated secretion of insulin
-Increased secretion of thyroxin
+Increased secretion of growth hormone
-Reduced secretion of growth hormone
-Elevated secretion of vasopressin

#A boy aged 9 is in endocrinology department because of increase bone fragility. What endocrine organ is
impaired?
-Adrenal glands
-Thymus
-Thyroid gland
+Parathyroid gland
-Epiphysis

#A child has infringements of teeth enamel and dentin formation because of reduced content of calcium in
his blood. What hormone deficiency may evoke these changes?
-Growth hormone
-Triiodthyronin
+Parathormone
-Thyroxin
-Thyrocalcitonin

#By X-ray examination of scull bones enlargement of cella turcica, thinned anterior inclined appendices, and
destruction of different places of cella turcica were revealed. What endocrine gland tumor may lead to such
bone destruction?
-Thyroid gland
-Epiphysis
-Adrenal gland
-Thymus
+Pituitary gland

#After the operation on thyroid gland tetany developed in a patient that manifested in convulsions of striped
muscles. What does this complication may be connected to?
+Mistaken removal of parathyroid glands
-Insufficient removal of thyroid tissue
-Hyperfunction of adrenal glands
-Hypofunction of adrenal glands
-Elevated blood content of thyroid-stimulating hormone

#A female patient with hyperthyroidism has increased body temperature. What is the mechanism of body
temperature elevation?
+Separation of oxidation and oxidative phosphorilation
-Increase utilization of glucose by tissues
-Intensification of glycogenolysis
-Intensification of protein catabolism
-Elevation of oxidation of fat in liver

#A woman has fatigue, sleepiness, apathy, worsening of memory, and edemas. These symptoms appeared in
several years after change of dwelling. “Endemic goiter” is diagnosed in this patient. What substance deficit
in water and food may lead to this disease?
+Iodine
-Fluoride
-Iron
-Calcium
-Magnesium
#A 38-years-old patient complains of thirst (he drinks 8 liters of water daily), polyuria, loss of weight, and
general malaise. He has been sick for 6 months. Urinalysis of the patient shows: special gravity – 1.001,
leukocytes – 1 to 2 in field of vision, protein – trace amount. What reason evokes the constant polyuria in
this patient?
+Reduction of ADH production
-Injury of renal glomerules
-Injury of renal tubules
-Elevation of osmotic pressure of urine
-Increase of oncotic pressure of urine

#When stomatological examining the patient aged 18 the following symptoms were revealed: woman
features of his face, anomalies of form of crowns in some teeth, catarrhal gingivitis, and resorption of
interdental junctures. Cytological examination of mucous membrane epithelium discovers that cells contain
1 Barr’s body. What endocrine pathology are these manifestations the most probably connected to?
+Hypogonadism
-Hypergonadism
-Cretinism
-Thyrotoxicosis
-Chronic hypoparathyroidism

#A 20-years-old patient has hypoplasia of enamel and impaired formation of dentin in his teeth after surgical
removal of thyroid gland. What hormone deficiency became the reason for indicated pathology?
+Parathormone+
-Thyroxin
-Thyroid-stimulating hormone
-Thyroid-stimulating hormone-releasing factor
-Thyrocalcitonin

#A patient, who lives in mountain region, was admitted to the hospital with diagnosis of endemic goiter.
What is characteristic for this disease?
+Decreasing of thyroxin and triiodthyronin synthesis
-Increase of thyroid hormone production
-Toxic action of thyroxin and triiodthyronin
-Chronic hypoparathyroidism

#A child aged 2.5 has retardation in physical development, absence of appetite, thirst, and polyuria; he sleeps
badly. Sugar was not found in the patient’ urine. Which of endocrine pathologies is the reason for disorders
of water-salt metabolism?
+Hyposecretion of antidiuretic hormone
-Hypersecretion of antidiuretic hormone
-Secondary hyperaldosteronism
-Hyposecretion of growth hormone
-Hyposecretion of adrenocorticotropic hormone

#What pathological conditions the tumor growing from reticular zone of adrenal gland may lead to?
+Virilization of female organism
-Elevation in content of glucocorticoids in blood
-Tachycardia
-Elevation of content of mineralocorticoids in blood

#What changes result from separation in oxidation and oxidative phosphorilation?


+Elevation in heat production
-Decrease of free oxidation part
-Decrease in heat production
-Enhancement of functional activity of organs
-Increase in ATP production

#What changes in adult organism may be evoked by hypersecretion of growth hormone?


+Development of diabetes mellitus
-Intensification of lipolysis
-Intensification of urea production
-Reduction of transport of amino acids to the cell
-Increase in content of calcium in blood

#A 40-years-old patient complains of decreased capability of working, sleepiness, sensitivity to cold,


fragility and coming out of hairs, dryness of skin, and edema of face and extremities. There is no formation
of pit under the pressing at the anterior surface of patient’s calves. Patient has slow tendon reflexes and
bradicardia. What infringement do these changes result from?
+Hypothyroidism
-Adrenal insufficiency
-Hypoparathyroidism
-Vitamin C deficiency
-Iron deficiency

#Influence of unfavorable psycho-emotional factor upon a patient results in development of non-specific


pathological process, which course consists of 3 stages: 1) anxiety reaction; 2) stage of resistance; 3) stage of
exhaustion. This process involves system hypothalamus – pituitary – adrenal cortex. What is the name for
this process?
+Stress
-Parabiosis
-Adaptation
-Compensation
-Dominance

#A doctor suspects the hypothyroid goiter in a patient with enlarged thyroid gland. What sign has a decisive
importance for establishing the diagnosis?
+Decrease of basal metabolism
-Bradicardia
-Edemas
-Arterial hypotension
-Hypodynamism
#Arterial hypotension, muscular weakness, and periodic convulsions appear in the patient with hepatic
cirrhosis. Content of sodium is increased and content of potassium is decreased in patient’s blood. What kind
of endocrine disorders underlies this symptoms?
+Secondary hyperaldosteronism
-Hypopituitarism
-Primary hyperaldosteronism
-Hyperpituitarism
-Hypoaldosteronism
#Prolonged intake of mineralocorticoids led to development of muscular weakness in a patient. What gives
rise to these symptoms?
+Hypokalemia
-Hyperkalemia
-Hypernatremia
-Hyponatremia
-Hypervolemia

#What hormone increased secretion causes becoming thin at the period of enhanced growth?
+Growth hormone
-Prolactin
-Glucagon
-Progesterone
-Insulin

#What kind of endocrine disorders lead to obesity?


+Hypercortisolism
-Hypofunction of adrenal glands
-Hyperparathyroidism
-Hypergonadism
-Hyperaldosteronism

#A 50-years-old female patient, who was operated on thyroid gland for diffuse toxic goiter, begins
complaining that she is spiritless and slow, has fast fatigability, increased working ability, sleepiness,
worsening of memory, and increasing of body weight. At examination the patient has dry skin, edematous
face, and striped nails with broken edges. What is your assumable diagnosis?
+Myxedema
-Cushing’s disease
-Acromegaly
-Obesity
-Thyrotoxicosis

#A 25-years-old male patient, who endured influenza, complains of increased thirst (he drinks 5 to 6 liters of
water daily), frequent abundant urination, and loss of body weight. Patient has dry mucous membranes and
dry skin with reduced turgor. Urinalysis of this patient displays: urine is colorless, its special gravity –
1.000-1.004, leucocytes and erythrocytes – 1 to 2 in field of vision. What pathology does this patient suffers
from?
+Diabetes insipidus
-Diabetes mellitus
-Conn’s disease
-Pheochromocytoma
-Secondary hyperaldosteronism

#When prophylactic examining of the first-year pupils in two of them lag in their height by 2-3 sigma
deviations was indicated. No another disorders were revealed. What hormone deficiency could lead to this
pathology?
+Growth hormone
-Sexual hormones
-Insulin
-Thyroid hormones
-Glucocorticoids

Pathophysiology of the nervous system

#A 38-year-old man is going undertreatment in a hospital for schizophrenia. Blood contents of glucose,
ketone bodies, and urea in him are norm. Shock therapy with regular injections of insulin was led to the
development of insulin coma, after that the patient’s condition improved. What was the most possible
reason for development of insulin coma?
-Glucosuria
-Dehydratation of tissues
+Hypoglycemia
-Metabolic acidosis
-Ketonemia

#Smell of apples is felt from the patient in comatose state. The content of glucose is 18 mmol/L. Which
coma is more possible in this case?
-Hypoglycemic
-Toxic
-Hyperosmolar
+Ketoacidotic
-Lactatacidotic

#A patient is drowsy, with cloudiness of consciousness, his reaction to the strong stimuli is slow. His
skin is pale and dry. He has edemas, muscular fibrillar tremor, midriasis, Cheyne-Stokes respiration with
ammonium smell. Pericardial rub was revealed during auscultation. Which type of coma has been
developed in this patient?
-Ketoacidotic
+Renal
-Hyperosmolar
-Hepatic
-Apoplectic

#Increase of blood pressure and rapid pulse are noticed in a sportsman at the start before competitions.
Influence of which part of the CNS can above-mentioned changes be explained?
+Cortex of hemispheares
-Medulla oblongata
-Mesencephalon
-Diencephalons
-Hypothalamus

#The close clinical examination of the patient, admitted to the reanimation department in unconscious
condition allowed to make a conclusion that the patient was in uremic comatose state. Which symptom is
more characteristic for this type of coma?
-Hyperglycemic
-Acetone smell from his mouth
-Polyuria
+Hypernitremia
-Non-gas alkalosis

#The close clinical examination of the patient, admitted to the reanimation department in unconscious
condition allowed to make a conclusion that the patient was in state of diabetic coma. Which symptom is
more characteristic for this type of coma?
+Hyperglycemia
-Smell of ammonium from the mouth
-Anuria
-Metabolic alkalosis
-Decrease of content of nonprotein nitrogen in serum

#Ptyalism, bradycardia (heart rate 45 beats per minute), miosis are observed in a man. What is the most
possible reason for such changes?
-Increase of sympathetic influence
+Increase of parasympathetic influence
-Decrease of sympathetic influence
-Decrease of parasympathetic influence
-Decrease of vegetative influence

#The reactions, typical for stimulation of parasympathetic nerves, appeared after irritation of
throphogenic zone of hypothalamus, including preoptic nucleas and anterior hypothalamic area. What
appears in this case?
-Tachycardia
-Mydriasis
-Exophthalmus
+Decrease of BP
-Hyperglycemia

#After amputating the upper extremity a patient had a bad pain in it. Which mechanism of the pain
feeling formation is more possible in this case?
+Phantom
-Reflex
-Hyposecretion of endorphin
-Hypersecretion of endorphin
-Hyposecretion of encephalin

#A patient had hemiplegia after insult. What disorder is observed in this case?
-Taste
-Balance
+Movement
-Vision
-Hearing

You might also like